You are on page 1of 538

2009 PREP SA on CD-ROM

Question: 1

You are evaluating a 6-month-old child who has a ventricular septal defect and is scheduled for
cardiac surgery. The childs weight is 6 kg (3rd percentile), length is at the 30th percentile, and
head circumference is at the 50th percentile. His mother states she prepares the formula by
adding 1 scoop of powder to 2 oz of water. She estimates that he drinks 24 oz of formula per
day. You estimate the babys intake is approximately 500 kcal per day of cow milk formula,
which is the recommended dietary allowance (RDA) for his age. According to his mother, he
spits up three times a day and passes two soft stools daily. On physical examination, you hear a
3/6 holosystolic murmur and palpate the liver 1 cm below the right costal margin.
Of the following, the BEST explanation for the childs malnutrition is

A. caloric requirements exceeding the RDA


B. cow milk protein intolerance
C. incorrect preparation of the formula
D. pathologic gastroesophageal reflux
E. undiagnosed pancreatic insufficiency

Copyright 2009 by the American Academy of Pediatrics

page 1

2009 PREP SA on CD-ROM


Critique: 1

Preferred Response: A

Children who have large ventricular septal defects, such as described for the child in the
vignette, have increased pulmonary blood flow and may have ventricular hypertrophy and heart
failure. Because their hearts have to work harder, their caloric needs often are increased above
the recommended dietary allowance (RDA) for healthy children. The absence of significant
vomiting, diarrhea, or rectal bleeding suggests that the patient does not have significant
gastroesophageal reflux, cow milk protein intolerance, or pancreatic insufficiency. Incorrect
preparation of the formula always should be considered, but the mother's reported preparation is
correct.
The RDA is defined as "a nutrient intake level that is ... sufficient to meet the nutrient
requirements of 97% of healthy individuals" in a given group, categorized by sex and age. RDAs
are developed by the Food and Nutrition Board of the Institute of Medicine in collaboration with
Health Canada. A full list of RDAs may be found in many reference textbooks and at the Food
and Nutrition Information Center on the United States Department of Agriculture web site
(http://fnic.nal.usda.gov). RDAs estimate energy requirements of healthy children and do not
adjust for chronic illness states that increase caloric needs, such as cystic fibrosis or congenital
heart disease. Children who have chronic illnesses often require more calories than the RDA to
grow and develop.
The estimated daily caloric requirement of a patient is the aggregate of the patient's basal
metabolic rate and physical activity. Various mathematic equations can be used to estimate the
daily caloric intake of children and adults of different ages. Such equations usually take into
account the individual's age, sex, physical activity level, and either the weight and height or,
preferably, the body surface area. For children who have chronic illnesses, more accurate
estimates of daily caloric requirements can be obtained by using a laboratory-based technique,
such as indirect calorimetry.
References:
McDaniel NL. Ventricular and atrial septal defects. Pediatr Rev. 2001;22:265-270. Available at:
http://pedsinreview.aappublications.org/cgi/content/full/22/8/265
Sonneville K. Nutritional requirements: dietary reference intakes. In: Hendricks KM, Duggan C.
Manual of Pediatric Nutrition. 4th ed. Hamilton, Ontario, Canada: BC Decker; 2005:83-100

Copyright 2009 by the American Academy of Pediatrics

page 2

2009 PREP SA on CD-ROM


Question: 2

You are admitting a 750-g female infant to the neonatal intensive care unit (NICU) for treatment
of respiratory distress and presumed sepsis. The pregnancy was complicated by
chorioamnionitis and preterm labor. The infants trachea was intubated, a single dose of
exogenous surfactant administered, and both an umbilical venous catheter and umbilical arterial
catheter were placed successfully in the delivery room. In the NICU, the infant is placed on a
radiant warmer. The nurse caring for the infant asks if the infant will need to be transferred to an
isolette incubator.
Of the following, the MOST likely reason for this infant to be relocated into an isolette incubator is

A. avoidance of light damage to the eyes


B. bronzing of the skin under the radiant warmer
C. inability to maintain core temperature on a radiant warmer
D. increased risk for infection under the radiant warmer
E. increased transcutaneous water loss under the radiant warmer

Copyright 2009 by the American Academy of Pediatrics

page 3

2009 PREP SA on CD-ROM


Critique: 2

Preferred Response: E

The extremely low gestational-age newborn (ELGAN) who has a birthweight of less than 1,000 g
is at great risk for transcutaneous evaporative water loss in an open air, nonhumidified
environment such as a radiant warmer. This is most concerning in the first 24 to 72 hours of
postnatal life, diminishing thereafter as the infant's skin becomes cornified. The use of a
contained, convectively heated, and humidified environment can reduce transcutaneous
evaporative water loss best in the first few days of the ELGAN's postnatal life.
Ambient light is not a contributing factor to retinopathy of prematurity, for which the ELGAN
is also at risk. Although excessive thermal warming can occur under the radiant warmer,
bronzing of the skin is related to the use of phototherapy lights in newborns who have
conjugated hyperbilirubinemia. The ELGAN's core temperature can be maintained using a
radiant warmer, but evaporative and convective heat losses remain a concern over lengthy
periods of time, and differences in extremity, head, and core temperatures may affect perfusion
and acid-base status. There is no increased risk for infection on the radiant warmer compared
with the isolette incubator.
References:
Dollberg S, Hoath SB. Temperature regulation in preterm infants: role of the skin-environment
interface. NeoReviews. 2001;2:e282-e291. Available for subscription at:
http://neoreviews.aappublications.org/cgi/content/full/2/12/e282
Korones SB. An encapsulated history of thermoregulation in the neonate. NeoReviews.
2004;5:e78-e85. Available for subscription at:
http://neoreviews.aappublications.org/cgi/content/full/5/3/e78
Sedin G. The thermal environment of the newborn infant. In: Martin RJ, Fanaroff AA, Walsh MC,
eds. Fanaroff and Martin's Neonatal-Perinatal Medicine. 8th ed. Philadelphia, Pa: Mosby
Elsevier; 2006:585-596

Copyright 2009 by the American Academy of Pediatrics

page 4

2009 PREP SA on CD-ROM


Question: 3

You are seeing a 1-month-old girl for follow-up after a hospitalization for acute gastroenteritis
caused by rotavirus. Her diarrhea had decreased in the hospital while taking oral rehydration
solution, but when her mother resumed her usual cow milk formula, the girl began to have an
increased number of very watery stools. She appears well hydrated, and findings on her
abdominal examination are normal.
Of the following, the MOST appropriate approach to managing this infants diarrhea is to

A. change to a lactose-free formula for the next few days


B. dilute the cow milk formula with oral rehydration solution for the next few days
C. give her only oral rehydration solution until the diarrhea resolves
D. readmit her to the hospital for administration of intravenous fluids
E. repeat her stool studies to confirm the diagnosis of rotavirus infection

Copyright 2009 by the American Academy of Pediatrics

page 5

2009 PREP SA on CD-ROM


Critique: 3

Preferred Response: A

The infant described in the vignette most likely has lactase deficiency due to rotavirus infection.
Lactase is an enzyme found in the most superficial villous portion of the intestinal brush border,
which hydrolyzes lactose to glucose and galactose. Lactase deficiency may have several
causes in children and adults. Primary lactase deficiency, the most common type, is a
genetically determined condition that affects children and adults at different ages but is unusual
before 5 years. Symptoms include abdominal distention, bloating, flatulence, or nausea after the
ingestion of lactose, with the amount of lactose needed to cause such symptoms varying from
person to person. The diagnosis is made by breath hydrogen testing, and management consists
of removing some or all lactose from the diet. Congenital lactase deficiency is extremely rare.
Secondary lactase deficiency may develop after an infectious gastroenteritis, such as
rotavirus, giardiasis, or cryptosporidiosis. Other causes include celiac disease and enteropathy
related to immunodeficiency. Secondary lactase deficiency is suggested when a child who has a
recent diarrheal illness experiences worsening diarrhea or bloating after the reintroduction of
lactose into the diet, as described for the girl in the vignette. Most children who have
gastroenteritis do not develop lactase deficiency. For this reason, most infants can tolerate and
should continue taking human milk or standard lactose-containing formula throughout a diarrheal
illness. For very young infants (eg, <3 months old), such as the one described in the vignette, or
those who have significant fluid losses, a lactose-free formula may be attempted until the
diarrhea resolves. Infants who are breastfed should be encouraged to continue breastfeeding,
even if secondary lactase deficiency is suspected.
Giving full-strength formula or human milk is recommended to supply the child with sufficient
calories during the recovery phase of a diarrheal illness; therefore, diluting the formula or
providing only oral rehydration solution is inappropriate. If the child is not vomiting, oral hydration
is optimal, and intravenous hydration is not necessary. There is no need to confirm the
diagnosis of rotavirus infection; doing so would not alter management plans.
References:
Dalby-Payne J, Elliott E. Gastroenteritis in children. BMJ Clinical Evidence. 2007. Available for
subscription at: http://clinicalevidence.bmj.com/ceweb/conditions/chd/0314/0314.jsp
Heyman MB; Committee on Nutrition. Lactose intolerance in infants, children, and adolescents.
Pediatrics. 2006;118:1279-1286. Available at:
http://pediatrics.aappublications.org/cgi/content/full/118/3/1279
King CK, Glass R, Bresee, Duggan C. Managing acute gastroenteritis among children: oral
rehydration, maintenance, and nutritional therapy. MMWR Morbid Mortal Wkly Rep Recomm
Rep. 2003;52(RR-16):1-16. Available at:
http://www.cdc.gov/mmwr/preview/mmwrhtml/rr5216a1.htm

Copyright 2009 by the American Academy of Pediatrics

page 6

2009 PREP SA on CD-ROM


Question: 4

During a prenatal visit with expectant parents, they report that they are strict vegans. They ask
you to advise them on a healthy diet and any required supplements. The mother plans to
breastfeed the newborn exclusively for the first 6 months.
Of the following, you are MOST likely to tell them that their newborn may require supplemental

A. calcium
B. folate
C. iron
D. vitamin B6
E. vitamin B12

Copyright 2009 by the American Academy of Pediatrics

page 7

2009 PREP SA on CD-ROM


Critique: 4

Preferred Response: E

A vegan diet, by definition, excludes all foods derived from animal products. A lacto-ovovegetarian diet may include milk and eggs. Although a vegan diet may be healthy, there is a risk
for vitamin B12 deficiency because vitamin B12 is only found in foods of animal origin.
Breastfeeding vegan mothers may produce milk that is deficient in this vitamin and require
supplementation that generally is achieved by continuing the consumption of prenatal vitamins
containing vitamin B12.
The recommended supplementation for breastfed vegan infants to prevent vitamin B12
deficiency is 0.4 mcg/day during the first 6 postnatal months and 0.5 mcg/day from 6 months to
1 year of age. Vegan infants who are not breastfed should receive iron-fortified soy infant
formula until 1 year of age to avoid deficiencies in iron. Vegan infants require no other mineral or
vitamin supplementation.
Vegan diets in older children and adolescents may be low in calcium (similar to the typical
American "teenage diet" that contains less than the recommended intake of dairy products), and
the zinc consumption may be relatively low due to the absence of phytate, which renders zinc
more bioavailable. Children who follow vegan diets may have relatively diminished overall energy
intake because such diets commonly are low in fat and high in fiber.
Review of nutrient intake and energy intake in conjunction with growth curves of children
eating vegan diets in both the United States and the United Kingdom demonstrate no significant
health issues. Height and weight measured in vegan populations may be slightly lower than
average but not in the range of failure to thrive or short stature. Adolescents eating vegan diets
are more likely than adolescents eating a typical American diet to meet nutritional goals, including
recommended intake of fruits and vegetables. Vegan adolescents are less likely to be obese
because they consume fewer foods high in fat. However, they remain at risk for vitamin B12
deficiency and should consume at least a daily multivitamin. They are less likely to have anemia
but just as likely to have low calcium intake as their non-vegan peers. Adolescents who follow a
lacto-ovo-vegetarian diet are less likely to have deficiencies in vitamin B12, calcium, and iron.
Folate and vitamin B6 are not likely to be deficient in persons who consume vegan diets
because those nutrients are found in many legumes, fruits, and vegetables that are the
mainstays of the diet.
References:
Kleinman RE. Nutritional aspects of vegetarian diets. In: Pediatric Nutrition Handbook. 5th ed. Elk
Grove Village, Ill: American Academy of Pediatrics; 2003:191-208
Mangels AR, Messina V. Considerations in planning vegan diets: infants. J Am Diet Assoc.
2001:101:670-677. Abstract available at: http://www.ncbi.nlm.nih.gov/pubmed/11424546
Messina V, Mangels AR. Considerations in planning vegan diets: children. J Am Diet Assoc.
2001:101:661-669. Abstract available at: http://www.ncbi.nlm.nih.gov/pubmed/11424545
Moilanen BC. Vegan diets in infants, children and adolescents. Pediatr Rev. 2004:25:174-176.
Available at: http://pedsinreview.aappublications.org/cgi/content/full/25/5/174
Perry CL, McGuire MT, Neumark-Sztainer D, Story M. Adolescent vegetarians: how well do
their dietary patterns meet the Healthy People 2010 objectives? Arch Pediatr Adolesc Med.
2002; 156:431-437. Available at: http://archpedi.ama-assn.org/cgi/content/full/156/5/431

Copyright 2009 by the American Academy of Pediatrics

page 8

2009 PREP SA on CD-ROM


Question: 5

You are treating a child who has suffered a splenic injury and is being transfused with large
volumes of packed red blood cells for severe anemia. He weighs 10 kg and has received 4 units
thus far.
Of the following, the finding on electrocardiography that is MOST likely to represent a serious
complication of his therapy is

A. atrial flutter
B. delta waves
C. prominent U waves
D. supraventricular tachycardia
E. tall-peaked T waves

Copyright 2009 by the American Academy of Pediatrics

page 9

2009 PREP SA on CD-ROM


Critique: 5

Preferred Response: E

Administration of fluids and blood products can be essential for resuscitation of the trauma
patient but may lead to potentially dangerous electrolyte imbalances. Recognition of these
abnormalities, which may be subtle, can be important and even lifesaving. When large volumes
of blood are transfused, as reported for the child in the vignette, hyperkalemia may occur, which
is believed to be the result of extravasation of potassium from the red blood cells that have been
irradiated and stored over time. Hyperkalemia can be associated with paresthesias, weakness,
and tingling, although cardiac toxicity typically precedes such symptoms. Severe cardiac
rhythym changes may begin abruptly. The classic electrocardiographic sign of hyperkalemia is
tall, peaked T waves (Item C5A), particularly as the serum potassium concentration approaches
or exceeds 5.0 to 6.0 mEq/L (5.0 to 6.0 mmol/L). As hyperkalemia progresses, other changes in
the ECG, such as widening of the QRS complex, may be noted. The rhythm changes that occur
as a result of hyperkalemia, including ventricular arrhythmias, may develop abruptly, leading to
sudden changes in the patient's clinical condition.
Another potential electrolyte complication of fluid and blood administration is hypocalcemia,
which results from the citrate-containing anticoagulants that bind free calcium. Hypocalcemia
may manifest on ECG as a prolonged QT interval, which tends to widen as the ionized calcium
concentrations decrease.
Atrial flutter, a primary disease of the atrial tissue, would not be expected in the patient
described in the vignette. The delta wave refers to the ECG finding of pre-excitation, seen in
conditions such as the Wolff-Parkinson-White syndrome that have an associated "bypass" tract
allowing for excitement of the His-Purkinje system without passage through the atrioventricular
node (Item C5B).
The U wave may be seen in hypokalemia (Item C5C), particularly as concentrations
decrease below 2.7 mEq/L (2.7 mmol/L), or hypercalcemia (calcium concentrations exceeding
12.0 mg/dL [3.0 mmol/L]), which would not be expected in the patient described in the vignette.
Hypercalcemia also may lead to a diminished QT interval, and with more severe hypercalcemia,
second- or third-degree heart block (Item C5D) may develop. Supraventricular tachycardia
(Item C5E) would not be expected to occur as a result of a large-volume transfusion process.
References:
Galel SA, Naiman JL. Use of blood and blood products. In: Rudolph CD, Rudolph AM, eds.
Rudolph's Pediatrics. 21st ed. New York, NY: McGraw-Hill Medical Publishing Division;
2003:1576-1581
Vetter V. Arrhythmias. In: Moller JH, Hoffman JIE, eds. Pediatric Cardiovascular Medicine.
Philadelphia, Pa: Churchill Livingstone; 2000:833-884

Copyright 2009 by the American Academy of Pediatrics

page 10

2009 PREP SA on CD-ROM


Question: 6

A 15-year-old girl presents to the emergency department with a 4-week history of nasal
drainage and face pain and a 2-week history of frontal headaches and fatigue. Her mother
complains that her daughter has an "attitude" and has not been respectful or seemed to care
about anything for the past 2 weeks. The daughter awoke this morning with a headache and
vomited. On physical examination, the adolescent is afebrile and has normal vital signs. She
responds slowly to questions and is not oriented to the date. She complains of pain to palpation
of her cheeks and forehead. She has no nuchal rigidity and no focal weakness. The remainder
of the physical examination findings are normal.
Of the following, the BEST initial diagnostic procedure is

A. computed tomography scan of the head with intravenous contrast


B. emergent electroencephalography to rule out nonconvulsive status epilepticus
C. lumbar puncture to rule out meningitis
D. nasal swab for bacterial culture
E. urine drug screen for barbiturates, amphetamines, and cocaine

Copyright 2009 by the American Academy of Pediatrics

page 11

2009 PREP SA on CD-ROM


Critique: 6

Preferred Response: A

The subacute onset of mental status changes described for the adolescent in the vignette
warrants an emergency evaluation. In most cases, neuroimaging is indicated, along with
appropriate laboratory testing.
The relatively nonspecific pain and what her mother perceives as common emotional
problems (apathy in a teenager) probably represent early frontal lobe symptoms. The headache
on awakening and vomiting are concerning for increased intracranial pressure (ICP). Confusion
and psychomotor retardation on the mental status examination indicate involvement of the
central nervous system. A focal, ischemic, ictal, infectious/inflammatory, or toxic/metabolic
process must be identified urgently. A brain abscess is suggested by the prominent facial pain in
this setting; the sinuses are a common source of brain abscesses (Item C6). Brain abscesses
often present only with nonspecific pain and not with fever.
Head computed tomography (CT) scan is preferred for this patient because the
constellation of pain, confusion, and morning vomiting makes a focal intracranial mass a
possibility. Increased ICP is associated with morning vomiting because ICP is highest in the
morning. Contrast is recommended because of the insidious onset, which could indicate either a
neoplasm or infectious process. Intravenous contrast is not needed for all neuroimaging
procedures. However, it increases the diagnostic yield of imaging studies where either neoplasm
or infection is suspected because both typically involve some degradation of the blood-brain
barrier or hypervascularity, resulting in contrast enhancement at the site of the lesion. Magnetic
resonance imaging (MRI) with contrast also is a good choice. The advantage of MRI is higher
spatial and soft-tissue resolution. Disadvantages of MRI compared with CT include: 1) less
availability for emergency department studies; 2) need for pharmacologic sedation in agitated
patients because sedation affects mental status, thereby masking disease-related mental
status; 3) longer time in the scanner, which could delay treatment decisions; and 4) cost. Thus,
in most cases, a head CT scan with contrast is preferred as the initial study in the emergency
department.
Electroencephalography (EEG) is an important test for assessment of a patient who has
encephalopathy of unclear cause to rule out nonconvulsive status epilepticus (NCSE),
particularly if the patient is known to have epilepsy. If an EEG cannot be obtained rapidly,
intravenous administration of 0.1 mg/kg lorazepam can treat NCSE immediately, although this
would not clear confusion about other causes. For this adolescent, the facial pain makes the
diagnosis of NCSE less likely than a brain abscess.
A lumbar puncture may be needed to rule out meningitis or encephalitis, but the pain and
morning vomiting more strongly suggest the possibility of an intracranial mass. Lumbar puncture
prior to head CT is not advised in this case because it could reduce pressure below the foramen
magnum and result in herniation from the supratentorial mass. Toxicology screening and nasal
swabs are reasonable but not the preferred initial diagnostic tests because they will not affect
emergency management of the increased intracranial pressure.
References:
Goodkin HP, Harper MB, Pomeroy SL. Intracerebral abscess in children: historical trends at
Children's Hospital Boston. Pediatrics. 2004;113:1765-1770. Available at:
http://pediatrics.aappublications.org/cgi/content/full/113/6/1765
Haslam RHA. Brain abscess. In: Kliegman RM, Behrman RE, Jenson HB, Stanton BF, eds.
Nelson Textbook of Pediatrics. 18th ed. Philadelphia, Pa: Saunders Elsevier; 2007:2524-2525
Kan L, Nagelberg J, Maytal J. Headaches in a pediatric emergency department: etiology,
imaging, and treatment. Headache. 2000;40:25-29. Abstract available at:
http://www.ncbi.nlm.nih.gov/pubmed/10759899

Copyright 2009 by the American Academy of Pediatrics

page 12

2009 PREP SA on CD-ROM


Question: 7

A 5-year-old girl who is new to your practice presents to the clinic for a prekindergarten physical
examination. Her primary caretaker, the maternal grandmother, reports that the childs mother
used multiple street drugs throughout her pregnancy as well as medications prescribed for
seizure and bipolar disorders. The grandmother is concerned that this childs speech
development is delayed. On physical examination, you note that the girl has wide-spaced eyes,
a short nose, and midface hypoplasia.
Of the following, the substance that is MOST likely to be associated with this childs dysmorphic
features is

A. lithium
B. lysergic acid diethylamide (LSD)
C. marijuana
D. methamphetamine
E. phenobarbital

Copyright 2009 by the American Academy of Pediatrics

page 13

2009 PREP SA on CD-ROM


Critique: 7

Preferred Response: E

The features described for the child in the vignette are most consistent with fetal anticonvulsant
syndrome, which can occur following exposure to numerous medications, including phenytoin,
carbamazepine, valproate, and phenobarbital. Multiple authors have observed a 10% to 20%
incidence of birth defects in infants exposed to phenobarbital in utero. Anomalies include midface
hypoplasia, ocular hypertelorism (Item C7A), nail hypoplasia (Item C7B), cleft lip+/-cleft palate,
and heart defects as well as developmental delay and pre- and postnatal growth failure.
Phenobarbital-exposed newborns may exhibit a withdrawal syndrome that is evidenced by
tremulousness and increased activity. Of interest, such infants are likely to have lower serum
bilirubin concentrations than nonexposed neonates.
Prenatal exposure to lithium is associated with an increased risk of cardiac malformations
(eg, Ebstein anomaly) in the fetus. If the mother takes lithium near term, the exposed neonate
may have cyanosis, hypotonia, abnormalities of cardiac rhythm, goiter, hypothyroidism, and
nephrogenic diabetes insipidus. Lithium exposure is not associated with dysmorphic features or
developmental delays.
Despite popular belief, lysergic acid diethylamide (LSD) generally is not associated with birth
defects or withdrawal symptoms in prenatally exposed infants. Although there are isolated case
reports of birth defects in exposed neonates, an increased risk for anomalies is not borne out by
epidemiologic studies. It is important to note, however, that LSD users often abuse other
substances, underscoring the importance of taking an in-depth drug/substance abuse history in
pregnant women.
Marijuana use during pregnancy is not known to be associated with an increased incidence
of birth defects, dysmorphic features, or developmental delay in exposed offspring, although
further study is needed in this regard. Some investigations have shown reduced fetal growth in
exposed pregnancies, but this is not confirmed. Prenatally exposed newborns may have
tremulousness, increased irritability, and abnormal visual response to light stimulus.
Although there does not appear to be an increase in congenital anomalies associated with
methamphetamine use during pregnancy, further study is necessary. There are reports of
decreased birthweight in exposed neonates. A neonatal withdrawal syndrome consisting of
abnormal sleep patterns, tremulousness, poor feeding, and increased tone has been observed
commonly. Once again, it is important to consider polydrug abuse in these instances.
As with all teratogens, the timing of exposure is critical, with the most vulnerable period of
embryonic development occurring between 18 and 60 days after conception, during
organogenesis. Dosage of the offending agent also is important, as are route of administration,
modifying environmental factors, and genetic background of the mother and fetus.
References:
Gallagher RC, Kingham K, Hoyme HE. Fetal anticonvulsant syndrome. In: Cassidy SB, Allanson
JE, eds. Management of Genetic Syndromes. 2nd ed. Hoboken, NJ: Wiley-Liss; 2005:239-250
Phenobarbital, lithium, LSD, marijuana, methamphetamine. Reprotox. Available for subscription
at: http://www.reprotox.org
Phenobarbital, lithium, LSD, marijuana, methamphetamine. Teris. Available for subscription at:
http://depts.washington.edu/terisweb/teris

Copyright 2009 by the American Academy of Pediatrics

page 14

2009 PREP SA on CD-ROM


Question: 8

A 15-year-old girl comes to the urgent care clinic complaining of lower abdominal pain for 48
hours. She is nauseated but has had no fever, vomiting, or diarrhea. She is afebrile and denies
abdominal trauma. She localizes the pain to the left lower quadrant and describes it as
intermittent, stabbing pain episodes separated by intervals of more continuous dull pain. She has
never been sexually active. Her last menstrual period was 1 week ago. She has had no vaginal
discharge or itching. On physical examination, she has left lower quadrant guarding and rebound
tenderness. Her pelvic examination shows no vaginal discharge or uterine tenderness, although
there is an exquisitely tender mass in the left adnexal area.
Of the following, the MOST likely diagnosis is

A. appendicitis
B. endometritis
C. ovarian torsion
D. sacroiliitis
E. splenic rupture

Copyright 2009 by the American Academy of Pediatrics

page 15

2009 PREP SA on CD-ROM


Critique: 8

Preferred Response: C

Torsion of any adnexal structure, including the ovary, can result in an acute pelvic mass (Item
C8). The most common causes of adnexal torsion in young women are cysts and neoplasms,
with about 6% of torsions in one series occurring in the setting of normal-appearing ovaries.
Histopathology was benign in more than 90% of cases in this series. Torsion occurs when
masses cause the ovary to swing on its vascular pedicle, and larger masses generally are
associated with a greater potential for torsion until the size of the mass impedes movement. The
onset of pain associated with torsion can be abrupt, sharp, and very severe. However, with
intermittent or partial torsion, intense periods of intermittent pain may be separated by
generalized aching during detorsion, as described for the teenager in the vignette. Pain can
occur for several days or weeks prior to a complete torsion. Nausea or vomiting can
accompany severe pain. If adnexal torsion is diagnosed, prompt intervention (untwisting the
adnexa usually via laparoscopy) is indicated to preserve ovarian function.
Appendicitis is in the differential diagnosis for lower abdominal pain, but is less likely when
pain occurs in the left lower quadrant. Splenic rupture is a surgical emergency, like ovarian
torsion, but the finding of an adnexal mass on examination is unlikely. Endometritis is not very
likely in a young woman who is not sexually active and more commonly presents with midline
uterine pain. An orthopedic cause for this patient's pain (eg, sacroiliitis) is unlikely, but it remains
in the differential diagnosis of pelvic pain. Sacroiliitis usually presents with low back or hip pain.
References:
Adams Hillard PJ. Pelvic masses. In: Neinstein LS, eds. Adolescent Health Care: A Practical
Guide. 5th ed. Philadelphia, Pa: Lippincott Williams & Wilkins; 2008:706-713
Growdon WB, Laufer MR. Ovarian torsion. UpToDate Online 15.3. 2008. Available for
subscription at: http://www.utdol.com/utd/content/topic.do?topicKey=gyn_surg/5273
Laufer MR, Goldstein DP. Gynecologic pain: dysmenorrhea, acute and chronic pelvic pain,
endometriosis, and premenstrual syndrome. In: Emans SJH, Laufer MR, Goldstein DP, eds.
Pediatric and Adolescent Gynecology. 5th ed. Philadelphia, Pa: Lippincott, Williams & Wilkins;
2005:417-476
Varras M, Tsikini A, Polyzos D, Samara Ch, Hadjopoulos G, Akrivis Ch. Uterine adnexal torsion:
pathologic and gray-scale ultrasonographic findings. Clin Exp Obstet Gynecol. 2004;31:34-38.
Abstract available at: http://www.ncbi.nlm.nih.gov/pubmed/14998184

Copyright 2009 by the American Academy of Pediatrics

page 16

2009 PREP SA on CD-ROM


Question: 9

A 2-year-old boy who has chronic renal failure is brought to the emergency department for
evaluation of nausea, fatigue, and muscle weakness. On physical examination, the boy has a
heart rate of 140 beats/min, decreased perfusion, and palpable pulses. You obtain
electrocardiography (Item Q9). Electrolyte measurements include a potassium concentration of
7.5 mEq/L (7.5 mmol/L) and a glucose value of 72.0 mg/dL (4.0 mmol/L).
Of the following, the MOST appropriate initial treatment is administration of

A. calcium chloride
B. insulin
C. normal saline 20 mL/kg
D. sodium polystyrene sulfonate
E. verapamil

Copyright 2009 by the American Academy of Pediatrics

page 17

2009 PREP SA on CD-ROM


Critique: 9

Preferred Response: A

Hyperkalemia can be asymptomatic or produce symptoms such as nausea, fatigue, and muscle
weakness, as reported for the boy in the vignette. The most concerning effect of hyperkalemia
is on cardiac membrane polarization. Electrocardiographic changes typically consist of peaked
T waves and progress to widening of the QRS complexes with dampening of P waves as the
hyperkalemia increases. Without treatment, the patient eventually develops ventricular fibrillation
or asystole.
The electrocardiographic tracing of the boy described in the vignette demonstrates peaked
T waves (Item C9), and he has decreased perfusion. The immediate priority is to stabilize his
cardiac membrane potential and decrease his risk of arrhythmias, which is accomplished best
by administration of intravenous calcium chloride, an agent that has a rapid onset of action.
Insulin causes potassium to shift to the intracellular spaces, thereby decreasing serum
concentrations, but it always should be administered in conjunction with glucose to avoid
hypoglycemia. Sodium bicarbonate administered intravenously also produces intracellular shifts
of potassium and may be useful in patients who have metabolic acidosis.
Treatments to enhance the elimination of potassium often are indicated in the treatment of
hyperkalemia, but they have a slower onset of action and, therefore, should not be the first line
of treatment in life-threatening hyperkalemia. Sodium polystyrene sulfonate, which can be
administered orally or rectally, exchanges sodium for potassium, which subsequently is
eliminated from the body. Loop diuretics also can enhance potassium elimination. Dialysis may
be indicated when ongoing elevated potassium concentrations are expected, such as in renal
failure, or with very high acute concentrations, as seen with tumor lysis syndrome or
rhabdomyolysis.
Administration of verapamil is not indicated in the treatment of hyperkalemia; it has been
reported to be associated with the development of complete heart block in this setting. Normal
saline has a minimal effect on hyperkalemia.
References:
Greenbaum LA. Electrolyte and acid-base disorders: potassium. In: Kliegman RM, Behrman RE,
Jenson HB, Stanton BF, eds. Nelson Textbook of Pediatrics. 18th ed. Philadelphia, Pa:
Saunders Elsevier; 2007:279-284
Hauser GJ, Kulick AF. Electrolyte disorders in the pediatric intensive care unit. In: Wheeler DS,
Wong HR, Shanley TP, eds. Pediatric Critical Care Medicine: Basic Science and Clinical
Evidence. New York, NY: Springer-Verlag; 2007:1156-1175

Copyright 2009 by the American Academy of Pediatrics

page 18

2009 PREP SA on CD-ROM


Question: 10

A 13-year-old girl who has just moved to the United States from Brazil comes to your office
because her mother is worried that she is not "developing yet." On physical examination, her
height is 50 inches, and she has a triangular face, a low hairline, high-arched palate, and a shieldshaped chest (Item Q10). Breast tissue is not visible or palpable, but there is Sexual Maturity
Rating 3 pubic hair. You obtain bone age radiography and a karyotype and measure serum
luteinizing hormone and follicle-stimulating hormone.
Of the following, the MOST appropriate additional laboratory measurement is

A. adrenocorticotropic hormone
B. prolactin
C. 17-hydroxyprogesterone
D. testosterone
E. thyroid-stimulating hormone

Copyright 2009 by the American Academy of Pediatrics

page 19

2009 PREP SA on CD-ROM


Critique: 10

Preferred Response: E

The clinical findings described for the girl in the vignette are characteristic of Turner syndrome
(gonadal dysgenesis) associated with an abnormality of one X chromosome. Girls who have this
disorder usually are short (mean adult height, approximately 55 inches without growth hormone
treatment); have poorly developed ovaries; and often have dysmorphisms, including a triangular
facies, low hairline, high-arched palate, hypoplastic nipples, and an increased carrying angle.
They may have left heart disorders such as coarctation of the aorta as well as horseshoe
kidney or other renal malformations.
Initial screening studies to diagnose Turner syndrome include a karyotype and
measurement of luteinizing hormone (LH) and follicle-stimulating hormone (FSH). Most girls who
have Turner syndrome do not initiate normal puberty. Concentrations of LH and FSH rise as
they reach pubertal age range because they have ovarian failure. Although concentrations of
estradiol and other estrogens are low, clinical estradiol assays are not designed to provide
accurate values in the low-normal range expected in early puberty. Therefore, physical findings
such as breast development are a better marker of estrogen effect than measurements of
estrogen.
Adolescents who have Turner syndrome are at higher risk of developing chronic
lymphocytic thyroiditis and hypothyroidism than the general population. Approximately 20% of
affected adolescent girls have antibody-positive autoimmune chronic lymphocytic thyroiditis, and
5% to 10% develop overt hypothyroidism. Accordingly, measurement of thyroid-stimulating
hormone is an appropriate laboratory test for patients such as the girl described in the vignette.
An elevated value indicates primary hypothyroidism and the need for confirmatory assessment
of free thyroxine and antithyroid antibodies (thyroperoxidase, antimicrosomal, or
antithyroglobulin).
Abnormalities of the hypothalamic-pituitary-adrenal axis are unusual in patients who have
Turner syndrome. Therefore, measurement of adrenocorticotropic hormone is not useful.
Measurement of prolactin would be useful if the girl had a pituitary or hypothalamic problem, but
her clinical findings strongly point to Turner syndrome. A 17-hydroxyprogesterone value would
be elevated in the presence of an adrenal biosynthetic defect leading to the development of the
most common form of congenital adrenal hyperplasia (cyp21 or 21-hydroxylase deficiency) as
well as some of the less common disorders of adrenal biosynthesis. Measuring testosterone
would be reasonable if there were evidence of inappropriate masculinization, such as
clitoromegaly and a growth spurt. Some girls who have Turner syndrome have functioning Y
chromosomal DNA and could have androgenization, but this is unusual. The presence of Y
chromosomal DNA does increase the risk of gonadal malignancy, and girls who have significant
Y chromosomal DNA on testing often require prophylactic gonadectomy.
References:
Chiovato L, Larizza D, Bendinelli G, et al. Autoimmune hypothyroidism and hyperthyroidism in
patients with Turner's syndrome. Eur J Endocrinol. 1996;134:568-575. Abstract available at:
http://www.ncbi.nlm.nih.gov/pubmed/8664977
Doswell BH, Visootsak J, Brady AN, Graham JM Jr. Turner syndrome: an update and review for
the primary pediatrician. Clin Pediatr. 2006;45:301-313. Abstract available at:
http://www.ncbi.nlm.nih.gov/pubmed/16703153
Frias JL, Davenport ML, Committee on Genetics and Section on Endocrinology. Health
supervision for children with Turner syndrome. Pediatrics. 2003;111:692-702. Available at:
http://pediatrics.aappublications.org/cgi/content/full/111/3/692
Matura LA, Ho VB, Rosing DR, Bondy CA. Aortic dilatation and dissection in Turner syndrome.
Circulation. 2007;116:1663-1670. Abstract available at:
http://www.ncbi.nlm.nih.gov/pubmed/17875973

Copyright 2009 by the American Academy of Pediatrics

page 20

2009 PREP SA on CD-ROM

Sybert VP, McCauley E. Turner's syndrome. N Engl J Med. 2004;351:1227-1238. Extract


available at: http://content.nejm.org/cgi/content/extract/351/12/1227
Turner Syndrome Society Website. Available at: http://www.turnersyndrome.org

Copyright 2009 by the American Academy of Pediatrics

page 21

2009 PREP SA on CD-ROM


Question: 11

A 2-month-old infant has lost the vision in both of his eyes due to bilateral retinoblastoma. His
distressed parents ask how the infants blindness will affect his behavior and development.
Of the following, the child MOST likely will

A. begin saying single words at 16 to 20 months


B. begin walking between 18 and 22 months
C. display behaviors of an autism spectrum disorder
D. have a language-based learning disorder
E. have significant cognitive impairments

Copyright 2009 by the American Academy of Pediatrics

page 22

2009 PREP SA on CD-ROM


Critique: 11

Preferred Response: B

Legal blindness is defined as central visual acuity with corrective lenses of 20/200 or less in the
strongest eye or a limited visual field that extends to an angle of 20 degrees. Congenital
blindness occurs in 30 per 100,000 births. More than 50% of children who have visual
impairment also have developmental disabilities, such as cognitive-adaptive disability, seizures,
hearing impairments, and learning disorders. In many of these cases, the disabilities result from
central nervous system pathology. Postnatal blindness, which accounts for approximately 8% to
11% of all childhood blindness, can be caused by infections, trauma, or tumors. Retinoblastoma
is the most common primary malignant intraocular tumor of childhood. The initial finding in most
cases is a white pupillary reflex (leukokoria) (Item C11). Advanced tumors may be treated with
enucleation.
Children who have congenital or acquired (eg, due to retinoblastoma) blindness without
associated neurologic abnormalities should not be at increased risk for motor or cognitive
impairment. They are not at increased risk for language-based learning disabilities or autism
spectrum disorders. However, children who have significant visual impairment may begin to
walk at an older age (18 to 22 months) than sighted children due to different exposure to motor
exploration. They typically develop language skills at the same time (12 months) as sighted
children. Children who have visual impairments should be provided with much physical contact
that includes hugging and comforting. They should be encouraged to partake in self-help skills
and exploration of their environment.
References:
Davidson PW, Burns CM. Visual impairment and blindness. In: Levine MD, Carey WB, Crocker
AC, eds. Developmental- Behavioral Pediatrics. 3rd ed. Philadelphia, Pa: WB Saunders
Company; 1999:571-578
Msall ME. Visual impairment. In: Parker S, Zukerman B, Augustyn M. Developmental and
Behavioral Pediatrics: A Handbook for Primary Care. 2nd ed. Philadelphia, Pa: Lippincott
Williams & Wilkins; 2005:366-369
Olitsky SE, Hug D, Smith LP. Disorders of vision. In: Kleigman RM, Behrman RE, Jenson HB,
Stanton BF, eds. Textbook of Pediatrics. 18th ed. Philadelphia, Pa: Saunders Elsevier;
2007:2573-2576

Copyright 2009 by the American Academy of Pediatrics

page 23

2009 PREP SA on CD-ROM


Question: 12

A 15-year-old boy presents to the clinic because of a persistent cough. According to his mother,
his cough has been present for approximately 2 weeks, but it seems to be getting worse. He
does not cough all the time, but the coughing episodes tend to come in bursts. This morning she
became very worried because he passed out during a coughing spell. Physical examination
reveals a healthy-appearing male in no apparent distress. He is afebrile, and his vital signs are
normal. He has petechiae on his face but no other skin lesions. His lungs are clear.
Of the following, the MOST appropriate antimicrobial agent to prescribe for this patient is

A. azithromycin
B. clarithromycin
C. doxycycline
D. erythromycin
E. trimethoprim-sulfamethoxazole

Copyright 2009 by the American Academy of Pediatrics

page 24

2009 PREP SA on CD-ROM


Critique: 12

Preferred Response: A

Any patient who has episodic coughing episodes that conclude in syncope or vomiting, such as
the boy described in the vignette, should be considered to have pertussis. Other complications
from pertussis described in adolescents include urinary incontinence, sleep interruption, rib
fractures, and pneumonia. Despite universal immunization of children against pertussis, a
marked increase in disease incidence has been demonstrated among adolescents of 11 to 18
years of age. In an attempt to address this problem, the American Academy of Pediatrics
recommends that adolescents in this age group receive a single dose of tetanus toxoid, reduced
diphtheria toxoid, and acellular pertussis (Tdap) for booster immunization. For those who
received only a tetanus toxoid and reduced diphtheria toxoid (Td) booster, a booster with Tdap
is suggested if the interval since the Td administration is at least 2 years or if the adolescent is
living in a setting of increased disease, has a risk of a complicated course if he or she acquires
the disease, or possibly can transmit infection to a vulnerable contact.
Treatment of pertussis with antimicrobial agents once the cough has started does not affect
the course of the illness but is recommended to limit the spread of disease to others. In the past,
the treatment of choice was erythromycin, but recent data have demonstrated that the
effectiveness of other macrolides such as azithromycin or clarithromycin is similar to that of
erythromycin, and they have fewer adverse effects. The most common complaints of patients
who take erythromycin preparations are gastrointestinal irritation, including epigastric distress,
abdominal cramping, nausea, vomiting, and diarrhea. The occurrence of these adverse effects
can result in poor adherence to treatment regimens. In addition, erythromycin cannot be used in
children younger than 1 month of age due to its association with the development of hypertrophic
pyloric stenosis. Azithromycin and clarithromycin attain higher tissue concentrations than
erythromycin, have longer half-lives, and can be administered less often and for a shorter total
duration. With the advent of these newer macrolides, erythromycin rarely is indicated as a firstline agent for any illness.
Azithromycin has become the agent of choice for treatment of pertussis because of the
ease of administration (once daily for 5 days) and the fact that it does not inhibit cytochrome P450, as erythromycin and clarithromycin do. Therefore, it does not interact with other
medications that are metabolized by this system (eg, digoxin, carbamazepine). Trimethoprimsulfamethoxazole also is effective against pertussis and is considered an alternative for patients
who cannot tolerate a macrolide or have a macrolide-resistant isolate if they are older than 2
months of age. Doxycycline is not recommended for the treatment of pertussis.
References:
American Academy of Pediatrics. Pertussis (whooping cough). In: Pickering LK, Baker CJ, Long
SS, McMillan JA, eds. Red Book: 2006 Report of the Committee on Infectious Diseases. 27th ed.
Elk Grove Village, Ill: American Academy of Pediatrics; 2006:498-520
Committee on Infectious Diseases. Prevention of pertussis among adolescents:
recommendations for use of tetanus toxoid, reduced diphtheria toxoid, and acellular pertussis
(Tdap) vaccine. Pediatrics. 2006;117:965-978. Available at:
http://pediatrics.aappublications.org/cgi/content/full/117/3/965
Tiwari T, Murphy TV, Moran J. Recommended antimicrobial agents for the treatment and
postexposure prophylaxis of pertussis: 2005 CDC guidelines. MMWR Recomm Rep.
2005;54(RR14):1-16. Available at: http://www.cdc.gov/mmwr/preview/mmwrhtml/rr5414a1.htm

Copyright 2009 by the American Academy of Pediatrics

page 25

2009 PREP SA on CD-ROM


Question: 13

A 15-year-old male presents for evaluation of a progressively enlarging lesion on his left forearm
that began 5 days ago. He explains that the lesion initially looked like a "spider bite" with a blister,
but over the last several days, a black scab has developed in the center of the lesion, and there
is a large area of redness around the scab. The lesion has been pruritic but not painful. Except
for low-grade fevers for the last 2 days, he has had no other systemic symptoms. He returned 1
week ago from a school trip to Morocco, where he visited a leather tannery, went shopping in
the large outdoor marketplace, visited some historic sites, and took a camel ride in the desert.
He states that the students stayed in a hostel in Morocco, but there were no screens on the
windows, and spiders, ants, and other insects were visible in the rooms. On physical
examination, the boy is afebrile, and his left forearm is edematous, with a 3x3-cm black eschar
surrounded by a 5-cm area of erythema and induration (Item Q13). The lesion is not tender to
palpation, and there is no drainage. There are several 1.5-cm tender lymph nodes in his left
axilla. Findings on the remainder of his examination are within normal limits.
Of the following, the MOST likely cause of this patients lesion is

A. Bacillus anthracis
B. Francisella tularensis
C. Loxosceles laeta
D. methicillin-resistant Staphylococcus aureus
E. Yersinia pestis

Copyright 2009 by the American Academy of Pediatrics

page 26

2009 PREP SA on CD-ROM


Critique: 13

Preferred Response: A

Anthrax is a zoonotic disease caused by Bacillus anthracis, which is a gram-positive,


encapsulated, spore-forming rod that occurs in many areas of the world. B anthracis spores can
remain viable in the soil for decades (Item C13A), representing a major reservoir of infection for
herbivorous livestock through ingestion. Human infection occurs through contact with infected
animals or contaminated animal products, including carcasses, hides, hair, wool, meat, bone
meal, and other contaminated foodstuffs.
Depending on the route of infection, anthrax disease may manifest in three different forms:
cutaneous, inhalational, and gastrointestinal. Approximately 95% of all human anthrax cases are
cutaneous anthrax. The incubation period of cutaneous anthrax is 1 to 12 days. The initial skin
lesion is a pruritic papule that resembles an insect or spider bite, as described for the boy in the
vignette. The papule progresses to the development of a central vesicular or bullous lesion that
becomes necrotic and hemorrhagic and forms a central black painless eschar, which is the
classic lesion of anthrax (Item C13B). There is marked surrounding edema, swelling, induration,
and erythema of the involved area but no associated tenderness. Regional lymphadenopathy,
fever, malaise, and headache also may be present. In most cases, the eschar falls off in 1 to 2
weeks, and total resolution occurs in 6 weeks.
Infections caused by methicillin-resistant Staphylococcus aureus (MRSA), the bite of
Loxosceles laeta, Francisella tularensis, and Yersinia pestis are associated with pain, rapid
progression, and in most cases, systemic symptoms. Skin and soft-tissue infections due to
MRSA have been described as resembling a "spider bite" that is very tender, with progressive
worsening of the erythema, warmth, and tenderness (Item C13C). It is not associated with the
development of a bullous, necrotic lesion with eschar. The bite of Loxosceles laeta (the brown
recluse spider) may resemble the cutaneous lesion of anthrax, but it is very painful (Item C13D).
Also, symptoms develop shortly after the bite and progress rapidly within 24 hours. Francisella
tularensis is the agent that causes tularemia. Signs and symptoms develop within 3 to 5 days of
exposure and include the abrupt onset of fever, chills, headache, malaise, and fatigue as well as
the development of a progressively enlarging, tender, localized lymphadenopathy and a red,
painful papule (Item C13E) in a region draining into the involved lymph nodes. The signs and
symptoms of Yersinia pestis (plague) develop abruptly and include fever; chills; weakness;
headache; and extremely tender, rapidly enlarging swelling of the lymph nodes of the groin,
axilla, or neck.
References:
American Academy of Pediatrics. Anthrax. In: Pickering LK, Baker CJ, Long SS, McMillan JA,
eds. Red Book: 2006 Report of the Committee on Infectious Diseases. 27th ed. Elk Grove
Village, Ill: American Academy of Pediatrics; 2006:208-211
Butler T, Dennis DT. Yersinia species, including plague. In: Mandell GL, Bennett JE, Dolin R,
eds. Mandell, Douglas and Bennett's Principles and Practice of Infectious Diseases. 6th ed. New
York, NY: Elsevier Churchill Livingstone; 2005:2691-2700
Inglesby TV, Henderson DA, Bartlett JG, et al. Anthrax as a biological weapon: medical and
public health management. JAMA. 1999;281:1735-1745. Available at: http://jama.amaassn.org/cgi/content/full/281/18/1735
King MD, Humphrey BJ, Wang YF, Kourbatova EV, Ray SM, Blumberg HM. Emergence of
community-acquired methicillin-resistant Staphylococcus aureus USA 300 clone as the
predominant cause of skin and soft-tissue infections. Ann Intern Med. 2006;144:309-317.
Abstract available at: http://www.ncbi.nlm.nih.gov/pubmed/16520471
Penn RL. Francisella tularensis (tularemia). In: Mandell GL, Bennett JE, Dolin R, eds. Mandell,
Douglas and Bennett's Principles and Practice of Infectious Diseases. 6th ed. New York, NY:
Elsevier Churchill Livingstone; 2005:2674-2685

Copyright 2009 by the American Academy of Pediatrics

page 27

2009 PREP SA on CD-ROM

Swanson DL, Vetter RS. Bites of brown recluse spiders and suspected necrotic arachnidism. N
Engl J Med. 2005;352:700-707. Extract available at:
http://content.nejm.org/cgi/content/extract/352/7/700

Copyright 2009 by the American Academy of Pediatrics

page 28

2009 PREP SA on CD-ROM


Question: 14

A 3-month-old infant who has a history of renal dysplasia associated with obstructive uropathy
has marked polyuria. He is breastfeeding and receiving supplemental cow milk-based formula.
In an effort to reduce the high urine output, you consider reducing the renal solute load by
changing feedings from the milk-based formula currently being used.
Of the following, the MOST appropriate change is to

A. a hydrolyzed formula containing medium-chain triglycerides


B. a more concentrated (24-kcal) milk-based formula
C. human milk exclusively
D. soy milk-based formula
E. whole cow milk

Copyright 2009 by the American Academy of Pediatrics

page 29

2009 PREP SA on CD-ROM


Critique: 14

Preferred Response: C

The infant described in the vignette has polyuria caused by a urinary concentrating defect. The
concentrating defect is the result of tubular damage due to the obstructive uropathy. The inability
to concentrate the urine causes the kidneys to create an "excessive" volume of urine to excrete
the solute load presented to them.
One strategy to reduce polyuria is to reduce the solute burden placed on the kidneys.
Potential renal solute load is affected by intake of protein, sodium, potassium, chloride, and
phosphorus. The protein and phosphorus content are the most important variables when
comparing infant feeding regimens.
Human milk possesses a lower potential renal solute load than cow milk or cow milk-based
formulas. Accordingly, the most appropriate change in feeding for the infant in the vignette is to
recommend that the mother stop cow milk formula supplementation and exclusively breastfeed.
If human milk is not available, a "low-solute" cow milk-based formula can be used. A low calciumphosphorus formula has the next lowest potential renal solute load compared with human milk.
Cow milk, soy milk-based formula, hydrolyzed formula with medium-chain triglycerides, and 24kcal milk-based formula all have greater renal solute loads than human milk.
Renal solute load should also be considered in nephrogenic diabetes insipidus.
References:
Fiorino KN, Cox J. Nutrition and growth. In: Robertson J, Shilkofski N, eds. Harriet Lane
Handbook: A Manual for Pediatric House Officers. 17th ed. Philadelphia, Pa: Elsevier Mosby;
2005:525-608
Hall RT, Carroll RE. Infant feeding. Pediatr Rev. 2000;21:191-200. Available at:
http://pedsinreview.aappublications.org/cgi/content/full/21/6/191
Linshaw MA. Congenital nephrogenic diabetes insipidus. Pediatr Rev. 2007;28:372-380.
Available at: http://pedsinreview.aappublications.org/cgi/content/full/28/10/372
Ziegler EE, Fomon SJ. Potential renal solute load of infant formulas. J Nutr. 1989;119 (12
suppl):1785-1788. Available at: http://jn.nutrition.org/cgi/reprint/119/12_Suppl/1785

Copyright 2009 by the American Academy of Pediatrics

page 30

2009 PREP SA on CD-ROM


Question: 15

A mother brings in her 13-month-old daughter for evaluation because her girl developed a
perioral rash and "hives" on two occasions last week. One episode occurred while eating yogurt
and another happened immediately after eating a bagel with cream cheese. She states that her
daughter has eaten other foods such as eggs and bread without problems but is breastfeeding
and never has been given milk-based formulas or cow milk. The infant has been given rice milk,
but she became fussy and seems to prefer breastfeeding. The mother is concerned that her
daughter may be allergic to milk but would like to stop breastfeeding.
Of the following, the BEST advice is to recommend

A. a cow milk food challenge in the clinic


B. avoidance of milk, egg, soy, and wheat products
C. breastfeeding until the child is 3 years old
D. switching to an amino acid-based formula
E. switching to soy milk

Copyright 2009 by the American Academy of Pediatrics

page 31

2009 PREP SA on CD-ROM


Critique: 15

Preferred Response: E

Milk protein allergy is an immunoglobulin (Ig) E-mediated food reaction that affects 2% to 3% of
infants within the first postnatal year. Typical symptoms include urticaria, angioedema, atopic
dermatitis, and anaphylaxis. With IgE-mediated reactions, the quantity of milk required to result in
a reaction often is minimal (eg, milk touching the face, a taste of ice cream). Taking a detailed
history about the specific food(s) involved, timing of the onset of symptoms, and type of
symptoms is important to distinguish IgE-mediated reactions, as described for the child in the
vignette, from other adverse milk reactions, such as milk protein enterocolitis and lactose
intolerance. Once an IgE-mediated food allergy is suspected, the clinician should consider
allergy skin testing or serum IgE testing for the suspected food.
While awaiting results from either blood testing or allergy consultation for skin testing, the
first reasonable action is to switch to a soy-based formula. Approximately 10% to 15% of infants
and children who have IgE-mediated milk protein allergies may not tolerate soy formula, but this
risk applies to infants younger than 6 months of age. For infants older than 6 months, the risk is
closer to 5%. Nonetheless, the initial soy formula trial should be performed in the clinic. Other
acceptable formula options in this scenario include an extensively hydrolyzed or an amino acidbased formula, although the unpleasant taste and significantly higher cost can be limiting for
many families. Also, because almost all affected infants can be fed successfully with a soy or
extensively hydrolyzed formula, switching initially to an amino acid-based formula is not required.
Food challenges often are used to assess adverse food reactions, but they generally are
reserved for foods that are unlikely allergens or if the clinical history is inconsistent or vague (eg,
a patient who eats a particular food and does not always have a reaction). Food challenges may
result in anaphylaxis and generally are avoided when the history and testing results support an
IgE-mediated reaction.
Infants who have a specific food allergy sometimes are placed incorrectly on restricted diets
that avoid multiple foods. Without a specific history of other adverse food reactions, avoidance of
other foods such as egg or wheat is not recommended. However, parents should be counseled
that children can develop other food allergies and should monitor their children during ingestion
of other common food allergens.
Breastfeeding until age 3 years old is an option that is not preferred by the mother in the
vignette. Although most IgE-mediated cow milk allergies resolve by 3 years of age, milk protein
allergy can persist past 5 years of age in up to 20% of affected children.
References:
American Academy of Pediatrics Committee on Nutrition. Soy protein-based formulas:
recommendations for use in infant feeding. Pediatrics. 1998;101:148-153. Available at:
http://pediatrics.aappublications.org/cgi/content/full/101/1/148
Bhatia J, Greer F, and the Committee on Nutrition. The use of soy protein-based formulas in
infant feeding. Pediatrics. 2008;121:1062-1068. Available at:
http://pediatrics.aappublications.org/cgi/content/full/121/5/1062
Klemola T, Vanto T, Juntunen-Backman K, Kalimo K, Korpela R, Varjonen E. Allergy to soy
formula and to extensively hydrolyzed whey formula in infants with cow's milk allergy: a
prospective, randomized study with a follow-up to the age of 2 years. J Pediatr. 2002;140:219224. Abstract available at: http://www.ncbi.nlm.nih.gov/ pubmed/11865274
Saarinen KM, Pelkonen AS, Mkel MJ, Savilahti E. Clinical course and prognosis of cow's milk
allergy are dependent on milk-specific IgE status. J Allergy Clin Immunol. 2005;116:869-875.
Abstract available at: http://www.ncbi.nlm.nih.gov/pubmed/16210063
Sampson HA, Leung DYM. Adverse reactions to foods. In: Kleigman RM, Behrman RE, Jenson
HB, Stanton BF, eds. Nelson Textbook of Pediatrics. 18th ed. Philadelphia, Pa: Saunders
Elsevier; 2007:986-989

Copyright 2009 by the American Academy of Pediatrics

page 32

2009 PREP SA on CD-ROM


Question: 16

A 16-year-old girl is brought to the emergency department after being found unresponsive in her
bedroom. Her parents report finding a note in which she wrote of "wanting to end the pain." In
addition, they found several empty, unlabeled pill vials on her dresser. On physical examination,
the girl is responsive only to painful stimuli. Her heart rate is 60 beats/min, respiratory rate is 16
breaths/min, blood pressure is 90/60 mm Hg, and oxygen saturation is 92%. Her pupils are 3
mm, equal in size, and sluggishly reactive. The remainder of findings on her physical
examination are normal.
Of the following, the MOST important diagnostic test to obtain when evaluating this patient is a

A. carboxyhemoglobin concentration
B. complete blood count
C. serum acetaminophen concentration
D. serum ammonia concentration
E. serum osmolality

Copyright 2009 by the American Academy of Pediatrics

page 33

2009 PREP SA on CD-ROM


Critique: 16

Preferred Response: C

Treatment of a patient who has ingested an unknown substance or substances, such as the
one described in the vignette, should focus initially on stabilization of vital functions; treatment of
the patient's symptoms; and identification of agents that are potentially fatal, have delayed clinical
toxicity, or for which antidotal therapy is indicated. The evaluation should include a screening
history to identify circumstances surrounding the event, potential environmental exposures,
available medications or toxins, and previous medical or psychiatric history. The physical
examination should focus on vital sign abnormalities, pupillary size and reactivity, skin findings,
and mental status. These components are likely to yield the most useful clues to toxins that
have identifiable symptom complexes (Item C16A).
Laboratory testing should assess the patient's acid/base status, oxygenation and ventilation,
glucose concentration, and anion gap. Qualitative urine drug testing may be obtained but has
significant limitations, including the small number of drugs tested (typically drugs of abuse) and
screening thresholds that may produce false-negative results. Therefore, the results of urine
toxicologic screening rarely affect treatment plans. Quantitative drug concentrations should be
measured based on the information gleaned from the initial history, physical examination, and
screening laboratory testing, although acetaminophen and salicylate concentrations should be
measured for every patient who has an unknown, mixed, or intentional ingestion. Acetaminophen
is of particular concern because it causes few initial symptoms, is a common agent in
adolescent and adult intentional ingestions, and may require antidotal therapy to prevent potential
fatal liver damage. Abdominal radiographs may be considered to examine the patient for radioopaque substances (Item C16B).
A complete blood count is not likely to provide information leading to the identification of a
toxin. Measurement of serum ammonia may be indicated if acetaminophen is identified as the
ingested agent because toxicity can lead to hepatic damage. Carboxyhemoglobin should be
measured in the patient whose history is suggestive of carbon monoxide exposure. Increased
serum osmolality may provide indirect evidence of alcohol ingestion, although a quantitative
serum test for alcohols is more useful in this setting.
References:
Erikson TB, Thompson TM, Lu JJ. The approach to the patient with an unknown overdose.
Emerg Med Clin North Am. 2007;25:249-281. Abstract available at:
http://www.ncbi.nlm.nih.gov/pubmed/17482020
Lavallee M, Olsson J Jr, Cheng TL. In brief: unknown poison. Pediatr Rev. 2004;25:370-371.
Available at: http://pedsinreview.aappublications.org/cgi/content/full/25/10/370
McKay CA Jr. Can the laboratory help me? Toxicology laboratory testing in the possibly
poisoned pediatric patient. Clin Pediatr Emerg Med. 2005;6:116-122
Valez LI, Shepherd JG, Goto CS. Approach to the child with occult toxic exposure. UpToDate
Online 15.3. 2008. Available for subscription at:
http://www.utdol.com/utd/content/topic.do?topicKey=ped_tox/3023&selectedTitle=4~150&source
=search_result

Copyright 2009 by the American Academy of Pediatrics

page 34

2009 PREP SA on CD-ROM


Question: 17

You are following a 3-month-old infant who was born at 30 weeks gestation, underwent a distal
ileal resection for necrotizing enterocolitis at 2 weeks of age, and subsequently was placed on
parenteral nutrition for 2 months. The baby has residual cholestasis from the parenteral nutrition
(total bilirubin, 5.0 mg/dL [85.5 mcmol/L]; direct bilirubin, 3.0 mg/dL [51.3 mcmol/L]). Currently,
she is receiving a cow milk protein hydrolysate formula concentrated to 24 kcal/oz (0.8
kcal/mL). You are considering adding a dietary supplement to increase the caloric density of the
formula.
Of the following, the supplement that is the MOST likely to be tolerated and cause less diarrhea
in this infant is

A. flaxseed oil
B. medium-chain triglyceride oil
C. olive oil
D. omega-3 polyunsaturated fatty acid (fish oil)
E. soybean oil

Copyright 2009 by the American Academy of Pediatrics

page 35

2009 PREP SA on CD-ROM


Critique: 17

Preferred Response: B

Infants who have chronic illnesses may have specialized nutritional requirements and often do
not tolerate the standard 20-kcal/oz formula given to healthy term infants. For example, children
who have some forms of congenital heart disease or renal disease may require a more
concentrated formula because the standard formula may lead to volume overload. Children who
have intestinal disease or malabsorption, such as the child described in the vignette, also may
require a more concentrated formula to decrease the likelihood of feeding intolerance or
diarrhea.
Formula may be concentrated by increasing the concentration of protein, carbohydrate, or
fat. Perhaps the easiest method of increasing the caloric density of a formula is to mix more
powder with the same amount of water. For example, four scoops of most commercially
available formulas mixed with 8 oz of water yields standard 0.67-kcal/mL (20-kcal/oz) formula,
but mixing five scoops in 8 oz results in 0.83-kcal/mL (25-kcal/oz) formula. However, exceeding
25-kcal/oz formula by increasing the amount of powder may yield too high a concentration of
protein, which could result in an excessive renal solute load. For this reason, carbohydrate and
fat supplements are available to concentrate infant and toddler feedings further. The most
common adverse effect of carbohydrate supplements is diarrhea, and lipid supplements may
cause either diarrhea or delayed gastric emptying. For these reasons, caution is recommended
when increasing the caloric density of a formula, especially when the caloric density is increased
to greater than 1 kcal/mL (30 kcal/oz).
Because the patient in the vignette has had an ileal resection and cholestasis, the best fat
supplement for him is oil composed of medium-chain triglycerides (MCT oil). MCT oil can be
absorbed directly across the enterocyte and does not require intraluminal digestion by bile acids.
In contrast, soy, olive, flaxseed, and fish oils are long-chain fatty acids that require bile acids for
digestion and might cause diarrhea in a child who has cholestasis and ileal resection.
References:
Courtney E, Grunko A, McCarthy T. Enteral nutrition. In: Hendricks KM, Duggan C. Manual of
Pediatric Nutrition. 4th ed. Hamilton, Ontario, Canada: BC Decker; 2005:252-316
Suchy FJ. Neonatal cholestasis. Pediatr Rev. 2004;25:388-396. Available at:
http://pedsinreview.aappublications.org/cgi/content/full/25/11/388

Copyright 2009 by the American Academy of Pediatrics

page 36

2009 PREP SA on CD-ROM


Question: 18

You have admitted a 750-g male infant to the neonatal intensive care unit (NICU) for treatment of
respiratory distress and presumed sepsis. The Apgar scores were 1, 5, and 7 at 1, 5, and 10
minutes, respectively. The infant received one dose of exogenous surfactant in the delivery
room. In the NICU, the infant is being cared for on a radiant warmer. At 4 hours after birth,
physical examination reveals a temperature of 97.0F (36.1C), heart rate of 180 beats/min,
respiratory rate of 40 breaths/min (assisted breaths on the ventilator), blood pressure of 45/27
mm Hg, mean arterial blood pressure of 30 mm Hg, and pulse oximetry of 92%. The infant is
receiving synchronized intermittent mechanical ventilation with a peak inflation pressure of 18
cm H2O over a positive end-expiratory pressure of 4 cm H2O at a rate of 40 breaths/min and an
FiO2 of 0.40. Umbilical catheters are present in the umbilical artery and vein. On physical
examination, you note a soft, flat anterior fontanelle. You auscultate equal mechanical breath
sounds bilaterally over the chest and note minimal subcostal retractions. The skin is thin and
somewhat moist, and many veins are visible through it. The ears are flattened against the
cranium and lack any cartilage or recoil. There is a small phallus and an empty scrotum. The
infant is laying on the warmer with legs and arms extended. The neuromotor tone is decreased,
and the infant does not appear to be very active, but he does respond to tactile stimuli with
movement of the arms and legs in seemingly random and purposeless activity.
Of the following, the MOST important next step is to

A. administer another dose of exogenous surfactant


B. consult the urologist for cryptorchidism
C. increase the inspired oxygen concentration
D. move the infant into an isolette incubator
E. obtain head ultrasonography to evaluate for any hemorrhage

Copyright 2009 by the American Academy of Pediatrics

page 37

2009 PREP SA on CD-ROM


Critique: 18

Preferred Response: D

The extremely low gestational-age newborn (ELGAN) whose birthweight is less than 1,000 g
may present with physical examination findings indicative of marked physical and
neurodevelopmental immaturity. These findings have been studied and correlated with
gestational age in scoring systems such as those published by Dubowitz in 1980 and Ballard in
1979 and (revised) 1991. Even the best efforts to estimate the gestational age of very lowbirthweight and immature infants at dates less than 28 completed weeks of gestation by
maternal last menstrual period and obstetric ultrasonography may be imprecise and tend to
overestimate maturity (gestational age) by 2 weeks. Nevertheless, the thin, moist, transparent
skin; flattened thin ears without cartilage or recoil; and small phallus and empty scrotum
described for the infant in the vignette indicate a degree of physical immaturity consistent with 28
weeks' gestation or less. Similarly, the low neuromotor tone, absence of flexed posture, relative
inactivity, and random purposeless flailing of extremities with tactile stimulation indicate
significant neurologic immaturity.
Moving the infant into an isolette incubator is an important next step in the care of this
ELGAN. The absolute benefit of using a radiant warmer bed (access to the infant to conduct
early examinations, admission nursing care, and procedures such as placement of umbilical
catheters) is eclipsed by its associated increased ambient convection, evaporative heat loss,
and transcutaneous water loss compared with the contained, convectively heated, and
humidified environment of the isolette. The isolette environment still allows access, visibility, and
treatment with phototherapy, but with less insensible water loss or variability in patient
temperature.
This ELGAN is being treated for respiratory distress syndrome and has had a good
response to exogenous surfactant administration, oxygen, and assisted ventilation. At 4 hours of
postnatal age, it is too early to administer a second dose of surfactant. Developmentally
cryptorchid testes may yet descend into the scrotal sac in coming weeks. An oxygen saturation
of 92% is good for this infant, and increasing the inspired oxygen concentration may be harmful,
contributing to oxidative injury of pulmonary tissues or retinopathy of prematurity. Cranial
ultrasonography is indicated in this sick and preterm infant, but without clinical indications of
hemorrhage, shock, and seizure or encephalopathy, such a study is performed best at 3 to 7
days of postnatal life.
References:
Donovan EF, Tyson JE, Ehrenkranz RA, et al. Inaccuracy of Ballard scores before 28 weeks'
gestation. National Institute of Child Health and Human Development Neonatal Research
Network. J Pediatr. 1999;135:147-152. Abstract available at:
http://www.ncbi.nlm.nih.gov/pubmed/10431107
Marn GMA, Martn Moreiras J, Llitera Fleixas G, et al. Assessment of the new Ballard score to
estimate gestational age [in Spanish]. An Pediatr (Barc). 2006;64:140-145. English abstract
available at: http://www.ncbi.nlm.nih.gov/pubmed/16527066
Sedin G. The thermal environment of the newborn infant. In: Martin RJ, Fanaroff AA, Walsh MC,
eds. Fanaroff and Martin's Neonatal-Perinatal Medicine. 8th ed. Philadelphia, Pa: Mosby
Elsevier; 2006:585-596
Thilo EH, Rosenberg AA. The newborn infant. In: Hay WW Jr, Levin MJ, Sondheimer JM,
Deterding RR, eds. Current Pediatric Diagnosis & Treatment. 18th ed. New York, NY: The
McGraw-Hill Companies, Inc; 2007:chap 1

Copyright 2009 by the American Academy of Pediatrics

page 38

2009 PREP SA on CD-ROM


Question: 19

You are called to the newborn nursery to evaluate a 1-day-old girl whose hands and feet are
blue. She was born at term via a cesarean section, and there were no complications. Apgar
scores were 9 at both 1 and 5 minutes. Her respiratory rate is 40 breaths/min, heart rate is 140
beats/min, and blood pressure is normal. Pulse oximetry is 98% on room air. Her lungs are
clear, and there is no murmur. Her lips are pink, but her hands and feet are cyanotic (Item Q19),
and capillary refill is less than 2 seconds.
Of the following, the MOST likely cause of her peripheral cyanosis is

A. cold environment
B. polycythemia
C. retained fetal lung fluid
D. sepsis
E. transposition of the great arteries

Copyright 2009 by the American Academy of Pediatrics

page 39

2009 PREP SA on CD-ROM


Critique: 19

Preferred Response: A

Cyanosis is a common finding in newborns. The first step in the evaluation of an infant who
exhibits cyanosis is to determine whether it is central or peripheral (acrocyanosis). The infant
described in the vignette has clinical features suggestive of acrocyanosis, a bluish discoloration
of the hands and feet (Item C19A) in response to vasomotor instability or cold environment. In
this condition, the lips and mucous membranes are spared, although the perioral area may be
affected. The extremities may be cool to touch. Acrocyanosis is believed to be due to
vasoconstriction of small arterioles and resolves in the first few postnatal months. Infants who
have acrocyanosis require no further evaluation; parental reassurance is all that is needed.
Central cyanosis is the bluish discoloration of the tissues best observed in the lips, tongue,
mucous membranes, and occasionally nail beds (Item C19B). It occurs when the concentration
of deoxygenated (reduced) hemoglobin exceeds 4.0 to 6.0 g/dL (40.0 to 60.0 g/L) within the
capillary bed. In an individual who has normal hemoglobin, cyanosis usually becomes apparent
when the oxygen saturation drops below 75%-80%. Causes of central cyanosis in the neonatal
period typically are related to cyanotic heart disease, such as transposition of the great arteries,
or respiratory disease, such as pneumonia and retained fetal lung fluid (transient tachypnea of
the newborn). The absence of central cyanosis, a cardiac murmur, and respiratory distress for
the infant in the vignette makes these diagnoses less likely than acrocyanosis of infancy.
Polycythemia, defined as a venous hematocrit of at least 65% (0.65), causes a deep
reddish-purple discoloration of the skin and mucous membranes. Infants may be otherwise
asymptomatic or may exhibit lethargy, anorexia, or respiratory distress. Polycythemia does not
cause either peripheral or central cyanosis per se, but it does affect the percent saturation at
which cyanosis becomes clinically apparent. Cyanosis in infants who have polycythemia occurs
at higher oxygen saturations than in those who have anemia. For example, if an infant has a
hemoglobin of 24.0 g/dL (240.0 g/L), he is likely to appear cyanotic when his oxygen saturation
reaches 87%. In an infant who has a hemoglobin of 12.0 g/dL (120.0 g/L), cyanosis may not be
seen until the oxygen saturation falls to 75%. Newborns who have sepsis may develop cyanosis
as a component of multisystem involvement, but other signs, such as respiratory distress, poor
peripheral perfusion, lethargy, or poor feeding, also are evident.
References:
Bernstein D. Evaluation of the cardiovascular system: history and physical examination. In:
Kliegman RM, Behrman RE, Jenson HB, Stanton BF, eds. Nelson Textbook of Pediatrics. 18th
ed. Philadelphia, Pa: Saunders Elsevier; 2007:1857-1863
Sasidharan P. An approach to diagnosis and management of cyanosis and tachypnea in term
infants. Pediatr Clin North Am. 2004;51:999-1021. Abstract available at:
http://www.ncbi.nlm.nih.gov/pubmed/15275985

Copyright 2009 by the American Academy of Pediatrics

page 40

2009 PREP SA on CD-ROM


Question: 20

A frustrated mother requests referral of her 15-month-old child to an allergy and asthma
specialist because the boy never seems to have stopped coughing and wheezing over the 6
months of the past respiratory virus season. During the history taking, the mother states that the
baby only occasionally is exposed to wood smoke at the familys barbecue restaurant and to
cosmetic chemicals used at the grandmothers hair salon. Both the father and grandfather
smoke cigarettes in the home.
Of the following, the environmental exposure that is MOST likely to be causing the childs
respiratory symptoms is exposure to

A. cigarette smoke
B. cleaning fluids
C. dust mites
D. hairspray
E. wood smoke

Copyright 2009 by the American Academy of Pediatrics

page 41

2009 PREP SA on CD-ROM


Critique: 20

Preferred Response: A

The child described in the vignette may have multiple triggers for his wheezing exacerbations,
although tobacco smoke probably is the one to which he is exposed most often that should be
diminished. Well-known tobacco toxins include nicotine, carbon monoxide, formaldehyde,
hydrogen cyanide, sulfur dioxide, nitrogen oxide, ammonia, polycyclic aromatic hydrocarbons,
and the nitrosamines. These substances produce both irritant and immunologic effects on the
respiratory tract. It is important to note that smoking cessation is difficult, and most pediatricians
have little training in the process. However, there is evidence that a brief discussion of smoking
cessation in the context of medical visits is effective.
Wood smoke is another significant irritant to the respiratory tract; in addition to chemical
irritants, wood smoke contains particulate matter. Assuming that the family's restaurant kitchen
meets current standards for air filtration/exhaust, the child's exposure should be minimal.
However, outdoor barbecue pits that discharge smoke directly into the environment should be
avoided.
Exposure to cooking spray/oils at the restaurant and cosmetic chemicals such as hairspray
at the salon also should be avoided, although the use of volatile organic compounds, including
chlorofluorocarbons in hairspray, permanent wave solution, cleaning solutions, and cooking
sprays, has decreased sharply since passage of the Clean Air Act and other Environmental
Protection Agency measures in the 1990s. Material Safety Data Sheets for most hairsprays, as
well as other cosmetics and cleaning fluids, reveal little inhalation toxicity, except in the case of
prolonged exposure under improper ventilation.
Although dust mite exposure may contribute to asthma exacerbation, there is no evidence
that environmental control can reduce the concentration of mites or exacerbations related to
them. Finally, serial respiratory virus infections themselves may contribute significantly to this
child's history of wheezing and cough.
The child in the vignette may benefit from therapeutic advances in asthma management
such as those contained in the National Heart, Lung and Blood Institute Guidelines for asthma,
including attention to number of symptomatic days, use of inhaled corticosteroids, and use of a
metered dose inhaler with spacer for proper delivery of medication, which may be more effective
than nebulization in older infants and children.
References:
Brunnhuber K, Cummings KM, Feit S, Sherman S, Woodcock J.Putting evidence into practice:
smoking cessation. BMJ Clinical Evidence. 2007. Available for subscription at:
http://clinicalevidence.bmj.com/ceweb/resources/index.jsp
Kum-Nji P, Meloy L, Herrod HG. Environmental tobacco smoke exposure: prevalence and
mechanisms of causation of infections in children. Pediatrics. 2006;117:5:1745-1754. Available
at: http://pediatrics.aappublications.org/cgi/content/full/117/5/1745
Roseby R, Waters E, Polnay A, Campbell R, Webster P, Spencer N. Family and carer smoking
control programmes for reducing children's exposure to environmental tobacco smoke.
Cochrane Database Syst Rev. 2003;3:CD001746. Available at:
http://www.cochrane.org/reviews/en/ab001746.html
Stein RT, Holberg CJ, Sherrill D, et al. Influence of parental smoking on respiratory symptoms
during the first decade of life: The Tucson Children's Respiratory Study. Am J Epidemiol.
1999;149:1030-1037. Available at: http://aje.oxfordjournals.org/cgi/reprint/149/11/1030
U.S. Environmental Protection Agency. Health effects of wood smoke. Available at:
http://www.epa.gov/woodstoves/healtheffects.html
U.S. Environmental Protection Agency. National volatile organic compound emission standards
for consumer products. Available at: http://www.epa.gov/fedrgstr/EPA-AIR/1998/September/Day-

Copyright 2009 by the American Academy of Pediatrics

page 42

2009 PREP SA on CD-ROM

11/a22660.htm

Copyright 2009 by the American Academy of Pediatrics

page 43

2009 PREP SA on CD-ROM


Question: 21

You are evaluating a newborn 6 hours after his birth. Labor and delivery were uncomplicated,
but amniocentesis performed during the pregnancy revealed trisomy 21. Fetal
echocardiography at 20 weeks gestation showed normal findings. The infant currently is
sleeping and is well-perfused, with a heart rate of 140 beats/min and no audible murmurs. His
physical features are consistent with Down syndrome.
Of the following, the MOST appropriate diagnostic study to perform is

A. barium swallow
B. cervical spine radiography
C. echocardiography
D. head ultrasonography
E. radiography of the abdomen

Copyright 2009 by the American Academy of Pediatrics

page 44

2009 PREP SA on CD-ROM


Critique: 21

Preferred Response: C

Congenital heart defects (CHDs) are the most common of the congenital anomalies, occurring
with an incidence of approximately 5 to 8 per 1,000 live births (0.5% to 0.8%). The incidence of
CHD is greater in stillbirths and there is an increased incidence of CHD in those who have
aneuploidy. It is believed by some that the abnormal chromosomal composition rather than the
cardiac abnormality is responsible for the fetal demise of those who have CHDs. The strong
association between chromosomal abnormality and CHD has been shown in a number of
studies that focus on identification of fetal cardiac abnormality. A number of well-defined
chromosomal anomalies are associated with CHD (Item C21).
Early and accurate diagnosis of CHD is important in counseling parents of children in whom
a chromosomal abnormality or syndrome is suspected. Such diagnoses may have significant
effects on the health and well-being of the newborns because some CHDs may require ductal
patency for perfusion of either the systemic or pulmonary circulation. Therefore,
echocardiography now is considered an important component of the routine health supervision
of infants who have Down syndrome and other syndromes predisposing to structural heart
defects.
Although results of fetal echocardiography for the infant in the vignette were interpreted as
normal, not all CHDs can be diagnosed routinely with fetal echocardiography because of the
shunting pathways of the fetal circulation. For example, persistent patency of the ductus
arteriosus and secundum atrial septal defects are diagnosed postnatally. Small ventricular septal
defects frequently are not seen during fetal echocardiography because the pressure in the right
and left ventricles are equal due to the ductus arteriosus, resulting in minimal flow across the
defect prenatally.
Given the strong association between Down syndrome and CHD, echocardiography should
be performed in the newborn described in the vignette. Although children born with Down
syndrome have an increased incidence of duodenal atresia and other types of gastrointestinal
obstruction, a barium swallow is not an appropriate initial test for an asymptomatic newborn.
Cervical radiography, beginning at age 3 years, is important because of the risk of atlantoaxial
(C1-C2) subluxation. There is no indication for routine head ultrasonography or abdominal
radiography in an otherwise asymptomatic newborn who has Down syndrome.
References:
Committee on Genetics. Health supervision for children with Down syndrome. Pediatrics.
2001;107:442-449. Available at: http://pediatrics.aappublications.org/cgi/content/full/107/2/442
Silberbach M, Hannon D. Presentation of congenital heart disease in the neonate and young
infant. Pediatr Rev. 2007;28:123-131. Available at:
http://pedsinreview.aappublications.org/cgi/content/full/28/4/123
Tennstedt C, Chaoui R, Krner H, Dietel M. Spectrum of congenital heart defects and
extracardiac malformations associated with chromosomal abnormalities: results of a seven year
necropsy study. Heart. 1999;82:34-39. Abstract available at:
http://www.ncbi.nlm.nih.gov/pubmed/10377306

Copyright 2009 by the American Academy of Pediatrics

page 45

2009 PREP SA on CD-ROM


Question: 22

A 4-year-old boy presents with headache and difficulty walking. On physical examination, he is
afebrile, all growth parameters are within normal limits, and his mentation appears normal. The
optic discs are clearly visible and appear normal. He has normal eye position in primary gaze
but cannot abduct his right eye fully. He has normal tone, strength, and reflexes in his upper
limbs, but has bilateral hyperreflexia at the knees and ankle clonus. On gait examination, he toewalks.
Of the following, the MOST important next step is to obtain

A. computed tomography scan of the head


B. electromyography/nerve conduction studies of the legs
C. lumbar puncture
D. magnetic resonance imaging of the thoracolumbar spine
E. visual evoked potentials

Copyright 2009 by the American Academy of Pediatrics

page 46

2009 PREP SA on CD-ROM


Critique: 22

Preferred Response: A

The boy described in the vignette presents with a very concerning constellation of symptoms
and signs requiring urgent evaluation of the central nervous system. The first step in the
diagnostic process is to localize the problem to the proper level of the nervous system: brain,
brainstem/cerebellum, spinal cord, nerve, junction, or muscle. Such a determination not only
allows for urgent appropriate diagnosis and treatment but also reduces unnecessary discomfort,
risks, and costs of inappropriate diagnostic testing. Consultation with a neurologist can be useful
before ordering testing. Headache, gait disturbance (toe-walking), and acquired ocular
misalignment (right eye cannot move to the right) localizes the problem to the central nervous
system and raises concerns for both hydrocephalus and a brainstem lesion.
Computed tomography (CT) scan of the head is the preferred neuroimaging technique for
this patient. Although brain magnetic resonance imaging (MRI) can document the
brainstem/posterior fossa better, it may not be readily available. Moreover, a head CT scan in
the emergency department is adequate to rule out hydrocephalus that requires emergent
neurosurgical consultation. Unfortunately, a common cause for hydrocephalus in a child of this
age is a brainstem or cerebellar neoplasm such as an astrocytoma, glioma, medulloblastoma, or
ependymoma. When these tumors enlarge in the posterior fossa, they can obstruct the flow of
cerebrospinal fluid and cause acute hydrocephalus, which is a neurosurgical emergency.
Primary headache disorders such as migraine do not often present before the age of 6
years and are not accompanied by the neurologic findings described in the vignette. Toe-walking
is a nonspecific finding that can occur in the context of relatively benign delayed development,
although if this represents a change for a child, an upper motor neuron (brain/spinal cord) lesion
must be ruled out. The ankle clonus described for the boy in the vignette mandates this
approach. Similarly, inability to abduct one eye can be congenital, but if this is a new finding, a
brain or brainstem lesion must be ruled out. In this case, headache and the oculomotor findings
make a brain or brainstem lesion likely.
Gait impairment and hyperreflexia only in the legs can result from hydrocephalus or can
localize to the mid- to lower spinal cord. A spinal cord lesion would not explain the headache, and
bilateral motor findings without sensory, bowel, or bladder involvement are uncommon for a
spinal cord lesion. Therefore, MRI of the spine is not needed. Similarly, the hyperreflexia is not
evidence of a nerve or muscle problem, obviating the need for electromyography and nerve
conduction studies. Lumbar puncture is therapeutic for headache due to pseudotumor cerebri,
which could present with this constellation of symptoms, but this degree of gait abnormality
would be uncommon in pseudotumor, and the presence of true hydrocephalus or an intracranial
mass must be excluded with a head CT or brain MRI prior to lumbar puncture. Visual evoked
potentials occasionally are used to determine whether the axons from eye to occipital cortex
function normally, but such a test generally is performed when demyelinating disorders are
suspected.
References:
Avellino AM. Hydrocephalus. In: Singer HS, Kossoff EH, Hartman AL, Crawford TO, eds.
Treatment of Pediatric Neurologic Disorders. Boca Raton, Fla: Taylor & Francis; 2005:25-36
Garton HJ, Piatt JH Jr. Hydrocephalus. Pediatr Clin North Am. 2004;51:305-325. Abstract
available at: http://www.ncbi.nlm.nih.gov/pubmed/15062673
Kestle JR. Pediatric hydrocephalus: current management. Neurol Clin. 2003;21:883-895.
Abstract available at: http://www.ncbi.nlm.nih.gov/pubmed/14743654
Kuttesch J Jr, Ater JL. Brain tumors in childhood. In: Behrman RE, Kliegman RM, Jenson HB,
Stanton BF, eds. Nelson Textbook of Pediatrics. 18th ed. Philadelphia, Pa: Saunders Elsevier;
2007:2128-2136
Piatt JH Jr. Recognizing neurosurgical conditions in the pediatrician's office. Pediatr Clin North

Copyright 2009 by the American Academy of Pediatrics

page 47

2009 PREP SA on CD-ROM

Am. 2004;51:237-270. Abstract available at: http://www.ncbi.nlm.nih.gov/pubmed/15062671

Copyright 2009 by the American Academy of Pediatrics

page 48

2009 PREP SA on CD-ROM


Question: 23

Parents who are new to your area bring in their 3-year-old daughter for evaluation because they
are concerned about her delayed speech. They say that she uses about 50 single words. The
girl has had tetralogy of Fallot repaired surgically and recurrent upper respiratory tract infections
with otitis media, for which tympanostomy tubes have been placed. Findings on physical
examination include microcephaly, underfolded pinnae, a broad nasal bridge, cleft uvula, and a
small chin. In addition, the childs speech has a hypernasal quality. The family history is
negative for birth defects and developmental delays.
Of the following, the contiguous gene deletion syndrome that BEST fits this childs features is

A. Angelman
B. Beckwith-Wiedemann
C. 4pD. Prader-Willi
E. 22q11

Copyright 2009 by the American Academy of Pediatrics

page 49

2009 PREP SA on CD-ROM


Critique: 23

Preferred Response: E

The contiguous gene deletion syndromes comprise a group of disorders defined by


characteristic dysmorphisms or behaviors in association with a submicroscopic chromosome
deletion that is detected using fluorescence in situ hybridization (FISH) or microarray
technologies. In each case, the deleted segment contains multiple genes that play varying roles
in producing a particular phenotype. Angelman, Beckwith-Wiedemann, Wolf-Hirschhorn (4p-),
Prader-Willi, and 22q11 deletion syndromes are among the best delineated contiguous gene
deletion syndromes.
The child described in the vignette has features most consistent with 22q11 (the long arm of
chromosome 22 at band 11) deletion syndrome, which occurs in approximately 1 in 4,000 births,
making it the most common of the contiguous gene deletion syndromes. Confusion has arisen
regarding the relationship of 22q11 deletion syndrome to the DiGeorge and velo-cardio-facial
syndromes. It now is known that the condition described by DiGeorge in 1965, consisting of
hypoplastic-to-absent thymus and parathyroid glands together with conotruncal heart defects, is
due to 22q11 deletion in most cases. Similarly, the condition described by Shprintzen and
associates in 1978 that has come to be called velo-cardio-facial syndrome, which includes cleft
palate, speech disorder, broad nasal root, immune dysfunction, and learning difficulties, is also
caused by 22q11 deletion. The two disorders represent different manifestations of the same
submicroscopic deletion.
22q11 deletion syndrome should be suspected in any child who has a conotruncal heart
defect (eg, tetralogy of Fallot, double-outlet right ventricle, truncus arteriosus), especially if
accompanied by developmental delay, dysmorphic features, or recurrent upper respiratory tract
infections. Testing for the condition involves sending blood for FISH (it must be specified that the
region of interest is 22q11) or for microarray analysis, assuming the array covers the 22q11
region.
Angelman and Prader-Willi syndromes (AS and PWS, respectively) are caused by
aberrations (usually contiguous gene deletions) on the long arm of chromosome 15 at bands 11
through 13. This region of the human genome is "imprinted," such that when the paternal region
is deleted, the result is PWS, and when the maternal region is deleted, the result is AS. Children
who have PWS typically present in infancy with poor feeding, hypotonia, mild facial
dysmorphisms, and undescended testes with or without micropenis in boys. At the age of 2 to 3
years, affected children start eating excessively and gain weight rapidly, becoming obese (Item
C23A). They are mildly to moderately mentally retarded and often exhibit excessive skin picking
and a facility for jigsaw puzzles. Children who have AS typically appear normal at birth, but by 1
to 2 years of age, they have acquired microcephaly and severe speech delay (Item C23B).
During this same period, most (~85%) develop generalized seizures. Children who have AS
have an ataxic gait and jerky arm movements, resulting in a "puppet-like" gait. They are typically
nonverbal and severely mentally retarded and have paroxysms of inappropriate laughter.
4p- (4p minus), previously called Wolf-Hirschhorn syndrome, is a multiple congenital
anomaly syndrome characterized by prenatal-onset growth deficiency, prominent glabella,
microcephaly, and cleft lip+/-cleft palate. Profound mental retardation is the rule. Affected
individuals may have a cytogenetically detectable deletion on the short arm of chromosome 4 or
they may have a submicroscopic deletion involving 4p16.3.
Beckwith-Wiedemann syndrome (BWS) is characterized by pre- and postnatal overgrowth,
hypoglycemia in the newborn period, large tongue (Item C23C), unusual earlobe creases,
omphalocele in some (Item C23D), and organomegaly. Affected children are at increased risk
for certain neoplasias, including hepatoblastoma and Wilms tumor, making it important to follow a
tumor surveillance protocol up to 7 to 10 years of age. Intelligence typically is normal. The
genetics of BWS are complex; the condition is caused by submicroscopic deletions/alterations in
genes located at 11p15, which is a highly imprinted region.
The management of each of the contiguous gene deletion syndromes involves referrals to
multiple specialists, depending on the systems involved, together with anticipatory guidance
based on natural history information.
References:

Copyright 2009 by the American Academy of Pediatrics

page 50

2009 PREP SA on CD-ROM

Battaglia A, Carey JC, Wright TJ. Wolf-Hirschhorn syndrome. GeneReviews. 2006. Available at:
http://www.geneclinics.org/servlet/access?db=geneclinics&site=gt&id=8888891&key=OvKiicpzcf
vnc&gry=&fcn=y&fw=vH7o&filename=/profiles/whs/index.html
Bishara N, Clericuzio CL. Common dysmorphic syndromes in the NICU. NeoReviews.
2008;9:e29-e38. Available for subscription at:
http://neoreviews.aappublications.org/cgi/content/full/9/1/e29
Cassidy SB, Schwartz S. Prader-Willi syndrome. GeneReviews. 2006. Available at:
http://www.geneclinics.org/servlet/access?db=geneclinics&site=gt&id=8888891&key=OvKiicpzcf
vnc&gry=&fcn=y&fw=vWzr&filename=/profiles/pws/index.html
Lin RJ, Cherry AM, Bangs CD, Hoyme HE. FISHing for answers: the use of molecular
cytogenetic techniques in neonatology. NeoReviews. 2003;4:e94-e98. Available for subscription
at: http://neoreviews.aappublications.org/cgi/content/full/4/4/e94
McDonald-McGinn DM, Emanuel BS, Zackai EH. 22q11.2 deletion syndrome. GeneReviews.
2005. Available at:
http://www.ncbi.nlm.nih.gov/bookshelf/br.fcgi?book=gene&part=gr_22q11deletion
Shprintzen RJ. Velo-cardio-facial syndrome. In: Cassidy SB, Allanson JE, eds. Management of
Genetic Syndromes. 2nd ed. Hoboken, NJ: Wiley-Liss; 2005:615-632
Shuman C, Amith AC, Weksberg R. Beckwith-Wiedemann syndrome. GeneReviews. 2005.
Available at:
http://www.geneclinics.org/servlet/access?db=geneclinics&site=gt&id=8888891&key=OvKiicpzcf
vnc&gry=&fcn=y&fw=0F2J&filename=/profiles/bws/index.html
Weksberg R, Shuman C. Beckwith-Wiedemann syndrome and hemihyperplasia. In: Cassidy SB,
Allanson JE, eds. Management of Genetic Syndromes. 2nd ed. Hoboken, NJ: Wiley-Liss;
2005:101-116
Williams CA. Angelman syndrome. In: Cassidy SB, Allanson JE, eds. Management of Genetic
Syndromes. 2nd ed. Hoboken, NJ: Wiley-Liss; 2005:53-62
Williams CA, Driscoll DJ. Angelman syndrome. GeneReviews. 2007. Available at:
http://www.geneclinics.org/servlet/access?db=geneclinics&site=gt&id=8888891&key=OvKiicpzcf
vnc&gry=&fcn=y&fw=tkPG&filename=/profiles/angelman/index.html

Copyright 2009 by the American Academy of Pediatrics

page 51

2009 PREP SA on CD-ROM


Question: 24

A 13-year-old girl presents with severe lower abdominal pain of 24 hours duration. She states
that the pain is sharp and constant and that she has had similar pain for several days
approximately monthly over the past 4 months. She has no vomiting or diarrhea with the pain,
but she is constipated frequently, having a bowel movement about every 3 to 4 days. She feels
that her jeans are getting tighter around the waist, although she remains active, playing soccer
daily. She has never had a menstrual period and denies ever being sexually active. On physical
examination, she is afebrile, her heart rate is 85 beats/min, and her blood pressure is 110/70 mm
Hg. Her weight is at the 60th percentile and her height at the 50th percentile for age. Her breasts
and genitalia are at Sexual Maturity Rating 5. Abdominal examination reveals a firm and tender
midline mass that is inferior to the umbilicus.
Of the following, the MOST likely diagnosis is

A. bladder obstruction
B. endometriosis
C. hematocolpos
D. megacolon
E. ovarian cyst

Copyright 2009 by the American Academy of Pediatrics

page 52

2009 PREP SA on CD-ROM


Critique: 24

Preferred Response: C

The adolescent described in the vignette has a clinical history and physical examination findings
compatible with an imperforate hymen, which probably is the most common obstructive anomaly
of the female reproductive tract. An adolescent patient who has an imperforate hymen may be
asymptomatic or may have a history of cyclic abdominal pain that may occur for several years
before the diagnosis is made. A bluish, bulging hymen may be seen on genital inspection (Item
C24), and a distended vagina may be palpated on rectoabdominal or abdominal examination. If
the vagina becomes substantially enlarged with accumulated blood, the patient may experience
back pain, pain with defecation that can result in constipation, nausea and vomiting, or difficulty
in urinating.
Bladder outlet obstruction occurs rarely, and although it produces a suprapubic mass, it
does not cause cyclic abdominal pain. Megacolon also is unlikely and does not cause cyclic
pain, although colonic irritation may develop from the pressure produced by the mass. An
ovarian cyst typically causes a right- or left-sided (not midline) mass, and endometriosis is an
unlikely cause of a palpable mass, although it can cause cyclic and acyclic pain in adolescents.
References:
Adams Hillard PJ, Deitch HF. Gynecologic disorders. In: Osborn LM, DeWitt TG, First LR, Zenel
JA, eds. Pediatrics. Philadelphia, Pa: Elsevier Mosby;2005:1461-1471
Laufer MR, Goldstein DP, Hendren WH. Structural abnormalities of the female reproductive
tract. In: Emans SJH, Laufer MR, Goldstein DP, eds. Pediatric and Adolescent Gynecology. 5th
ed. Philadelphia, Pa: Lippincott, Williams & Wilkins; 2005:334-416

Copyright 2009 by the American Academy of Pediatrics

page 53

2009 PREP SA on CD-ROM


Question: 25

During teaching rounds, the pediatric ward resident reports on a 4-month-old circumcised male
infant who was admitted to the pediatric ward for fever that morning. The infant is now afebrile
and has had respiratory rates of 40 breaths/min while sleeping and greater than 60 breaths/min
when awake. The infant has a soft, flat fontanelle on physical examination and is not irritable.
The only diagnostic studies obtained on admission were a urinalysis and complete blood count,
the results of which were normal, except for a white blood cell count of 16.0x103/mcL
(16.0x109/L).
Of the following, the MOST appropriate next step is

A. administration of 100 mL normal saline


B. chest radiography
C. lumbar puncture
D. reassurance of the resident that this represents normal respiratory variation
E. urine culture

Copyright 2009 by the American Academy of Pediatrics

page 54

2009 PREP SA on CD-ROM


Critique: 25

Preferred Response: B

Respiratory rates vary across a relatively wide range in pediatric patients, depending on factors
such as age and activity status. Therefore, strict definitions of tachypnea and bradypnea are
difficult to determine and always must be considered in association with other factors such as
current clinical status and individual history. Because tachypnea is a sensitive indicator of lower
airway disease, patients who have elevated respiratory rates deserve a clinical evaluation in the
context of other associated symptoms.
The tachypnea, history of fever, and elevated white blood cell count described for the boy in
the vignette warrant chest radiography. Although lumbar puncture and a urine culture often are
indicated to evaluate infants who have fever, the elevated respiratory rate combined with
reassuring neurologic examination results and normal urinalysis make pneumonia a more likely
diagnosis. Administration of a normal saline bolus would not be expected to improve the
abnormal respiratory rate.
References:
Bloomfield D. In brief: tachypnea. Pediatr Rev. 2002;23:294-295. Available at:
http://pedsinreview.aappublications.org/cgi/content/full/23/8/294
Sectish TC, Prober CG. Pneumonia. In: Kliegman RM, Behrman RE, Jenson HB, Stanton BF,
eds. Nelson Textbook of Pediatrics. 18th ed. Philadelphia, Pa: Saunders Elsevier;2007:17951799

Copyright 2009 by the American Academy of Pediatrics

page 55

2009 PREP SA on CD-ROM


Question: 26

A 6-year-old boy who has severe vomiting and dehydration is admitted to the hospital. Initial
laboratory studies demonstrate a serum sodium concentration of 126.0 mEq/L (126.0 mmol/L),
potassium of 5.3 mEq/L (5.3 mmol/L), and pH of 7.26. After 24 hours of rehydration with 0.9%
saline, his serum sodium concentration is 129.0 mEq/L (129.0 mmol/L) and potassium is 4.9
mEq/L (4.9 mmol/L). On physical re-examination, you note that his knees, elbows, dorsal
fingers, and tongue are somewhat pigmented (Item Q26), and his skin is darker than that of
other family members.
Of the following, the MOST useful diagnostic laboratory study at this time is measurement of
serum

A. antidiuretic hormone (ADH) and alpha-melanocortin-stimulating hormone


B. cortisol and adrenocorticotropic hormone (ACTH)
C. cortisol and dehydroepiandrosterone
D. dehydroepiandrosterone and ACTH
E. insulin-like growth factor 1 and ADH

Copyright 2009 by the American Academy of Pediatrics

page 56

2009 PREP SA on CD-ROM


Critique: 26

Preferred Response: B

Acute or chronic vomiting and dehydration associated with hyponatremia and elevated
potassium concentrations, as described for the boy in the vignette, suggest adrenocortical
insufficiency. Primary adrenal insufficiency is associated with skin pigmentation (Item C26)
because of ACTH overproduction. ACTH acts directly on the melanocortin receptors of skin to
activate melanin production. Normally, ACTH controls the amount of cortisol produced by the
adrenal cortex by stimulating adrenal steroidogenesis. Cortisol then feeds back both to the
pituitary and hypothalamus to inhibit pituitary ACTH release. Measurement of low serum cortisol
and markedly elevated serum ACTH concentrations at any time of the day usually confirms the
diagnosis of primary adrenal insufficiency. The presence of skin pigmentation in the boy
described in the vignette suggests that identifying an elevated ACTH value will be easy, but
because there is diurnal variation in ACTH and cortisol, with highest concentrations during the
early morning hours and lowest in the late afternoon and evening, children who have less severe
adrenal insufficiency should have these hormones assessed in the early morning. In addition, an
ACTH stimulation test might be necessary for diagnostic confirmation. In this test, synthetic
ACTH1-24 is administered intravenously, and the adrenal cortisol response is measured before
injection and at 1 hour postinjection. An adequate cortisol response at 1 hour rules out adrenal
insufficiency.
Although ADH concentrations might be elevated in primary adrenal insufficiency because of
loss of intravascular fluid volume, measurement of this hormone does not help in the diagnosis
of a child who has low serum sodium and somewhat elevated potassium values. Low serum
sodium is associated with inappropriate ADH release, but the potassium would not be elevated.
ACTH and melanocyte-stimulating hormone (a melanocortin) both are produced from enzymatic
cleavage of a larger molecule, proopiomelanocortin. Although melanocyte-stimulating hormone
might be overproduced to some extent in the presence of excess ACTH, elevated
concentrations of this hormone are not necessary for skin pigmentation with ACTH excess.
Dehydroepiandrosterone (DHEA) is a weak androgen precursor produced by the fetal
adrenal initially; production increases again in mid-childhood with the onset of adrenarche. DHEA
values rise slowly from age 4 to 6 years. Elevated values may be found with some adrenal
tumors and some relatively rare types of congenital adrenal hyperplasia, but these disorders are
associated with some degree of early puberty. DHEA-S, the sulfated product of DHEA, usually
has stable serum values without diurnal variation. Therefore, normal DHEA or DHEA-S values in
an adolescent or adult confirm the presence of a functioning adrenal gland. This test is less
useful in a young child because onset of adrenarche is variable. Cortisol is low in primary or
secondary adrenal insufficiency but tends toward low ranges in most people after the early
morning hours.
Insulin-like growth factor 1 is a marker for growth hormone sufficiency, and normal
concentrations vary with age and sex. Because this child does not seem to have a growth
problem, the result of this assay should be normal for age.
References:
Auchus RJ, Rainey WE. Adrenarche-physiology, biochemistry and human disease. Clin
Endocrinol. 2004;60:288-296. Available at: http://www.blackwellsynergy.com/doi/full/10.1046/j.1365-2265.2003.01858.x
Coco G, Dal Pra XC, Presotto F, et al. Estimated risk for developing autoimmune Addison's
disease in patients with adrenal cortex antibodies. J Clin Endocrinol Metab. 2006;91:1637-1645.
Available at: http://jcem.endojournals.org/cgi/content/full/91/5/1637
Donohoue PA. Diagnosis of adrenal insufficiency in children. UpToDate Online 15.3. 2008.
Available for subscription at:
http://www.uptodateonline.com/utd/content/topic.do?topicKey=pediendo/20697
Perry R, Kecha O, Paquette J, Huot C, van Vliet G, Deal C. Primary adrenal insufficiency in
Copyright 2009 by the American Academy of Pediatrics

page 57

2009 PREP SA on CD-ROM

children: twenty years experience at the Sainte-Justine Hospital, Montreal. J Clin Endocrinol
Metab. 2005;90:3243-3250. Available at: http://jcem.endojournals.org/cgi/content/full/90/6/3243
Wilson TA, Speiser P. Adrenal insufficiency. eMedicine Specialties, Pediatrics: General Medicine,
Endocrinology. 2007. Available at: http://www.emedicine.com/ped/TOPIC47.HTM

Copyright 2009 by the American Academy of Pediatrics

page 58

2009 PREP SA on CD-ROM


Question: 27

A mother of a 6-year-old boy in your practice is concerned that her son may have dyslexia. She
has brought a sample of his printing to the visit in which the boy wrote "ded" instead of "bed" and
"dad" instead of "bad." She wants your advice on what she should do to help her son learn how
to write properly.
Of the following, the MOST appropriate response is to

A. reassure the mother that letter reversal can be normal through 7 years of age
B. recommend a comprehensive psychoeducational evaluation for a learning disability
C. recommend neurologic evaluation
D. refer the child for an occupational therapy evaluation and services to improve his writing skills
E. refer the child for vision therapy

Copyright 2009 by the American Academy of Pediatrics

page 59

2009 PREP SA on CD-ROM


Critique: 27

Preferred Response: A

Letter reversal in writing can be normal in children through 7 years of age. Dyslexia, a word
recognition defect, is a specific learning disability that is neurobiologically based. It is
characterized by problems with the ability to recognize words accurately and poor spelling and
decoding skills. Its prevalence is as high as 17.4% of the school-age population. Affected
children have problems attaching the correct labels or names to letters and words. They may
call a "b" a "d" or read "saw" as "was." Because the problem is linguistic, not visual, affected
children do not have problems copying letters.
Backward writing and letter reversal occur commonly in early development for all children
whether or not they have learning disabilities. All children should receive routine vision
screening, but a visual acuity problem would not be the cause of the letter reversal for the boy
described in the vignette. There is no scientific evidence that vision therapy (eye exercise) is
effective in the remediation of language-based learning disorders. Because letter reversal still
can be considered in the normal range of development at 6 years of age, psychoeducational
evaluation, neurologic evaluation, and occupational therapy are not indicated for this child.
References:
Committee on Children With Disabilities, American Academy of Pediatrics (AAP) and American
Academy of Ophthalmology (AAO), and American Association for Pediatric Ophthalmology and
Strabismus (APOS). Learning disabilities, dyslexia, and vision: a subject review. Pediatrics.
1998;102:1217-1219. Available at:
http://pediatrics.aappublications.org/cgi/content/full/102/5/1217
Fletcher JM, Lyon GR, Fuchs LS, Barnes MA. Reading disabilities: word recognition. In:
Learning Disabilities: From Identification to Intervention. New York, NY: The Guilford Press:
2007:85-163
Shaywitz SE, Shawitz BA. Dyslexia (specific reading disability). Pediatr Rev. 2003;24:147-153.
Available at: http://pedsinreview.aappublications.org/cgi/content/full/24/5/147

Copyright 2009 by the American Academy of Pediatrics

page 60

2009 PREP SA on CD-ROM


Question: 28

A 14-year-old girl presents to the emergency department with a 2-day history of fever and a
rash. The rash has been progressive, and now her mouth and eyes hurt. Upon further
questioning, she reports that she was started on an antibiotic 7 days ago for some complaints of
dysuria, but she does not remember its name. Physical examination reveals a moderately toxicappearing female whose temperature is 102.6F (39.2C), respiratory rate is 25 breaths/min,
heart rate is 105 beats/min, and blood pressure is 105/70 mm Hg. Her bulbar conjunctivae are
erythematous (Item Q28A), and she has some early bullous lesions developing in her mouth.
She has right upper quadrant tenderness and multiple target lesions (Item Q28B) on her chest,
abdomen, arm, back, upper thighs, buttocks, and face.
Of the following, the antimicrobial agent that is MOST likely to be associated with these clinical
findings is

A. amoxicillin
B. azithromycin
C. cefdinir
D. clindamycin
E. trimethoprim-sulfamethoxazole

Copyright 2009 by the American Academy of Pediatrics

page 61

2009 PREP SA on CD-ROM


Critique: 28

Preferred Response: E

Stevens-Johnson syndrome, the condition described in the vignette, may be caused by


infectious agents (eg, Mycoplasma pneumoniae, herpes simplex virus) or medications such as
nonsteroidal anti-inflammatory agents (eg, ibuprofen, salicylates), anticonvulsants (eg,
phenytoin, carbamazepine), and other antimicrobial agents (eg, trimethoprim-sulfamethoxazole
[TMP-SXT], cephalosporins). Although generally well tolerated, TMP-SXT has several adverse
effects. From 3% to 8% of patients may experience mild gastrointestinal symptoms, including
nausea, vomiting, anorexia, diarrhea, glossitis, and stomatitis. Approximately 3% to 4% of
patients who receive TMP-SXT develop skin lesions. These drug eruptions include
maculopapular rashes, urticaria (Item C28A), pruritus, photodermatitis (Item C28B), exfoliative
dermatitis, erythema multiforme, toxic epidermal necrolysis (Item C28C), and Stevens-Johnson
syndrome. Although any of the antimicrobial agents listed in the vignette (eg, amoxicillin,
azithromycin, cefdinir, clindamycin) can cause these skin reactions, sulfa-containing agents
such as TMP-SXT most commonly are responsible. Patients infected with the human
immunodeficiency virus and taking TMP-SXT for Pneumocystis prophylaxis also are at
increased risk for the development of such reactions. Bone marrow suppression can occur with
prolonged administration of TMP-SXT, resulting in pancytopenia, agranulocytosis, anemia, or
thrombocytopenia. TMP-SXT also competes with bilirubin for plasma protein binding sites and is
not recommended for infants younger than 2 months of age. TMP-SXT can potentiate the effects
of warfarin, phenytoin, methotrexate, and oral hypoglycemic agents, leading to bleeding,
phenytoin toxicity, severe pancytopenia, and hypoglycemia, respectively. The half-life of digoxin
also is increased, resulting in elevated concentrations. Oral contraception metabolism can be
accelerated when these medications are administered in conjunction with TMP-SXT, leading to
ineffective contraception.
TMP-SXT is recommended as first-line therapy for acute, uncomplicated urinary tract
infections; urinary tract prophylaxis; selected bacterial gastrointestinal infections (eg, Shigella);
and for treatment and prophylaxis of Pneumocystis infections. The use of amoxicillin for the
empiric treatment of urinary tract infections is limited due to drug resistance among Escherichia
coli. Although cefdinir has very good activity against most community-acquired isolates of E coli
and other gram-negative organisms, its use is often cost prohibitive. Clindamycin should be
used only in certain situations (eg, when the isolated organism is Staphylococcus aureus), and
azithromycin is not indicated for the treatment of urinary tract infections.
References:
Libecco JA, Powell KR. Trimethoprim/sulfamethoxazole: clinical update. Pediatr Rev.
2004;25:375-380. Available at: http://pedsinreview.aappublications.org/cgi/content/full/25/11/375
Mittmann N, Chan B, Knowles S, Cosentino L, Shear N. Intravenous immunoglobulin use in
patients with toxic epidermal necrolysis and Stevens-Johnson syndrome. Am J Clin Dermatol.
2006;7:359-368. Abstract available at: http://www.ncbi.nlm.nih.gov/pubmed/17173470
Saidinejad M, Ewald MB, Shannon MW. Transient psychosis in an immune-competent patient
after oral trimethoprim-sulfamethoxazole administration. Pediatrics. 2005;115:e739-e741.
Available at: http://pediatrics.aappublications.org/cgi/content/full/115/6/e739

Copyright 2009 by the American Academy of Pediatrics

page 62

2009 PREP SA on CD-ROM


Question: 29

A 14-year-old girl presents for evaluation after 4 days of a temperature to 103.0F (39.5C),
nausea, abdominal cramping, and profuse bloody diarrhea. She reports that she has not
traveled anywhere, has no pets, and has had no ill contacts or unusual food exposures. One
week ago, she was diagnosed with a methicillin-sensitive Staphylococcus aureus chronic
osteomyelitis of her distal radius and has been receiving intravenous cefazolin therapy via a
peripherally inserted central catheter line. Physical examination reveals an uncomfortable
teenager who complains of severe abdominal pain and has a temperature of 102.8F (39.4C)
and moist mucous membranes. Her abdomen is diffusely tender, with voluntary guarding but no
rebound tenderness on palpation. Rectal examination demonstrates normal sphincter tone with
no fissures or other lesions. Laboratory findings include a peripheral white blood cell count of
15.0x103/mcL (15.0x109/L); hemoglobin of 13.0 g/dL (130.0 g/L); platelet count of
300.0x103/mcL (300.0x109/L); and a differential count of 65% neutrophils, 25% lymphocytes,
and 10% monocytes. Her stool appears watery and grossly bloody.
Of the following, the MOST appropriate treatment for this patients condition is

A. ceftriaxone
B. clindamycin
C. metronidazole
D. trimethoprim-sulfamethoxazole
E. vancomycin

Copyright 2009 by the American Academy of Pediatrics

page 63

2009 PREP SA on CD-ROM


Critique: 29

Preferred Response: C

The patient described in the vignette has findings of antibiotic-associated Clostridium difficile
disease due to her cefazolin therapy. C difficile accounts for approximately 20% to 30% of
cases of antibiotic-associated diarrhea, 50% to 70% of antibiotic-associated colitis cases, and
more than 90% of antibiotic-associated pseudomembranous colitis cases. The incidence of
antibiotic-associated diarrhea and colitis varies greatly, depending on the antibiotic and its
spectrum of activity and pharmacokinetic properties. Almost all antibiotic classes have been
associated with disease, but clindamycin, the penicillins, and the cephalosporins are implicated
most commonly.
The pathogenesis of C difficile-mediated diarrhea and colitis includes: 1) disruption and
eradication of normal colonic organisms by antibacterial agents; 2) colonization with toxigenic C
difficile; and 3) production of toxin A or toxin B by C difficile, both of which mediate cytoskeletal
damage of target cells, resulting in mucosal injury and inflammation.
Infection with toxigenic C difficile causes a spectrum of disease, ranging from asymptomatic
carriage to a fulminant, relapsing, and occasionally fatal colitis. Signs and symptoms may
develop as early as the first day of therapy to as late as 10 weeks after therapy has ended, with
a typical range of 5 to 10 days after the initiation of antibiotic therapy. The gastrointestinal
manifestations of C difficile are variable, ranging from watery diarrhea to colitis with blood. Other
findings include fever (seen in 30% to 50% of patients), leukocytosis (mean peripheral leukocyte
count of more than 15.0 x 103/mcL [15.0 x 109/L), nausea, malaise, abdominal pain, and
cramping. Oral metronidazole and oral vancomycin are equally effective for the treatment of C
difficile-associated diarrhea and colitis, but metronidazole is considered the drug of choice
because of cost and concerns regarding the emergence of vancomycin-resistant enterococci.
Ceftriaxone, clindamycin, and trimethoprim-sulfamethoxazole are not effective in the treatment of
this infection.
References:
American Academy of Pediatrics. Clostridium difficile. In: Pickering LK, Baker CJ, Long SS,
McMillan JA, eds. Red Book: 2006 Report of the Committee on Infectious Diseases. 27th ed. Elk
Grove Village, Ill: American Academy of Pediatrics; 2006:261-263
Benson L, Song X, Campos J, Singh N. Changing epidemiology of Clostridium difficile-associated
disease in children. Infect Control Hosp Epidemiol. 2007;28:1233-1235. Abstract available at:
http://www.ncbi.nlm.nih.gov/pubmed/17926272
Klein EJ, Boster DR, Stapp JR, et al. Diarrhea etiology in a children's hospital emergency
department: a prospective cohort study. Clin Infect Dis. 2006;43:807-813. Abstract available at:
http://www.ncbi.nlm.nih.gov/pubmed/16941358
Thielman NM, Wilson KH. Antibiotic-associated colitis. In: Mandell GL, Bennett JE, Dolin R, eds.
Mandell, Douglas and Bennett's Principles and Practice of Infectious Diseases. 6th ed. New
York, NY: Elsevier Churchill Livingstone; 2005:1249-1263

Copyright 2009 by the American Academy of Pediatrics

page 64

2009 PREP SA on CD-ROM


Question: 30

A 14-year-old girl who has a history of insulin-dependent diabetes mellitus (IDDM) presents with
vomiting, increased urination, and decreased energy. Physical examination reveals Kussmaul
breathing and delayed capillary refill. Laboratory findings include:
Sodium, 136.0 mEq/L (136.0 mmol/L)
Potassium, 5.2 mEq/L (5.2 mmol/L)
Chloride, 100.0 mEq/L (100.0 mmol/L)
Bicarbonate, 10.0 mEq/L (10.0 mmol/L)
Blood urea nitrogen, 24.0 mg/dL (8.6 mmol/L)
Creatinine, 0.9 mg/dL (79.6 mcmol/L)
Glucose, 550.0 mg/dL (30.5 mmol/L)
The patient receives initial hydration with 20 mL/kg of normal saline.
Of the following, a TRUE statement regarding this patient is that the

A. initial maintenance fluid regimen should avoid supplemental potassium


B. serum potassium concentration reflects adequate intracellular potassium
C. serum potassium concentration should increase with correction of the acidosis
D. serum potassium concentration should increase with correction of the hyperglycemia
E. total body potassium content is depleted

Copyright 2009 by the American Academy of Pediatrics

page 65

2009 PREP SA on CD-ROM


Critique: 30

Preferred Response: E

Sodium and potassium are cations that often are accompanied by chloride as an anion when
consumed in the diet. Within the body, these cations play very different roles. The cell
membrane is permeable to both sodium and potassium, but due to Na+-K+-ATPase, they are
contained primarily on opposite sides of the cell membrane, with sodium distributed almost
exclusively extracellularly and potassium contained intracellularly. Sodium is the principal
extracellular cation and acts as the primary osmole to maintain extracellular volume-including
plasma volume, which is important for tissue perfusion.
Sodium balance is regulated in the kidney. Excessive sodium intake results in reduced
sodium conservation within the nephron and increased excretion to keep balance with the
increased intake. One tissue in which sodium is contained is the bone, and growing children who
have normal kidney function require 2 to 3 mEq/kg per day of sodium for growth.
Potassium is important for maintaining the resting membrane potential of cells. The typical
intracellular concentration of potassium (140.0 mEq/L [140.0 mmol/L]) is in marked contrast to
the 3.5 to 5.5 mEq/L (3.5 to 5.5 mmol/L) of potassium in the extracellular compartment. The ratio
of extracellular-to-intracellular potassium determines the resting membrane potential and
subsequent action potentials in tissues such as skeletal/cardiac muscle and neuronal tissues.
Accordingly, hypokalemia and hyperkalemia can result in muscle paralysis or cardiac
dysrhythmias such as ventricular tachycardia, fibrillation, and ultimately cardiac arrest. The
growing child requires 1 to 2 mEq/kg per day of potassium for cellular growth and to avoid
potassium deficiency.
Certain conditions alter the measured extracellular potassium concentrations, often
providing misleading information about intracellular stores. In the setting of acidosis, potassium
shifts from intracellular to extracellular locations, increasing measured extracellular potassium
values. Conversely, alkalosis or correction of metabolic acidosis results in a lowering of
measured potassium values. Therapies that result in intracellular shifting of potassium include:
insulin, beta-2 agonists such as albuterol or epinephrine, and sodium bicarbonate (due to
quenching of acidosis). The patient described in the vignette has severe metabolic acidosis with
increased urine output, and the serum potassium value of 5.2 mEq/L (5.2 mmol/L) is misleading.
In fact, potassium should be added to the intravenous fluids because this girl's total body
potassium is depleted, which will become very apparent upon correction of the acidosis and of
the hyperglycemia (with insulin).
References:
Plotnick L. Insulin-dependent diabetes mellitus. Pediatr Rev. 1994;15:137-148. Available at:
http://pedsinreview.aappublications.org/cgi/reprint/15/4/137
Rose BD, Post TW. Potassium homeostasis. In: Clinical Physiology of Acid-base and Electrolyte
Disorders. 5th ed. New York, NY: McGraw-Hill Medical Publishing Division; 2001:372-375
Rose BD, Post TW. The total body water and the plasma sodium concentration. In: Clinical
Physiology of Acid-base and Electrolyte Disorders. 5th ed. New York, NY: McGraw-Hill Medical
Publishing Division; 2001:241-243

Copyright 2009 by the American Academy of Pediatrics

page 66

2009 PREP SA on CD-ROM


Question: 31

You have just assisted in the delivery of a 38-week gestational age male infant who was born via
cesarean section to a 25-year-old woman. As you are completing the infants initial physical
examination, the father mentions that he and his wife have allergic rhinitis and asthma. He asks
whether his son is at increased risk for allergies and how they can reduce the boys chance for
developing such allergic disorders.
Of the following, the MOST appropriate next step is to

A. explain that because both parents have asthma, breastfeeding will not reduce the risk of
eczema

B. explain that breastfeeding or formula choices do not matter now because the mother did not
restrict her diet during pregnancy

C. measure the cord blood immunoglobulin E concentration to help establish the newborns risk
for atopic disorders

D. recommend exclusive breastfeeding for 4 months with the addition of a hypoallergenic formula
if needed

E. start the newborn on a cow milk formula for the first month, then switch to strict breastfeeding
if he develops eczema

Copyright 2009 by the American Academy of Pediatrics

page 67

2009 PREP SA on CD-ROM


Critique: 31

Preferred Response: D

The incidence of atopy (allergic rhinitis, asthma, eczema) has increased significantly over the
past few decades. The ability to intervene and either delay or prevent atopic disease in infants
born to atopic parents has been the subject of numerous studies. Application of these studies to
the population as a whole is difficult because the specific interventions and endpoints for each
study often differ. However, one aspect that is agreed on is that atopy risk for infants increases
significantly when both parents have a history of atopy (30% to 60%) compared with a history
for just one parent (20% to 40%) or neither parent (10% to 15%).
Prior to delivery, two prevention strategies have been studied: maternal diet restriction and
supplementation with probiotics. Currently, no evidence supports maternal dietary restriction to
common allergenic foods. Some studies have supported administration of probiotics (eg,
Lactobacillus rhamnosus) to the mother 2 to 4 weeks before delivery and to the infant for 6
months after birth. One study demonstrated a reduction in eczema at 2 years but no reduction in
asthma, immunoglobulin (Ig) E concentrations, or allergen sensitization. Further, the dose and
type of probiotic has differed in various investigations, making generalized recommendations
difficult.
Even if both parents have atopy, as described in the vignette, breastfeeding or formula
choices may affect atopy outcomes for the infant. In "high-risk" newborns (ie, both parents have
atopy or one parent and one sibling have atopy), the American Academy of Pediatrics
Committee on Nutrition recommends exclusive breastfeeding for at least 4 months, with
supplementation of a hypoallergenic formula if needed. Although it is difficult to compare studies
because the duration of breastfeeding and atopic outcome (ie, eczema, allergic rhinitis, asthma)
differ, breastfeeding for at least 3 months reduces the risk for eczema. The protective benefit
becomes more complex when controlling for the specific maternal atopic condition. For "highrisk" infants born to women who choose not to breastfeed, most studies and experts support
starting an extensively hydrolyzed formula. Starting a cow or soy milk formula, compared with
an extensively hydrolyzed formula, increases the risk for early eczema. Or note, interventions
resulting in decreased atopy early in life may not predict later atopic outcomes.
Cord IgE concentrations can be used to assess a newborn's risk for atopy, but its
measurement currently is not recommended as a routine screening tool. Furthermore, because
both parents in the vignette have a history of atopy, the child already is considered "high risk."
The ability to predict atopy based on cord IgE concentrations also depends on the cutoff value
used. In one study, 80% of newborns whose cord IgE concentrations were greater than 0.9
kU/L subsequently developed atopy by 5 years of age, but the specific IgE value did not
correlate with atopy severity.
References:
American Academy of Pediatrics Committee on Nutrition. Hypoallergenic infant formulas.
Pediatrics. 2000;106: 346-349. Available at:
http://pediatrics.aappublications.org/cgi/content/full/106/2/346
Greer FR, Sicherer SH, Burks AW, Committee on Nutrition and Section on Allergy and
Immunology. Effects of early nutritional interventions on the development of atopic disease in
infants and children: the role of maternal dietary restriction, breastfeeding, timing of introduction
of complementary foods and hydrolyzed formulas. Pediatrics. 2008;121:183-191. Available at:
http://pediatrics.aappublications.org/cgi/content/full/121/1/183
Mihrshahi S, Ampon R, Webb K, et al for the CAPS Team. The association between infant
feeding practices and subsequent atopy among children with a family history of asthma. Clin Exp
Allergy. 2007;37:671-679. Abstract available at: http://www.ncbi.nlm.nih.gov/pubmed/17456214
Prescott SL, Bjrkstn B. Probiotics for the prevention or treatment of allergic disease. J Allergy
Clin Immunol. 2007;120:255-262. Abstract available at:
http://www.ncbi.nlm.nih.gov/pubmed/17544096

Copyright 2009 by the American Academy of Pediatrics

page 68

2009 PREP SA on CD-ROM

Snijders BEP, Thijs C, Dagnelie PC, et al. Breast-feeding duration and infant atopic
manifestations, by maternal allergic status, in the first two years of life (KOALA study). J Pediatr.
2007;151:347-351. Abstract available at: http://www.ncbi.nlm.nih.gov/pubmed/17889066

Copyright 2009 by the American Academy of Pediatrics

page 69

2009 PREP SA on CD-ROM


Question: 32

A 2-year-old girl who has a 4-day history of varicella presents to the office with agitation. Her
mother reports that she treated the fever, rash, and pruritus with acetaminophen and
diphenhydramine regularly, which provided some relief. This morning her daughter seemed
more irritable, had a higher fever than yesterday, and "seemed delirious." On physical
examination, the agitated and inconsolable child has a temperature of 104.2F (40.1C), heart
rate of 160 beats/min, respiratory rate of 36 beats/min, and blood pressure of 135/87 mm Hg.
Her pupils are dilated and sluggishly reactive. Examination of the skin reveals numerous small,
crusted erosions without surrounding erythema. Neurologic examination demonstrates no focal
findings, and the patient is not ataxic.
Of the following, the MOST likely explanation for these symptoms is

A. diphenhydramine overdose
B. hypoglycemia
C. intracranial hemorrhage
D. Reye syndrome
E. varicella cerebellitis

Copyright 2009 by the American Academy of Pediatrics

page 70

2009 PREP SA on CD-ROM


Critique: 32

Preferred Response: A

The patient described in the vignette is exhibiting symptoms of anticholinergic toxicity. The
classic mnemonic "hot as a hare, dry as a bone, blind as a bat, red as a beet, and mad as a
hatter" aptly describes a number of the commonly seen signs and symptoms, including
hyperpyrexia, dry skin, dilated pupils, flushing, and delirium. In addition, affected patients
typically exhibit tachycardia and hypertension. The clinical syndrome caused by anticholinergic
toxicity is due to competitive inhibition of acetylcholine binding to the postganglionic
parasympathetic muscarinic receptors by the offending agent. Many commonly used
medications have anticholinergic properties, including antihistamines, tricyclic antidepressants,
antispasmodics, and mydriatics. For this patient, diphenhydramine is the most likely culprit.
Treatment of anticholinergic poisoning is primarily supportive, with stabilization of vital
functions (the ABCs) of greatest importance, followed by appropriate decontamination (eg,
activated charcoal for ingestions, skin cleansing and removal of transdermal delivery devices for
dermal absorption). Agitation and seizures may be treated with benzodiazepines.
Physostigmine, a cholinergic agent that competitively inhibits acetylcholinesterase, may be more
effective for the treatment of severe agitation than benzodiazepines, but significant toxicity risks
make its use controversial.
Hypoglycemia can cause altered mental status but typically produces bradycardia,
hypotension, and coma, rather than agitation. Intracranial hemorrhage can be seen in patients
who have varicella-related thrombocytopenia, although affected patients frequently present with
hemorrhagic skin lesions (Item C32). In addition, patients who have intracranial hemorrhage
typically present with focal neurologic findings and bradycardia with hypertension, if intracranial
pressure is increased.
The classic presentation of Reye syndrome includes sudden onset of protracted vomiting
without fever in association with lethargy that progresses to delirium, seizures, stupor, and
coma. Reye syndrome typically occurs 5 to 7 days after the onset of a viral illness, following
apparent improvement in the child's initial symptoms. It results from mitochondrial dysfunction
leading to liver failure and cerebral edema. Ninety percent of cases have been reported to follow
an upper respiratory tract infection; varicella has been associated with 5% of cases.
Central nervous system complications of varicella are uncommon and may include
cerebellitis, transverse myelitis, peripheral neuritis, and optic neuritis. Cerebellitis is the most
common presentation of encephalitis and typically presents on the third to eighth day of illness
with ataxia. Meningoencephalitis presents with fever, meningeal signs, seizures, and altered
mental status.
References:
Burns JJ Jr. Toxicity, anticholinergic. eMedicine Specialties, Emergency Medicine, Toxicology.
2006. Available at: http://www.emedicine.com/EMERG/topic36.htm
Burns MJ, Linden CH, Graudins A, Brown RM, Fletcher KE. A comparison of physostigmine and
benzodiazepines for the treatment of anticholinergic poisoning. Ann Emerg Med. 2000;35:374381. Abstract available at: http://www.ncbi.nlm.nih.gov/pubmed/10736125
Carey RG, Balisteri WF. Mitochondrial hepatopathies. In: Kleigman RM, Behrman RE, Jenson
HB, Stanton BF, eds. Nelson Textbook of Pediatrics. 18th ed. Philadelphia, Pa: Saunders
Elsevier; 2007:1696-1697
Gershon AA, LaRussa P. Varicella-zoster virus infections. In: Gershon AA, Hotez PJ, Katz SL,
eds. Krugman's Infectious Diseases of Children. 11th ed. Philadelphia, Pa: Mosby; 2004:785-816
Su M, Goldman M. Anticholinergic poisoning. UpToDate Online 15.3. 2008. Available for
subscription at:
http://www.utdol.com/utd/content/topic.do?topicKey=ad_tox/13958&selectedTitle=1~376&source
=search_result

Copyright 2009 by the American Academy of Pediatrics

page 71

2009 PREP SA on CD-ROM


Question: 33

A 7-month-old child presents for a follow-up office visit after undergoing a Kasai procedure for
biliary atresia at 6 weeks of age. The mother states that the boy is irritable when his right arm is
moved. On physical examination, the infant is jaundiced. You detect tenderness in the anterior
radial head. Radiography of the affected region demonstrates metaphyseal fraying (Item Q33)
and a fracture.
Of the following, the MOST appropriate laboratory studies to obtain next are

A. calcium and phosphorus measurement and bone densitometry (DEXA scan)


B. calcium and phosphorus measurement and urinary calcium-to-creatinine ratio
C. calcium, phosphorus, and 25-hydroxyvitamin D measurement
D. calcium, phosphorus, and magnesium measurement
E. magnesium, phosphorus, and parathyroid hormone measurement

Copyright 2009 by the American Academy of Pediatrics

page 72

2009 PREP SA on CD-ROM


Critique: 33

Preferred Response: C

Chronic cholestasis due to biliary atresia results in decreased bile flow into the intestine. The
absence of intraluminal bile acids, in turn, causes decreased digestion of lipids, leading to fat
malabsorption. In addition, absorption of fat-soluble vitamins (A, D, E, and K) is impaired, which
may lead to clinical sequelae of fat-soluble vitamin deficiency. Finally, steatorrhea may impair
calcium absorption because intraluminal free fatty acids may bind calcium.
The clinical presentation of the patient in the vignette strongly suggests the presence of
rickets from vitamin D deficiency. Therefore, the most helpful initial laboratory testing is
determination of calcium, phosphorus, and 25-hydroxyvitamin D concentrations. The 25hydroxyvitamin D assay is the best measure of hepatic stores of vitamin D and is a better
marker of vitamin D status than either serum vitamin D or 1,25-dihydroxyvitamin D. Although
bone density testing, measurement of serum magnesium and parathyroid hormone, and
determination of the urinary calcium-to-creatinine ratio may provide useful additional information,
they will not help establish the diagnosis of vitamin D-deficient rickets.
Rickets is a potentially preventable complication of biliary atresia, but requires monitoring of
calcium, phosphorus, and 25-hydroxyvitamin D concentrations two to four times a year. Infants
who have biliary atresia routinely receive supplementation with approximately 8,000 IU of
ergocalciferol (vitamin D2) daily. This dose of vitamin D is approximately 20 times the
recommended dietary allowance for a healthy toddler. If rickets develops or the vitamin D
concentration cannot be maintained within the normal range, the patient should receive either
calcitriol (1,25-dihydroxyvitamin D3) or intramuscular vitamin D.
References:
Campbell KM, Bezerra JA. Biliary atresia. In: Walker WA, Goulet O, Kleinman RE, Sherman PM,
Shneider BL, Sanderson IR, eds. Pediatric Gastrointestinal Disease. 4th ed. Hamilton, Ontario,
Canada: BC Decker; 2004:1122-1138
Suchy FJ. Neonatal cholestasis. Pediatr Rev. 2004;25:388-396. Available at:
http://pedsinreview.aappublications.org/cgi/content/full/25/11/388

Copyright 2009 by the American Academy of Pediatrics

page 73

2009 PREP SA on CD-ROM


Question: 34

You are examining a 3.5-kg term infant 48 hours after his birth. Results of the physical
examination are normal, and you are considering discharging him from the hospital. He is being
fed formula from a bottle, and the nurses report intakes of 30 mL every 3 hours. He has wet at
least six diapers daily for the past 2 days, but he has not passed any meconium or expressed
any stool since birth.
Of the following, the MOST likely diagnosis is

A. ileal atresia
B. imperforate anus
C. meconium ileus
D. meconium plug syndrome
E. neonatal small left colon syndrome

Copyright 2009 by the American Academy of Pediatrics

page 74

2009 PREP SA on CD-ROM


Critique: 34

Preferred Response: D

Ninety-five percent of term infants express meconium or pass a stool in the first 24 hours of
postnatal life. The infant described in the vignette is term and appropriately grown and has been
feeding and voiding well, but he has failed to pass meconium or any stool in the first 2 days of
postnatal life. Although he does not have any abdominal distention, emesis, or systemic illness,
the pediatric clinician should be concerned about potential bowel, particularly colonic,
obstruction.
The most likely explanation for the symptoms described for the infant in the vignette is
meconium plug syndrome, which typically is an isolated phenomenon that is not associated with
anatomic obstruction (eg, atresia). It occurs commonly in term and preterm infants and may be
associated with maternal magnesium sulfate treatment for pre-eclampsia/eclampsia. Meconium
plug obstruction generally is related to hypomotility. Clinically, there may be no abdominal
findings or a gradual increase in girth but no other signs of illness. Plain radiographs of the
abdomen generally provide nonspecific findings, but may show a paucity of gas in the
rectosigmoid. A contrast enema characteristically illuminates the plugs of meconium and
facilitates their evacuation. On occasion, a firm, paraffin-like formed plug may be expressed
spontaneously by affected infants during the second postnatal day. Although some infants who
have retained meconium may exhibit a small left colon on contrast enema, colonic motility
usually is normal upon evacuation of the meconium plug(s).
Although meconium plug syndrome is the most common cause of delayed passage of stool,
the clinician also should consider Hirschsprung disease, a congenital absence of ganglion cells.
A failure to pass meconium in the first 24 hours of postnatal life characterizes 95% of affected
infants. The area of affected bowel typically is in the rectosigmoid, where a transition zone may
be observed on contrast enema, although this finding is less common in neonates. If
Hirschsprung disease is considered, diagnostic rectal biopsy should be performed.
The neonatal small left colon syndrome is seen in infants of diabetic mothers and is
diagnosed using a contrast enema. The enema may be both diagnostic and therapeutic, as seen
in meconium plug syndrome. Gradual feeding and monitoring of the stooling pattern generally
results in resolution of the condition over the early weeks of postnatal life.
Imperforate anus occurs in about 1 in 4,000 to 5,000 births, and typically is apparent on
physical examination. In some cases, a fistulous tract may exist, and the expression of
meconium may occur anywhere along the perineal-scrotal-urethral line. Imperforate anus may
be an isolated finding or seen in conjunction with other anomalies such as vertebral
malformations, cardiac malformation, tracheoesophageal fistula/esophageal atresia, renal
anomalies, and limb malformation (VACTERL association).
Ileal atresia (proximal or distal) occurs as part of a spectrum of jejunal-ileal bowel atresia
that likely reflects a mesenteric vascular defect or interruption in development. Its absolute
frequency is not well reported, although it commonly is diagnosed prenatally (dilated bowel,
polyhydramnios) on obstetric ultrasonography. With a distal obstruction, the newborn may take
early feedings well, but becomes ill, with bile-stained emesis and abdominal distention, in the first
24 to 48 postnatal hours. Plain radiographic findings may include multiple stacked loops of airfilled bowel and air-fluid levels.
Meconium ileus is a condition of intestinal obstruction related to thickened, inspissated
mucus mixed with meconium that is characteristic of cystic fibrosis and is related to altered
chloride and water balance in mucus. The meconium may be beadlike, in small, dense pellets,
and even visible on prenatal obstetric ultrasonography. A microcolon may exist distal to the
small bowel obstruction. Affected infants may have visible and palpable loops of bowel on
examination, in addition to abdominal distention, bilious emesis, and failure to pass meconium in
the first 24 to 48 hours of postnatal life. Plain radiographs may reveal a soap-bubble appearance
characteristic of meconium stool. A contrast enema may reveal a microcolon and failure to see
contrast reflux past the ileocecal valve.
References:
Albanese CT, Sylvester KG. Pediatric surgery. In: Doherty GM, Way LW, eds. Current Surgical

Copyright 2009 by the American Academy of Pediatrics

page 75

2009 PREP SA on CD-ROM

Diagnosis and Treatment. 12th ed. New York, NY: The McGraw-Hill Companies, Inc; 2006:chap
45
Burge D, Drewett M. Meconium plug obstruction. Pediatr Surg Int. 2004;20:108-110. Abstract
available at: http://www.ncbi.nlm.nih.gov/pubmed/14760494
Casaccia G, Trucchi A, Spirydakis I, et al. Congenital intestinal anomalies, neonatal short bowel
syndrome, and prenatal/neonatal counseling. J Pediatr Surg. 2006;41:804-807. Abstract
available at: http://www.ncbi.nlm.nih.gov/pubmed/16567197
Hajivassiliou CA. Intestinal obstruction in neonatal/pediatric surgery. Semin Pediatr Surg.
2003;12:241-253. Abstract available at: http://www.ncbi.nlm.nih.gov/pubmed/14655163
Magnuson DK, Parry RL, Chwals WJ. Selected abdominal gastrointestinal anomalies. In: Martin
RJ, Fanaroff AA, Walsh MC, eds: Fanaroff and Martin's Neonatal-Perinatal Medicine. 8th ed.
Philadelphia, Pa: Mosby Elsevier; 2006:1381-1402
Nurko S. Motility of the colon and anorectum. NeoReviews. 2006;7:e34-e48. Available for
subscription at: http://neoreviews.aappublications.org/cgi/content/full/7/1/e34
Sutton TL. Index of suspicion in the nursery. NeoReviews. 2006;7:e269-e271. Available for
subscription at: http://neoreviews.aappublications.org/cgi/content/full/7/5/e269
Thilo EH, Rosenberg AA. The newborn infant. In: Hay WW Jr, Levin M, Sondheimer JM,
Deterding RR, eds. Current Pediatric Diagnosis & Treatment. 18th ed. New York, NY: The
McGraw-Hill Companies, Inc; 2007:chap 1

Copyright 2009 by the American Academy of Pediatrics

page 76

2009 PREP SA on CD-ROM


Question: 35

A 4-year-old boy who recently emigrated from Central America is brought to your clinic because
of 2 weeks of colicky abdominal pain that recently has worsened. His vital signs are normal, and
he is afebrile. Physical examination reveals mild diffuse tenderness, but there is no rebound or
guarding. After your examination, he has an episode of vomiting. Examination of the vomitus
reveals long, slim objects that resemble worms (Item Q35).
Of the following, the BEST treatment choice is

A. albendazole
B. iodoquinol
C. metronidazole
D. praziquantel
E. voriconazole

Copyright 2009 by the American Academy of Pediatrics

page 77

2009 PREP SA on CD-ROM


Critique: 35

Preferred Response: A

Ascariasis is caused by infestation with the roundworm Ascaris lumbricoides. It occurs most
commonly in tropical regions and areas that have poor sanitation, although many cases also
occur in the United States each year. Adult worms (Item C35A) live in the small intestine and
produce eggs (Item C35B) that are excreted in the stool into the soil. Infestation occurs when
the eggs in the contaminated soil are ingested. Larvae pass from the small intestine into the
bloodstream, traveling to the liver and lungs. They migrate from the lung to the pharynx, where
they are swallowed. The worms mature in the small intestine, where they produce their eggs,
completing the cycle.
Many patients who have ascariasis are asymptomatic, but symptoms such as nonspecific
gastrointestinal complaints and abdominal pain can occur, as described for the boy in the
vignette. Intestinal obstruction and symptoms related to the migration of the larvae, such as
obstructive jaundice and peritonitis, also may be seen. Adult worms may pass from the rectum,
nose, or mouth if there is heavy worm burden. The diagnosis is made by seeing either ova on
microscopic stool examination or the adult worm itself, as described for the boy in the vignette.
Treatment of ascariasis that is not associated with intestinal obstruction consists of a single
dose of either albendazole or pyrantel pamoate or a 3-day course of mebendazole. Intestinal
obstruction may require treatment with piperazine citrate solution or, rarely, surgical intervention.
Iodoquinol is a luminal amebicide that is effective for the treatment of asymptomatic amebic
cyst excreters, and metronidazole is the drug of choice for patients who have symptomatic
amebiasis as well as giardiasis. Neither is appropriate for the treatment of ascariasis.
Praziquantel is useful for the treatment of liver fluke infestations, schistosomiasis, and tapeworm
infestations. Voriconazole is an intravenous or oral medication used in the treatment of fungal
infections.
References:
American Academy of Pediatrics. Amebiasis. In: Pickering LK, Baker CJ, Long SS, McMillan JA,
eds. Red Book: 2006 Report of the Committee on Infectious Diseases. 27th ed. Elk Grove
Village, Ill: American Academy of Pediatrics; 2006:204-208
American Academy of Pediatrics. Ascaris lumbricoides infections. In: Pickering LK, Baker CJ,
Long SS, McMillan JA, eds. Red Book: 2006 Report of the Committee on Infectious Diseases.
27th ed. Elk Grove Village, Ill: American Academy of Pediatrics; 2006:218-219
Dent AE, Kazura JW. Ascariasis (Ascaris lumbricoides). In: Kliegman RM, Behrman RE,
Jenson HB, Stanton BF, eds. Nelson Textbook of Pediatrics. 18th ed. Philadelphia, Pa:
Saunders Elsevier; 2007:1495

Copyright 2009 by the American Academy of Pediatrics

page 78

2009 PREP SA on CD-ROM


Question: 36

An 8-month-old boy who has Down syndrome and a large ventriculoseptal defect has had
recurrent otitis media and sinusitis during the respiratory virus season that required four
separate courses of antibiotics in 4 months. At todays visit, his mother states that his rhinitis
and otitis media symptoms have resolved, but she is concerned about a recurrent diaper rash
that is unresponsive to both barrier creams and repeated use of the nystatin cream prescribed
last month. Examination reveals white plaques (Item Q36A) on the buccal mucosa just inside the
lips and a diaper rash (Item Q36B).
Of the following, the BEST therapeutic option for this child is

A. oral fluconazole
B. oral griseofulvin
C. oral itraconazole
D. topical clotrimazole alone
E. topical clotrimazole combined with triamcinolone

Copyright 2009 by the American Academy of Pediatrics

page 79

2009 PREP SA on CD-ROM


Critique: 36

Preferred Response: A

The child described in the vignette has recurrent Candida diaper dermatitis and thrush that is
probably due to his four courses of antibiotics. Candida infections are a common problem in
infants, toddlers, and children who have underlying medical conditions that require frequent
administration of antibiotics. The administration of antibiotics disrupts normal skin and mucous
membrane microbiologic flora. The boy described in the vignette has been prescribed topical
antifungal preparations without sustained improvement.
Prescribing alternative topical antifungal preparations such as clotrimazole for the diaper
rash might treat Candida sp that are resistant to nystatin, but it will not treat thrush. Once-daily
oral fluconazole has been proven superior to oral nystatin for resistant thrush and effective for
Candida diaper dermatitis. Recurrence rates are similar for both fluconazole and nystatin. Oral
itraconazole is another effective antifungal, but it may be associated with more gastrointestinal
adverse effects, including hepatotoxicity, and is dosed twice daily. Griseofulvin is not the drug of
choice for Candida infection, although it is useful in fungal hair and nail infections.
Combining antifungal agents with steroids may diminish inflammation temporarily, but
steroids also delay the resolution of fungal infection by decreasing T-cell response in the skin,
resulting in worsening of the dermatitis after a few days. Further, high systemic absorption of
steroids may occur in areas of thin and occluded skin, such as the diaper area.
References:
American Academy of Pediatrics. Candidiasis (moniliasis, thrush). In: Pickering LK, Baker CJ,
Long SS, McMillan JA, eds. Red Book: 2006 Report of the Committee on Infectious Diseases.
27th ed. Elk Grove Village, Ill: American Academy of Pediatrics; 2006:242-246
American Academy of Pediatrics. Drugs for invasive and other serious fungal infections in
children. In: Pickering LK, Baker CJ, Long SS, McMillan JA, eds. Red Book: 2006 Report of the
Committee on Infectious Diseases. 27th ed. Elk Grove Village, Ill: American Academy of
Pediatrics; 2006:780
American Academy of Pediatrics. Recommended doses of parenteral and oral antifungal drugs.
In: Pickering LK, Baker CJ, Long SS, McMillan JA, eds. Red Book: 2006 Report of the
Committee on Infectious Diseases. 27th ed. Elk Grove Village, Ill: American Academy of
Pediatrics; 2006:777-779
American Academy of Pediatrics. Topical drugs for superficial fungal infections. In: Pickering LK,
Baker CJ, Long SS, McMillan JA, eds. Red Book: 2006 Report of the Committee on Infectious
Diseases. 27th ed. Elk Grove Village, Ill: American Academy of Pediatrics; 2006:781-784
Goins RA, Ascher D, Waecker N, Arnold J, Moorefield E. Comparison of fluconazole and
nystatin oral suspensions for treatment of oral candidiasis in infants. Pediatr Infect Dis J.
2002;21:1165-1167. Abstract available at: http://www.ncbi.nlm.nih.gov/pubmed/12506950
Pankhurst CL. Antifungal treatment in immunocompetent or immunocompromised infants and
children. BMJ Clinical Evidence. 2007. Available for subscription at:
http://clinicalevidence.bmj.com/ceweb/conditions/orh/1304/1304_I4.jsp538REF22
Weisse ME, Aronoff SC. Candida. In: Kliegman RM, Behrman RE, Jenson HB, Stanton BF, eds.
Nelson's Textbook of Pediatrics. 18th ed. Philadelphia, Pa: Saunders Elsevier; 2007:1207-1310

Copyright 2009 by the American Academy of Pediatrics

page 80

2009 PREP SA on CD-ROM


Question: 37

You are called to the newborn nursery to evaluate a 2-hour-old male who was born at term. The
pregnancy was uncomplicated, but meconium staining was noted at delivery. The baby weighs
3.8 kg, is afebrile, and has a heart rate of 165 beats/min and a respiratory rate of 70
breaths/min. You note tachypnea and hyperpnea with clear breath sounds, no murmurs, and
strong distal pulses. His oxygen saturation in room air is 68%. You place a nonrebreather mask
to deliver an Fio2 of 1.0. After 5 minutes, the oxygen saturation is 72%.
Of the following, the BEST explanation for the findings of the hyperoxia test is

A. meconium aspiration syndrome


B. persistent pulmonary hypertension of the newborn
C. pneumonia
D. retained fetal lung liquid syndrome
E. transposition of the great arteries

Copyright 2009 by the American Academy of Pediatrics

page 81

2009 PREP SA on CD-ROM


Critique: 37

Preferred Response: E

Most commonly, the practitioner is alerted to hypoxemia in the newborn by the finding of a low
oxygen saturation value. Among the various causes of abnormal oxygenation in the newborn
are pulmonary pathologies, congenital cardiovascular malformations, persistent pulmonary
hypertension of the newborn, and disturbances of the hematologic and metabolic systems. Rightto-left shunting can be thought of as a diversion of desaturated blood away from the lungs and to
the systemic circulation. This can occur because blood does not perfuse the ventilated portions
of the lung (intrapulmonary right-to-left shunting). Intrapulmonary shunting resulting from
infection such as pneumonia, pneumothorax, retained fetal lung liquid, and pulmonary
prematurity is the most frequently encountered reason for desaturation in a newborn.
Conversely, abnormal oxygenation can result from the situations in which the desaturated blood
does not perfuse the pulmonary artery from the heart (intracardiac right-to-left shunting) or is
diverted from the pulmonary circuit through the ductus arteriosus (extracardiac right-to-left
shunting). Examples of these pathologies include pulmonary atresia, transposition of the great
arteries, tricuspid atresia, and pulmonary hypertension.
Whether the cause of the desaturation is intrapulmonary, intracardiac, or extracardiac rightto-left shunting, cyanosis (blue, maroon, or purple discoloration of the skin) is likely to be
present. Typically, clinicians discern cyanosis in patients who have oxygen saturations of less
than 85%, although it may be apparent to some when the saturation is 90% or less. If cyanosis
is suspected in the newborn, cyanotic heart disease must be considered. It is reasonable to
place the patient in a high-oxygen atmosphere (near FiO2 of 1.0) to determine if the high-dose
oxygen can overcome the shunting. If the degree of cyanosis improves and oxygen saturations
become normal, the problem likely is intrapulmonary shunting rather than cyanotic heart
disease. If hyperoxia does not lead to increased oxygen saturation and higher PaO2 (=150 torr),
cyanotic congenital heart disease should be considered and the infant should undergo further
cardiac evaluation.
The newborn described in the vignette has cyanosis but no murmurs. His tachypnea and
hyperpnea (deep breathing) represent the physiologic response to hypoxemia. His saturation
improves slightly with the delivery of high-dose oxygen. Transposition of the great arteries is the
best explanation for the infant's hypoxemia (Item C37). No amount of oxygen delivered to the
patient's alveoli can improve oxygenation of the pulmonary blood flow because the pulmonary
blood flow in transposition already is well saturated. The neonate remains desaturated until
oxygenated blood from the left atrium adequately crosses the atrial septum to be delivered to the
system through the aorta. In contrast, the oxygenation defect associated with meconium
aspiration syndrome, persistent pulmonary hypertension of the newborn, pneumonia, and
retained fetal lung liquid syndrome is improved with high-dose oxygen delivery to the alveoli.
References:
Driscoll D, Allen HD, Atkins DL, et al. Guidelines for evaluation and management of common
congenital cardiac problems in infants, children, and adolescents. A statement for healthcare
professionals from the Committee on Congenital Cardiac Defects of the Council on
Cardiovascular Disease in the Young, American Heart Association. Circulation. 1994;90:21802188. Available at: http://circ.ahajournals.org/cgi/reprint/90/4/2180
Ranjit MS. Common congenital cyanotic heart defects--diagnosis and management. J Indian
Med Assoc. 2003;101:71-72, 74. Abstract available at:
http://www.ncbi.nlm.nih.gov/pubmed/12841486
Silberbach M, Hannon D. Presentation of congenital heart disease in the neonate and young
infant. Pediatr Rev. 2007;28:123-131. Available at:
http://pedsinreview.aappublications.org/cgi/content/full/28/4/123

Copyright 2009 by the American Academy of Pediatrics

page 82

2009 PREP SA on CD-ROM


Question: 38

The mother of a 10-month-old child who has mild hypotonia brings him to the office after he has
an unprovoked seizure. On physical examination, you note several hypopigmented macules on
the trunk (Item Q38A). Magnetic resonance imaging of the brain reveals several thickened
areas of cerebral cortex (Item Q38B), with abnormal signal and abnormalities along the walls of
the lateral ventricles (Item Q38C).
Of the following, the MOST likely diagnosis is

A. incontinentia pigmenti
B. neurofibromatosis type 1
C. Sturge-Weber syndrome
D. tuberous sclerosis
E. von Hippel-Lindau syndrome

Copyright 2009 by the American Academy of Pediatrics

page 83

2009 PREP SA on CD-ROM


Critique: 38

Preferred Response: D

Low tone and seizures are relatively common neurologic problems. Low tone is a nonspecific
finding that may be due to disease in the central or peripheral nervous system, but the
occurrence of a seizure suggests a central cerebral cause. Neuroimaging with brain magnetic
resonance imaging (MRI) generally is recommended for any infant who has a seizure because
congenital brain malformations are more common at this age. The combination of such cerebral
symptoms and pigmentary abnormalities of the skin are an indication to obtain brain MRI to
assess for a possible neurocutaneous disorder. The seizures, hypotonia, hypopigmented
macules, and MRI findings described for the child in the vignette are most suggestive of
tuberous sclerosis complex (TSC) (Item C38A).
Incontinentia pigmenti is characterized by spasticity rather than hypotonicity, and skin
findings include swirled hyperpigmentation following the line Blaschko (Item C38B). Skin findings
associated with neurofibromatosis type 1 include caf au lait macules (Item C38C), axillary
freckling, and iris Lisch nodules (Item C38D). Although Sturge-Weber syndrome can be
associated with seizures in the first postnatal year, the primary skin finding is a port wine stain
(Item C38E). Neither seizures nor skin abnormalities are seen with von Hippel-Lindau
syndrome.
Initial management of this patient centers on the chief complaint, which is the seizure. In
most cases, no treatment is recommended in a child after a first unprovoked seizure. However,
the recurrence risk is much greater in TSC.
Subsequently, diagnostic assessment should be directed toward confirming whether this
child has TSC. In many cases, this is a clinical diagnosis based on the characteristic findings of
skin examination and the cerebral complications. However, many other organs may be involved
at presentation or during the child's lifetime, including the eyes, kidneys, lungs, and heart. In the
future, this child is at risk for developmental learning difficulties, behavior problems that can
include features of autistic spectrum disorders, and malignancies. Given the autosomal
dominant inheritance, proper management involves assessment of the parents and genetic
counseling. Commercial genetic testing is available and is helpful in cases where clinical
features, particularly early in the disease, do not confirm the diagnosis. Genetic testing may
have false-negative results due to mosaicism, ie, some organs may be affected due to TSC1 or
TSC2 mutations that are not present in blood. Given the complexity of this diagnosis, many
experts recommend that children who have TSC be cared for in multidisciplinary specialty
clinics.
References:
Ferner RE. Neurofibromatosis 1 and neurofibromatosis 2: a twenty first century perspective.
Lancet Neurol. 2007;6:340-351. Abstract available at:
http://www.ncbi.nlm.nih.gov/pubmed/17362838
Haslam RHA. Neurocutaneous syndromes. In: Kliegman RM, Behrman RE, Jenson HB, Stanton
BF, eds. Nelson Textbook of Pediatrics. 18th ed. Philadelphia, Pa: Saunders Elsevier; 2007:24832488
Kandt RS. Tuberous sclerosis complex. In: Singer HS, Kossoff EH, Hartman AL, Crawford TO,
eds. Treatment of Pediatric Neurologic Disorders. Boca Raton, Fla: Taylor & Francis; 2005:553560
Northrup H, Au K-S. Tuberous sclerosis complex. GeneReviews. 2005. Available at:
http://www.ncbi.nlm.nih.gov/bookshelf/br.fcgi?book=gene&part=tuberous-sclerosis

Copyright 2009 by the American Academy of Pediatrics

page 84

2009 PREP SA on CD-ROM


Question: 39

You are called to the emergency department to evaluate a 5-month-old boy who has new-onset
seizures. On physical examination, you note that he is thin and has marked hepatomegaly. The
mother tells you that he has been irritable the past several mornings when he awakened from a
full nights sleep. This morning, she found him seizing in his crib and called 911. Laboratory tests
performed on specimens taken prior to starting intravenous fluids reveal hypoglycemia, lactic
acidosis, hyperuricemia, and hyperlipidemia. You suspect a diagnosis of glycogen storage
disease.
Of the following, the MOST appropriate long-term management of this disorder includes

A. coenzyme Q10 administration


B. oral administration of cornstarch
C. oral carnitine supplementation
D. protein restriction
E. restriction of long-chain fats

Copyright 2009 by the American Academy of Pediatrics

page 85

2009 PREP SA on CD-ROM


Critique: 39

Preferred Response: B

The hepatomegaly, severe fasting hypoglycemia, lactic acidosis, hyperuricemia, hyperlipidemia,


and ketonuria described for the thin child in the vignette are most consistent with glycogen
storage disease type I (GSD I) (von Gierke disease). GSD I is an autosomal recessive disorder
resulting from deficiency of the enzyme glucose-6-phosphatase, and it is the most serious of all
the hepatic glycogenoses.
The laboratory findings result from complete blockage of the release of glycogen. Affected
children typically have massive hepatomegaly without splenomegaly on physical examination,
and they may have a wasted appearance. Kidneys are enlarged and may be palpable on
examination. Parents may give a history of irritability and pallor, especially prior to feedings (after
fasting). Some of the children develop seizures.
The mainstays of treatment for GSD I are the avoidance of fasting and frequent
administration of free glucose. The approaches that have been most successful include
continuous nocturnal nasogastric or gastrostomy feedings or administration of uncooked
cornstarch every 4 hours during sleep or other times of fasting. Maintenance of euglycemia
reverses clinical and biochemical abnormalities in most patients.
Coenzyme Q often is administered to individuals who have mitochondrial disorders and is of
unclear benefit, but it plays no role in the management of GSD I. Similarly, carnitine
supplementation and protein and long-chain fat restrictions are of no benefit in GSD I.
The management of disorders of carbohydrate metabolism, regardless of their cause, is
aimed at ensuring the availability of energy for cellular metabolism without compromising
necessary fat and protein stores. This requires frequent delivery of carbohydrates, and
gastrostomy tube placement or venous access may be necessary to ensure success.
References:
Hoffmann GF, Nyhan WL, Zschocke J, Kahler SG, Mayatepek E. Approach to the patient with
hepatic disease. In: Inherited Metabolic Diseases. Philadelphia, Pa: Lippincott Williams & Wilkins;
2002:191-214
Nyhan WL, Barshop BA, Ozand PT. Disorders of carbohydrate metabolism. In: Atlas of
Metabolic Diseases. 2nd ed. London, England: Hodder Arnold; 2005:371-402

Copyright 2009 by the American Academy of Pediatrics

page 86

2009 PREP SA on CD-ROM


Question: 40

A 13-year-old girl comes to your office because her menstrual periods are irregular. She
attained menarche at 12 years of age and states that she has had only four menstrual periods
over the past year. The periods last for 5 to 7 days and require the use of four pads per day.
She has never been sexually active. She plays no sports, but she swims in the summer for fun.
On physical examination, her weight and height are at the 50th percentile for age. She has
minimal facial acne and no hirsutism or other skin lesions. Her breast and genital development is
at Sexual Maturity Rating 5.
Of the following, the MOST appropriate management strategy for this patient is to

A. measure follicle-stimulating hormone, luteinizing hormone, and prolactin


B. measure free thyroxine and thyroid-stimulating hormone
C. monitor menses and reassure the girl
D. obtain a bone age by radiography
E. obtain pelvic ultrasonography

Copyright 2009 by the American Academy of Pediatrics

page 87

2009 PREP SA on CD-ROM


Critique: 40

Preferred Response: C

Young women of low gynecologic age (ie, a few years after the onset of menstruation) often
have anovulatory cycles due to immaturity of the hormonal feedback system of the
hypothalamic-pituitary-ovarian axis. During the adolescent years, when ovulation does not occur
with every cycle, both frequent and infrequent menstruation can result. By 2 years after
menarche, 55% to 82% of cycles are ovulatory, and by 5 years after menarche, 80% to 90%
are ovulatory, resulting in more regular menses. The girl described in the vignette has no signs
of androgen excess or evidence of weight loss that may be associated with an eating disorder
but does have scant irregular periods without cramps or heavy prolonged menses. Because
these findings suggest physiologic anovulatory cycles, no further laboratory studies, such as
measurement of follicle-stimulating hormone, luteinizing hormone, and prolactin, are necessary.
Her continued normal linear growth obviates the need for a bone age measurement. The
girl's normal growth coupled with no evidence of fatigue or skin dryness make thyroid disease
unlikely. Pelvic ultrasonography is not indicated in the absence of signs of androgen excess or
other symptoms suggestive of ovarian pathology. The possibility of pregnancy always should be
assessed in the presence of amenorrhea.
References:
Emans SJ. Amenorrhea in the adolescent. In: Emans SJH, Laufer MR, Goldstein DP, eds.
Pediatric and Adolescent Gynecology. 5th ed. Philadelphia, Pa: Lippincott, Williams & Wilkins;
2005:214-269
Ohlemeyer CL. Menstrual disorders. In Osborn LM, DeWitt TG, First LR, Zenel JA eds.
Pediatrics. Philadelphia, Pa: Elsevier Mosby; 2005:1455-1460

Copyright 2009 by the American Academy of Pediatrics

page 88

2009 PREP SA on CD-ROM


Question: 41

You are evaluating a 20-month-old boy who has a rectal temperature of 106F (41.1C) and a
history of coughing. His mother reports that the child has had a decrease in activity and eating
over the past 2 days. On physical examination, the boy appears moderately ill but is alert and
easily interacts with you. He occasionally grunts, has a heart rate of 140 beats/min, and has a
respiratory rate of 55 breaths/min. His neck is supple, he is circumcised, and he has no
evidence of otitis media.
Of the following, the BEST initial test in the evaluation of this child is

A. chest radiography
B. C-reactive protein measurement
C. erythrocyte sedimentation rate
D. lumbar puncture
E. urinalysis

Copyright 2009 by the American Academy of Pediatrics

page 89

2009 PREP SA on CD-ROM


Critique: 41

Preferred Response: A

Physical examination, a detailed history, and direct observation can help the clinician recognize
up to 90% of children who have a serious illness. Carefully selected laboratory and diagnostic
tests can enhance the detection of invasive bacterial infections in children who have fever.
Infants younger than 3 months of age who present with a fever are at high risk of invasive
bacterial infection due to their still-developing immune system. Specific high-risk factors,
including a history of prematurity, evidence of an abnormally elevated or depressed white blood
cell count, a focal source of infection such as otitis media or soft-tissue infection, or abnormal
chest findings on radiography are associated with serious bacterial infection in almost 25% of
cases. Infants younger than 3 months of age who do not have such high-risk factors still have a
nearly 3% incidence of a serious bacterial infection. Infants between the ages of 3 months and 3
years remain at risk for serious bacterial infections, with reports of a 3% incidence of bacteremia
associated with a temperature of 102.2F (39.0C) and 7% with a temperature of 104.0F
(40.0C). A temperature higher than 105.8F (41.0C) frequently is associated with invasive
bacterial infection.
The child described in the vignette has both an elevated temperature and signs and
symptoms suggestive of a primary respiratory infection that make chest radiography the best
initial diagnostic test. A lumbar puncture can be deferred because of the patient's age and
appropriate interaction with the examiner. The patient's age, sex, and circumcised status make
a urinary tract infection unlikely; as a result, urinalysis also could be deferred pending results of
chest radiography. Positive urinalysis can be suggestive of a urinary tract infection, but a urine
culture remains the definitive test when a urinary tract infection is suspected. Elevated
erythrocyte sedimentation rates and C-reactive protein measurements are indicative of the
presence of acute inflammatory processes, but they are nonspecific findings. Their elevation in
young children who have fever may be useful in guiding the clinician toward additional studies in
selected children.
References:
Brook I. Unexplained fever in young children: how to manage severe bacterial infection. BMJ.
2003;327:1094-1097. Available at: http://www.bmj.com/cgi/content/full/327/7423/1094
McCarthy PL. Evaluation of the sick child in the office and clinic. In: Kliegman RM, Behrman RE,
Jenson HB, Stanton BF, eds. Nelson Textbook of Pediatrics. 18th ed. Philadelphia, Pa:
Saunders Elsevier; 2007:363-365

Copyright 2009 by the American Academy of Pediatrics

page 90

2009 PREP SA on CD-ROM


Question: 42

You are called to the emergency department to see an 8-year-old girl in whom congenital
adrenal hyperplasia was diagnosed at birth. She is being treated with oral mineralocorticoid daily
(9-alpha-fludrocortisone 0.1 mg) and hydrocortisone 5 mg orally every 8 hours. She is febrile
(temperature of 102.0F [38.9C]) and has vomited twice. According to her mother, other family
members recently recovered from a gastrointestinal illness that started with fever and vomiting.
Of the following, the MOST appropriate treatment for this child is to

A. administer usual oral medication


B. administer parenteral hydrocortisone hemisuccinate
C. administer parenteral mineralocorticoid
D. double the usual oral dose of mineralocorticoid
E. triple the usual oral dose of hydrocortisone

Copyright 2009 by the American Academy of Pediatrics

page 91

2009 PREP SA on CD-ROM


Critique: 42

Preferred Response: B

The girl described in the vignette has adrenal insufficiency and is vomiting and febrile. She needs
stress doses of glucocorticoids but cannot keep down orally administered agents. No parenteral
preparation of mineralocorticoid is available. In contrast to synthetic glucocorticoids,
hydrocortisone has about 1% of the mineralocorticoid effect of aldosterone (the most important
natural mineralocorticoid), and a large dose of hydrocortisone can act as a mineralocorticoid.
Accordingly, this child needs a rapid-acting parenteral hydrocortisone preparation.
Hydrocortisone hemisuccinate administered subcutaneously, unless she has poor perfusion,
which would necessitate intramuscular or intravenous administration, can stabilize her course.
The girl's usual oral medication is not sufficient for the increased need of stress. Increasing
the oral medication may not be sufficient because she probably will not be able to keep it down
or absorb it if she is vomiting. Mineralocorticoid need generally does not increase during stress.
If she is vomiting and receiving parenteral hydrocortisone, she might require intravenous 0.9%
saline with glucose to maintain her sodium stores and her blood glucose in the normal range.
For children who have usual febrile illnesses or require surgery, administration of
glucocorticoid in doses that supply three to six times the usual cortisol secretion rate of 3 to 7
mg/M2 per day is reasonable treatment. However, children who have adrenal insufficiency and
develop varicella because of failure or lack of immunization must be managed very carefully.
There are no controlled studies, but some anecdotal data suggest that infection with varicella
virus can become devastating in glucocorticoid-treated individuals. For such children, careful
titration of hydrocortisone dose to only twice the usual need may provide the small amount of
needed additional glucocorticoid without compromising the children's immune responses.
References:
Donohoue PA. Treatment of adrenal insufficiency in children. UptoDate Online 15.3. 2008.
Available for subscription at:
http://www.uptodateonline.com/utd/content/topic.do?topicKey=pediendo/19876
Shulman DI, Palmert MR, Kemp SF, for the Lawson Wilkins Drug and Therapeutics Committee.
Adrenal insufficiency: still a cause of morbidity and death in childhood. Pediatrics. 2007;119:e484e494. Available at: http://pediatrics.aappublications.org/cgi/content/full/119/2/e484
Wilson TA, Speiser P. Adrenal insufficiency. eMedicine Specialties, Pediatrics: General Medicine,
Endocrinology. 2007. Available at:
http://www.emedicine.com/ped/TOPIC47.HTM

Copyright 2009 by the American Academy of Pediatrics

page 92

2009 PREP SA on CD-ROM


Question: 43

A 9-year-old girl has been evaluated by a learning consultant and found to have a slow reading
rate, weakness in short-term memory, and problems with reading comprehension. Her parents
ask you what subjects other than reading will be most challenging for her due to these learning
difficulties.
Of the following, the subject that this child should find MOST challenging is

A. art
B. creative writing
C. mathematics
D. music
E. social studies

Copyright 2009 by the American Academy of Pediatrics

page 93

2009 PREP SA on CD-ROM


Critique: 43

Preferred Response: E

A student who has a slow reading rate, reduced reading comprehension, and impaired shortterm memory, such as the girl described in the vignette, will encounter problems in "content"
classes, which include subjects such as science, history, and social studies. When children
read texts in these subjects, they need to read factual information and use the material in the
text to obtain knowledge about the subject. This requires comprehension of the text, which
involves identifying and understanding the words.
To determine the meaning of a word, a reader first must decode and identify the word on the
page. A slow reader takes much longer to complete assignments and test questions. Further,
individuals who have weakness in understanding and remembering the text will have much more
difficulty with homework assignments and on examinations. The girl in the vignette will not have
the same difficulty with art, music, creative writing, or mathematics because these subjects do
not demand the same emphasis on reading a text and recalling facts.
References:
Fletcher JM, Lyon GR, Fuchs LS, Barnes MA. Reading disabilities: comprehension. In: Learning
Disabilities: From Identification to Intervention. New York, NY: The Guilford Press; 2007:184-206
Shaywitz SE, Shaywitz BA. Dyslexia (specific reading disability). Pediatr Rev. 2003;24:147-153.
Available at: http://pedsinreview.aappublications.org/cgi/content/full/24/5/147

Copyright 2009 by the American Academy of Pediatrics

page 94

2009 PREP SA on CD-ROM


Question: 44

As you are leaving the supermarket, the cashier tells you that she is worried because her child
recently had a positive tuberculin skin test. She had to take him to the health department for skin
testing because he had been in contact with her father, who recently was diagnosed with active
pulmonary tuberculosis. They told her that the boys skin test was positive at "25," but his chest
radiograph was normal. She is concerned because the doctor told her that the case is a little
unusual because of the type of tuberculosis her father has. She asked the physician at the
health department to write it down, and she hands you a piece of paper that says "INH
resistant." The mother asks you what type of medication her boy should receive.
Of the following, the MOST appropriate antituberculous agent to prescribe for this boy is

A. ciprofloxacin
B. ethambutol
C. isoniazid
D. pyrazinamide
E. rifampin

Copyright 2009 by the American Academy of Pediatrics

page 95

2009 PREP SA on CD-ROM


Critique: 44

Preferred Response: E

With a positive tuberculin skin test, a negative chest radiograph, and no indications of active
disease, the patient described in the vignette meets the classification of a latent tuberculosis
infection (LTBI). LTBI usually is treated with isoniazid (INH) once daily for 9 months, but when
the source case is known to have INH resistance, this agent should not be used. Instead, a 6month course of rifampin is recommended. An exception to this approach would be if the source
case was known to be resistant to both INH and rifampin, in which case a tuberculosis expert
should be consulted to determine the best course of treatment. Ciprofloxacin, ethambutol, and
pyrazinamide are used in combination with other antituberculous agents for the treatment of
active tuberculous disease; they are not indicated for monotherapy in treating LTBI.
Although usually very well tolerated, patients who receive rifampin should be aware of
possible adverse effects. Urine, tears, and saliva change to a reddish-orange color, which may
stain clothes or contact lenses. Rifampin therapy also may be associated with mild "flulike"
symptoms (eg, myalgias) that resolve with continued therapy. Rifampin also induces
cytochrome P-450 activity and, therefore, decreases the half-life of medications such as
warfarin, digoxin, thyroxine, oral contraceptives, and some antimicrobial agents (eg,
chloramphenicol), making them less effective. When rifampin is used in combination with INH,
patients are twice as likely to develop hepatitis as are patients treated with rifampin in
combination with other antituberculous medications. Thrombocytopenia and leukopenia both
have been associated with rifampin therapy exceeding 1 month's duration.
References:
Alsayyed B, Adam HM. In brief: rifampin. Pediatr Rev. 2004;25:216-217. Available at:
http://pedsinreview.aappublications.org/cgi/content/full/25/6/216
American Academy of Pediatrics. Tuberculosis. In: Pickering LK, Baker CJ, Long SS, McMillan
JA, eds. Red Book: 2006 Report of the Committee on Infectious Diseases. 27th ed. Elk Grove
Village, Ill: American Academy of Pediatrics; 2006:678-698
Bliziotis IA, Ntziora F, Lawrence KR, Falagas ME. Rifampin as adjuvant treatment of Grampositive bacterial infections: a systemic review of comparative clinical trials. Eur J Clin Microbiol
Infect Dis. 2007;26:849-856. Abstract available at:
http://www.ncbi.nlm.nih.gov/pubmed/17712583

Copyright 2009 by the American Academy of Pediatrics

page 96

2009 PREP SA on CD-ROM


Question: 45

You are speaking to a group of medical students about different antibiotic classes that can be
used in the treatment of meningitis. One student asks you about chloramphenicol, a drug with
which he is not familiar.
Of the following, the MOST common adverse effect associated with chloramphenicol therapy is

A. dose-related bone marrow suppression


B. drug eruption
C. gray baby syndrome
D. idiosyncratic aplastic anemia
E. optic neuritis

Copyright 2009 by the American Academy of Pediatrics

page 97

2009 PREP SA on CD-ROM


Critique: 45

Preferred Response: A

Chloramphenicol is an antibiotic that inhibits protein synthesis by reversibly binding to the 50S
subunit of the 70S ribosome. Against most sensitive organisms, chloramphenicol produces a
static effect by blocking protein synthesis. However, it is bactericidal against some meningeal
pathogens, such as Haemophilus influenzae, Streptococcus pneumoniae, and Neisseria
meningitidis. Chloramphenicol is not bactericidal against group B streptococci or enteric gramnegative bacilli because drug concentrations in the cerebrospinal fluid sufficient for effective
eradication of these organisms cannot be achieved at therapeutic doses. With the possible
exception of typhoid fever in some developing areas of the world where cost and availability
make it the primary therapy, chloramphenicol no longer is the drug of choice for any specific
infection.
Chloramphenicol is extremely active against a variety of organisms, including bacteria,
spirochetes, rickettsiae, chlamydiae, and mycoplasmas. Most gram-positive and gram-negative
aerobic bacteria are inhibited by concentrations achievable in the serum, and chloramphenicol is
one of the most effective antibiotics against anaerobic bacteria, including Bacteroides strains.
Salmonellae, including S typhi, generally are susceptible, although chloramphenicol-resistant
strains have emerged rapidly since the late 1980s in India, Korea, Vietnam, Peru, Mexico, and
Thailand.
Plasma concentrations of chloramphenicol vary, depending on the time and route of
administration. Oral chloramphenicol is well absorbed from the gastrointestinal tract and
produces peak serum concentrations of 12 mcg/mL of active antibiotic after a 1-g dose. The
intravenous preparation of the drug produces active chloramphenicol concentrations in the
serum that are 70% of those obtained after an oral dose. Intramuscular administration is well
tolerated and produces peak serum values that are similar to those seen with intravenous
administration. However, for patients who have enteric fever, peak serum concentrations of the
drug after intramuscular administration are only one half to two thirds of those obtained by
intravenous administration due to delayed drug absorption from the injection site.
Metabolism and excretion of chloramphenicol vary widely in the pediatric population and are
age- and patient-dependent. Therefore, monitoring of serum concentrations is critical. Significant
drug-drug interactions occur between chloramphenicol and other agents. Chloramphenicol
prolongs the half-life of tolbutamide, chlorpropamide, phenytoin, cyclophosphamide, and warfarin
by inhibiting hepatic microsomal enzymes. Drugs such as phenytoin, rifampin, and phenobarbital
decrease the serum concentration and increase the total body clearance of chloramphenicol by
inducing hepatic microsomal enzymes. Chloramphenicol may delay the response of anemias to
iron, folic acid, and vitamin B12, and in vitro it antagonizes the bactericidal activity of the
penicillins, cephalosporins, and aminoglycoside antibiotics.
The most common adverse effect associated with chloramphenicol is reversible bone
marrow depression that is characterized by a combination of reticulocytopenia, anemia,
leucopenia, and thrombocytopenia. Other adverse effects include idiosyncratic aplastic anemia
that is rare but may be fatal; hemolytic anemia in patients who have the Mediterranean form of
glucose-6-phosphate dehydrogenase deficiency; gray baby syndrome in neonates, which is
characterized by abdominal distention, vomiting, flaccidity, cyanosis, circulatory collapse, and
death; and optic neuritis that may occur in patients receiving prolonged therapy. Hypersensitivity
reactions (eg, rashes, drug fevers, anaphylaxis) are rare.
References:
Kauffman RE, Miceti JN, Strebel L, Buckley JA, Done AK, Dajani AS. Pharmacokinetics of
chloramphenicol and chloramphenicol succinate in infants and children. J Pediatr. 1981;98:315320. Abstract available at: http://www.ncbi.nlm.nih.gov/pubmed/7463235
Myers B, Salvatore M. Tetracyclines and chloramphenicol. In: Mandell GL, Bennett JE, Dolin R,
eds. Mandell, Douglas and Bennett's Principles and Practice of Infectious Diseases. 6th ed. New
York, NY: Elsevier Churchill Livingstone, 2005:356-373

Copyright 2009 by the American Academy of Pediatrics

page 98

2009 PREP SA on CD-ROM

Pickering LK, Hoecker JL, Kramer WG, Kohl S, Cleary TG. Clinical pharmacology of two
chloramphenicol preparations in children: sodium succinate (IV) and palmitate (oral) esters. J
Pediatr. 1980;96:757-761. Abstract available at: http://www.ncbi.nlm.nih.gov/pubmed/6987361
Rahal JJ Jr, Simberkoff MS. Bactericidal and bacteriostatic action of chloramphenicol against
meningeal pathogens. Antimicrob Agents Chemother. 1979;16:13-18. Available at:
http://aac.asm.org/cgi/reprint/16/1/13?view=long&pmid=38742
Yunis AA. Chloramphenicol-induced bone marrow suppression. Semin Hematol. 1973;10:225234

Copyright 2009 by the American Academy of Pediatrics

page 99

2009 PREP SA on CD-ROM


Question: 46

A 2-year-old boy presents with fever and abdominal pain. Urinalysis reveals a specific gravity of
1.010, pH of 5.5, 2+ protein, no blood, and negative leukocyte esterase and nitrite tests.
Microscopy findings are negative.
Of the following, the MOST appropriate diagnostic test to assess the severity of proteinuria in
this child is

A. a random urine sample measurement for quantitative protein and creatinine concentrations
B. measurement of serum albumin concentration and correlation with urine protein concentration
measured by dipstick

C. measurement of urine microalbumin excretion


D. sulfosalicylic acid treatment of a freshly voided urine sample
E. 24-hour urine collection for protein and creatinine concentrations

Copyright 2009 by the American Academy of Pediatrics

page 100

2009 PREP SA on CD-ROM


Critique: 46

Preferred Response: A

The 2+ protein measurement on a dilute urine sample (urine specific gravity <1.015) described
for the boy in the vignette is abnormal and requires quantitation of the proteinuria. Although
traditional teaching favored 24-hour urine testing for quantitative assessment of proteinuria in the
past, this technique no longer is necessary. Spot/random urine samples for proteinuria correlate
closely with 24-hour urine collections and are much easier to collect via a urine bag, especially in
the child who is not yet toilet trained. A normal spot urine protein-to-creatinine ratio is less than
0.2.
The urine dipstick is a good screening test for proteinuria, but because it is a qualitative test,
it is a suboptimal indicator of the severity of proteinuria. Similarly, sulfosalicylic acid treatment of
freshly voided urine is a qualitative, not quantitative test. The sulfosalicylic acid induces
precipitation of proteins. Urine microalbumin is an ultrasensitive technique (using a
radioimmunoassay, enzyme-linked immunoassay, or nephelometry) to detect albumin excretion
in the urine that falls below that measured on a urine dipstick. Assessment of microalbuminuria is
used commonly in patients who have diabetes as a screening test for renal disease before overt
proteinuria occurs. Microalbuminuria screens also have been used in those who have
hypertension. Although this technique is quantitative, it has no role when overt proteinuria is
present, as measured on a urine dipstick. Measurement of serum albumin should be deferred
until the severity of the proteinuria can be quantified.
Although 24-hour urine testing for proteinuria no longer is necessary, the test remains useful
for evaluating children for risk factors for nephrolithiasis/urolithiasis or to screen for
catecholamine metabolites in children who have suspected neural crest tumors (neuroblastoma
or pheochromocytoma).
References:
Abitbol C, Zilleruelo G, Freundlich M, Strauss J. Quantitation of proteinuria with urinary
protein/creatinine ratios and random testing with dipsticks in nephrotic children. J Pediatr.
1990;116:243-247. Abstract available at: http://www.ncbi.nlm.nih.gov/pubmed/2299494
Chahar OP, Bundella B, Chahar CK, Purohit M. Quantitation of proteinuria by use of single
random spot urine collection. J Indian Med Assoc. 1993;91:86-87. Abstract available at:
http://www.ncbi.nlm.nih.gov/pubmed/8409488
Gregianin LJ, McGill AC, Pinheiro CM, Brunetto AL. Vanilmandelic acid and homovanillic acid
levels in patients with neural crest tumor: 24-hour urine collection versus random sample.
Pediatr Hematol Oncol. 1997;14:259-265. Abstract available at:
http://www.ncbi.nlm.nih.gov/pubmed/9185210
Pontremoli R, Leoncini G, Ravera M, et al. Microalbuminuria, cardiovascular, and renal risk in
primary hypertension. J Am Soc Nephrol. 2002;13:S169-S172. Available at:
http://jasn.asnjournals.org/cgi/content/full/13/suppl_3/S169

Copyright 2009 by the American Academy of Pediatrics

page 101

2009 PREP SA on CD-ROM


Question: 47

A 12-month-old girl presents with a 3-month history of a pruritic rash that involves her cheeks,
neck, anterior trunk, and antecubital and popliteal areas. The rash improves after use of an overthe-counter topical steroid cream but still is present most days, and the infant often wakes up at
night scratching. On physical examination, you observe a raised erythematous rash that has
areas of lichenification (Item Q47).
Of the following, the MOST helpful intervention is to

A. eliminate fruit and acidic juices from the diet


B. eliminate milk, eggs, soy, and wheat from the diet
C. perform aeroallergen allergy testing
D. perform food allergy testing
E. recommend a skin biopsy

Copyright 2009 by the American Academy of Pediatrics

page 102

2009 PREP SA on CD-ROM


Critique: 47

Preferred Response: D

Some 30% to 40% of infants who have moderate-to-severe atopic dermatitis (AD), such as
described for the infant in the vignette, may have an underlying immunoglobulin (Ig) E-mediated
food allergy exacerbating the AD. For some infants, food ingestion may result in immediate
worsening of AD severity, although most infants do not demonstrate this immediate reaction.
Many foods have been implicated in AD, but five (milk, eggs, soy, wheat, and peanut) account
for 90% of the causative allergens.
Both allergy skin testing and measurement of serum IgE concentrations to these foods can
help to identify and eliminate likely triggers. Either a negative IgE blood test (<0.35 kU/L) or a
negative skin test for a specific food provides a high negative predictive value. On the other
hand, the positive predictive value for a skin or blood test may be only 50%.
Although the most commonly implicated foods often are eliminated from the diet, such an
approach does not improve symptoms in most (60% to 70%) children because they do not have
IgE-mediated AD. The unnecessary elimination of multiple foods can have an adverse effect on
nutrition, and food avoidance should be guided by the dietary history, eczema severity, and skin
or blood testing.
Frequently, children experience perioral rashes after drinking fruit juice. Such rashes
typically are nonpruritic, limited to the area of contact, and resolve within a few hours. The
mechanism of such rashes is unknown, but children generally outgrow such reactions by age 4
years. In cases involving more widespread cutaneous symptoms, such as described in the
vignette, elimination of fruit or acidic juices is unnecessary.
Parents often request testing for environmental allergies. House dust mites have been
implicated in some cases of AD, although they are less likely a cause for moderate-to-severe
atopic dermatitis than food allergies. Climate changes such as cold, dry air or hot, humid
weather can worsen AD, but specific seasonal allergens such as oak tree or ragweed are not
associated with eczema in infants.
A skin biopsy can provide insight into the pathophysiology of chronic rashes or lesions.
Generally, skin biopsies neither are advised nor provide insight into the causes of typical AD
manifestations in infants, but atypical presentations or lack of expected improvement with
appropriate therapy should prompt consideration of a dermatology referral.
References:
Burks W. Skin manifestations of food allergy. Pediatrics. 2003;111:1617-1624. Available at:
http://pediatrics.aappublications.org/cgi/content/full/111/6/S2/1617
Greer FR, Sicherer SH, Burks AW, Committee on Nutrition and Section on Allergy and
Immunology. Effects of early nutritional interventions on the development of atopic disease in
infants and children: the role of maternal dietary restriction, breastfeeding, timing of introduction
of complementary foods and hydrolyzed formulas. Pediatrics. 2008;121:183-191. Available at:
http://pediatrics.aappublications.org/cgi/content/full/121/1/183
Sampson HA, Leung DYM. Adverse reactions to foods. In: Kleigman RM, Behrman RE, Jenson
HB, Stanton BF, eds. Nelson Textbook of Pediatrics. 18th ed. Philadelphia, Pa: Saunders
Elsevier; 2007:986-989

Copyright 2009 by the American Academy of Pediatrics

page 103

2009 PREP SA on CD-ROM


Question: 48

The mother of a 2-year-old boy calls you because she found her son holding an open bottle of
liquid dishwasher detergent. He is crying, drooling profusely, and has vomited three times. In
answer to your questions, she reports that he is not sleepy and did not seem to get it in his eyes
or on his skin. On examination in your office, you note an ulcer on his lower lip and several
ulcers on his tongue.
Of the following, the most appropriate next step in the evaluation and management of this boy
is to

A. administer activated charcoal


B. administer ipecac syrup
C. reassure the mother that the product is nontoxic
D. refer the boy to a gastroenterologist for urgent endoscopy
E. refer the boy to the emergency department for gastric lavage

Copyright 2009 by the American Academy of Pediatrics

page 104

2009 PREP SA on CD-ROM


Critique: 48

Preferred Response: D

Dishwasher detergent is a highly caustic alkali that can cause significant oropharyngeal, upper
airway, and esophageal injury if ingested. Alkaline agents cause deep liquefaction necrosis of
the affected tissues, with ulceration and perforation likely complications. Common household
products such as oven and drain cleaners, dishwasher detergents, and cleaning products that
contain ammonia pose a risk of caustic injury. Substances that have a pH greater than 11 are
likely to cause injury, even in small amounts.
Children who have ingested caustic or corrosive agents typically present with drooling,
dysphagia, or emesis, as reported for the boy in the vignette, although the presence and
severity of symptoms is not predictive of esophageal injury. The presence or absence of oral
lesions also is not predictive of esophageal injury.
The initial evaluation and treatment of patients who have ingested caustic substances
includes stabilization of vital functions, pain management, and other supportive measures.
Decontamination with activated charcoal is not indicated because it will not adsorb acids or
alkalis and is likely to impair endoscopic evaluation. Use of ipecac syrup or gastric lavage is
contraindicated because of the potential to cause further injury to the esophagus. In the
symptomatic patient, endoscopy should be undertaken within 6 to 24 hours of ingestion to
determine the presence of esophageal burns and the degree of severity.
Evaluation of the asymptomatic patient is controversial. Because the available data suggest
that esophageal burns may be found in as many as 45% of those who do not have oral lesions,
endoscopy is recommended in clear cases of ingestion. If the history is questionable and the
patient remains asymptomatic, with normal swallowing, after several hours of observation, some
authors have suggested that it is safe to forego endoscopy.
References:
Cordero B, Savage RR, Cheng TL. In brief: corrosive ingestions. Pediatr Rev. 2006;27:154-155.
Available at: http://pedsinreview.aappublications.org/cgi/content/full/27/4/154
Ferry GD. Caustic esophageal injury in children. UpToDate. 2008. Available for subscription at:
http://www.utdol.com/utd/content/topic.do?topicKey=pedigast/11441&view=print
Gaudreault P, Parent M, McGuigan MA, Chicoine L, Lovejoy FH Jr. Predictability of esophageal
injury from signs and symptoms: a study of caustic ingestion in 378 children. Pediatrics.
1983;71:767-770. Available at: http://pediatrics.aappublications.org/cgi/content/full/71/5/767

Copyright 2009 by the American Academy of Pediatrics

page 105

2009 PREP SA on CD-ROM


Question: 49

A 12-year-old boy has had cholestasis since infancy from Alagille syndrome. He has been lost
to medical follow-up for the last several years. He now presents to your office with pain in his
right upper thigh after a fall. His thigh is intensely tender, and ultrasonography demonstrates a
large hematoma in his quadriceps. The parents state that he has tended to bruise easily in the
past few months.
Of the following, the condition MOST likely to account for this patients symptoms is

A. factor VIII deficiency


B. idiopathic thrombocytopenic purpura
C. vitamin C deficiency
D. vitamin K deficiency
E. von Willebrand disease

Copyright 2009 by the American Academy of Pediatrics

page 106

2009 PREP SA on CD-ROM


Critique: 49

Preferred Response: D

Alagille syndrome is characterized by cardiac disease (especially peripheral pulmonary


stenosis), vertebral anomalies, ocular anomalies (posterior embryotoxon), facial dysmorphism
(triangular facies, macrocephaly, large ears) (Item C49A), and paucity of the intrahepatic bile
ducts. The hepatic manifestations of this syndrome account for much of the medical morbidity.
Specifically, impaired bile flow results in chronic cholestasis, which leads to severe pruritus,
jaundice, malabsorption of nutrients, and malabsorption of fat-soluble vitamins (Item C49B).
Although most affected children have their jaundice improve as they grow older, a subset
progresses to cirrhosis and requires liver transplantation.
Patients who have hepatic disease must have their nutritional status monitored carefully.
Chronic anorexia, recurrent illnesses, and fat malabsorption may result in caloric deficiency and
growth failure. Caloric supplementation by nasogastric tube or gastrostomy may be necessary
to ensure adequate caloric intake. In addition, patients who have cholestasis are at risk for fatsoluble vitamin deficiency. Vitamin D deficiency typically causes osteopenia and rickets, vitamin
E deficiency causes peripheral neuropathy and ataxia, and vitamin A deficiency may cause night
blindness or corneal lesions. The bruising described for the patient in the vignette most likely is
due to vitamin K deficiency. Vitamin K is a cofactor essential in posttranscriptional carboxylation
of the clotting factors II, VII, IX, and X. Thus, vitamin K deficiency leads to prolonged prothrombin
and partial thromboplastin time, which predisposes to bruising. Although factor VIII deficiency,
vitamin C deficiency, von Willebrand disease, and idiopathic thrombocytopenia purpura also may
cause bruising, the patient who has Alagille syndrome is not at increased risk for developing
these conditions.
In addition to supplementing patients who have chronic liver disease with fat-soluble
vitamins, the clinician caring for these patients also must supply adequate calories. Patients who
have advanced chronic liver disease may have both anorexia and increased caloric
requirements. In addition, patients who have portal hypertension and ascites may need to have
total fluid intake restricted, which, in turn, means that they may require a more concentrated and
less palatable formula. For these reasons, nasogastric or gastrostomy feedings sometimes are
necessary to achieve optimal growth, especially when preparing a patient for liver
transplantation.
References:
Cranenburg ECM, Shurgers LJ, Vermeer C. Vitamin K: the coagulation vitamin that became
omnipotent. Thromb Haemost. 2007;98:120-125. Available at:
http://www.schattauer.de/index.php?id=1268&pii=th07070120&no_cache=1
Kamath BM, Piccoli DA. Heritable disorders of the bile ducts. Gastroenterol Clin North Am.
2003;32:857-875. Abstract available at: http://www.ncbi.nlm.nih.gov/pubmed/14562578
Sokol RJ. Fat-soluble vitamins and their importance in patients with cholestatic liver diseases.
Gastroenterol Clin North Am. 1994;23:673-705. Abstract available at:
http://www.ncbi.nlm.nih.gov/pubmed/7698827

Copyright 2009 by the American Academy of Pediatrics

page 107

2009 PREP SA on CD-ROM


Question: 50

A term infant is delivered by emergency cesarean section following the acute onset of maternal
vaginal bleeding and profound fetal bradycardia on electronic fetal heart rate monitoring. The
Apgar scores are 1, 2, and 3 at 1, 5, and 10 minutes, respectively. Resuscitation includes
intubation and assisted ventilation, chest compressions, and intravenous epinephrine. The infant
is admitted to the neonatal intensive care unit and has seizures at 6 hours of age.
Of the following, a TRUE statement about infants who have seizures following perinatal asphyxia
is that most

A. develop epilepsy
B. develop microcephaly
C. do not have severe long-term neurodevelopmental delay
D. experience hearing loss
E. require multiple anticonvulsant medications

Copyright 2009 by the American Academy of Pediatrics

page 108

2009 PREP SA on CD-ROM


Critique: 50

Preferred Response: C

The infant described in the vignette has concerning perinatal events, low Apgar scores through
10 minutes of postnatal age, the need for vigorous resuscitation, and subsequent seizure
activity 6 hours after birth. Seizures occurring in the first 24 hours after birth in newborns who
have perinatal asphyxia are considered evidence of hypoxic-ischemic encephalopathy.
Accompanied by profound acidemia, these seizures also may reflect significant cerebral cellular
injury or cell death.
The causes of neonatal seizure broadly include hypoxia-ischemia, which may be global (as
in perinatal asphyxia) or focal (as in arterial or venous thrombosis); hemorrhagic injury; brain
malformation; transient metabolic disorders (such as hypoglycemia or hypocalcemia); infectious
(viral, bacterial, or fungal); inborn errors of metabolism; or unknown (Item C50A). As
demonstrated in Item C50B, most term infants who have neonatal seizures do not manifest longterm neurodevelopmental sequelae. Further, even though motor abnormalities may be found on
examination in a slight majority (53%) of newborns having a seizure for any cause, few have
cerebral palsy.
However, when seizures occur in newborns due to asphyxia (generalized hypoxic-ischemic
injury), the prevalence of abnormal neurodevelopmental outcomes is more concerning. Although
severe impairment in neurodevelopmental outcomes may occur in fewer than 50% of these
infants, the presence of mild-to-moderate neurodevelopmental impairment in cognitive and motor
function as well as some (~33%) children having long-term epilepsy brings the level of abnormal
outcomes to approximately 50% (Item C50B). Early predictions of outcome for such children
may be determined by evaluating the worst early electroencephalography (EEG) finding, the
follow-up 1-week EEG, and the findings evident on cranial magnetic resonance imaging.
Following perinatal asphyxia and resultant seizures, most newborns can be treated with a
single anticonvulsant drug. Longstanding epilepsy and microcephaly occur in fewer than 50% of
patients. Hearing loss, though more common in asphyxiated infants who have neurologic injury
than in uncomplicated term infants, still occurs in fewer than 10% of infants.
References:
Hill A. Neonatal seizures. Pediatr Rev. 2000;21:117-121. Available at:
http://pedsinreview.aappublications.org/cgi/content/full/21/4/117
Ronen GM, Buckley D, Penney S, Streiner DL. Long-term prognosis in children with neonatal
seizures: a population-based study. Neurology. 2007;69:1816-1822. Abstract available at:
http://www.ncbi.nlm.nih.gov/pubmed/17984448
Tekgul H, Gauvreau K, Soul J, et al. The current etiologic profile and neurodevelopmental
outcome of seizures in term newborn infants. Pediatrics. 2006;117:1270-1280. Available at:
http://pediatrics.aappublications.org/cgi/content/full/117/4/1270

Copyright 2009 by the American Academy of Pediatrics

page 109

2009 PREP SA on CD-ROM


Question: 51

A 16-year-old girl who is new to your practice complains of a nearly constant headache for the
past year. She describes the pain as a band around her head that often is throbbing and is
worse during the middle of the day. She denies nausea or vomiting but reports occasional
fatigue. There is no family history of headaches. She has missed more than 20 days of school
this year because of the headache, and she is struggling to maintain a C average. She admits to
hating school and does not participate in extracurricular activities because she "doesnt like
anything." Findings on her physical examination, including complete neurologic and funduscopic
evaluation, are normal.
Of the following, the BEST next step in the management of this girls headaches is to

A. advise her to keep a headache diary and return in 2 months


B. obtain a lumbar puncture
C. obtain computed tomography scan of the brain
D. prescribe oral sumatriptan
E. refer her for psychosocial evaluation and counseling

Copyright 2009 by the American Academy of Pediatrics

page 110

2009 PREP SA on CD-ROM


Critique: 51

Preferred Response: E

Chronic headache is a common complaint in children and adolescents. A careful history and
complete neurologic examination are indicated in the evaluation of headache. The most
important initial consideration is to identify characteristics of the headache that suggest serious
diagnoses, such as brain tumors or other diseases causing increased intracranial pressure.
Such characteristics include worsening pain at night or immediately upon awakening,
association with vomiting, and worsening pain with coughing or straining. Papilledema or focal
neurologic findings may be found in patients who have increased intracranial pressure or the
examination results may be normal.
Migraine headaches are periodic, may be accompanied by an aura, and typically are
relieved by sleep. A family history of migraines usually can be elicited. The neurologic
examination typically yields normal results, although complicated migraines can be accompanied
by focal neurologic deficits such as hemiparesis, cranial nerve palsies, and visual disturbances.
Pain from stress-related or tension headache generally is diffuse and may be described as
"bandlike" or throbbing, as reported for the girl in the vignette. Pain usually occurs on most days,
and school absence is frequent. The neurologic examination yields normal results.
It is also very important for the clinician to obtain further information regarding any emotional,
social, or academic difficulties the patient may be experiencing. Such data can help to determine
both an underlying cause for the headache and the effect the headache is having on the child's
quality of life. Family stressors and depression are known causes of headache, and questions
regarding sleep patterns, anhedonia, school performance, and relationships with family and
friends can help to screen for these conditions. Studies have shown that children and
adolescents who have frequent or severe headaches have greater impairment in academic and
social functioning than those who do not have headaches. A thorough psychosocial evaluation
can aid in evaluating patients for depression and assessing the effect of headaches on their
daily functioning. Nonpharmacologic therapies, such as rest, relaxation techniques, and removal
of stress from the environment, can be effective once the stress is identified.
Advising the girl in the vignette to keep a diary for 2 months likely would prolong her
difficulties and not address the underlying cause of her headaches. Lumbar puncture and
computed tomography scan of the brain may play some role in the evaluation of an acute
headache, especially if meningitis or a mass lesion of the brain is suspected, but these
diagnoses are very unlikely in a patient who has had a headache for a year and normal results
on neurologic examination. Oral sumatriptan is used in the treatment of migraine, but this girl's
history points to depression or another psychological cause for her headache, and prescribing
medications without a psychosocial evaluation probably would not be effective.
References:
Forsyth R, Farrell K. Headache in childhood. Pediatr Rev. 1999;20:39-45. Available at:
http://pedsinreview.aappublications.org/cgi/content/full/20/2/39
Silver N. Headache (chronic, tension type). BMJ Clinical Evidence. 2007. Available for
subscription at: http://clinicalevidence.bmj.com/ceweb/conditions/nud/1205/1205_guidelines.jsp
Strine TW, Okoro CA, McGuire LC, Balluz LS. The associations among childhood headaches,
emotional and behavioral difficulties, and health care use. Pediatrics. 2006;117: 1728-1735.
Available at: http://pediatrics.aappublications.org/cgi/content/full/117/5/1728

Copyright 2009 by the American Academy of Pediatrics

page 111

2009 PREP SA on CD-ROM


Question: 52

When a 14-year-old girl had frequent complaints of shoulder pain made worse by pitching
softball a few months ago, you diagnosed overuse injury. Nonsteroidal anti-inflammatory drugs
and rest have provided some relief. She presents today with complaints of recurrent upper arm
pain that is unrelated to exercise and sometimes awakens her from sleep. Physical examination
reveals a slightly larger circumference of the left proximal humerus compared with the right.
There is minimal tenderness on palpation over the area, although the girl reports a constant
ache. She has full range of motion of the arm at the shoulder and elbow. You obtain a shoulder
radiograph (Item Q52).
Of the following, the MOST likely diagnosis is

A. acromioclavicular separation
B. acute osteomyelitis
C. chronic osteomyelitis
D. osteosarcoma
E. supracondylar fracture of the humerus

Copyright 2009 by the American Academy of Pediatrics

page 112

2009 PREP SA on CD-ROM


Critique: 52

Preferred Response: D

The girl described in the vignette has radiographic findings and clinical history that are most
suggestive of osteogenic sarcoma of the humerus. The most common presenting symptom of
osteosarcoma is pain, particularly with activity, as described for the girl. The affected patient
may have a history of swelling, depending on the size of the lesion and its location. Patients or
their parents may complain of a "sprain," "arthritis," or so-called "growing pains." Symptoms may
be present for weeks, months, or occasionally longer before osteosarcoma is diagnosed. The
child often has a history of trauma, as in this patient, because trauma occurs frequently in
teenagers. Pathologic fractures are not particularly common with this tumor compared with
leukemia and other malignancies. Systemic symptoms, such as fever and night sweats, are
rare.
Osteosarcoma is the third most common cancer in adolescence, after lymphomas and brain
tumors. Peak age at diagnosis is 10 to 25 years. The incidence is 400 cases per year in the
United States (4.8 cases per 1 million persons younger than 20 years) and is slightly higher in
African Americans than in whites. An increased incidence during adolescence corresponds with
the growth spurt.
Osteosarcoma occurs in the long bones near metaphyseal growth plates (Item C52A). Most
are high-grade intramedullary osteosarcomas, with only about 5% being low-grade lesions. The
most common sites are the femur (42%, with 75% of femoral tumors occurring distally), tibia
(19%, with 80% in the proximal tibia), and humerus (10%, with 90% in the proximal humerus).
The overall 5-year survival rate for patients whose condition was diagnosed between 1974 and
1994 was 63% (59% for males, 70% for females). The current 5-year survival rate is estimated
to be 65%. The mainstay of therapy is excision of the lesion. Chemotherapy is required to treat
micrometastatic disease, which is present but not detectable in most patients at diagnosis.
Metastatic spread to the lungs only rarely results in respiratory symptoms; such symptoms
usually indicate extensive lung involvement. Metastasis to other sites is extremely rare.
The exact cause of osteosarcoma is unknown, but it is believed to be a tumor of
osteoprogenitor cells, which are multipotential, hormone-responsive stromal cells in the
periosteum and marrow that are capable of differentiating into many lineages, depending on their
environment. Among the known risk factors are rapid bone growth, exposure to radiation, and
potentially a genetic predisposition. Children who have a prior personal or family history of
retinoblastoma and those who have received radiation therapy for a previous malignancy are at
higher risk of developing osteosarcoma.
Initial evaluation of an adolescent or older child presenting with bone pain and swelling,
especially with a palpable mass, should include:
o Plain radiographs (two views) of the suspected lesions, although no single feature on
radiographs is diagnostic. Osteosarcomatous lesions can be purely osteolytic (about 30% of
patients), purely osteoblastic (about 45% of patients), or a mixture of both. Elevation of the
periosteum may appear as the characteristic Codman triangle. Extension of tumor through the
periosteum may result in a so-called "sunburst appearance" (about 60% of patients).
o Both magnetic resonance imaging of the primary lesion and computed tomography scan of
the chest are necessary to confirm the diagnosis and for staging purposes. Such scans
frequently are performed at the tertiary center using their protocols.
The differential diagnosis of bone pain and swelling includes stress fracture, hematoma,
bone cysts, and other bony tumors such as Ewing sarcoma.
Although acute osteomyelitis is common in children, it is relatively less common in
teenagers; approximately 50% of cases occur in preschool-age children. Infection frequently is
characterized by overlying erythema, warmth, and more acute systemic signs such as fever
and malaise. The white blood cell count is elevated in 50% of patients, and the erythrocyte
sedimentation rate or C-reactive protein value is increased. Plain radiographs may be read as
normal early in the disease course.
Chronic osteomyelitis also may cause pain. However, results of blood tests often are
normal. The radiograph for the patient in the vignette, however, shows a classic sunburst
pattern, which is not consistent with chronic osteomyelitis.
Acromioclavicular (AC) separation typically results in sudden pain and limited range of

Copyright 2009 by the American Academy of Pediatrics

page 113

2009 PREP SA on CD-ROM

motion. The diagnosis can be made by an anteroposterior radiograph, which can demonstrate
excessive separation of the AC joint (Item C52B). Supracondylar fracture of the humerus
usually is caused by falling onto the extremity (often outstretched) and may be associated with
acute pain, swelling, and deformity near the elbow (Item C52C).
References:
Arndt CAS. Neoplasms of bone. In: Kliegman RM, Behrman RE, Jenson HB, Stanton BF, eds.
Nelson's Textbook of Pediatrics. 18th ed. Philadelphia, Pa: Saunders Elsevier; 2007:2146-2150
Cripe TP. Osteosarcoma. eMedicine Specialties, Pediatrics, Oncology. 2006. Available at:
http://www.emedicine.com/ped/topic1684.htm
Gorlick R, Anderson P, Andrulis I, et al. Biology of childhood osteogenic sarcoma and potential
targets for therapeutic development. Clin Cancer Res. 2003;9:5442-5453. Available at:
http://clincancerres.aacrjournals.org/cgi/content/full/9/15/5442
Gurney JG, Swensen AR, Bulterys M. Malignant bone tumors. In: Ries LAG, Smith MA, Gurney
JG, et al, eds. Cancer Incidence and Survival Among Children and Adolescents: United States
SEER Program 1975-1995. Bethesda, Md: National Cancer Institute, SEER Program; 1999:88110. Available at: http://seer.cancer.gov/publications/childhood/bone.pdf
Kalyoussef S, Tolan RW Jr. Osteomyelitis. eMedicine Specialties, Pediatrics, Infectious
Diseases. 2006. Available at: http://www.emedicine.com/PED/topic1677.htm
Seade LE, Bryan, WJ, Bartz RL, Josey R. Acromioclavicular joint injury. eMedicine Specialties,
Sports Medicine, Shoulder. 2006. Available at: http://www.emedicine.com/sports/TOPIC3.HTM

Copyright 2009 by the American Academy of Pediatrics

page 114

2009 PREP SA on CD-ROM


Question: 53

A 7-month-old female has undergone the second stage of surgical palliation (Glenn operation)
for hypoplastic left heart syndrome. She was discharged from the hospital 1 week ago, and her
mother brings her to the office because of irritability that began this morning. On physical
examination, the infant is awake and irritable, with a heart rate of 150 beats/min and a respiratory
rate of 50 breaths/min. She has cyanosis of the face and mucosal surfaces and swelling of the
arms and head.
Of the following, the BEST explanation for this patients clinical presentation is

A. polycythemia
B. postpericardiotomy syndrome
C. protein-losing enteropathy
D. superior vena cava syndrome
E. thoracic duct injury

Copyright 2009 by the American Academy of Pediatrics

page 115

2009 PREP SA on CD-ROM


Critique: 53

Preferred Response: D

The child described in the vignette has had surgery involving her superior vena cava, which has
been sewn by an end-to-side anastomosis to her right pulmonary artery (Glenn operation). She
is at risk for stenosis at the surgical site, thrombosis within the superior vena cava, and altered
hemodynamics if the pulmonary vascular resistance (and, thus, the pulmonary artery pressure)
increases, which raises the pressure against which the venous drainage must occur. Her
symptoms and physical findings are consistent with superior vena cava syndrome (Item C53),
and she should undergo an aggressive evaluation and rapid institution of treatment.
Obstruction of the systemic venous return may result from one of three primary causes:
extrinsic compression of either the superior or inferior vena cava, intrinsic obstruction of
systemic return, or abnormal hemodynamics with significantly elevated right atrial pressures.
The systemic veins are thin-walled vascular structures that typically drain at low pressure into
the superior and inferior vena cavae. Normally, these large veins drain into the right atrium at low
pressure. Flow in any system moves from high to low pressure, and the cardiovascular system
is no exception. The right atrium in the healthy individual typically has a pressure of less than 10
mm Hg, often less than 5 mm Hg. Mechanisms that facilitate venous drainage to the right atrium
include gravity for the vessels of the upper body and muscular contraction of the lower
extremities, which serves to "push blood up" the valved veins of the caudal portion of the body.
As long as the pressure in the right atrium ("downstream") remains lower than the pressures in
the veins, forward flow ensues. Any process that increases the pressure in the right atrium
raises the pressure needed to ensure forward drainage of the systemic veins. Similarly, any
obstruction of the superior or inferior vena cava raises the pressure "upstream" and may limit
normal venous drainage.
Extrinsic compression can result from a mediastinal mass or tumor that physically
compresses the vena cava, thereby raising the pressure needed to pass blood through the
narrowing and into the right atrium. Intrinsic obstructions can result from surgical anastomotic
sites, baffle stenosis, thrombosis (eg, from an indwelling catheter), or cardiac tumors that
physically obstruct blood return through the vena cava.
When blood return from the superior vena cava into the right atrium is obstructed
significantly, patients may demonstrate signs of superficial venous distention, venous
congestion, and facial and upper body edema, as described for the infant in the vignette. As the
venous pressure increases proximal to the obstruction, the venous drainage of the brain may
become engorged, leading to discomfort, irritability, and seizure and brain injury.
Polycythemia (elevated hemoglobin concentrations) might lead to sluggish blood flow
through the small capillaries, but would not cause the findings described in this child.
Postpericardiotomy syndrome, which can occur in children who have had cardiac surgery,
generally presents with fever and systemic symptoms. Protein-losing enteropathy is a serious
complication that can occur in patients who have increased pressure in the venous drainage of
the gut, including those who undergo single ventricle palliation. Such patients typically present
with diarrhea and edema of the entire body, not localized to the upper compartments, as in this
patient. Thoracic duct injury can occur in any patient undergoing cardiac surgery and often
leads to a chylothorax.
References:
Moore P. Obstructive lesions. In: Rudolph CD, Rudolph AM, eds. Rudolph's Pediatrics. 21st ed.
New York, NY: McGraw-Hill Medical Publishing Division; 2003:1800-1813
Silberbach M, Hannon D. Presentation of congenital heart disease in the neonate and young
infant. Pediatr Rev. 2007;28:123-131. Available at:
http://pedsinreview.aappublications.org/cgi/content/full/28/4/123

Copyright 2009 by the American Academy of Pediatrics

page 116

2009 PREP SA on CD-ROM


Question: 54

A 6-year-old boy presents with a sudden-onset loss of awareness characterized by staring,


drooling, and chewing movements for more than 15 minutes, followed by confusion, then deep
sleep. On physical examination in the emergency department, the child is afebrile and appears
to be returning to normal. Vital signs and general examination findings are normal, and there are
no focal findings. Head computed tomography scan shows a large, contrast-enhancing cerebral
mass (Item Q54) without edema or midline shift.
Of the following, the MOST likely diagnosis is

A. arteriovenous malformation
B. brain damage from a seizure
C. ependymoma
D. glioblastoma multiforme
E. herpes encephalitis

Copyright 2009 by the American Academy of Pediatrics

page 117

2009 PREP SA on CD-ROM


Critique: 54

Preferred Response: A

The most common cause of hemorrhagic stroke in children is vascular malformation, of which
there are two types: arteriovenous malformations (AVMs) and cavernous malformations. These
can present in childhood with hemorrhage that leads to headache and seizures, as described for
the boy in the vignette. Severe, "worst-ever" headache; seizure; and mental status change are
indications for emergency evaluation and imaging of the central nervous system. The computed
tomography scan obtained for this boy shows a contrast-enhancing tortuous vascular mass
without surrounding edema or midline shift (Item C54A).
Seizures are not believed to cause brain damage in children, except in rare cases. A large
hemorrhage can cause seizures, but seizures cannot cause a large hemorrhage.
Ependymomas are one of the more common brain tumors in children. They emerge from the
ependymal lining of ventricles, do not hemorrhage, and typically occur in the posterior fossa of
young children (Item C54B). Therefore, they present more indolently, with headache and cranial
nerve findings. Glioblastoma multiforme, the highest grade astrocytoma, is uncommon in young
children. Although glioblastoma multiforme can hemorrhage, it does not have a tortuous vascular
appearance on imaging and has surrounding edema. Herpes encephalitis causes hemorrhagic
necrosis (Item C54C) and can present with seizures, but usually the patient is febrile and the
characteristic imaging appearance does not involve a tortuous vascular-appearing mass.
References:
Huang J, Gailloud PH, Tamargo RJ. Vascular malformations. In: Singer HS, Kossoff EH,
Hartman AL, Crawford TO, eds. Treatment of Pediatric Neurologic Disorders. Boca Raton, Fla:
Taylor & Francis; 2005:409-414
Klimo P Jr, Rao G, Brockmeyer D. Pediatric arteriovenous malformations: a 15-year experience
with an emphasis on residual and recurrent lesions. Childs Nerv Syst. 2007;23:31-37. Abstract
available at: http://www.ncbi.nlm.nih.gov/pubmed/17053936
Thai Q, Moriarty JL, Tamargo RJ. Central nervous system vascular malformations in pediatric
patients. In: Maria BL, ed. Current Management in Child Neurology. 3rd ed. Hamilton, Ontario,
Canada: BC Decker Inc; 2005:595-605

Copyright 2009 by the American Academy of Pediatrics

page 118

2009 PREP SA on CD-ROM


Question: 55

While examining a newborn, you note a persistent curve in the spine regardless of the babys
position. You order spine radiographs, which reveal multiple vertebral malformations and
segmentation defects (Item Q55).
Of the following, the MOST appropriate studies to guide further management are

A. chromosome analysis and renal ultrasonography


B. echocardiography and chromosome analysis
C. echocardiography and renal ultrasonography
D. head ultrasonography and ophthalmology consultation
E. renal and head ultrasonography

Copyright 2009 by the American Academy of Pediatrics

page 119

2009 PREP SA on CD-ROM


Critique: 55

Preferred Response: C

Congenital scoliosis, as described for the infant in the vignette, is caused by errors in vertebral
development during embryogenesis, resulting in abnormal segmentation or formation (Item C55).
It is associated with other malformations in more than 50% of affected individuals, most
commonly involving the heart, kidneys, and spinal cord. Most cases of congenital scoliosis are
not hereditary, although numerous syndromes include spinal malformations.
The child who has congenital scoliosis should receive serial posteroanterior and lateral
spine radiographs beginning, ideally, at birth. After 1 year of age, spine radiographs should be
taken with the child standing and the pelvis maintained in a level position. Computed tomography
scan with thin slices and three-dimensional reconstruction may be helpful, especially if
malformations are multiple or complex. Magnetic resonance imaging (MRI) with cartilage
sequences is recommended to determine growth potential. Significant progression of scoliosis
usually warrants surgery. Bracing typically is not helpful as a primary treatment modality but
may be recommended after surgery.
Every child who has congenital scoliosis and vertebral anomalies should be evaluated
clinically for associated malformations. Up to 20% of affected individuals have abnormalities of
the urinary tract, and up to 25% have cardiac anomalies. Therefore, children should receive
screening renal ultrasonography and echocardiography. Up to 40% of affected children may
have abnormalities of the spinal cord such as syrinx and tethered cord, warranting spinal MRI.
Chromosome analysis, ophthalmology evaluation, and head ultrasonography typically are not
helpful in directing management but must be considered on a case-by-case basis.
References:
Arlet V, Odent T, Aebi M. Congenital scoliosis. Eur Spine J. 2003;12:456-463. Abstract available
at: http://www.ncbi.nlm.nih.gov/pubmed/14618384
Ferguson RL. Medical and congenital comorbidities associated with spinal deformities in the
immature spine. J Bone Joint Surg Am. 2007;89:34-41
Hedequist D, Emans J. Congenital scoliosis. J Am Acad Orthop Surg. 2004;12:266-275.
Abstract available at: http://www.ncbi.nlm.nih.gov/pubmed/15473678

Copyright 2009 by the American Academy of Pediatrics

page 120

2009 PREP SA on CD-ROM


Question: 56

A community group asks you to speak at a forum on teenage pregnancy. The number of
pregnancies among young adolescents at the local middle school has increased this year, and
several community members want more information about adolescent pregnancy and its longterm effects.
Of the following, the MOST appropriate statement to include in your talk about pregnant and
parenting adolescents in the United States is that

A. adolescent fathers do not have increased school drop-out rates


B. adolescents who become pregnant have the same vocational opportunities as their
nonpregnant female peers

C. most adolescent pregnancies occur in 14- to 16-year-old young women


D. poverty is correlated significantly with adolescent pregnancy
E. the adolescent pregnancy rate is increasing in the United States

Copyright 2009 by the American Academy of Pediatrics

page 121

2009 PREP SA on CD-ROM


Critique: 56

Preferred Response: D

Although the proportion of teenagers reporting sexual activity has dropped and adolescent
pregnancy rates have declined in recent years, the United States still has the highest adolescent
birth rate among comparable developed countries. Most adolescent pregnancies occur among
older adolescents (ie, 18 and 19 year olds), and young women in this age group have a lower
risk of medical complications involving the mother or child than those younger than 17 years of
age. The adverse consequences of adolescent pregnancy are myriad, including school
interruption, persistent poverty, limited vocational opportunities, separation from the father of the
baby, divorce, and repeat pregnancy. Poverty is an important factor correlated with adolescent
pregnancy, with as many as 83% of adolescents who gave birth and 61% of adolescents who
had abortions coming from poor or low-income homes compared with 38% of nonpregnant
adolescent females. Adolescent fathers have been shown to have similar outcomes to
adolescent mothers, with a higher likelihood of having poor academic performance, higher
school drop-out rates, and decreased income potential compared with their peers who are not
fathers.
In an attempt to decrease adolescent child-bearing rates as well as unsafe sexual practices,
every pediatrician should integrate sexuality education into clinical practice with children from
early childhood through adolescence. Pediatricians also should consider participating in
development and implementation of sexuality education curricula for schools or public efforts.
References:
American Academy of Pediatrics Committee on Psychosocial Aspects of Child and Family
Health and Committee on Adolescence. Sexuality education for children and adolescents.
Pediatrics. 2001;108:498-502. Available at:
http://pediatrics.aappublications.org/cgi/content/full/108/2/498
Klein JD, and the AAP Committee on Adolescence. Adolescent pregnancy: current trends and
issues. Pediatrics. 2005;116:281-286. Available at:
http://pediatrics.aappublications.org/cgi/content/full/116/1/281

Copyright 2009 by the American Academy of Pediatrics

page 122

2009 PREP SA on CD-ROM


Question: 57

You are treating a 14-year-old boy in the pediatric intensive care unit who suffered a traumatic
brain injury in a motor vehicle crash earlier today and underwent surgery to drain a right-sided
epidural hematoma. He is currently receiving mechanical ventilation and is sedated. The nurse
calls you to the bedside because the intraventricular catheter is clotted and no intracranial
pressure waveform is seen on the monitor. On physical examination, you note that his right pupil
is dilated and unresponsive to light, which differs from findings on your examination immediately
after surgery.
Of the following, the MOST appropriate immediate next step is

A. administration of fentanyl
B. administration of mannitol
C. cerebral angiography
D. replacement of the intraventricular catheter
E. ophthalmology consultation

Copyright 2009 by the American Academy of Pediatrics

page 123

2009 PREP SA on CD-ROM


Critique: 57

Preferred Response: B

Increased intracranial pressure can cause herniation of the cerebral contents, which is a
neurologic emergency. Transtentorial herniation results from downward or upward displacement
of the brain through the tentorium at the level of the incisura. Temporal lobe (uncal) herniations
are a subcategory of transtentorial herniations that are characterized by dilation of a unilateral
pupil due to compression of the oculomotor nerve, as described for the boy in the vignette (Item
C57). Uncal herniations usually occur with rapid expansion of the contents of the temporal lobe
fossa, such as seen with epidural hematomas, focal injury, or infection.
Findings for the boy in the vignette suggest rebleeding of the epidural hematoma, which
requires prompt action to reduce the associated swelling and reverse the herniation. Osmotic
agents such as mannitol or hypertonic saline should be administered while calling the
neurosurgeon and planning for emergent radiologic evaluation.
Pain or anxiety may produce bilateral, not unilateral, pupillary dilation. Therefore, treatment
with fentanyl would not address this patient's neurologic emergency. Ischemic stroke can
produce pupil dilation but is less likely than uncal herniation in this scenario. Cerebral
angiography to evaluate blood vessel characteristics usually is performed after stroke is
diagnosed by computed tomography scan or magnetic radiographic imaging. Replacement of
the clotted intraventricular catheter is indicated for overall management, but acute measures to
reverse the uncal herniation are of much higher priority. Ophthalmology consultation for
anisocoria is not indicated at this time because the most likely cause of the patient's signs and
symptoms is acute herniation.
References:
Avner JR. Altered states of consciousness. Pediatr Rev. 2006:27:331-338. Available at:
http://pedsinreview.aappublications.org/cgi/content/full/27/9/331
Frankel LR. Neurological emergencies and stabilization. In: Kliegman RM, Behrman RE, Jenson
HB, Stanton BF, eds. Nelson Textbook of Pediatrics. 18th ed. Philadelphia, Pa: Saunders
Elsevier; 2007:405-410

Copyright 2009 by the American Academy of Pediatrics

page 124

2009 PREP SA on CD-ROM


Question: 58

The mother of a 10-year-old boy, whom you have been following since he was 3 years old,
complains that he is always hungry and is gaining weight. The mother, who is overweight,
reports that the boy refuses to exercise, and she cannot control his diet. She just read an article
in a magazine about weight gain from Cushing syndrome and wonders if he could have this
condition.
Of the following, the growth chart shown in Item 58 that suggests Cushing syndrome is

A. Growth chart A
B. Growth chart B
C. Growth chart C
D. Growth chart D
E. Growth chart E

Copyright 2009 by the American Academy of Pediatrics

page 125

2009 PREP SA on CD-ROM


Critique: 58

Preferred Response: C

Weight gain from exogenous obesity can be confused with Cushing syndrome, but
glucocorticoid excess, as seen in Cushing syndrome, almost always is associated with
attenuation of normal growth, as documented with Growth Chart C (Item C58A). The other
growth charts are more typical for exogenous obesity, with height either enhanced or
unchanged in the presence of weight gain. Other signs and symptoms of Cushing syndrome
include hypertension, violaceous skin striae (Item C58B), "buffalo hump" and muscle weakness
because of loss of muscle mass, centripetal obesity, cushingoid facies, easy bruisability,
hirsutism, failure of pubertal progression or amenorrhea in women, loss of libido in men,
headache, depression, and dysphoria. Comparison of school photographs from past years can
be a useful exercise, although the clinical diagnosis of pituitary Cushing syndrome (Cushing
disease) can be difficult.
Documentation of several elevated 24-hour urine free cortisol measurements as well as
elevated overnight dexamethasone-suppressed serum cortisol, evening salivary cortisol, or
midnight serum cortisol values aid in diagnosis. However, children who are very obese,
stressed, or depressed may have inappropriate elevations in serum or urine cortisol values, and
children who have mild Cushing disease may have normal values on one or more occasions.
References:
Batista DL, Riar J, Keil M, Stratakis CA. Diagnostic tests for children who are referred for the
investigation of Cushing syndrome. Pediatrics. 2007;120:e575-e586. Available at:
http://pediatrics.aappublications.org/cgi/content/full/120/3/e575
Greening JE, Storr HL, McKensie SA, et al. Linear growth and body mass index in pediatric
patients with Cushing's disease or simple obesity. J Endocrinol Invest. 2006;29:885-887.
Abstract available at: http://www.ncbi.nlm.nih.gov/pubmed/17185896
Klish WJ. Clinical evaluation of the obese child and adolescent. UpToDate Online 15.3. 2008.
Available for subscription at:
http://www.uptodateonline.com/utd/content/topic.do?topicKey=pedigast/11089
Magiakou MA, Mastorakas G, Oldfield EH, et al. Cushing's syndrome in children and
adolescents. Presentation, diagnosis, and therapy. N Engl J Med. 1994;331:629-636. Abstract
available at: http://www.ncbi.nlm.nih.gov/pubmed/8052272
Nieman L, Ilias I. Evaluation and treatment of Cushing's syndrome. Am J Med. 2005;118:13401346. Abstract available at: http://www.ncbi.nlm.nih.gov/pubmed/16378774

Copyright 2009 by the American Academy of Pediatrics

page 126

2009 PREP SA on CD-ROM


Question: 59

The parents of an 8-year-old boy are concerned because he recently has begun to struggle in
school. In the past, he always had been an attentive and motivated student. His current teacher
reports that at times when he is speaking in class, he stops speaking abruptly, stares with
glassy eyes, then resumes speaking. At home, his parents note that he "spaces out" when
eating dinner. His parents ask your input and the best approach to treat his issues.
Of the following, the MOST appropriate initial evaluation is

A. computed tomography scan of the brain


B. educational evaluation
C. electroencephalography
D. formal audiologic evaluation
E. magnetic resonance imaging of the brain

Copyright 2009 by the American Academy of Pediatrics

page 127

2009 PREP SA on CD-ROM


Critique: 59

Preferred Response: C

The child described in the vignette appears to be experiencing absence seizures. These
present as multiple daily episodes of abrupt arrest of consciousness and voluntary movement
that last a few seconds (Item C59), followed by immediate return to normal consciousness.
Decline in school function is common at the onset of this disorder. Electroencephalography
(EEG) confirms the diagnosis, showing a pattern of spike and wave electrical complexes that
have a frequency of 3 Hz (3/second). Computed tomography scan and magnetic resonance
imaging are not indicated in the evaluation of children who have uncomplicated absence
seizures.
An audiologic evaluation would be indicated if the child was suspected of not hearing
properly. Decisions regarding an educational evaluation should be deferred until the diagnosis of
epilepsy is clarified. Occasionally, inattention associated with attention-deficit/hyperactivity
disorder (ADHD) may mimic absence epilepsy. However, children who have ADHD do not have
a sudden cessation of motor activity, blank facial expression, or flickering of the eyelids
characteristic of absence seizures. Computed tomography scan, magnetic resonance imaging,
and EEG generally are not indicated for the diagnosis of either ADHD or a learning disability.
Children who have learning disabilities are identified by results from standardized
psychoeducational testing. ADHD is diagnosed if the child meets American Psychiatric
Association Diagnostic and Statistical Manual of Mental Disorders, fourth edition, text revision
(DSM-IV-TR) criteria and has symptoms of inattention and hyperactivity/impulsivity that have
lasted for more than 6 months and were observed prior to age 7 years. The behaviors cause
"clinically significant impairment in social, academic or occupational functioning" and present in
more that one setting. Symptoms do not include the type of "staring spells" described for the boy
in the vignette.
References:
American Academy of Pediatrics. Committee on Quality Improvement, Subcommittee on
Attention-Deficit/Hyperactivity Disorder. Clinical practice guideline: diagnosis and evaluation of
the child with attention-deficit/hyperactivity disorder. Pediatrics. 2000;105:1158-1170. Available
at: http://pediatrics.aappublications.org/cgi/content/full/105/5/1158
American Psychiatric Association. Diagnostic criteria for ADHD. In: Diagnostic and Statistical
Manual of Mental Disorders. 4th ed. Text revision. Arlington, Va: American Psychiatric
Association; 2000:85-94
American Psychiatric Association. Diagnostic criteria for learning disability. In: Diagnostic and
Statistical Manual of Mental Disorders. 4th ed. Text revision. Arlington, Va: American Psychiatric
Association; 2000:49-56
Posner E. Absence seizures in children. BMJ Clinical Evidence. 2007. Available for subscription
at: http://clinicalevidence.bmj.com/ceweb/conditions/chd/0317/0317_background.jsp
Pritchard D. Attention deficit hyperactivity disorder in children. BMJ Clinical Evidence. 2006.
Available for subscription at:
http://clinicalevidence.bmj.com/ceweb/conditions/chd/0312/0312_background.jsp

Copyright 2009 by the American Academy of Pediatrics

page 128

2009 PREP SA on CD-ROM


Question: 60

You are seeing a young girl for a health supervision visit. Her older brother recently underwent a
bone marrow transplant, and you inquire about his health. The mother is tearful as she tells you
it has been difficult, explaining that he has had fever for about 10 days, his "counts are still
down," and they are planning to start amphotericin B just in case he has a fungal infection. She is
concerned because she was told about potential adverse effects of the medication and how they
need to watch the "electrolytes in his blood" very closely. She doesnt know what "electrolytes"
are, but asks what parameter in his blood might be affected.
Of the following, the MOST clinically important parameter to monitor during the initiation of
amphotericin B therapy is

A. bicarbonate
B. creatinine
C. glucose
D. potassium
E. sodium

Copyright 2009 by the American Academy of Pediatrics

page 129

2009 PREP SA on CD-ROM


Critique: 60

Preferred Response: D

The major metabolic abnormality that occurs early in therapy with amphotericin B administration
is hypokalemia, which can be difficult to correct, especially if there is concurrent
hypomagnesemia. Amphotericin B augments potassium and magnesium excretion by the
kidney. Therefore, potassium concentrations cannot be corrected in most patients until the
hypomagnesemia also is corrected. The hypokalemia can occur whether amphotericin B
deoxycholate or the lipid formulations of amphotericin B are used. Problems with sodium,
glucose, and bicarbonate usually do not occur with amphotericin B therapy. Elevation of the
blood urea nitrogen or creatinine values associated with amphotericin B therapy occurs slowly
and is due to the cumulative renal effects of the medication. Once the medication is stopped,
renal function returns to normal in most patients.
Other common adverse reactions associated with amphotericin B therapy include fever,
chills, rigors, malaise, nausea, vomiting, and headaches. Younger children and infants generally
do not have these reactions. Older children and adolescents may require premedication with
ibuprofen or acetaminophen to lessen such reactions. In addition, lengthening of infusion times to
between 4 and 6 hours can blunt such symptoms. In severe cases, patients may require
antihistamines, antiemetics, meperidine, or the addition of hydrocortisone to the infusion before
administration of amphotericin B.
References:
American Academy of Pediatrics. Antifungal drugs for systemic fungal infections. In: Pickering
LK, Baker CJ, Long SS, McMillan JA, eds. Red Book: 2006 Report of the Committee on
Infectious Diseases. 27th ed. Elk Grove Village, Ill: American Academy of Pediatrics; 2006:774776
Zaoutis TE, Benjamin DK, Steinbach WJ. Antifungal treatment in pediatric patients. Drug Resist
Update. 2005;8:235-245. Abstract available at: http://www.ncbi.nlm.nih.gov/pubmed/16054422

Copyright 2009 by the American Academy of Pediatrics

page 130

2009 PREP SA on CD-ROM


Question: 61

You are evaluating an 8-year-old boy who has acute lymphoblastic leukemia and is in septic
shock caused by Klebsiella pneumoniae. The antibiotic susceptibilities for the organism reveal
that it is resistant to ampicillin, cefazolin, ceftriaxone, and gentamicin.
Of the following, the MOST appropriate antibiotic to use in this patient is

A. cefuroxime
B. clindamycin
C. meropenem
D. penicillin G
E. piperacillin

Copyright 2009 by the American Academy of Pediatrics

page 131

2009 PREP SA on CD-ROM


Critique: 61

Preferred Response: C

Imipenem and meropenem are carbapenems, a type of beta-lactam antibiotic that has a very
broad antibacterial spectrum of activity. These agents are beta-lactamase-stable and bind with
high affinity to the penicillin-binding proteins of gram-positive and gram-negative bacteria.
Imipenem is slightly more active against gram-positive bacteria than meropenem; meropenem is
slightly more active against gram-negative aerobic organisms. Both have excellent activity
against aerobic hemolytic streptococci, Streptococcus pneumoniae (including resistant strains),
methicillin-susceptible S aureus and S epidermidis, Listeria sp, and Bacillus sp. Penicillinsusceptible strains of Enterococcus faecalis are susceptible to imipenem (bacteriostatic), but
resistant to meropenem. As a class, carbapenems also are highly active against Neisseria
gonorrhoeae and N meningitidis, Haemophilus influenzae and other Haemophilus sp, Morganella
sp, Proteus sp, most Enterobacteriaceae, Citrobacter sp, Enterobacter sp, Providencia sp,
Pseudomonas sp, and most anaerobic species, including anaerobic gram-positive cocci. They
also are active against Nocardia sp and Actinomyces sp.
Carbapenems are useful for the treatment of a wide variety of infections. Imipenem and
meropenem are most appropriate for treatment of infections caused by cephalosporin-resistant
Enterobacteriaceae, especially Serratia marcescens, Providencia sp, C freundii, and
Enterobacter sp; as empiric therapy for serious infections in patients previously treated with
multiple antibiotics; as single agents in the treatment of febrile neutropenia; and as treatment for
polymicrobial and nosocomial infections. Thus, meropenem is appropriate therapy for the boy
described in the vignette, who is infected with a multiple drug-resistant strain of Klebsiella
pneumoniae (a gram-negative organism) that is resistant to the cephalosporins (including the
third-generation cephalosporins).
An infrequent adverse effect of imipenem (<1%) is seizures. This effect has been reported
in patients receiving very high doses who have underlying central nervous system pathology
and in patients who have decreased renal function in whom the dose has not been adjusted.
Seizures are not a major problem with meropenem, even in patients being treated for meningitis.
Cefuroxime is a second-generation cephalosporin to which the organism infecting the patient
in the vignette is resistant. Clindamycin only has activity against gram-positive organisms.
Because the K pneumoniae strain is resistant to ampicillin, it also is resistant to penicillin G. The
antibiotic resistance pattern of the identified pathogen indicates that one of its major resistance
mechanisms is the production of extended-spectrum beta-lactamases, which are enzymes that
hydrolyze many different penicillins and cephalosporins. Because of the production of these
enzymes, piperacillin is not an effective treatment option.
References:
Balfour JA, Bryson HM, Brogden RN. Imipenem/cilastatin: an update of its antibacterial activity,
pharmacokinetics and therapeutic efficacy in the treatment of serious infections. Drugs.
1996;51:99-136. Abstract available at: http://www.ncbi.nlm.nih.gov/pubmed/8741235
Nicolau DP. Carbapenems: a potent class of antibiotics. Expert Opin Pharmacother. 2008;9:2337. Abstract available at: http://www.ncbi.nlm.nih.gov/pubmed/18076336
Norrby SR. Carbapenems. Med Clin North Am. 1995;79:745-759. Abstract available at:
http://www.ncbi.nlm.nih.gov/pubmed/7791421
Wiseman LR, Wagstaff AJ, Brogden RN, Bryson HM. Meropenem: a review of its antibacterial
activity, pharmacokinetic properties and clinical efficacy. Drugs. 1995;50:73-101. Abstract
available at: http://www.ncbi.nlm.nih.gov/pubmed/7588092
Zhanel GG, Wiebe R, Diley L, et al. Comparative review of the carbapenems. Drugs.
2007;67:1027-1052. Abstract available at: http://www.ncbi.nlm.nih.gov/pubmed/17488146

Copyright 2009 by the American Academy of Pediatrics

page 132

2009 PREP SA on CD-ROM


Question: 62

A 10-year-old boy has marked fluid intake, frequent urination, and decreased visual acuity. On
physical examination, the boy is short (<5th percentile), neurologic evaluation findings are
normal, and no edema is present. His electrolyte values are normal. Other laboratory results
include:
Blood urea nitrogen, 36.0 mg/dL (12.9 mmol/L)
Creatinine, 2.0 mg/dL (176.8 mcmol/L)
Hemoglobin, 6.5 g/dL (65.0 g/L)
Urine specific gravity, 1.005
Urine pH, 6
Urine protein, 1+
Ophthalmologic evaluation reveals features of retinitis pigmentosa.
Of the following, the MOST likely diagnosis is

A. Alport syndrome
B. diabetic nephropathy
C. juvenile nephronophthisis
D. Lowe syndrome
E. nephropathic cystinosis

Copyright 2009 by the American Academy of Pediatrics

page 133

2009 PREP SA on CD-ROM


Critique: 62

Preferred Response: C

The symptoms of polyuria and polydipsia; clinical finding of short stature; and laboratory findings
of azotemia, anemia, and proteinuria described for the patient in the vignette point to the
diagnosis of juvenile nephronophthisis (NPH). This autosomal recessive inherited disorder is
characterized by chronic tubulointerstitial disease. NPH begins with tubular dysfunction, which
progresses to azotemia and ultimately, end-stage renal disease (ESRD). The clinical
presentation includes symptoms consistent with a urinary concentrating defect (polyuria and
polydipsia) at 2 to 3 years of age that progresses to more overt symptoms in the next few
years. Tubular wasting of sodium can lead to hyponatremia and salt craving. Signs of acidosis,
anemia, and azotemia appear next. The median age of onset of ESRD is 13 years. NPH is
estimated to account for 5% to 10% of all cases of ESRD in children. The diagnosis is based
primarily on clinical symptoms; renal ultrasonography usually is not helpful because the finding of
mild increased echogenicity of the kidneys is nonspecific. Medullary cysts, which characterize
the disease, may be missed on ultrasonography but usually are seen on computed tomography
scan. Renal biopsy, which is not always performed, demonstrates tubular damage, with
interstitial fibrosis and normal glomeruli. Cysts may be seen in advanced disease.
NPH may be associated with other disorders, such as the Senior-Loken syndrome, which is
seen in 15% of cases. This syndrome involves the eye, with tapetoretinal degeneration, retinitis
pigmentosa (Item C62A), and the development of blindness. Other findings include nystagmus,
coloboma (Item C62B), and cataracts (Item C62C). NPH also is associated with Joubert
syndrome (NPH with aplasia of the cerebellar vermis causing ataxia and retinal
coloboma/retinitis pigmentosa), hepatic fibrosis, and skeletal defects (cone-shaped epiphyses).
Four genes have been identified in association with NPH. Most cases of juvenile NPH are
related to the NPHP1 gene, which is located on chromosome 2q12.3 and encodes for
nephrocystin 1. NPHP2 and NPHP3 encode for proteins for the infantile and adolescent forms of
NPH, respectively.
Alport syndrome is an X-linked disorder associated with a defect of the alpha 5 chain of type
IV collagen (Col4A5). It manifests with hematuria, high-frequency sensorineural hearing loss,
and anterior lenticonus (and occasionally cataracts). Diabetic nephropathy can result in retinal
and renal disease, but the patient in the vignette lacks a history of diabetes and has no
glycosuria. Lowe syndrome is an X-linked disorder characterized by Fanconi syndrome
(glycosuria, metabolic acidosis, aminoaciduria, and hypophosphatemia), mental retardation, and
congenital cataracts, but it does not result in azotemia, anemia, or retinal abnormalities.
Nephropathic cystinosis is a lysosomal storage disorder that has an autosomal recessive
inheritance pattern and involves the Fanconi syndrome, failure to thrive, and cystine crystal
deposition within the cornea (Item C62D), resulting in severe photophobia.
References:
Hildebrandt F. Nephronophthisis-medullary cystic kidney disease. In: Avner ED, Harmon WE,
Niaudet P, eds. Pediatric Nephrology. 5th ed. Philadelphia, Pa: Lippincott Williams & Wilkins;
2004:665-673
Niaudet P. Inherited nephropathies. In: Kher KK, Schnaper HW, Makker SP, eds. Clinical
Pediatric Nephrology. 2nd ed. London, England: Informa Healthcare; 2007:195-212.
Saunier S, Calado J, Benessy F, et al. Characterization of the NPHP1 locus: mutational
mechanism involved in deletions in familial juvenile nephronophthisis. Am J Hum Genet.
2000;66:778-789. Available at:
http://www.pubmedcentral.nih.gov/articlerender.fcgi?tool=pubmed&pubmedid=10712196

Copyright 2009 by the American Academy of Pediatrics

page 134

2009 PREP SA on CD-ROM


Question: 63

A mother brings in her 11-month-old son after he broke out in "hives" today during breakfast.
The infant had stayed home from child care with a low-grade fever, and the mother had let him
eat eggs for the first time. Immediately after breakfast, the mother noted a diffuse erythematous,
pruritic rash covering the boys trunk and extremities. She is concerned that her son may have
an egg allergy.
Of the following, the BEST statement regarding immunoglobulin E-mediated egg food allergy is
that

A. cooking the egg eliminates its allergic potential


B. egg is the most common food allergy in the first postnatal year
C. egg white is more allergenic than egg yolk
D. most children do not outgrow their egg allergy
E. the measles-mumps-rubella vaccine is contraindicated in children who have egg allergy

Copyright 2009 by the American Academy of Pediatrics

page 135

2009 PREP SA on CD-ROM


Critique: 63

Preferred Response: C

Immunoglobulin (Ig) E-mediated egg allergy is one of the more common childhood food allergies,
affecting approximately 1% to 2% of children. As described in the vignette, cutaneous features
are common, including atopic dermatitis, urticaria, and pruritus. Once the diagnosis of egg
allergy is determined, patients generally are advised to avoid all egg food products with the hope
that most children will outgrow their egg allergy within 3 to 5 years (Item C63).
The primary allergenic egg protein is ovomucoid, a protein predominantly in the egg white.
Approximately 50% of children may be able to tolerate small amounts of egg protein that has
been heated extensively (eg, baked goods). Prolonged heating at high temperatures can
denature proteins from a conformational form to a linear form. Some children who are allergic to
eggs do not recognize the linear protein form as an allergen and, therefore, do not experience a
reaction. Of note, the brief cooking used to make scrambled eggs will not denature heat-stable
proteins.
The relationship between egg allergy and vaccination is a common question. The measlesmumps-rubella vaccine is safe for children who have egg allergy and should be administered
without special precautions. The trivalent influenza and live attenuated influenza vaccines
contain small amounts of egg protein and are contraindicated for patients who have egg allergy.
However, studies have supported a two-dose protocol for the administration of the influenza
vaccine in egg-allergic patients. The two-dose protocol involves administering one tenth of the
vaccine, observing the recipient for a period of time, and administering the rest of the vaccine,
followed by a similar observation period.
In westernized countries, milk generally is regarded as the most common food allergen in
infants, with an incidence of 2.5%, compared with an incidence of 1.5% for egg allergy.
References:
Boyano-Martnez, T, Garca-Ara C, Daz-Pena JM, and Martn-Esteban M. Prediction of
tolerance on the basis of quantification of egg white-specific IgE antibodies in children with egg
allergy. J Allergy Clin Immunol. 2002;110:304-309. Abstract available at:
http://www.ncbi.nlm.nih.gov/pubmed/12170273
Piquer-Gibert M, Plaza-Martn A, Martorell-Aragons A, et al. Recommendations for
administering the triple viral vaccine and antiinfluenza vaccine in patients with egg allergy.
Allergol Immunopathol (Madr). 2007;35:209-212. Available at: http://db.doyma.es/cgibin/wdbcgi.exe/doyma/mrevista.pubmed_full?inctrl=05ZI0102&rev=105&vol=35&num=5&pag=20
9
Sampson HA, Leung DYM. Adverse reactions to foods. In: Kleigman RM, Behrman RE, Jenson
HB, Stanton BF, eds. Nelson Textbook of Pediatrics. 18th ed. Philadelphia, Pa: Saunders
Elsevier; 2007:986-989

Copyright 2009 by the American Academy of Pediatrics

page 136

2009 PREP SA on CD-ROM


Question: 64

A 12-year-old boy is brought to the emergency department by emergency medical services


after sustaining a lower leg injury sliding into home plate during a baseball game. He tells you that
he thinks his leg twisted when he slid. He reports that he had immediate pain in his right ankle
and has been unable to walk since the injury occurred. Prior to transport, the paramedics
splinted his right lower leg. On physical examination, he has significant swelling and ecchymosis
around his distal tibia and fibula. Following the administration of analgesia, radiographs are
obtained (Item Q64).
Of the following, the MOST likely complication of this injury is

A. avascular necrosis of the distal tibia


B. osteochondritis desiccans
C. osteomyelitis
D. tibial growth arrest
E. unicameral bone cyst

Copyright 2009 by the American Academy of Pediatrics

page 137

2009 PREP SA on CD-ROM


Critique: 64

Preferred Response: D

Pediatric fracture patterns differ from those in adults for a variety of biomechanical, physiologic,
and anatomic reasons. Most importantly, children are at risk of orthopedic injuries involving the
growth plates (physes) that may lead to adverse sequelae if not diagnosed acutely and
managed appropriately. Physeal injuries occur most commonly in school-age children and
account for 15% to 30% of all pediatric fractures.
The physis, or growth plate, of the pediatric bone is located between the metaphysis and the
epiphysis. This region of the pediatric skeleton is composed predominantly of cartilage cells that
multiply rapidly, calcify along the metaphyses, and lead to longitudinal bone growth. Although the
growth activity at the physes is responsible for rapid fracture healing, damage to this area can
impair healing and future growth. This region of the bone is particularly vulnerable to injury
because the weaker physeal cartilage is less resistant to traumatic forces than the mineralized
bone and ligaments.
Epiphyseal fractures typically are categorized using the five-tier Salter-Harris classification,
a scale based on the degree of physeal, bone, and joint involvement (Item C64A and Item
C64B). This categorization largely determines fracture management and prognosis. The
radiograph of the boy described in the vignette indicates that he has suffered a type IV tibial
fracture (Item C64C). His fracture is unique in that it occurs through three anatomic planes:
coronally through the distal tibial metaphysis, transversely through the physis, and sagittally
through the epiphysis (ie, a triplane fracture). These fractures occur because the physis has not
fused completely. An associated fibular fracture is a common finding.
Osteochondritis desiccans most commonly affects the medial femoral condyle of the knee.
Caused by repetitive stress, it is characterized by bone fragment development and separation at
the posterolateral aspect of the distal femur (Item C64D).
Unicameral, or simple, bone cysts usually are found in the metaphyses of long bones of
children between 4 and 12 years (Item C64E). They may lead to pathologic fractures but most
often are asymptomatic and stop expanding after skeletal maturity is reached.
Unless the fracture is open, osteomyelitis is not a likely complication. Distal tibial avascular
necrosis has been reported as an extremely rare complication of severe fractures in adults
whose physes already are closed.
References:
Dinolfo EA, Adam HM. In brief: fractures. Pediatr Rev. 2004;25:218-219. Available at:
http://pedsinreview.aappublications.org/cgi/content/full/25/6/218
Gholve PA, Hosalkar HS, Wells L. Common fractures. In: Kleigman RM, Behrman RE, Jenson
HB, Stanton BF, eds. Nelson Textbook of Pediatrics. 18th ed. Philadelphia, Pa: Saunders
Elsevier; 2007:2834-2841
Perron AD, Miller MD, Brady WJ. Orthopedic pitfalls in the ED: pediatric growth plate injuries. Am
J Emerg Med. 2002;20:50-54. Abstract available at:
http://www.ncbi.nlm.nih.gov/pubmed/11781914

Copyright 2009 by the American Academy of Pediatrics

page 138

2009 PREP SA on CD-ROM


Question: 65

A 4-month-old male infant presents for his initial examination. The family recently immigrated to
the United States from southeast Asia. They describe progressive abdominal distention (Item
Q65A) in the infant over the past 2 months. Physical examination demonstrates a firm liver edge
2 cm below the right costal margin and a spleen tip palpable 3 cm below the costal margin.
Abdominal ultrasonography demonstrates a fluid-filled abdomen (Item Q65B).
Of the following complications from his underlying disorder, this child is MOST at risk for

A. acute intussusception
B. emphysematous chronic lung disease
C. gastrointestinal bleeding
D. pneumococcal meningitis
E. renal failure

Copyright 2009 by the American Academy of Pediatrics

page 139

2009 PREP SA on CD-ROM


Critique: 65

Preferred Response: C

The firm liver, enlarged spleen, and fluid in the abdomen seen on ultrasonography described for
the infant in the vignette is consistent with a number of clinical scenarios, including infection and
malignancy. However, the findings strongly suggest chronic liver disease, with secondary portal
hypertension resulting in ascites and splenomegaly. Accordingly, he is at risk for esophageal
variceal bleeding. Acute intussusception, emphysematous chronic lung disease, pneumococcal
meningitis, and renal failure are not associated directly with portal hypertension.
Portal hypertension may occur from either hepatic venous thrombosis (Budd-Chiari
syndrome), portal venous thrombosis ("cavernous transformation of the portal vein"), or hepatic
cirrhosis. Hepatic cirrhosis is the most common cause in both adults and children. In adults,
alcoholic cirrhosis is the most common cause. In children, cirrhosis may occur from a number of
congenital, hereditary, and infectious conditions. These include biliary atresia, Alagille syndrome,
alpha-1-antitrypsin deficiency, cystic fibrosis, hepatitis C, congenital hepatic fibrosis, and
congestive heart failure.
The portal vein drains the intestinal blood and lymph that subsequently enter the liver, where
blood and nutrients are filtered through the hepatic sinusoids. The cirrhotic liver has increased
resistance to the flow of portal blood, resulting in increased portal venous pressure. This
increase in pressure is transmitted to the splenic vein (resulting in splenomegaly) and the
esophageal and hemorrhoidal venous systems (resulting in esophageal varices and
hemorrhoids). The changes in oncotic pressure also may result in hypoalbuminemia and
accumulation of ascitic fluid in the abdominal cavity, as seen in this infant.
The earliest clinical sign that suggests portal hypertension usually is an enlarged spleen. As
liver disease progresses, ascites, prominent abdominal veins, and esophageal or gastric
varices may develop. Many patients who have portal hypertension exhibit laboratory signs of
chronic liver disease, including transaminase elevation and jaundice. However, patients who
have some conditions associated with portal hypertension, including congenital hepatic fibrosis
and alpha-1-antitrypsin deficiency, may not have jaundice. In such patients, portal hypertension
should be suspected if the liver or spleen is chronically enlarged.
References:
Molleston JP. Variceal bleeding in children. J Pediatr Gastroenterol Nutr. 2003;37:538-545
Shepherd RW, Ramm GA. Liver function and dysfunction: fibrogenesis and cirrhosis. In: Walker
WA, Goulet O, Kleinman RE, Sherman PM, Shneider BL, Sanderson IR. Pediatric
Gastrointestinal Disease. 4th ed. Hamilton, Ontario, Canada: BC Decker; 2004:80-88

Copyright 2009 by the American Academy of Pediatrics

page 140

2009 PREP SA on CD-ROM


Question: 66

A 2.1-kg, 34-week gestation infant is delivered to a mother who has chorioamnionitis and had a
positive group B streptococcal urinary tract infection at 30 weeks of gestation. Four hours after
birth, the infant requires admission to the intensive care nursery because of respiratory distress.
Physical examination reveals a temperature of 96.8F (36.0C), heart rate of 160 beats/min,
respiratory rate of 80 breaths/min, blood pressure of 60/30 mm Hg, mean arterial pressure of 40
mm Hg, and pulse oximetry of 82% on room air. The infant audibly grunts, has flaring of the ala
nasi and intercostal and subcostal chest wall retractions, and is poorly perfused, with a capillary
refill time of 4 seconds and mild acrocyanosis. There is no heart murmur.
Of the following, the MOST likely radiographic findings expected for this infant are

A. air bronchograms, diffusely hazy lung fields, and low lung volume
B. cardiomegaly, hazy lung fields, and pulmonary vascular engorgement
C. fluid density in the horizontal fissure, hazy lung fields with central vascular prominence, and
normal lung volume

D. gas-filled loops of bowel in the left hemithorax and opacification of the right lung field
E. patchy areas of diffuse atelectasis, focal areas of air-trapping, and increased lung volumes

Copyright 2009 by the American Academy of Pediatrics

page 141

2009 PREP SA on CD-ROM


Critique: 66

Preferred Response: A

Infants delivered at any gestational age are at risk for respiratory distress from many causes.
Respiratory distress syndrome (RDS) is due to surfactant deficiency and characteristically
affects newborns delivered at less than 32 weeks' gestation. In the term or late preterm infant,
such as the infant described in the vignette, additional or alternative causes of respiratory
distress should be considered, including infection, aspiration, cardiac disease, or congenital
anomalies (eg, tracheoesophageal fistula, diaphragmatic hernia). Infants born to mothers who
have a history of prolonged rupture of membranes, chorioamnionitis, or known maternal group B
streptococcal carriage have a high risk for infection, including congenital pneumonia.
Respiratory distress may be seen anytime after birth but characteristically is present in the first
24 hours. Tachypnea, tachycardia, and increased work of breathing with a supplemental oxygen
requirement are the most common clinical signs. Temperature instability, irritability, lethargy, and
poor feeding also may be present. The chest radiograph for a newborn who has congenital
pneumonia may resemble that of RDS with hyaline membrane disease (air bronchograms,
diffusely hazy lung fields, and low lung volumes) (Item C66A). Rarely, a lobar infiltrate may be
seen.
The most likely infectious agents in congenital pneumonia include those that ascend from
the genital tract before or during labor: group B streptococci, Escherichia coli, and Klebsiella sp.
No specific clinical sign or laboratory finding in the sick newborn confirms the presence or
absence of pneumonia. Therefore, all newborns who exhibit respiratory distress should have
blood cultures obtained and be started on broad-spectrum antibiotic therapy (eg, ampicillin and
gentamicin) until a diagnosis of bacterial infection can be excluded. Obtaining a Gram stain of
tracheal aspirates from newborns in whom pneumonia is suspected and who have required
tracheal intubation and assisted ventilation may be helpful.
The chest radiograph depicting cardiomegaly, hazy lung fields, and pulmonary vascular
engorgement is characteristic of congenital heart disease with pulmonary overcirculation (eg,
critical aortic stenosis, total anomalous pulmonary venous return with obstruction, truncus
arteriosus, transposition of the great arteries) (Item C66B). Fluid densities in the horizontal
fissure, hazy lung fields with central vascular prominence, and normal lung volumes are
characteristic of retained fetal lung fluid, referred to clinically as transient tachypnea of the
newborn (Item C66C). Gas-filled loops of bowel in either hemithorax are suspicious for a
diaphragmatic hernia, more commonly seen on the left, with a mediastinal shift to the right and
compression atelectasis and opacification of the right lung field (Item C66D). Patchy atelectasis
with focal areas of air-trapping and increased lung volumes are characteristic of aspiration
syndromes, most notably meconium (Item C66E).
References:
Aly H. Respiratory disorders in the newborn: identification and diagnosis. Pediatr Rev.
2004;25:201-208. Available at: http://pedsinreview.aappublications.org/cgi/content/full/25/6/201
Herting E, Gefeller O, Land M, van Sonderen L, Harms K, Robertson B, and Members of the
Collaborative European Multicenter Study Group. Surfactant treatment of neonates with
respiratory failure and group B streptococcal infection. Pediatrics. 2000;106:957-964. Available
at: http://pediatrics.aappublications.org/cgi/content/full/106/5/957
Sivit CJ. Diagnostic imaging. In: Martin RJ, Fanaroff AA, Walsh MC, eds. Fanaroff and Martin's
Neonatal-Perinatal Medicine. 8th ed. Philadelphia, Pa: Mosby Elsevier; 2006:713-732
Thilo EH, Rosenberg AA. The newborn infant. In: Hay WW Jr, Levin M, Sondheimer JM,
Deterding RR, eds. Current Pediatric Diagnosis & Treatment. 18th ed. New York, NY: The
McGraw-Hill Companies, Inc; 2007:chap 1

Copyright 2009 by the American Academy of Pediatrics

page 142

2009 PREP SA on CD-ROM


Question: 67

A 16-month-old boy is brought to your clinic because his mother says he is "walking funny"
today. She states that he has been walking for 4 months and is very active, but she is unaware
of any trauma or falls. She denies fever or other symptoms. He appears well and has normal
vital signs. Physical examination reveals mild tenderness to palpation over the medial aspect of
the lower leg just above the ankle. There is no overlying bruising, erythema, or edema, and you
can elicit full range of motion in the hips, knees, and ankles.
Of the following, the MOST likely diagnosis is

A. aneurysmal bone cyst


B. ankle sprain
C. fracture
D. osteomyelitis
E. transient synovitis

Copyright 2009 by the American Academy of Pediatrics

page 143

2009 PREP SA on CD-ROM


Critique: 67

Preferred Response: C

The differential diagnosis of the child who has an acute limp includes infectious causes such as
osteomyelitis and septic arthritis, postinfectious entities such as transient synovitis and
rheumatic fever, and injuries related to accidental or nonaccidental trauma. The presence of
fever or other signs and symptoms, including erythema and limitation of joint motion, should lead
the clinician to investigate infectious or postinfectious causes. However, in the absence of
constitutional symptoms and typical external signs of infection, such as described for the boy in
the vignette, an occult fracture should be considered, even if no history of trauma is elicited.
Careful questioning regarding any falls or injuries, observation of the parent-child interaction,
and a complete physical examination that includes the skin are important because many children
who have fractures from inflicted trauma come to medical attention because of vague limb pain
or limp. However, the presence of a fracture in a toddler does not necessarily indicate inflicted
injury (eg, child abuse) because children who are just learning to walk and run can fall and
sustain injury without the parent's awareness of the event.
Anteroposterior and lateral radiographs of the lower extremities are indicated if a fracture is
suspected, and the type of fracture seen can provide some clues as to the mechanism of injury.
One of the most common fractures in children is a nondisplaced spiral fracture of the tibia,
otherwise as known as a "toddler's fracture" (Item C67A). The mechanism of injury is a twisting
movement of the lower leg that occurs as the toddler learns to move around his or her
environment. Examination often reveals only a limp or tenderness over the affected area. Initial
radiographs can appear normal, with a periosteal reaction appearing in follow-up films. Torus
(buckle) and greenstick (Item C67B) fractures also can be seen with relatively minor trauma
because children's bones are more porous and more likely to "buckle" or bend with trauma than
adult bones. Fractures of the growth plate are common with trauma and are particularly
concerning because of the risk of growth retardation at the site of the fracture. The Salter-Harris
system is used to classify types of growth plate fractures and identify patients who are at high
risk of growth failure. Finally, some types of fractures should alert the clinician immediately to the
possibility of child abuse, including multiple fractures in different stages of healing, metaphyseal
"chip" fractures (Item C67C), femoral fracture in children younger than 1 year of age (Item
C67D), and scapular fracture.
The minimal external signs and no constitutional symptoms such as fever reported for the
child in the vignette make the diagnosis of osteomyelitis unlikely. Transient synovitis occurs after
a viral illness and generally affects the knees and hips. Some limitation of motion at the joint is
expected, but external signs such as erythema typically are absent. Aneurysmal bone cysts are
metaphyseal lesions (Item C67E) that usually occur in adolescence and cause pain with or
without swelling. The ligaments and tendons in children generally are stronger than the growing
bones underneath them, in contrast to adults, whose bones are stronger. The amount of injuring
force required to cause a sprain in an adult is more likely to cause a fracture in a child.
References:
Dinolfo EA. Fractures. Pediatr Rev. 2004;25:218-219. Available at:
http://pedsinreview.aappublications.org/cgi/content/full/25/6/218
Eiff MP, Hatch RL. Boning up on common pediatric fractures. Contemp Pediatr. 2003;20:30-59
Kellogg ND and the Committee on Child Abuse and Neglect. Evaluation of suspected child
physical abuse. Pediatrics. 2007;119:1232-1241. Available at:
http://pediatrics.aappublications.org/cgi/content/full/119/6/1232

Copyright 2009 by the American Academy of Pediatrics

page 144

2009 PREP SA on CD-ROM


Question: 68

An 11-year-old girl presents 2 weeks after an office visit for a presumed viral illness
characterized by fever, malaise, and flushing of the cheeks. Today, her mother notes that she
no longer has a fever, but she complains of pain in her knees and elbows. On physical
examination, the left knee is slightly swollen and warm but not erythematous. The girl reports
pain on movement of both elbows, but there are no physical findings on examination of the
elbows or other joints. The remainder of the physical examination findings are normal, except for
an oral temperature of 100.6F (38.1C). Results of laboratory studies include a white blood cell
count of 8.9x103/mcL (8.9x109/L) with 40% polymorphonuclear leukocytes, 45% lymphocytes,
and 15% monocytes; hemoglobin of 11.0 g/dL (110.0 g/L); platelet count of 472.0x103/mcL
(472.0x109/L); and erythrocyte sedimentation rate of 20 mm/hr.
Of the following, the MOST likely pathogen to cause this childs joint complaints is

A. Borrelia burgdorferi
B. Coxsackievirus
C. group A beta-hemolytic streptococci
D. influenza A virus
E. parvovirus B19

Copyright 2009 by the American Academy of Pediatrics

page 145

2009 PREP SA on CD-ROM


Critique: 68

Preferred Response: E

The girl described in the vignette has developed swelling of the knee and arthralgias following a
recent febrile illness, which is typical of a postinfectious arthritis. The flushing of her cheeks
suggests that her febrile illness was due to human parvovirus B19 infection, commonly known
as erythema infectiosum (EI) (Item C68). Parvovirus B19 infection is a common cause of
postinfectious arthritis. Multiple viruses can cause postinfectious arthritis, including influenza,
hepatitis B, rubella, and Epstein-Barr.
Arthralgias may occur in 10% of children who have clinical or laboratory evidence of EI.
Older children, particularly girls and young women, frequently experience involvement of the
knees, although involvement of both large and small joints has been reported. Parvovirus B19
has been detected in synovial fluid and serum samples of such patients. Some clinical features
of parvovirus B19 infection are similar to those of autoimmune connective tissue diseases, and
some children who have EI may develop positive antinuclear antibody serum test results or
rheumatoid factor-positive serology.
Lyme disease results from infection with the spirochete Borrelia burgdorferi, which is
transmitted by deer tick bites. In North America, Lyme disease is most prevalent in the
northeastern, midwestern, and southern and western coastal areas of the United States as well
as in Ontario, Canada. School-age children are affected most commonly, with boys and girls
affected equally. Arthritis is the second most frequent presentation of Lyme disease, following
the cutaneous signs of erythema migrans. Arthralgias usually develop in the early phase, but the
onset of arthritis may occur months to years after the original infection. Initially, the arthritis is
episodic, but it may evolve to a recurrent and prolonged condition. Two thirds of affected
children present with monoarthritis of the knee, but oligoarticular involvement of the large joints
and, rarely, a polyarthritis of the small joints also can occur.
The diagnosis of Lyme arthritis is based on history and physical examination findings as well
as laboratory tests to document infection with B burgdorferi. The immunoglobulin G titers to B
burgdorferi can remain positive for years and, therefore, cannot be used to monitor treatment
response or failure. In contrast to adults, the prognosis for Lyme arthritis in children generally is
good, and symptoms resolve over time without permanent damage to joints.
Group A beta-hemolytic streptococcal infection can be associated with arthritis that may
occur during the acute illness or after the acute illness has resolved (poststreptococcal reactive
arthritis). Other bacterial causes of postinfectious arthritis include Neisseria gonorrhoeae,
Staphylococcus aureus, and other streptococcal species. Coxsackieviruses are not often
associated with arthritis. Enterovirus, hepatitis B, rubella, and mumps infections may cause
transient arthritis. Many other infections result in malaise and myalgias associated with a
prodrome, but they do not cause true arthritis.
References:
American Academy of Pediatrics. Enterovirus (nonpoliovirus) infections (group A and B
Coxsackieviruses, echoviruses, numbered enteroviruses. In: Pickering LK, Baker CJ, Long SS,
McMillan JA, Eds. Red Book: 2006 Report of the Committee on Infectious Diseases. 27th ed. Elk
Grove Village, Ill: American Academy of Pediatrics; 2006:284-285
American Academy of Pediatrics. Group A streptococcal infections. In: Pickering LK, Baker CJ,
Long SS, McMillan JA, Eds. Red Book: 2006 Report of the Committee on Infectious Diseases.
27th ed. Elk Grove Village, Ill: American Academy of Pediatrics; 2006:610-620
American Academy of Pediatrics. Influenza. In: Pickering LK, Baker CJ, Long SS, McMillan JA,
Eds. Red Book: 2006 Report of the Committee on Infectious Diseases. 27th ed. Elk Grove
Village, Ill: American Academy of Pediatrics; 2006:401-411
American Academy of Pediatrics. Parvovirus B19/erythema infectiosum, fifth disease). In:
Pickering LK, Baker CJ, Long SS, McMillan JA, Eds. Red Book: 2006 Report of the Committee
on Infectious Diseases. 27th ed. Elk Grove Village, Ill: American Academy of Pediatrics;

Copyright 2009 by the American Academy of Pediatrics

page 146

2009 PREP SA on CD-ROM

2006:484-487
Khouqeer R, Cohen M. Viral arthritis. eMedicine Specialties, Rheumatology, Infectious Arthritis.
2006. Available at: http://www.emedicine.com/med/topic3414.htm
Koch WC. Parvovirus B19. In: Kliegman RM, Behrman RE, Jenson HB, Stanton BF, eds.
Nelson's Textbook of Pediatrics. 18th ed. Philadelphia, Pa: Saunders Elsevier; 2007:1357-1359
Lehman HW, Knll, A, Kster RM, Modrow S. Frequent infection with a viral pathogen,
parvovirus B19, in rheumatic diseases of childhood. Arthritis Rheum. 2003;48:1631-1638.
Available at: http://www3.interscience.wiley.com/cgi-bin/fulltext/104536478/HTMLSTART
Siegel DM. In brief: antinuclear antibody (ANA) testing. Pediatr Rev. 2003;24:320-321. Available
at: http://pedsinreview.aappublications.org/cgi/content/full/24/9/320
Tse SML, Laxer RM. Approach to acute limb pain in childhood. Pediatr Rev. 2006;27:170-180.
Available at: http://pedsinreview.aappublications.org/cgi/content/full/27/5/170

Copyright 2009 by the American Academy of Pediatrics

page 147

2009 PREP SA on CD-ROM


Question: 69

You are prescribing atenolol for a 15-year-old boy in whom you diagnosed hypertrophic
cardiomyopathy. There is a family history of asthma. He is concerned about the potential
adverse effects of medicines.
Of the following, a TRUE statement about treatment with this drug is that

A. he cannot use atenolol because of his family history of asthma


B. he has an increased risk for developing diabetes mellitus
C. he may develop gynecomastia
D. his school performance may be adversely affected
E. tachyphylaxis may develop over time

Copyright 2009 by the American Academy of Pediatrics

page 148

2009 PREP SA on CD-ROM


Critique: 69

Preferred Response: D

Beta-blocking drugs have an antagonistic effect on the beta-adrenergic receptors. This diverse
and widely used group of medications can have multiple and various effects on the heart and
other organ systems. In the heart, they typically exhibit some degree of negative chronotropic
(slowing of the heart rate), negative dromotropic (slowing of the conduction through the
atrioventricular node), and negative inotropic (decrease in the ventricular force of contraction)
effects. As a result, this class of medications is used in the treatment of many pediatric
conditions, including hypertension, arrhythmias, hypertrophic cardiomyopathy, dilated
cardiomyopathy, Marfan syndrome, and migraine prophylaxis.
The adverse effects of beta-blocker drugs have been well-documented and may be divided
into three broad categories based on end-organ effects (Item C69).
The patient described in the vignette has been prescribed a beta blocker that has relative
cardiac selectivity. Some children and adolescents experience a decrease in school
performance while being treated with beta blockers, most likely related to the central nervous
penetration, and it may be difficult to differentiate this effect from the known fatigue or depression
that also can occur.
Although bronchospasm or asthma is a contraindication to the use of some beta blockers
(eg, propranolol), a family history alone is not a contraindication for their use. Some beta
blockers can cause a decreased reaction to hypoglycemia, leading to a relative contraindication
of their use in those who have diabetes, but their use does not increase the risk of developing
diabetes mellitus. Similarly, beta blocker use typically does not lead to gynecomastia. Although
reflex tachycardia may be noted when chronic beta blocker use is discontinued, these drugs are
not associated with tachyphylaxis.
References:
Feld LG, Corey H. Hypertension in childhood. Pediatr Rev. 2007;28:283-298. Available at:
http://pedsinreview.aappublications.org/cgi/content/full/28/8/283
Opie LH, Sonnenblick EH, Frishman WH, Thadani U. Beta-blocking agents. In: Opie LH, ed.
Drugs for the Heart. 4th ed. Philadelphia, Pa: W.B. Saunders Co; 1995:1-30

Copyright 2009 by the American Academy of Pediatrics

page 149

2009 PREP SA on CD-ROM


Question: 70

A 14-year-old girl is brought to the emergency department because she has back pain and a
sudden inability to walk. Neurologic examination shows normal upper limb strength. However,
her legs are flaccid, relatively symmetrically weak, areflexic, and numb to pinprick. Vibratory and
position sense in the legs persists. A sensory deficit exists below the sixth thoracic dermatome.
Rectal examination shows low rectal tone. The remainder of her physical examination findings,
including vital signs, are normal.
Of the following, the MOST appropriate next diagnostic study is emergent

A. brain magnetic resonance imaging


B. lumbar puncture
C. nerve conduction/electromyography of the legs
D. somatosensory evoked potentials
E. spine magnetic resonance imaging

Copyright 2009 by the American Academy of Pediatrics

page 150

2009 PREP SA on CD-ROM


Critique: 70

Preferred Response: E

The child described in the vignette has findings consistent with an acute spinal cord lesion,
which is a neurologic emergency. In some cases, emergency neurosurgery is needed for
lesions causing acute spinal cord compression, and the initial diagnostic emphasis should be
directed toward identifying such lesions. The first step is to localize the problem to the proper
level of the nervous system: brain, brainstem/cerebellum, spinal cord, nerve, junction, or
muscle. Such localization not only allows for urgent appropriate diagnosis and treatment but can
reduce discomfort, risks, and costs of inappropriate diagnostic testing. Urgent phone or inperson consultation with a neurologist or neurosurgeon can be useful before ordering testing.
The back pain, acute bilateral flaccid weakness, and sensory loss below the level of the
lesion described for the girl in the vignette localize this problem to the spinal cord. In this case,
the rapid onset of symptoms and the preservation of vibratory and proprioceptive sensation
point to the anterior cord, bilaterally, which is consistent with an anterior spinal artery stroke.
Spinal cord strokes in children are uncommon but can occur after aortic surgery or as a
consequence of thrombotic disorders, infection, inflammatory diseases, or trauma.
Based on the history and examination findings, magnetic resonance imaging (MRI) of the
spine is the test of choice. This should aid in determining the specific location of the lesion and
whether emergency neurosurgery to decompress the spinal cord is needed.
Brain MRI is not the initial modality of choice because of the urgency of the presentation and
the localization of the lesion suggested by clinical findings. If a spinal cord lesion is radiologically
consistent with the diagnosis of transverse myelitis, a subsequent brain MRI with gadolinium
should be obtained to look for evidence of demyelinating lesions in the brain, which is supportive
of a diagnosis of multiple sclerosis.
A subsequent lumbar puncture also may help identify the cause of the patient's symptoms,
but it may not be necessary and should not delay obtaining the spine MRI. Lumbar puncture is
the appropriate test for suspected Guillain-Barr syndrome, an acute inflammatory
demyelinating polyneuropathy, which can present with weakness beginning in the proximal legs
but not with the sensory findings seen in this child. Lumbar puncture is also helpful for identifying
inflammatory causes of spinal cord disease. Somatosensory evoked potentials are
neurophysiologic tests that document the integrity of sensory pathways from peripheral nerves
through the spinal cord to the brain. They are not part of standard emergency management in
this clinical setting. Nerve conduction studies and electromyography are not needed to assess
the nerves because the problem localizes to spinal cord.
References:
Hakimi KN, Massagli TL. Anterior spinal artery syndrome in two children with genetic thrombotic
disorders. J Spinal Cord Med. 2005;28:69-73. Abstract available at:
http://www.ncbi.nlm.nih.gov/pubmed/15832907
Haslam RHA. Spinal cord disorders. In: Kliegman RM, Behrman RE, Jenson HB, Stanton BF,
eds. Nelson Textbook of Pediatrics. 18th ed. Philadelphia, Pa: Saunders Elsevier; 2007:25262530
Menkes JH, Ellenbogen RC. Traumatic brain and spinal cord injuries in children. In: Maria BL,
ed. Current Management in Child Neurology. 3rd ed. Hamilton, Ontario, Canada: BC Decker Inc;
2005:515-527
Nance JR, Golomb MR. Ischemic spinal cord infarction in children without vertebral fracture.
Pediatr Neurol. 2007;36:209-216. Abstract available at:
http://www.ncbi.nlm.nih.gov/pubmed/17437902

Copyright 2009 by the American Academy of Pediatrics

page 151

2009 PREP SA on CD-ROM


Question: 71

An infant in the newborn nursery is normally grown and normally formed, except for a
preauricular pit (preauricular sinus) bilaterally (Item Q71). He has passed his newborn hearing
screening. When you meet the babys mother, you learn that she has progressive, bilateral
sensorineural hearing loss for which she uses hearing aids.
Of the following, the MOST helpful test to aid in diagnosis and management of this babys
condition is

A. chromosome analysis
B. head ultrasonography
C. ophthalmology consultation
D. radiographic skeletal survey
E. renal ultrasonography

Copyright 2009 by the American Academy of Pediatrics

page 152

2009 PREP SA on CD-ROM


Critique: 71

Preferred Response: E

For many years, the association between ear anomalies and other malformations, especially
involving the kidneys, have been noted in the medical literature. Based on numerous studies, the
documented frequency of defects in other organ systems is 5% to 40% in children who have ear
anomalies.
Largely due to the association between crumpled, deformed ears and renal agenesis (as
seen in Potter sequence), it has become customary in many centers to evaluate the kidneys in
the newborn, even if only a preauricular pit/sinus (PPS) is noted (Item C71). However, recent
studies have shown that PPS alone is not associated with an increased risk for renal anomalies,
and renal ultrasonography is not warranted in these cases.
It is critical to evaluate newborns and children who have auricular anomalies carefully for
dysmorphic features and evidence of organ malformations. It is important to know if the mother
has diabetes because diabetic embryopathy is associated with ear anomalies. In addition, a
family history should be obtained, with special attention to any individuals who have ear
anomalies, deafness, and renal malformations. Should such a history be elicited, as for the
mother in the vignette, renal ultrasonography is warranted, and further evaluations may be in
order. Chromosome analysis, head ultrasonography, ophthalmology consultation, and
radiographic skeletal survey are not necessary initial tests for a child who has auricular
anomalies.
References:
Adam M, Hudgins L. The importance of minor anomalies in the evaluation of the newborn.
NeoReviews. 2003;4:e99-e104. Available for subscription at:
http://neoreviews.aappublications.org/cgi/content/full/4/4/e99
Arora RS, Pryce R. Is ultrasonography required to rule out renal malformations in babies with
isolated preauricular tags? Arch Dis Child. 2004;89:492-493
Huang XY, Tay GS, Wansaicheong GK-L, Low WK. Preauricular sinus: clinical course and
associations. Arch Otolaryngol Head Neck Surg. 2007;133:65-68. Available at:
http://archotol.ama-assn.org/cgi/content/full/133/1/65
Wang RY, Earl DL, Ruder RO, Graham JM Jr. Syndromic ear anomalies and renal ultrasounds.
Pediatrics. 2001;108:e32. Available at:
http://pediatrics.aappublications.org/cgi/content/full/108/2/e32

Copyright 2009 by the American Academy of Pediatrics

page 153

2009 PREP SA on CD-ROM


Question: 72

A 16-year-old girl comes to your office with complaints of a thick white vaginal discharge. She is
sexually active with one partner with whom she always uses condoms. She has no complaints
of fever or abdominal pain, but she reports external "burning" of the vaginal area when she
urinates. On physical examination, she is afebrile. Pelvic examination reveals fiery red labia
majora and minora and an adherent white discharge on the vaginal walls, with a moderate
amount of white discharge in the vaginal vault. The speculum examination is uncomfortable for
her, but there is no cervical motion, uterine, or adnexal tenderness, and the cervix shows no
friability or discharge.
Of the following, the MOST likely pathogen responsible for this patients symptoms is

A. Candida albicans
B. Chlamydia trachomatis
C. group A Streptococcus
D. Neisseria gonorrhoeae
E. Trichomonas vaginalis

Copyright 2009 by the American Academy of Pediatrics

page 154

2009 PREP SA on CD-ROM


Critique: 72

Preferred Response: A

Candidal vulvovaginitis is a common problem for young women and usually is caused by
Candida albicans, although other candidal species also may be involved. Typical symptoms of
vulvovaginal candidiasis include a thick, white, creamy vaginal discharge; pruritus; vaginal
discomfort; dyspareunia; and external dysuria. The diagnosis is suggested clinically by the
previously noted symptoms and the presence of vulvar swelling, erythema, and fissures or
erosions, as described for the girl in the vignette. The diagnosis may be confirmed by a wet
preparation or Gram stain showing pseudohyphae or yeasts (Item C72A) or by culture.
Chlamydia trachomatis and Neisseria gonorrhoeae, both of which may present with
abnormal vaginal discharge, produce cervical, not vaginal infections. C trachomatis and N
gonorrhoeae infections can be asymptomatic, but they often present with a yellowish purulent or
mucopurulent endocervical discharge, friability of the endocervix, and cervical motion
tenderness; vulvar inflammation does not occur. Group A Streptococcus (S pyogenes) is a
respiratory pathogen that can cause vaginitis in prepubertal girls, but rarely causes vaginal
discharge in the adolescent. Infection with Trichomonas vaginalis usually causes a diffuse,
malodorous, yellow-green vaginal discharge (Item C72B) with vulvar irritation, although some
affected women can have minimal or no symptoms.
References:
American Academy of Pediatrics. Chlamydia trachomatis. In: Pickering LK, Baker CJ, Long SS,
McMillan JA, eds. Red Book: 2006 Report on the Committee on Infectious Diseases. 27th ed.
Elk Grove Village, Ill: American Academy of Pediatrics; 2006:252-257
American Academy of Pediatrics. Gonococcal infections. In: Pickering LK, Baker CJ, Long SS,
McMillan JA, eds. Red Book: 2006 Report on the Committee on Infectious Diseases. 27th ed.
Elk Grove Village, Ill: American Academy of Pediatrics; 2006:301-309
American Academy of Pediatrics. Group A streptococcal infections. In: Pickering LK, Baker CJ,
Long SS, McMillan JA, eds. Red Book: 2006 Report on the Committee on Infectious Diseases.
27th ed. Elk Grove Village, Ill: American Academy of Pediatrics; 2006:610-620
Emans SJ. Office evaluation of the child and adolescent. In: Emans SJH, Laufer MR, Goldstein
DP, eds. Pediatric and Adolescent Gynecology. 5th ed. Philadelphia, Pa: Lippincott, Williams &
Wilkins; 2005:1-50
Workowski KA, Berman SM, Centers for Disease Control and Prevention. Sexually transmitted
diseases treatment guidelines, 2006. MMWR Recomm Rep. 2006;55(RR11):1-94. Available at:
http://www.cdc.gov/mmwr/preview/mmwrhtml/rr5511a1.htm

Copyright 2009 by the American Academy of Pediatrics

page 155

2009 PREP SA on CD-ROM


Question: 73

An 18-month-old boy fell into a swimming pool 12 hours ago. He had no heart rate when he was
pulled from the pool, and cardiopulmonary resuscitation (CPR) was initiated at the scene. The
CPR was continued for 30 minutes until spontaneous circulation was restored in the emergency
department. He is now in the pediatric intensive care unit, receiving mechanical ventilation with
maximal intensive care support. Over the past several hours, his blood pressure has increased,
he has developed persistent bradycardia, and he exhibits no movement in response to
stimulation. He has not received any neuromuscular blockers or sedation. In addition, his pupils
are dilated bilaterally and do not respond to light. Bedside electroencephalography demonstrates
generalized burst suppression with loss of reactivity to external stimuli.
In discussion with his parents, you inform them that these recent changes are MOST likely a
result of

A. agitation
B. increasing intracranial pressure
C. myocardial failure
D. ongoing seizure activity
E. physiologic response to the ventilator

Copyright 2009 by the American Academy of Pediatrics

page 156

2009 PREP SA on CD-ROM


Critique: 73

Preferred Response: B

Hypoxic-ischemic injury, which occurs after periods of impaired perfusion, such as with
cardiopulmonary arrest, produces a broad spectrum of organ failure. Unfortunately, the brain is
one of the organs most vulnerable to decreased oxygen and nutrient delivery, and hypoxicischemic central nervous system (CNS) injury is common following asphyxia. Irreversible CNS
injury may occur after as little as 3 to 5 minutes of interrupted blood flow or oxygen delivery.
Both ischemia and hypoxia trigger numerous pathophysiologic processes that result in cellular
injury, cell death, and subsequent development of cerebral edema that compromises blood flow
to adjacent areas of the brain. These areas are initially either potentially recoverable or uninjured
but at risk due to impaired perfusion.
The boy described in the vignette is exhibiting evidence of increased intracranial pressure
due to significant global hypoxic-ischemic injury. His hypertension and bradycardia represent
the Cushing reflex (the raising of systemic arterial pressure to increase cerebral perfusion with
associated stimulation of the carotid bodies and subsequent bradycardia). Cushing triad
(systemic arterial hypertension, bradycardia, and depressed or irregular respirations) is a late
sign of increased intracranial pressure that often develops just prior to cerebral herniation. The
lack of spontaneous movements and dilated, unresponsive pupils described for the boy are
consistent with severe CNS injury and a poor outcome.
Agitation or pain due to underlying injuries, ventilator asynchrony, or anxiety can produce
hypertension, but this typically is associated with tachycardia and increased motor movements.
Myocardial hypoxic-ischemic injury normally is manifested by decreased ventricular function
and hypotension. Although patients who have hypoxic-ischemic injuries are at risk for the
development of seizures, the electroencephalographic findings in this patient are consistent with
a severe hypoxic-ischemic encephalopathy and portend a poor prognosis. Abnormal motor
movements or vital signs in combination with neurologic examination results that are not
consistent with a known cause should prompt the consideration of seizure and subsequent
evaluation and treatment.
References:
Doherty DR, Hutchison JS. Hypoxic ischemic encephalopathy after cardiorespiratory arrest. In:
Wheeler DS, Wong HR, Shanley T, eds. Pediatric Critical Care Medicine: Basic Science and
Clinical Evidence. New York, NY: Springer-Verlag; 2007:935-946
Kallas HJ. Drowning and submersion injury. In: Kliegman RM, Behrman RE, Jenson HB, Stanton
BF, eds. Nelson Textbook of Pediatrics. 18th ed. Philadelphia, Pa: Saunders Elsevier; 2007:438449
Meyer RJ, Theodorou AA, Berg RA. Childhood drowning. Pediatr Rev. 2006;27:163-169.
Available at: http://pedsinreview.aappublications.org/cgi/content/full/27/5/163

Copyright 2009 by the American Academy of Pediatrics

page 157

2009 PREP SA on CD-ROM


Question: 74

On the initial health supervision visit of a 7-year-old boy who is new to your practice, you note
that his height is 43 inches, which is at the 50th percentile for a 5-year-old, and that his weight is
appropriate for his age. His parents say that he has been wearing the same size clothes for at
least the past year. The boy also has dry skin. You suspect he has hypothyroidism and decide
to measure thyroid-stimulating hormone concentrations.
Of the following, the MOST appropriate additional study needed to evaluate this child for
hypothyroidism is

A. bone age radiography


B. measurement of free thyroxine
C. measurement of total thyroxine
D. measurement of tri-iodothyronine
E. thyroid ultrasonography

Copyright 2009 by the American Academy of Pediatrics

page 158

2009 PREP SA on CD-ROM


Critique: 74

Preferred Response: B

The initial evaluation of the boy described in the vignette should be directed at determining
whether he has hypothyroidism and defining its severity. The assay of free thyroxine (FT4)
generally provides the best estimate of circulating active concentrations of thyroxine. Total
thyroxine (TT4) concentrations are dependent on circulating thyroxine and on circulating binding
proteins, including thyroxine-binding globulin (TBG) and thyroxine-binding prealbumin (TBPA). In
general, TT4 is an accurate measure of thyroid hormone sufficiency, but about 1 in 8,000
individuals has familial sex-linked deficiency or excess of TBG and, therefore, has a TT4 value
that cannot be interpreted properly. Although hypothyroidism in children usually is caused by
primary thyroid gland failure, a substantial proportion of affected individuals could have a
hypothalamic or pituitary disease, in which case the FT4 value is low and the thyroid-stimulating
hormone value is normal or low.
Tri-iodothyronine (T3) is the active form of intracellular thyroid hormone. Only about 15% of
T3 is produced by the thyroid gland; the rest is produced by the enzymatic deiodination of T4 in
most cells of the body, including the hepatocyte. Because the process of enzymatic deiodination
is nutritionally controlled, individuals who are particularly well-fed or obese or small children who
eat frequently have elevated T3 values compared with adult normal values. In contrast, sick
people or people who are not eating well have very low values. Therefore, T3 cannot be used to
diagnose hypothyroidism.
Children who have hypothyroidism often have delayed bone age, which may be used as an
indicator of how long the hypothyroidism has been present. Although a bone age radiograph
might be part of the extended evaluation of hypothyroidism, the diagnosis first must be
established by measuring FT4.
Most children who have acquired primary hypothyroidism have chronic lymphocytic
thyroiditis, with measurable serum antibodies directed against the thyroperoxidase enzyme, the
thyroid microsomes that contain these antibodies, or thyroglobulin. In addition, they often have
palpable thyroid glands. If they do not have positive antithyroid antibodies, thyroid
ultrasonography can aid in assessing the shape and position of the thyroid gland. Some affected
children could have congenitally abnormal thyroid glands with maldescent but enough thyroid
hormone production not to become hypothyroid until early childhood. Assessment using
technetium or radioactive iodine imaging also might be appropriate.
References:
Hunter I, Greene SA, MacDonald TM, Morris AD. Prevalence and aetiology of hypothyroidism in
the young. Arch Dis Child. 2000;83:207-210. Abstract available at:
http://www.ncbi.nlm.nih.gov/pubmed/10952634
LaFranchi S. Acquired hypothyroidism in childhood and adolescence. UpToDate Online 15.3.
2008. Available for subscription at:
http://www.uptodateonline.com/utd/content/topic.do?topicKey=pediendo/4633
Ferry RJ Jr, Bauer AJ. Hypothyroidism. eMedicine Specialties, Pediatrics, Endocrinology. 2006.
Available at: http://www.emedicine.com/ped/TOPIC1141.HTM

Copyright 2009 by the American Academy of Pediatrics

page 159

2009 PREP SA on CD-ROM


Question: 75

An 8-year-old boy has difficulty in academics and a short attention span. His father states that
he had the same problems when he was a child. Physical examination reveals macrocephaly,
multiple caf au lait macules (Item Q75A) and axillary freckles (Item Q75B). Upon questioning,
the father explains that he has similar skin findings.
Of the following, the MOST likely diagnosis is

A. fragile X syndrome
B. hypomelanosis of Ito
C. neurofibromatosis type 1
D. tuberous sclerosis
E. velocardiofacial syndrome

Copyright 2009 by the American Academy of Pediatrics

page 160

2009 PREP SA on CD-ROM


Critique: 75

Preferred Response: C

The frequency of learning difficulties is increased in individuals who have a positive family history
for learning disorders or genetic syndromes such as neurofibromatosis type 1 (NF-1), fragile X
syndrome, or velocardiofacial syndrome (VCFS). It is important to obtain a family history when
evaluating children who have suspected learning disorders to ascertain if there is a possible
genetic cause. A thorough family history also may identify speech or language problems in up to
30% of first-degree relatives of children who have language delays, a risk factor for learning
difficulties.
The multiple caf au lait macules and axillary freckling described for the boy in the vignette
meet diagnostic criteria for NF-1. The offspring of an affected individual has a 50% risk of
inheriting the altered NF1 gene, which can be detected by molecular genetic testing, although
this rarely is necessary for diagnosis. Learning disabilities occur with increased frequency in
those who have a family history of NF-1, ranging from 40% to 60%.
Fragile X syndrome, the most common genetic cause of intellectual disabilities, occurs in 1
in 1,200 males and 1 in 2,500 females. A mutation in the FMR1 gene (Xq27.3) leads to
expansion of the CGG trinucleotide repeat. Women who have alleles in the range of 29 to 200
CGG repeats are at risk for giving birth to affected children. Males who have fragile X syndrome
have large heads, long faces, large ears, and macro-orchidism but no skin findings. Males who
have the full mutation have mean intelligence quotient (IQ) scores of 40; affected females have
mean IQ scores in the low-average to borderline range.
Velocardiofacial syndrome affects 1 in 400 individuals, and 10% of cases are inherited in an
autosomal dominant pattern. The condition is caused by a microdeletion on chromosome
22q11.2 and may be associated with cardiac and facial anomalies, but not cutaneous
abnormalities. Learning difficulties are reported in 82% to 100% of affected children.
Hypomelanosis of Ito is a congenital skin disorder that is characterized by linear or whorled
hypopigmentation that follows the lines of Blaschko, the paths of embryonic neural crest cell
migration (Item C75A). There is no evidence for genetic transmission. Associated intellectual
disability has been reported in as many as 70% of cases, seizures in 40%, and microcephaly in
25%. However, these high percentages may be the result of selection bias, and the actual
frequency of associated abnormalities may be lower.
Tuberous sclerosis complex (TSC) is inherited in an autosomal dominant pattern, but as
many as two thirds of cases represent new mutations. TSC may present during infancy with
infantile spasms and a hypsarrhythmic electroencephalographic pattern. Autism is seen 1% of
affected individuals. The most common skin finding is hypopigmented macules that have been
likened to an ash leaf; they are seen in more than 90% of affected individuals (Item C75B).
References:
Haslam RHA. Neurocutaneous syndromes. In: Kliegman RM, Behrman RE, Jenson HB, Stanton
BF, eds. Nelson Textbook of Pediatrics. 18th ed. Philadelphia, Pa: Saunders Elsevier; 2007:24832488
Kates WR, Antshel KM, Femont W, Roizen, NJ, Shprintzen RJ. Velocardiofacial syndrome. In:
Accardo PJ, ed. Capute & Accardo's Neurodevelopmental Disabilities in Infancy and Childhood.
Volume II: The Spectrum of Neurodevelomental Disabilities. 3rd ed. Baltimore, Md: Paul H.
Brookes Publishing Co; 2008:363-373
Lyon GR, Shaywitz SE, Shaywitz BA. Dyslexia. In: Kliegman RM, Behrman RE, Jenson HB,
Stanton BF, eds. Nelson Textbook of Pediatrics. 18th ed. Philadelphia, Pa: Saunders Elsevier;
2007:150-151
Morelli JG. Hyperpigmented lesions. In: Kliegman RM, Behrman RE, Jenson HB, Stanton BF,
eds. Nelson Textbook of Pediatrics. 18th ed. Philadelphia, Pa: Saunders Elsevier; 2007:26822685

Copyright 2009 by the American Academy of Pediatrics

page 161

2009 PREP SA on CD-ROM

National Fragile X Foundation Web site. Available at: http://www.fragilex.org/


Nowicki ST, Hansen RL, Hagerman RJ. X-linked intellectual disabilities In: Accardo PJ, ed.
Capute & Accardo's Neurodevelopmental Disabilities in Infancy and Childhood. Volume II: The
Spectrum of Neurodevelomental Disabilities. 3rd ed. Baltimore, Md: Paul H Brookes Publishing
Co; 2008:331-351

Copyright 2009 by the American Academy of Pediatrics

page 162

2009 PREP SA on CD-ROM


Question: 76

A 6-year-old girl presents with a history of swelling on her jaw of 1 months duration. The mother
has been to a "couple of emergency rooms," but nobody can tell her what is wrong with the girl.
The childs father died about 3 years ago from pneumonia, and the mother reports that she has
"no energy," but she has not sought medical care. The mother states that her daughter has
been fairly healthy except for frequent ear infections. On physical examination, the girl is afebrile;
her weight is 16 kg (3rd percentile); her height is 105 cm (3rd percentile); and she has scarred
tympanic membranes, bilateral parotid swelling, mild clubbing, and some fine crackles on lung
examination.
Of the following, the MOST likely diagnosis is

A. bacterial parotitis
B. common variable immunodeficiency
C. human immunodeficiency virus infection
D. lymphoma
E. mumps

Copyright 2009 by the American Academy of Pediatrics

page 163

2009 PREP SA on CD-ROM


Critique: 76

Preferred Response: C

The chronic parotid swelling, clubbing of the digits, history of chronic otitis media, and paternal
and maternal illness described for the girl in the vignette are most characteristic of human
immunodeficiency virus (HIV) infection. Among HIV-infected children, digital clubbing and
wheezing are due to a chronic lung disease known as lymphocytic interstitial pneumonitis.
Patients who have bacterial parotitis usually are febrile and have purulent material coming
from Stensen's duct (Item C76A). Although patients who have HIV infection may have
lymphoma, the presentation usually is not bilateral and is much less common than chronic
parotitis. Sinopulmonary infections can occur with common variable immunodeficiency (CVID),
but HIV is more common and also is more likely to affect both parents, unlike CVID.
Mumps can present with parotid swelling (Item C76B) but is an uncommon infection in
developed countries, and the duration of the swelling is not long. The parotid swelling associated
with mumps usually peaks in 3 days and resolves over the next 7 days. If mumps is being
considered, the diagnosis can be confirmed by isolation of the virus in cell culture from throat
washings, saliva, urine, or cerebrospinal fluid. In addition, a positive mumps immunoglobulin M
titer, mumps polymerase chain reaction test, or a significant increase between acute and
convalescent serum mumps titers can help establish the diagnosis.
References:
American Academy of Pediatrics. Mumps. In: Pickering LK, Baker CJ, Long SS, McMillan JA,
eds. Red Book: 2006 Report of the Committee on Infectious Diseases. 27th ed. Elk Grove
Village, Ill: American Academy of Pediatrics; 2006:464-468
Burchett SK, Pizzo PA. HIV infection in infants, children, and adolescents. Pediatr Rev.
2003;24:186-194. Available at: http://pedsinreview.aappublications.org/cgi/content/full/24/6/186

Copyright 2009 by the American Academy of Pediatrics

page 164

2009 PREP SA on CD-ROM


Question: 77

You are evaluating a 2-year-girl who recently was adopted from Russia for a 4-day history of
temperature to 102.5F (39.2C), rash, coryza, malaise, conjunctivitis, and cough that have
worsened over the last 24 hours. She had nasal congestion and rhinorrhea for 5 days prior to
developing the fever, rash, and cough. The girl has been in the United States for 7 days. She
was adopted from a rural orphanage, where she was exposed to farm animals, but information
regarding her past medical history and immunizations is unavailable. Physical examination
shows a tired-appearing, irritable toddler who is clinging to her adopted mother. She has a
temperature of 103.0F (39.5C), bilateral conjunctival injection, profuse clear rhinorrhea, an
erythematous buccal mucosa with scattered whitish specks (Item Q77A) on the left side, and an
erythematous posterior pharynx with no tonsillar exudates. There is a confluent erythematous
maculopapular rash on her face, trunk, and abdomen (Item Q77B), with scattered patches on
her legs.
Of the following, the test MOST likely to confirm the diagnosis for this child is

A. blood culture
B. C-reactive protein measurement
C. serology
D. throat culture
E. urine culture

Copyright 2009 by the American Academy of Pediatrics

page 165

2009 PREP SA on CD-ROM


Critique: 77

Preferred Response: C

The patient described in the vignette has history and physical examination findings that are
classic for measles. Measles is caused by the measles (rubeola) virus, which is an RNA virus in
the Paramyxoviridae family. The simplest and preferred method for establishing the diagnosis is
by demonstrating a positive serologic test for measles immunoglobulin M (IgM) antibody on a
single serum specimen obtained during the first encounter with a person suspected of having
the disease. Measles IgM antibody is detectable for at least 1 month after the onset of the rash.
The measles vaccine alone does not give an IgM response. Other methods of diagnosis include
detection of measles virus antigen in respiratory epithelial cells or tissue by immunofluorescent
methods and detection of viral genome by polymerase chain reaction. Isolation of the measles
virus from blood, throat, and urine cultures is possible but is very difficult and is not readily
available. C-reactive protein is a nonspecific measurement of inflammation that is not specific for
the diagnosis of measles.
References:
American Academy of Pediatrics. Measles. In: Pickering LK, Baker CJ, Long SS, McMillan JA,
eds. Red Book: 2006 Report of the Committee on Infectious Diseases. 27th ed. Elk Grove
Village, Ill: American Academy of Pediatrics; 2006:441-452
Centers for Disease Control and Prevention. Measles, mumps, and rubella - vaccine use and
strategies for elimination of measles, rubella, and congenital rubella syndrome and control of
mumps: recommendations of the Advisory Committee on Immunization Practices (ACIP).
MMWR Recomm Rep. 1998;47(RR-8):1-57. Available at:
http://www.cdc.gov/mmwr/preview/mmwrhtml/00053391.htm
Maldonado YA. Rubeola virus (measles and subacute sclerosing panencephalitis). In: Long SS,
Pickering LK, Prober CG, eds. Principles and Practice of Pediatric Infectious Diseases. 2nd ed.
Philadelphia, Pa: Churchill Livingstone; 2003:1148-1155

Copyright 2009 by the American Academy of Pediatrics

page 166

2009 PREP SA on CD-ROM


Question: 78

You employ voiding cystourethrography (VCUG) to evaluate a 4-year-old girl who had a febrile
urinary tract infection 1 month ago. The study reveals a smooth-walled bladder, absence of
vesicoureteral reflux, and a mildly narrowed urethra.
Of the following, the MOST appropriate management in this situation is

A. administration of prophylactic antibiotics until 6 years of age


B. no treatment
C. placement of the child on a clean intermittent catheterization program
D. repeat VCUG in 6 months
E. urologic consultation for cystoscopic evaluation

Copyright 2009 by the American Academy of Pediatrics

page 167

2009 PREP SA on CD-ROM


Critique: 78

Preferred Response: B

The patient described in the vignette has had a febrile urinary tract infection (UTI), but voiding
cystourethrography (VCUG) failed to demonstrate vesicoureteral reflux. It did, however,
demonstrate a narrow urethra with an otherwise normal bladder. The question that arises is
whether the "narrow urethra" is associated with potential voiding dysfunction and development of
the UTI.
The recognition of a distal urethral ring in some females who had UTIs in the 1960s led to
the hypothesis that a narrowed urethra or "urethral stenosis" was responsible for UTIs.
However, it is now recognized that the previously termed "narrow urethra" in females is a normal
variant. Indeed, dilatations led to more invasive surgeries of the internal urethra to prevent
"restenosis." A recent survey of Fellows in the Section of Urology of the American Academy of
Pediatrics found that 61% of respondents never use dilatation under any circumstance, and only
2% perform this procedure regularly. Therefore, no treatment is needed for the child in the
vignette.
Children who have recurrent UTIs may have vesicoureteral reflux or dysfunctional voiding
with incomplete bladder emptying. More recently, greater emphasis has been placed on the
effects of pelvic floor muscle dysfunction in the child who has dysfunctional voiding and
symptoms of enuresis, UTI, and constipation. It is theorized that the pelvic floor muscles
responsible for continence in such children are hyperactive, resulting in abnormal bladder
emptying and high postvoid residual volumes. Biofeedback is being used to teach children how
to improve relaxation of pelvic floor muscles and reduce urinary retention and UTI risk.
There is no indication for prophylactic antibiotics for a child who has a single UTI and no
vesicoureteral reflux. Only children who have significant urinary retention, such as seen with
neurogenic bladder (myelomeningocele) or detrusor sphincter dyssynergia (Hinman syndrome),
warrant intermittent catheterization. Follow-up VCUG is not needed for the patient in the vignette.
Cystoscopy is not indicated in a child who has a smooth wall bladder and no urinary symptoms.
References:
Brock WA, Kaplan GW. Abnormalities of the lower urinary tract. In: Edelmann CM Jr, Bernstein
J, Meadow SR, Spitzer A, Travis LB, eds. Pediatric Kidney Disease. 2nd ed. Boston, Ma: Little,
Brown, and Company; 1992:2037-2076
McKenna PH, Herndon CD, Connery S, Ferrer FA. Pelvic floor muscle retraining for pediatric
voiding dysfunction using interactive computer games. J Urol. 1999;162:1056-1063. Abstract
available at: http://www.ncbi.nlm.nih.gov/pubmed/10458431
Metwalli AR, Cheng EY, Kropp BP, Pope JC 4th. The practice of urethral dilation for voiding
dysfunction among fellows of the Section on Urology of the American Academy of Pediatrics. J
Urol. 2002;168:1764-1767. Abstract available at: http://www.ncbi.nlm.nih.gov/pubmed/12352355

Copyright 2009 by the American Academy of Pediatrics

page 168

2009 PREP SA on CD-ROM


Question: 79

An 18-year-old girl is admitted to the hospital for intravenous therapy for a complicated urinary
tract infection that failed to respond to outpatient therapy with a sulfa-based antibiotic. Her urine
culture shows more than 100,000 colony-forming units/mL of Pseudomonas aeruginosa that is
sensitive to aztreonam and imipenem. As you take her medical history, she mentions she is
"highly allergic" to penicillin.
Of the following, a TRUE statement regarding penicillin drug reactions is that

A. a nonpruritic maculopapular rash that occurs in patients who receive amoxicillin during
mononucleosis is a contraindication for future penicillin therapy

B. aztreonam can be administered safely to patients who have a history of immunoglobulin E


(IgE)-mediated penicillin allergy

C. desensitization can be used to administer penicillin safely to patients who have experienced
Stevens-Johnson reactions to penicillin

D. skin testing to major and minor determinants of penicillin can exclude IgE-mediated and nonIgE-mediated reactions

E. a patient who can only recall a childhood history of penicillin allergy but does not remember the
details is very likely to react to future penicillin courses

Copyright 2009 by the American Academy of Pediatrics

page 169

2009 PREP SA on CD-ROM


Critique: 79

Preferred Response: B

Penicillin is the most common cause of drug-induced anaphylaxis. Because the likelihood of an
immunoglobulin (Ig) E-mediated penicillin allergy is less than 20% based on history alone,
patients often are referred to an allergist-immunologist for evaluation and skin testing. The
negative predictive value of penicillin skin testing for IgE-mediated reactions is 97% to 99%.
However, standardized skin testing materials such as benzylpenicilloyl polylysine or minor
determinants either no longer are available or are not approved by the United States Food and
Drug Administration, respectively. In addition, non-IgE-mediated penicillin reactions, such as
Stevens-Johnson syndrome (Item C79A), result in negative IgE skin tests, even though drug
administration causes adverse effects.
Because appropriate penicillin skin testing material is not available in most allergy clinics,
administration of a non-beta-lactam antibiotic is advised for the patient who has a history of
suspected IgE-mediated penicillin allergy. If a penicillin, cephalosporin, or carbapenem drug is
required, consultation with an allergist-immunologist should be considered. A detailed discussion
regarding avoidance, graded challenge, or drug desensitization usually ensues from the
consultation. Carbapenems have a high cross-reactivity with penicillin and should be avoided,
but monobactams such as aztreonam do not cross-react and may be administered safely.
Desensitization is a process that allows the gradual administration of a drug to a patient who
has an IgE-mediated drug allergy. Thus, desensitization cannot be successful for a patient who
experienced a non-IgE-mediated reaction such as Stevens-Johnson syndrome. Also, once the
desensitization is complete and the patient finishes the appropriate antibiotic therapy, he or she
still is considered "allergic" to that drug and must undergo repeat desensitizations to receive
subsequent courses of that antibiotic.
One common non-IgE-mediated reaction to penicillin occurs when an antibiotic such as
amoxicillin is administered to a patient who has an Epstein-Barr virus infection (eg, infectious
mononucleosis). Typical symptoms include a nonpruritic, maculopapular rash that occurs a few
days into therapy (Item C79B). Clinically, it may be difficult to distinguish this reaction from a true
drug allergy, but this rash is not due to an allergic reaction, and future drug use need not be
restricted.
References:
Boguniewicz M, Leung DYM. Adverse reactions to drugs. In: Kleigman RM, Behrman RE,
Jenson HB, Stanton BF, eds. Nelson Textbook of Pediatrics. 18th ed. Philadelphia, Pa:
Saunders Elsevier; 2007:990-994
Wolf R, Orion E, Marcos B, Matz H. Life-threatening acute adverse cutaneous drug reactions.
Clin Dermatol. 2005;23:171-181. Abstract available at:
http://www.ncbi.nlm.nih.gov/pubmed/15802211

Copyright 2009 by the American Academy of Pediatrics

page 170

2009 PREP SA on CD-ROM


Question: 80

A 5-year-old boy is brought to the emergency department because of a nose bleed that has
lasted 1} hours. His mother reports that he has had nose bleeds in the past that usually
stopped when she pinched his nose, but this time he continued to bleed. She says that he does
pick his nose and that he has had cold symptoms for the past 3 days. There is no family history
of bleeding disorders, and he had no excessive bleeding after circumcision. On physical
examination, the awake, alert, and anxious patient is holding a bloody washcloth to his nose. His
heart rate is 140 beats/min, respiratory rate is 24 breaths/min, blood pressure is 100/60 mm Hg,
and oxygen saturation is 98%. There is active bleeding from his right naris, but an active anterior
bleeding site is not visible. Bleeding is controlled with phenylephrine instillation and packing.
Of the following, the MOST appropriate further evaluation is

A. chest radiograph
B. computed tomography scan of the sinuses
C. magnetic resonance arteriography of the sinuses
D. nasopharyngoscopy
E. no further evaluation

Copyright 2009 by the American Academy of Pediatrics

page 171

2009 PREP SA on CD-ROM


Critique: 80

Preferred Response: D

Nosebleeds are common in children, most often associated with trauma (eg, nose-picking),
mucosal friability due to upper respiratory tract infections, and mucosal drying related to
environmental conditions. Most epistaxis episodes are self-limited and require only bleeding
control instructions and comfort care. Recurrent nosebleeds are seen in fewer than 10% of
patients and only rarely are related to an underlying anatomic or hematologic abnormality.
Evidence suggests that antibacterial creams instilled in the affected naris may decrease the
incidence of recurrent idiopathic epistaxis as the child "outgrows" the predisposition.
Patients who have nosebleeds that recur frequently or are difficult to control or localize,
have a family history suggestive of a bleeding disorder, or have signs and symptoms suggestive
of a blood dyscrasia (eg, petechiae) may require further evaluation for an underlying cause.
Evaluation should be directed at likely causes, including vascular anomalies, foreign bodies,
benign and malignant masses, and primary or secondary bleeding diatheses such as
thrombocytopenia, von Willebrand disease, platelet dysfunction, hemophilias, leukemia, liver
disease, or those related to medications.
The history and physical examination should guide further evaluation. The inability to localize
the bleeding to the anterior vestibule for the boy described in the vignette suggests bleeding from
the posterior nasopharynx. Ninety percent of epistaxis events result from injury to the
Kiesselbach vascular plexus in the anterior nasal septum, and the bleeding site should be visible
on nasal inspection. Bleeding from the anterior or posterior ethmoidal or sphenopalatine arteries
in the posterior nasopharynx is more difficult to see and control. Therefore, nasopharyngoscopy
is indicated to identify the site and to aid in directing treatment. If a mass or vascular anomaly is
identified, computed tomography scan or magnetic resonance angiography/magnetic resonance
venography should be undertaken. A chest radiograph is not likely to provide information that
would guide evaluation or treatment of epistaxis.
When a bleeding disorder is suspected, the initial hematologic evaluation should include
complete blood and platelet counts, with evaluation of the blood smear, prothrombin time, and
partial thromboplastin time and typing and cross-matching if transfusion is anticipated. If
abnormalities are identified, consultation with a hematologist can help to guide further evaluation,
which may include a closure test to evaluate for platelet dysfunction and von Willebrand factor or
ristocetin cofactor assays to assess for von Willebrand disease. Of note, in one study of
patients who had recurrent epistaxis and were referred to a hematologist, one third ultimately
were diagnosed with bleeding disorders, and 50% of these patients had von Willebrand disease.
Family history was the only predictor that identified patients who had primary bleeding diatheses.
References:
Haddad J Jr. Acquired disorders of the nose. In: Kleigman RM, Behrman RE, Jenson HB,
Stanton BF, eds. Nelson Textbook of Pediatrics. 18th ed. Philadelphia, Pa: Saunders Elsevier;
2007:1744-1745
McGarry G. Nosebleeds in children. BMJ Clinical Evidence. 2006. Available for subscription at:
http://clinicalevidence.bmj.com/ceweb/conditions/chd/0311/0311.jsp
Messner AH. Epidemiology and etiology of epistaxis in children. UpToDate Online 15.3. 2008.
Available for subscription at: http://www.utdol.com/utd/content/topic.do?topicKey=ped_lryn/5986
Messner AH. Evaluation of epistaxis in children. UpToDate Online 15.3. 2008. Available for
subscription at:
http://www.utdol.com/utd/content/topic.do?topicKey=ped_lryn/6248&selectedTitle=4~150&sourc
e=search_result
Sandoval C, Dong S, Visintainer P, Ozkaynak MF, Jayabose S. Clinical and laboratory features
of 178 children with recurrent epistaxis. J Pediatr Hematol Oncol. 2002;24:47-49. Abstract
available at: http://www.ncbi.nlm.nih.gov/pubmed/11902740

Copyright 2009 by the American Academy of Pediatrics

page 172

2009 PREP SA on CD-ROM


Question: 81

A 4-month-old infant who has gastroschisis underwent surgical repair on the first day after birth,
but continues to require support with parenteral nutrition and lipids. He now has developed poor
feeding, irritability, and progressive diarrhea. Radiography demonstrates metaphyseal fraying,
but calcium, phosphorus, and 25-hydroxyvitamin D concentrations are normal. When you
review his prior laboratory studies, you note he has had neutropenia for the past 4 weeks.
Of the following, this childs symptoms are MOST consistent with

A. copper deficiency
B. magnesium deficiency
C. vitamin A deficiency
D. vitamin B6 deficiency
E. zinc deficiency

Copyright 2009 by the American Academy of Pediatrics

page 173

2009 PREP SA on CD-ROM


Critique: 81

Preferred Response: A

The poor feeding, irritability, metaphyseal irregularity, normal vitamin D value, and neutropenia
described for the infant in the vignette are consistent with copper deficiency, a rare disorder that
develops when children receive parenteral nutrition without trace element supplementation.
Copper is a trace element that has many important biochemical functions. It is an important
component of respiratory chain enzymes (eg, cytochrome C) and lysyl oxidase (an enzyme
critical to collagen production and bone formation). Menkes disease, a rare syndrome involving
a defect in copper transport and profoundly low serum copper concentrations, is characterized
by hypotonia, developmental delay, seizures, and "steely hair." Copper deficiency also may be
seen in preterm infants who do not receive appropriate trace element supplementation. The
clinical presentation of copper deficiency in the preterm infant is more subtle, but can include
pallor, poor feeding, hypochromic anemia, neutropenia, and skeletal changes (including
metaphyseal fraying and osteoporosis). The patient's symptoms are more consistent with
copper deficiency than with magnesium, vitamin B6, zinc, or vitamin A deficiency. Specifically,
magnesium deficiency can cause hypotonia and apnea, vitamin B6 deficiency can cause
hypotonia and seizures, zinc deficiency can cause diarrhea and skin rashes, and vitamin A
deficiency can lead to corneal lesions and impaired vision.
To prevent copper deficiency, most parenteral nutrition is supplemented with 200 mcg/L of
copper. Infants who have cholestasis may have impaired copper excretion into the bile,
necessitating a decrease in the concentration of copper in the parenteral nutrition to prevent
copper overload.
References:
Collier S, Gura KM, Richardson D, Duggan C. Parenteral nutrition. In: Hendricks KM, Duggan C.
Manual of Pediatric Nutrition. 4th ed. Hamilton Ontario, Canada: BC Decker; 2005:317-375
Giles E, Doyle LW. Copper in extremely low-birthweight or very preterm infants. NeoReviews.
2007;8:e159-e164. Available for subscription at:
http://neoreviews.aappublications.org/cgi/content/full/8/4/e159

Copyright 2009 by the American Academy of Pediatrics

page 174

2009 PREP SA on CD-ROM


Question: 82

You are counseling a 23-year-old woman who has diabetes mellitus and has been your patient
for the past 18 years. She recently found out that she is pregnant and asks you about potential
complications for her unborn child.
Of the following, the MOST likely complications to expect for this womans child are

A. hyperacusis, hypercalcemia, hydronephrosis


B. hyperbilirubinemia, hypercalcemia, polydactyly
C. hyperglycemia, hypocalcemia, polysplenia
D. hypoglycemia, hypocalcemia, polycythemia
E. hypogonadism, hypocalcemia, polyuria

Copyright 2009 by the American Academy of Pediatrics

page 175

2009 PREP SA on CD-ROM


Critique: 82

Preferred Response: D

Disorders of glucose regulation such as diabetes mellitus may complicate as many as 5% of


pregnancies. This includes women who develop diabetes (insulin resistance) during pregnancy,
known as gestational diabetes; those who have pre-existing insulin resistance (type 2 diabetes);
and those who have pre-existing insulin-dependent diabetes mellitus (IDDM) (type 1 diabetes).
The pregnant woman described in the vignette has IDDM. Risks posed to her developing fetus
and newborn are numerous (Item C82).
Postnatal hypoglycemia frequently is encountered in the first 4 to 12 hours in infants of
diabetic mothers (IDMs) and is related to macrosomia, an increased metabolic rate, and fetal
hyperinsulinemia that takes a few days after birth to diminish. Postnatal hypocalcemia results
from the effects of poor late-trimester transfer of calcium across the placenta in pregnancies
affected by diabetes, a delay in normal postnatal parathyroid hormone elevation, and poor fetal
and neonatal bone mineralization (ie, poor calcium stores). If hypomagnesemia is found in the
IDM, it must be corrected to allow normal parathyroid function to resume. Polycythemia
(hematocrit >65% [0.65]) represents the fetal response to its increased metabolic rate and a
relative fetal hypoxemia in utero when pregnancy is complicated by diabetes and fetal
macrosomia. Resultant hyperbilirubinemia needs to be anticipated.
Hyperacusis is not seen in IDMs, whose risk for hearing impairment is similar to that of other
newborns requiring intensive care. Hypercalcemia is seen in Williams syndrome, but not in
IDMs. Hydronephrosis, seen on prenatal ultrasonography, may be present in a number of highrisk pregnancies, but not IDMs. Polydactyly may be seen in trisomies and some other congenital
syndromes but does not occur with greater frequency in IDMs. Polysplenia is seen in defects of
left-right asymmetry but does not have an increased incidence in IDMs. Hypogonadism is
characteristic of congenital adrenal hyperplasia, Prader-Willi syndrome, and Turner syndrome; it
is not more common in IDMs.
References:
Cowett RM. The infant of the diabetic mother. NeoReviews. 2002;3:e173-e189. Available for
subscription at: http://neoreviews.aappublications.org/cgi/content/full/3/9/e173
Sivit CJ. Diagnostic imaging. In: Martin RJ, Fanaroff AA, Walsh MC, eds. Fanaroff and Martin's
Neonatal-Perinatal Medicine. 8th ed. Philadelphia, Pa: Mosby Elsevier; 2006:713-732

Copyright 2009 by the American Academy of Pediatrics

page 176

2009 PREP SA on CD-ROM


Question: 83

A 17-year-old girl complains of an itchy rash all over her back and trunk for 2 weeks. Topical
hydrocortisone has not relieved the rash or itching. She denies fever or other symptoms, and
her vital signs are normal. Examination of the skin reveals multiple 5- to 8-mm salmon-colored
thin scaling plaques over her trunk (Item Q83). There is one similar lesion on her abdomen that
measures 2x3 cm. There are no other lesions, and the remaining findings of her physical
examination are normal.
Of the following, the BEST approach to managing this girls rash is to

A. administer intramuscular penicillin


B. administer topical coal tar lotion
C. administer topical selenium sulfide shampoo
D. prescribe oral corticosteroids
E. reassure her that the rash will resolve in several weeks

Copyright 2009 by the American Academy of Pediatrics

page 177

2009 PREP SA on CD-ROM


Critique: 83

Preferred Response: E

The rash described for the girl in the vignette is most consistent with the diagnosis of pityriasis
rosea, a self-limited condition believed to have a viral etiology, although this remains unproven. A
nonspecific prodrome of fever and malaise is seen occasionally. Typical features include a
salmon-colored patch surrounded by a darker rim and a fine scale, known as a "herald patch"
(Item C83A). Several days after the appearance of the herald patch, multiple similar but smaller
lesions develop over the trunk and back along lines of skin cleavage, giving the impression of a
"Christmas tree" pattern (Item C83B). The palms and soles typically are spared. Some pruritus
occurs in approximately 25% of patients. The rash is generally present for 2 to 12 weeks and
resolves spontaneously. Exposure to sunlight may hasten resolution of the lesions.
Numerous therapies have been prescribed for the treatment of pityriasis rosea, including
oral antibiotics, oral antihistamines, and oral steroids, but most have not proven to be effective.
Therefore, reassurance is all that is necessary. Topical antipruritic agents may relieve itching in
some cases. Coal tar preparations are useful in the treatment of psoriasis and other chronic
dermatoses. Topical selenium sulfide shampoo may be helpful for tinea versicolor and
seborrheic dermatitis, but these conditions are not suggested by the appearance of the girl's
rash.
One of the most important diagnoses to consider in the differential diagnosis of pityriasis
rosea is secondary syphilis. The skin lesions can look similar to those of pityriasis rosea and
may follow the same lines of distribution, but the palms and soles generally are affected (Item
C83C). Patients also may complain of vague constitutional symptoms, such as headaches,
fatigue, and lymph node swelling. It is critical to obtain an accurate sexual history in any
adolescent who presents with this type of rash. Intramuscular penicillin is the drug of choice for
secondary syphilis.
References:
Chuh AAT, Dofitas BL, Comisel GG, et al. Interventions for pityriasis rosea. Cochrane Database
Syst Rev. 2007;2:CD005068. Available at:
http://www.mrw.interscience.wiley.com/cochrane/clsysrev/articles/CD005068/frame.html
Morelli JG. Diseases of the epidermis. In: Kliegman RM, Behrman RE, Jenson HB, Stanton BF,
eds. Nelson Textbook of Pediatrics. 18th ed. Philadelphia, Pa: Saunders Elsevier; 2007:27022707
Wolfrey JD, Billica WH, Gulbranson SH, et al. Pediatric exanthems. Clin Fam Pract. 2003;5:557588

Copyright 2009 by the American Academy of Pediatrics

page 178

2009 PREP SA on CD-ROM


Question: 84

A 4-year-old boy who has had mild eczema in the past that was treated successfully with
emollients presents with the worst exacerbation he ever has had. He has multiple lichenified
lesions, especially in the antecubital fossae (Item Q84) and popliteal fossa, which is usual for
him, but he also has nummular lesions on the trunk. His mother reports no changes in
detergents or personal hygiene products. The boy has been going to a summer day camp at the
local community center for the first time.
Of the following, the factor MOST likely involved in his eczema exacerbation is

A. activities in the air-conditioned gymnasium


B. craft activities using water-soluble paint and glue
C. daily snacks of popcorn and fruit juice
D. outdoor soccer practice every afternoon
E. wearing a cotton t-shirt

Copyright 2009 by the American Academy of Pediatrics

page 179

2009 PREP SA on CD-ROM


Critique: 84

Preferred Response: D

A number of factors may worsen atopic dermatitis by causing itching and scratching. Exposure
to heat (as may occur during warm weather months or because of overdressing during cold
weather months), sunlight, chemicals (including swimming pool chemicals such as chlorine), and
sweat retention cause exacerbations. Cold weather and low humidity may cause skin to become
dry, leading to increased pruritus. Wool or synthetic materials, fragrances, harsh soaps or
detergents, and some fabric softeners also may cause itching.
For the child described in the vignette, overheating during outdoor play is the most likely
activity that is exacerbating his eczema. Activities in an air-conditioned gymnasium would be
beneficial. Although the use of glues and craft materials may cause irritation, they would be
anticipated to produce a rash on the hands at sites of exposure. The snacks the child is
receiving are unlikely to worsen atopic dermatitis. Food allergy as an exacerbating factor is
observed most often in infants, not older children. The allergens most commonly implicated are
milk, eggs, soy, wheat, and peanuts.
References:
Ashcroft DM, Chen L-C, Garside R, Stein K, Williams HC. Topical pimecrolimus for eczema.
Cochrane Database Syst Rev. 2007;4:CD005500. Available at:
http://www.mrw.interscience.wiley.com/cochrane/clsysrev/articles/CD005500/frame.html
Bath-Hextall F, Williams H. Eczema (atopic). BMJ Clinical Evidence. 2006. Available for
subscription at: http://clinicalevidence.bmj.com/ceweb/conditions/skd/1716/1716_I15.jsp
Beattie PE, Lewis-Jones MS. A comparative study of impairment of quality of life in children with
skin disease and children with other chronic childhood diseases. Br J Dermatol. 2006;155:145151. Abstract available at: http://www.ncbi.nlm.nih.gov/pubmed/16792766
Byremo G, Rd G, Carlsen KH. Effect of climatic change in children with atopic eczema. Allergy.
2006;61:1403-1410. Abstract available at: http://www.ncbi.nlm.nih.gov/pubmed/17073869
Ersser SJ, Latter S, Sibley A, Satherley PA, Welbourne S. Psychological and educational
interventions for atopic eczema in children. Cochrane Database Syst Rev. 2007;3:CD004054.
Available at:
http://www.mrw.interscience.wiley.com/cochrane/clsysrev/articles/CD004054/frame.html
Kramer MS, Kakuma R. Maternal dietary antigen avoidance during pregnancy or lactation, or
both, for preventing or treating atopic disease in the child. Cochrane Database Syst Rev.
2006;3:CD000133. Available at:
http://www.mrw.interscience.wiley.com/cochrane/clsysrev/articles/CD000133/frame.html
Osborn DA, Sinn J. Formulas containing hydrolysed protein for prevention of allergy and food
intolerance in infants. Cochrane Database Syst Rev. 2003;4:CD003664. Available at:
http://www.mrw.interscience.wiley.com/cochrane/clsysrev/articles/CD003664/frame.html
Osborn DA, Sinn JK. Probiotics in infants for prevention of allergic disease and food
hypersensitivity. Cochrane Database Syst Rev. 2007;4:CD006475. Available at:
http://www.mrw.interscience.wiley.com/cochrane/clsysrev/articles/CD006475/frame.html
Osborn DA, Sinn J. Soy formula for prevention of allergy and food intolerance in infants.
Cochrane Database Syst Rev. 2006;4:CD003741. Available at:
http://www.mrw.interscience.wiley.com/cochrane/clsysrev/articles/CD003741/frame.html

Copyright 2009 by the American Academy of Pediatrics

page 180

2009 PREP SA on CD-ROM


Question: 85

You are called by the mother of 3-year-old girl because the child appears confused and is pale
and sweating. The mother thinks the child may have taken some of her grandmothers
imipramine. You advise her to contact emergency medical services for immediate transport to
the emergency department, where you plan to meet them.
Of the following, the MOST appropriate action to take in the emergency department is

A. chest radiography to evaluate for pulmonary edema


B. continuous cardiac monitoring for dysrhythmias
C. echocardiography to assess cardiac function
D. electroencephalography to identify a seizure focus
E. measurement of the serum concentration of imipramine

Copyright 2009 by the American Academy of Pediatrics

page 181

2009 PREP SA on CD-ROM


Critique: 85

Preferred Response: B

The child described in the vignette has symptoms suggestive of tricyclic antidepressant (TCA)
ingestion. This class of antidepressants is used less frequently today for treatment of
depression because of the availability of selective serotonin reuptake inhibitors (SSRIs). TCAs
are not the drugs of choice for treatment of depression in children and adolescents but are used
occasionally in the treatment of other disorders (eg, enuresis, narcolepsy). SSRIs are much
safer to use than TCAs because they have fewer adverse effects and are unlikely to result in
death when overdose occurs.
When TCAs are ingested in toxic amounts, they primarily affect the central nervous and
cardiovascular systems. Central nervous system signs and symptoms of TCA toxicity include
irritability, euphoria, seizures, and unresponsiveness. Autonomic nervous system symptoms
such as mydriasis, dry skin, dry mouth, urinary retention, and tachycardia also may be evident.
Among the direct effects on the cardiac system are a delay in signal conduction through the
bundle of His, depression of myocardial contractile function, and prolongation of the QRS and
the QT intervals. These latter cardiac effects may potentiate arrhythmia formation.
Continuous cardiac monitoring for the possible occurrence of arrhythmia is essential in
patients who are suspected of toxic TCA ingestion. Electrocardiography is the best tool for
assessing the function of the conduction system. The voltage intervals should be measured,
with particular attention to the QRS duration and the QT interval. A QRS duration of greater than
100 msec is associated with the development of seizures; a QRS duration of more than 160
msec is associated with ventricular dysrhythmias that may be particularly difficult to treat.
Echocardiography is an excellent tool to evaluate cardiac structure and function but is not
necessarily indicated as part of the emergency department evaluation of a child in whom toxic
TCA ingestion is suspected and does not obviate the need for continuous cardiac monitoring.
Similarly, chest radiography does not play a role in the acute management of such a patient.
Electroencephalography has no place in the initial evaluation and management of children in
whom a TCA toxic ingestion is suspected. Seizures, when they occur, usually do so early in the
course and often resolve by the time anticonvulsants are administered. Measuring serum drug
concentrations of TCAs is not helpful for the prognosis or management of TCA ingestion. Serum
electrolyte concentrations may be measured, but they have no predictive value in the
management of TCA toxicity.
References:
Boehnert MT, Lovejoy FH Jr. Value of the QRS duration versus the serum drug level in
predicting seizures and ventricular arrhythmias after an acute overdose of tricyclic
antidepressants. N Engl J Med. 1985;313:474-479. Abstract available at:
http://www.ncbi.nlm.nih.gov/pubmed/4022081
Hatcher-Kay C, King CA. Depression and suicide. Pediatr Rev. 2003;24:363-371. Available at:
http://pedsinreview.aappublications.org/cgi/content/full/24/11/363
Liebelt EL, Francis PD, Woolf AD. ECG lead aVR versus QRS interval in predicting seizures and
arrhythmias in acute tricyclic antidepressant toxicity. Ann Emerg Med. 1995;26:195-201.
Abstract available at: http://www.ncbi.nlm.nih.gov/pubmed/7618783
McGuigan ME. Poisoning potpourri. Pediatr Rev. 2001;22:295-302. Available at:
http://pedsinreview.aappublications.org/cgi/content/full/22/9/295
Prez-Fontn J, Lister G. The acutely ill infant and child. In: Rudolph CD, Rudolph AM, eds.
Rudolph's Pediatrics. 21st ed. New York, NY: McGraw-Hill Medical Publishing Division; 2003:364365

Copyright 2009 by the American Academy of Pediatrics

page 182

2009 PREP SA on CD-ROM


Question: 86

A 4-year-old boy who has neuroblastoma presents with back pain and an inability to urinate. He
is alert, with normal general examination findings and normal mental status. Strength and tone in
the arms are normal, but tone is low in the legs, and patellar reflexes are diminished.
Of the following, the MOST appropriate next step for diagnosis is

A. lumbar puncture
B. magnetic resonance imaging with contrast of the spine
C. postvoid bladder residual measurement
D. radiograph of the spine
E. voiding cystourethrography

Copyright 2009 by the American Academy of Pediatrics

page 183

2009 PREP SA on CD-ROM


Critique: 86

Preferred Response: B

Back pain in a young child is unusual and requires prompt evaluation. When there are additional
signs that can localize to the spinal cord, such as inability to urinate, low tone, and diminished
reflexes, as described for the boy in the vignette, immediate evaluation in the emergency
department is needed. Lesions within or near the spinal cord that are causing symptoms can
progress to irreversible damage that may be avoided with emergency neurosurgery.
The child described in the vignette has a history of neuroblastoma, an aggressive tumor that
is prone to metastasis, and emerging flaccid paralysis. Therefore, magnetic resonance imaging
(MRI) of the spine with gadolinium contrast is indicated to assess the nature of the problem and
determine the location of the lesion more exactly (Item C86). Hypervascularity or vascular wall
breakdown in a metastasis may result in contrast enhancement, improving visualization of the
lesion.
Lumbar puncture may be needed subsequently to evaluate for other causes or for tumor
cytology, but this procedure should be deferred until after imaging the spine and, if necessary,
emergent neurosurgical consultation. Similarly, spine imaging takes precedence over urologic
dynamic studies such as postvoid bladder residual measurement or voiding
cystourethrography. A plain radiograph may provide some information, but if MRI is available, it
is preferred because it can assist in emergency surgical planning.
Management of spinal cord compression due to a tumor includes immediate neurosurgical
consultation. Intravenous dexamethasone typically is used, along with a proton pump inhibitor or
histamine2 blocker. Aggressive pain management may be needed.
References:
Haslam RHA. Spinal cord disorders. In: Kliegman RM, Behrman RE, Jenson HB, Stanton BF,
eds. Nelson Textbook of Pediatrics. 18th ed. Philadelphia, Pa: Saunders Elsevier; 2007:25262530
Kim S, Chung DH. Pediatric solid malignancies: neuroblastoma and Wilms' tumor. Surg Clin
North Am. 2006;86:469-487

Copyright 2009 by the American Academy of Pediatrics

page 184

2009 PREP SA on CD-ROM


Question: 87

The mother of a boy in your practice is contemplating another pregnancy and asks for your
advice. The woman is tall and thin and works as a model part-time. She had previously reported
to you a history of bulimia. She is extremely concerned about any "extra" weight she may gain
during the pregnancy, and she confides that she sometimes smokes cigarettes to avoid eating.
Additionally, she occasionally has taken her sons methylphenidate to suppress her appetite.
When asked about alcohol use, she describes herself as a "social drinker."
Of the following, the MOST accurate statement to make in counseling this woman is that

A. bulimia during pregnancy is not associated with birth defects


B. cigarette smoking increases the risk of sudden infant death syndrome in the exposed infant
C. one or two alcoholic beverages per day will do no harm to the embryo/fetus
D. prenatal methylphenidate exposure is associated with craniofacial malformations
E. vitamin supplements reduce the risk of defects associated with prenatal alcohol exposure

Copyright 2009 by the American Academy of Pediatrics

page 185

2009 PREP SA on CD-ROM


Critique: 87

Preferred Response: B

The extent of the impact that environmental exposures have on pregnancy outcome is still
largely unknown. However, some large, well-designed epidemiologic studies offer insight into
complications that can arise from fetal exposure to maternal eating disorders, tobacco smoking,
and substances of abuse. Undoubtedly, genetic and other factors modify outcomes.
The impact of cigarette smoking on fetal development and pregnancy has been, and
continues to be, an active area of study. Intrauterine growth restriction is the most consistent
negative effect of maternal smoking on the fetus, with an average 200-g reduction in the
birthweights of term infants born to mothers who smoke during pregnancy. The more a woman
smokes, the greater the reduction in fetal weight.
Cigarette smoking during pregnancy also has been associated with increased health risks
to exposed children as they grow. Among the reported complications are abnormal pulmonary
function, increased cancer risk, and lower sperm counts. Numerous studies have found an
increased risk for sudden infant death in prenatally exposed infants.
Studies of the effects of eating disorders on pregnancy outcome are limited. However, both
prospective, controlled studies and literature reviews indicate that pregnant women who have
past or active eating disorders, including bulimia, are at increased risk for delivering babies of
significantly lower-than-average birthweight and head circumference (including microcephaly).
Prenatal exposure to alcohol may result in varied outcomes collectively termed fetal alcohol
spectrum disorder. To date, no safe quantity of alcohol has been established for consumption
during pregnancy. Therefore, the strong recommendation is that women abstain from drinking
alcoholic beverages from at least the time of conception and throughout pregnancy.
The effects of methylphenidate on pregnancy outcome have been studied in experimental
animals and a limited number of humans. To date, there does not appear to be an increase in
congenital anomalies associated with prenatal exposure to methylphenidate.
The question of the potential benefits of vitamin and micronutrient supplementation for
women who smoke cigarettes and drink alcohol during pregnancy and their babies is under
investigation. At present, it is not clear whether supplementation can mitigate poor outcomes,
and no specific recommendations exist. However, it is clear that good nutrition and vitamin
supplementation during pregnancy reduce the risk for pregnancy complications.
References:
Cigarette smoking, methamphetamine. Reprotox. Available for subscription at:
http://www.reprotox.org
Cigarette smoking, methamphetamine. Teris. Available for subscription at
http://depts.washington.edu/terisweb/teris/
Cogswell ME, Weisberg P, Spong C. Cigarette smoking, alcohol use and adverse pregnancy
outcomes: implications for micronutrient supplementation. J Nutr. 2003;133:1722S-1731S.
Available at: http://jn.nutrition.org/cgi/content/full/133/5/1722S
Kouba S, Hllstrm T, Londholm C, Lindn Hirschbe A. Pregnancy and neonatal outcomes in
women with eating disorders. Obstet Gynecol. 2005;105:255-260. Available at:
http://www.greenjournal.org/cgi/content/full/105/2/255
Kunz LH, King JC. Impact of maternal nutrition and metabolism on health of the offspring. Semin
Fetal Neonatal Med. 2007;12;71-77. Abstract available at:
http://www.ncbi.nlm.nih.gov/pubmed/17200031
Micali N, Simonoff E, Treasure J. Risk of major adverse outcomes in women with eating
disorders. Br J Psychiatry. 2007;190:255-259. Abstract available at:
http://www.ncbi.nlm.nih.gov/pubmed/17329747

Copyright 2009 by the American Academy of Pediatrics

page 186

2009 PREP SA on CD-ROM


Question: 88

An 18-year-old young man comes to your office with complaints of burning pain with urination
over the past 24 hours. He has seen a small amount of yellowish discharge from his penis
during this time. He also complains of some lower back pain over the past 48 hours. He denies
fever or rashes, but his eyes are a little irritated. He is sexually active and uses condoms "most
of the time." On physical examination, he is afebrile, his palpebral and bulbar conjunctivae are
mildly injected (Item Q88), and his back is tender at the lower lumbar area, but there is no
costovertebral angle tenderness. Genital examination reveals no scrotal tenderness and scant
yellow discharge at the urethral orifice.
Of the following, the MOST likely cause of this patients symptoms is

A. Chlamydia trachomatis
B. Gardnerella vaginalis
C. Neisseria gonorrhoeae
D. Treponema pallidum
E. Trichomonas vaginalis

Copyright 2009 by the American Academy of Pediatrics

page 187

2009 PREP SA on CD-ROM


Critique: 88

Preferred Response: A

Urethritis can have infectious and noninfectious causes. Symptoms include mucopurulent or
purulent discharge, dysuria, and urethral pruritus. Several organisms, including Neisseria
gonorrhoeae and Chlamydia trachomatis, cause urethritis. Ureaplasma urealyticum,
Mycoplasma genitalium, Gardnerella vaginalis, herpes simplex virus, adenovirus, and
Trichomonas vaginalis are implicated in nonchlamydial, nongonococcal urethritis (NGU), but
they are more difficult to detect than N gonorrhoeae and C trachomatis.
The constellation of conjunctivitis, urethritis, and arthritis reported for the young man in the
vignette represents the classic symptoms of a form of reactive arthritis once called Reiter
syndrome. The term reactive arthritis refers to rheumatic disorders that appear after an
infection, but in which the responsible pathogen is not detected in the affected joint. C
trachomatis is the only genital pathogen commonly accepted to be a cause of reactive arthritis.
N gonorrhoeae can cause septic arthritis or disseminated gonococcal infection (ie, a rash and
tenosynovitis) but does not produce reactive arthritis. Although T vaginalis and G vaginalis may
cause urethritis, they do not produce the other symptoms exhibited by the boy described in the
vignette. Syphilis, caused by infection with Treponema pallidum, may affect bones congenitally
(osteochondritis) or in late stages of the disease (with gummas, granulomatous lesions that
involve bones as well as soft tissue or viscera) but does not produce urethral discharge or
conjunctivitis.
Reactive arthritis caused by C trachomatis is treated with a single 1-g oral dose of
azithromycin or with 100 mg doxycycline orally twice a day for 7 days, after testing for both N
gonorrhoeae and C trachomatis is completed. This is also the recommended regimen for all
NGUs. First-line treatment of uncomplicated gonococcal urethritis is accomplished with
ceftriaxone 125 mg intramuscularly or cefixime 400 mg orally, both in a single dose.
References:
Centers for Disease Control and Prevention. Update to CDC's sexually transmitted diseases
treatment guidelines, 2006: fluoroquinolones no longer recommended for treatment of
gonococcal infections. MMWR Morbid Mortal Wkly Rep. 2007;56:332-336. Available at:
http://www.cdc.gov/mmwr/preview/mmwrhtml/mm5614a3.htm
Fortenberry JD, Neinstein LS. Syphilis. In: Neinstein LS, ed. Adolescent Health Care: A Practical
Guide. 5th ed. Philadelphia, Pa: Lippincott Williams & Wilkins; 2008:825-833
Workowski KA, Berman SM, Centers for Disease Control and Prevention. Sexually transmitted
diseases treatment guidelines, 2006. MMWR Recomm Rep. 2006;55(RR11):1-94. Available at:
http://www.cdc.gov/mmwr/preview/mmwrhtml/rr5511a1.htm
Yu DT. Reactive arthritis (formerly Reiter syndrome): definition, diagnosis, and management.
UpToDate Online 15.3. 2008. Available for subscription at:
http://www.utdol.com/utd/content/topic.do?topicKey=spondylo/7349

Copyright 2009 by the American Academy of Pediatrics

page 188

2009 PREP SA on CD-ROM


Question: 89

You are evaluating an 18-month-old girl for vomiting. She has a history of febrile seizures and
recurrent ear infections. She is receiving no medications. Over the past several weeks, her
parents have noticed that she has been "increasingly clumsy." She has vomited each of the last
three mornings but has had no diarrhea or fever. Physical examination findings are normal
except for an ataxic gait and hyperreflexia.
Of the following, the MOST appropriate next step is

A. administration of an antiemetic
B. computed tomography scan of the head
C. electroencephalography
D. lumbar puncture
E. reassurance and re-evaluation in 3 to 5 days

Copyright 2009 by the American Academy of Pediatrics

page 189

2009 PREP SA on CD-ROM


Critique: 89

Preferred Response: B

Initial symptoms of increased intracranial pressure often consist of headaches and confusion
that may be accompanied by lethargy. The child described in the vignette exhibits signs of a
progressive increase in pressure, such as vomiting (especially in the morning) and changes in
motor tone. Physical examination findings include a full or bulging fontanelle and widened
sutures. These signs, as well as pupillary changes and papilledema, should prompt rapid
evaluation to prevent permanent neurologic injury and progression of the increased pressure
with potential neurologic catastrophe.
Intracranial pressure is maintained by the balance of the contents of the cranial vault, which
includes brain, blood, and cerebrospinal fluid. Increases in intracranial pressure can occur with a
wide variety of disease processes, such as brain tumors, hydrocephalus, infections, head
trauma, and hypoxic-ischemic injury. Intracranial pressure increases when the compensatory
mechanisms of the cranial vault are exceeded and produce numerous symptoms, depending on
the age of the patient and the underlying pathology.
Computed tomography scan or magnetic resonance imaging of the head is the first priority
in evaluating suspected increased intracranial pressure. Meningitis is unlikely in this patient due
to the chronicity of symptoms and absence of fever. Accordingly, lumbar puncture is not
indicated at this time. Electroencephalography might be indicated if atypical migraines or
seizures were suspected, but the initial priority is to evaluate the patient for potential lifethreatening disease processes. The child has no evidence of viral infection, and reassurance or
administration of antiemetics is not appropriate.
References:
Frankel LR. Neurological emergencies and stabilization. In: Kliegman RM, Behrman RE, Jenson
HB, Stanton BF, eds. Nelson Textbook of Pediatrics. 18th ed. Philadelphia, Pa: Saunders
Elsevier; 2007:405-412
Larsen GY, Goldstein B. Consultation with the specialist: increased intracranial pressure. Pediatr
Rev. 1999;20:234-239. Available at:
http://pedsinreview.aappublications.org/cgi/content/full/20/7/234

Copyright 2009 by the American Academy of Pediatrics

page 190

2009 PREP SA on CD-ROM


Question: 90

A 16-year-old girl comes to your office complaining that her menstrual periods have been
irregular and scanty. Her last period was 3 months ago and lasted for only 2 days. Among the
findings on physical examination are fine, moist skin; firm, palpable thyroid gland (Item Q90); and
finger tremor. Results of laboratory studies include a thyroid-stimulating hormone value of less
than 0.05 mIU/L (normal, 0.5 to 5.0 mIU/L) and free thyroxine value of 1.9 ng/dL (24.5 pmol/L)
(normal, 0.6 to 1.3 ng/dL [7.7 to 16.8 pmol/L]).
Of the following, the additional physical examination finding that BEST supports the diagnosis of
hyperthyroidism is

A. abdominal obesity
B. atrophy of lingual papillae
C. hepatomegaly
D. hirsutism
E. muscle weakness

Copyright 2009 by the American Academy of Pediatrics

page 191

2009 PREP SA on CD-ROM


Critique: 90

Preferred Response: E

The clinical findings of hyperthyroidism in children usually are obvious, but the diagnosis can be
subtle in mild disease. Classic clinical findings include weight loss, increased appetite,
decreased strength and sports performance, hyperactivity, tremors, sweating, nocturnal
sleeplessness (sometimes with daytime somnolence), irritability, decreased school
performance, pruritus, and nocturia. Menarcheal girls may have scant, infrequent menses, as
described for the girl in the vignette. On physical examination, most children have a palpable,
firm thyroid gland (Item C90A) that has an audible bruit. Some children may have fine, moist
skin; a visible tremor; slight skin darkening; fine scalp hair, with some hair loss at the temples;
muscle weakness; and some loss of muscle mass that can be identified by examining the thenar
and hypothenar eminences. A "thyrotoxic stare" accompanies hyperthyroidism. Exophthalmos
(Item C90B) may be found in thyrotoxicosis due to Graves disease. Hirsutism, hepatomegaly,
abdominal obesity, and changes in the lingual papillae are not findings of hyperthyroidism.
Other laboratory studies that are of use in caring for the girl in the vignette include a
measurement of triiodothyronine, which often is substantially elevated in hyperthyroidism, and
measures of thyroid-stimulating immunoglobulins, which are elevated in Graves disease.
Radioactive iodine or technetium uptake imaging can distinguish between subacute thyroiditis
(low uptake) and Graves disease (high uptake). This distinction is important because treatments
for Graves disease, including use of antithyroid drugs, radioactive iodine, and surgery, are not
appropriate for transient subacute thyroiditis.
References:
Fenton CL, Gold JG. Hyperthyroidism. eMedicine Specialties, Pediatrics, Endocrinology. 2006.
Available at: http://www.emedicine.com/ped/topic1099.htm
Ferry RJ Jr, Levitsky LL. Graves disease. eMedicine Specialties, Pediatrics, Endocrinology.
2006. Available at: http://www.emedicine.com/ped/topic899.htm
LaFranchi S. Clinical manifestations and diagnosis of hyperthyroidism in children and
adolescents. UpToDate Online 15.3. 2008. Available for subscription at:
http://www.uptodateonline.com/utd/content/topic.do?topicKey=pediendo/5570

Copyright 2009 by the American Academy of Pediatrics

page 192

2009 PREP SA on CD-ROM


Question: 91

An infant in the newborn nursery does not appear to respond to visual or auditory input. On
physical examination, he shows evidence of intrauterine growth restriction (IUGR), absent red
reflexes, and numerous bluish papules (Item Q91). The mother, who immigrated to the United
States during her third trimester, did not receive prenatal care. She denies use of alcohol, drugs,
or tobacco products during pregnancy. She reports that she had a low-grade fever and rash
during the second month of the pregnancy.
Of the following, the MOST likely infectious cause of the findings in this infant is

A. cytomegalovirus
B. human immunodeficiency virus
C. rubella virus
D. Toxoplasma gondii
E. varicella-zoster virus

Copyright 2009 by the American Academy of Pediatrics

page 193

2009 PREP SA on CD-ROM


Critique: 91

Preferred Response: C

Congenital infections may lead to developmental sequelae in infancy and childhood, including
visual impairment, hearing loss, and intellectual disabilities (Item C91A). The infant described in
the vignette has clinical findings most consistent with congenital rubella syndrome (CRS). These
findings include intrauterine growth restriction (IUGR), absent red reflexes (due to cataracts),
and bluish papules (Item C91B). Other clinical findings associated with CRS include nerve
deafness, microphthalmia, cardiac defects, meningoencephalitis, hepatomegaly, and
microcephaly.
Maternal cytomegalovirus (CMV) infection is common, but 90% of infants who have CMV
infection are asymptomatic at birth. Maternal symptoms include a flulike illness that may involve
fever, but a rash is not seen, as reported by the mother in the vignette. Among the clinical
findings of congenital CMV infection are hepatomegaly, splenomegaly, jaundice, petechiae,
chorioretinitis, IUGR, purpura, and microcephaly. CMV infection is the leading nongenetic reason
for sensorineural hearing loss and the most common congenital infection to cause intellectual
disability.
Congenital varicella infection may result in zigzag scarring of the skin, limb deformities, or
cataracts. Occasionally, severely affected infants may have central nervous system
involvement with necrotizing cerebral lesions or microcephaly.
Congenital toxoplasmosis presents with IUGR, anemia, jaundice, hepatosplenomegaly,
intracranial calcifications, hydrocephalus, microcephaly, and chorioretinitis, but not skin lesions.
A newborn infected perinatally with human immunodeficiency virus exhibits no symptoms or
signs. Later, the infant may develop subtle clinical findings, such as lymphadenopathy and
hepatosplenomegaly, or nonspecific symptoms, such as failure to thrive, chronic or recurrent
diarrhea, interstitial pneumonia, or oral thrush. Central nervous system involvement is variable,
ranging from mild learning disabilities to severe mental retardation.
References:
Adler SP, Marshall B. Cytomegalovirus infections. Pediatr Rev. 2007;28:92-100. Available at:
http://pedsinreview.aappublications.org/cgi/content/full/28/3/92
Mason W. Rubella. In: Kliegman RM, Behrman RE, Jenson HB, Stanton, BF, eds. Nelson
Textbook of Pediatrics. 18th ed. Philadelphia, Pa: Saunders Elsevier; 2007:1337-1341
McLeod R, Remington JS. Toxoplasmosis (Toxoplasma gondii). In: Kliegman RM, Behrman RE,
Jenson HB, Stanton BF, eds. Nelson Textbook of Pediatrics. 18th ed. Philadelphia, Pa:
Saunders Elsevier; 2007:1486-1495
Myers MG, Seward J, La Russa P. Varicella-zoster virus. In: Kliegman RM, Behrman RE,
Jenson HB, Stanton BF, eds. Nelson Textbook of Pediatrics. 18th ed. Philadelphia, Pa:
Saunders Elsevier; 2007:1366-1372
Stagno S. Cytomegalovirus. In: Kliegman RM, Behrman RE, Jenson HB, Stanton BF, eds.
Nelson Textbook of Pediatrics. 18th ed. Philadelphia, Pa: Saunders Elsevier; 2007:1377-1379
Yogev R, Gould Chadwick E. Acquired immunodeficiency syndrome (human immunodeficiency
virus). In: Kliegman RM, Behrman RE, Jenson HB, Stanton BF, eds. Nelson Textbook of
Pediatrics. 18th ed. Philadelphia, Pa: Saunders Elsevier; 2007:1427-1442

Copyright 2009 by the American Academy of Pediatrics

page 194

2009 PREP SA on CD-ROM


Question: 92

A 6-month-old boy presents to the emergency department with a 2-day history of fever and a 1day history of left cheek swelling. You discover that his parents do not believe in providing their
children with immunizations. Despite this, the boy has never been ill. He has two older siblings,
and nobody is sick at home. The mother denies any recent bug bites or trauma to the area on
his cheek. Physical examination reveals a mildly toxic-appearing child who has a temperature of
103.0F (39.4C), heart rate of 145 beats/min, respiratory rate of 26 breaths/min, and blood
pressure of 80/45 mm Hg. His anterior fontanelle is slightly bulging, his tympanic membranes are
erythematous, his left cheek is indurated and appears erythematous to slightly violaceous (Item
Q92), and he is irritable.
Of the following, the MOST likely organism to cause this childs illness is

A. Haemophilus influenzae type b


B. Neisseria meningitidis
C. Staphylococcus aureus
D. Streptococcus pneumoniae
E. Streptococcus pyogenes

Copyright 2009 by the American Academy of Pediatrics

page 195

2009 PREP SA on CD-ROM


Critique: 92

Preferred Response: A

Children who have not received the Haemophilus influenzae type b (Hib) vaccine are at risk for
illnesses commonly caused by this organism, including buccal and periorbital cellulitis (as
described for the boy in the vignette) (Item C92A), pyogenic arthritis, epiglottitis (Item C92B),
and bacterial meningitis. Neisseria meningitidis usually does not cause a facial cellulitis, and
Staphylococcus aureus and Streptococcus pyogenes are less likely pathogens in the absence
of a history of a break in the skin. S pneumoniae can be the cause of a nontraumatic facial
cellulitis in Hib-vaccinated children, but in an unvaccinated child, Hib would be the most likely
pathogen.
Hib disease can be verified by recovery of the organism from a sterile site (eg, blood,
cerebrospinal fluid, joint fluid) or by urine antigen testing. Once the organism is isolated,
antimicrobial susceptibility testing is important because approximately 30% to 40% of Hib
isolates produce beta-lactamase, making these organisms resistant to ampicillin.
References:
American Academy of Pediatrics. Haemophilus influenzae infections. In: Pickering LK, Baker CJ,
Long SS, McMillan JA, eds. Red Book: 2006 Report of the Committee on Infectious Diseases.
27th ed. Elk Grove Village, Ill: American Academy of Pediatrics; 2006:310-318
Tzanakaki G, Mastrantonio P. Aetiology of bacterial meningitis and resistance to antibiotics of
causative pathogens in Europe and in the Mediterranean region. Int J Antimicrob Agents.
2007;29:621-629. Abstract available at: http://www.ncbi.nlm.nih.gov/pubmed/17368858

Copyright 2009 by the American Academy of Pediatrics

page 196

2009 PREP SA on CD-ROM


Question: 93

You are seeing a 5-year-old boy who has developed diplopia, dysphagia, dry mouth, diarrhea,
weakness in his arms, and shortness of breath over the past 18 hours. According to his
records, he received his diphtheria, tetanus, acellular pertussis (DTaP), poliovirus inactivated
(IPV), measle-mumps-rubella (MMR), and varicella booster immunizations about 1 month ago.
He attended a class picnic 3 weeks ago that was held in a state park. He has no history of
unusual exposures or ill contacts, and except for falling off his bike 5 days ago and scraping his
arm, he has had no other trauma. Physical examination reveals an awake and alert boy who
complains of "seeing double" and of pain with swallowing. His pupils are 3 mm bilaterally and
sluggish, and his mucous membranes are dry. He takes shallow breaths, but his lungs are
clear, and his abdomen is mildly distended. His left arm has a 4x4-cm abrasion that is mildly
swollen, erythematous, and tender, with some serosanguineous drainage. His left arm has 2/5
strength and decreased tone. He has 1+ reflexes in the upper and lower extremities.
Of the following, the MOST likely cause of this patients condition is

A. botulism
B. cerebral vascular accident
C. Guillain-Barr syndrome
D. tetanus
E. tick paralysis

Copyright 2009 by the American Academy of Pediatrics

page 197

2009 PREP SA on CD-ROM


Critique: 93

Preferred Response: A

Clostridium botulinum spores are found worldwide in soil and marine sediments. Botulism results
from the absorption of botulinum toxins, most commonly produced by C botulinum (a large, grampositive, anaerobic bacillus that has a subterminal spore), into the circulation from a wound or
mucosal surface. Of the seven antigenic toxin types that the organism produces, human
botulism is caused by neurotoxins A, B, E, and occasionally F. The clinical forms of botulism
include: foodborne, infant, wound, and that of undetermined cause, which is rare. Almost all
cases of infant botulism and wound botulism are caused by types A and B toxin. The onset of
symptoms may be abrupt (within hours) or evolve gradually over several days. Foodborne
botulism occurs with the ingestion of food that is contaminated with spores of C botulinum and
has been stored improperly under anaerobic conditions, allowing for toxin production. In wound
botulism, the spores are introduced into the wound at the time of trauma, where they germinate
and produce toxin. Infant botulism results from ingestion of C botulinum spores that germinate,
multiply, and produce toxin in the intestine. Most cases of infant disease occur in breastfed
infants at the time of introduction of nonhuman milk substances, and the source of the spores
usually is not identified.
The classic presentation of botulism is acute, bilateral cranial nerve palsies associated with
symmetric descending weakness. Fever is absent, sensory deficits do not occur, and the
patient remains responsive, with a normal level of consciousness. Foodborne disease usually
develops between 12 and 36 hours after toxin ingestion. The initial symptoms include nausea,
dry mouth, and diarrhea. Disease progresses to cranial nerve dysfunction, most commonly
starting with the eyes (diplopia, blurry vision), and descending to include dysphagia, upper
extremity weakness, respiratory dysfunction, and lower extremity weakness. The symptoms of
wound botulism are similar to that of food-borne disease, although the incubation period is 4 to
14 days from the time of injury until the onset of symptoms. Also, in many cases, the wound may
not appear to be healing, as described for the boy in the vignette. Infant botulism develops 3 to
30 days from the time of exposure to the spore-containing material. Infants present with
constipation, which is followed by feeding difficulties, hypotonia, increased drooling, a weak cry,
diminished gag reflex, truncal weakness, cranial nerve palsies, and generalized weakness with
ventilatory failure. Progression occurs for 1 to 2 weeks, followed by stabilization for another 2 to
3 weeks before recovery starts.
An important element in the treatment of all forms of botulism is meticulous supportive care.
In addition, patients who have suspected foodborne and wound botulism should be treated with
equine trivalent antitoxin (types A, B, and E) available from the Centers for Disease Control and
Prevention through state health departments. Immediate administration of antitoxin is critical for
successful therapy because it arrests the progression of paralysis but does not reverse it.
Patients who have wound botulism also should undergo wound debridement, even if the wound
appears to be healing. The role of antibiotic therapy is unknown, but penicillin G or metronidazole
frequently is recommended. Human-derived botulinum antitoxin (botulism immune globulin
intravenous) (BIGIV) is used in the treatment of infant botulism caused by type A or type B C
botulinum toxin. BIGIV therapy should be initiated as early in the illness as possible. Antibiotic
therapy is not recommended for infant botulism.
Patients who have cerebral vascular accidents usually do not present with bilateral
symmetric weakness or the other symptoms demonstrated by the patient in the vignette.
Patients who have tetanus have persistent, painful tonic spasms of the muscles of the neck,
jaw, and trunk and are very rigid. The paralysis of tick paralysis is ascending, beginning in the
lower extremities and ascending symmetrically to involve the trunk, upper extremities, and head
within a few hours. Patients who have Guillain-Barr syndrome typically present with numbness
and paresthesias of the hands and feet, followed by progressive weakness involving all four
extremities. Motor impairment begins in the lower extremities and progresses in an ascending
pattern to involve the upper extremities, trunk, and cranial nerves.
References:
American Academy of Pediatrics. Botulism and infant botulism (Clostridium botulinum). In:

Copyright 2009 by the American Academy of Pediatrics

page 198

2009 PREP SA on CD-ROM

Pickering LK, Baker CJ, Long SS, McMillan JA, eds. Red Book: 2006 Report of the Committee
on Infectious Diseases. 27th ed. Elk Grove Village, Ill: American Academy of Pediatrics;
2006:257-260
American Academy of Pediatrics. Tetanus (lockjaw). In: Pickering LK, Baker CJ, Long SS,
McMillan JA, eds. Red Book: 2006 Report of the Committee on Infectious Diseases. 27th ed. Elk
Grove Village, Ill: American Academy of Pediatrics; 2006:648-653
Bleck TP. Clostridium botulinum (botulism). In: Mandell GL, Bennett JE, Dolin R, eds. Mandell,
Douglas, and Bennett's Principles and Practice of Infectious Diseases. 6th ed. Philadelphia, Pa:
Elsevier Churchill Livingstone; 2005:2822-2828
Mathieu ME, Wilson BB. Ticks (including tick paralysis). In: Mandell GL, Bennett JE, Dolin R,
eds. Mandell, Douglas, and Bennett's Principles and Practice of Infectious Diseases. 6th ed.
Philadelphia, Pa: Churchill Livingstone; 2005:3312-3315
Parke JT. Peripheral neuropathies. In: McMillan JA, Feigin RD, DeAngelis CD, Jones MD Jr,
eds. Oski's Pediatrics Principles and Practice. 4th ed. Philadelphia, Pa: Lippincott Williams &
Wilkins; 2006:2310-2316
Schlagger B, Kornberg AJ, Prensky AL. Cerebrovascular disease in childhood. In: McMillan JA,
Feigin RD, DeAngelis CD, Jones MD Jr, eds. Oski's Pediatrics Principles and Practice. 4th ed.
Philadelphia, Pa: Lippincott Williams & Wilkins; 2006:2270-2279

Copyright 2009 by the American Academy of Pediatrics

page 199

2009 PREP SA on CD-ROM


Question: 94

An 8-month-old girl who has a history of cardiomyopathy following viral myocarditis presents
with poor weight gain. She is receiving a 20-kcal/oz milk-based formula and has no history of
vomiting or diarrhea. Her only medication is furosemide. Physical examination findings include a
heart rate of 130 beats/min, respiratory rate of 60 breaths/min, and blood pressure of 88/44 mm
Hg.
Of the following, the MOST appropriate initial strategy to increase weight gain for this girl is to

A. change to a 24-kcal/oz formula


B. discontinue furosemide therapy
C. increase the volume of 20-kcal/oz formula
D. place a gastrostomy feeding tube
E. start parenteral nutrition

Copyright 2009 by the American Academy of Pediatrics

page 200

2009 PREP SA on CD-ROM


Critique: 94

Preferred Response: A

Young infants who have cardiac dysfunction, such as the girl described in the vignette, often
have difficulty ingesting sufficient calories for growth because they frequently have increased
caloric expenditure with feeding. Increased caloric intake may place the infants at risk of fluid
overload, necessitating the use of diuretics. Because an increased volume of feedings with a 20kcal/oz formula may place the infant at risk of fluid overload, use of a more concentrated formula
is required. Accordingly, the infant described in the vignette should be changed to a 24-kcal/oz
formula. Increasing the caloric density of feedings can meet the goals of increased calories and
"relative" fluid restriction.
Discontinuation of furosemide likely would lead to weight gain from fluid retention, not a true
weight gain. A gastrostomy tube provides a conduit for feeding and probably plays a role in a
child incapable of taking in sufficient calories (eg, chronic renal failure or severe developmental
delay), but the child in the vignette deserves a trial of high-calorie feedings before subjecting her
to this surgical procedure. Finally, parenteral nutrition is not a suitable option because this child
has a functional gastrointestinal tract and can tolerate enteral nutrition.
References:
Kelleher DK, Laussen P, Teixeira-Pinto A, Duggan C. Growth and correlates of nutritional status
among infants with hypoplastic left heart syndrome (HLHS) after stage 1 Norwood procedure.
Nutrition. 2006; 22:237-244. Abstract available at: http://www.ncbi.nlm.nih.gov/pubmed/16500550
Pillo-Blocka F, Adatia I, Sharieff W, McCrindle BW, Zlotkin S. Rapid advancement to more
concentrated formula in infants after surgery for congenital heart disease reduces duration of
hospital stay: a randomized clinical trial. J Pediatr. 2004;145:761-766. Abstract available at:
http://www.ncbi.nlm.nih.gov/pubmed/15580197
Yahav J, Avigad S, Frand M, et al. Assessment of intestinal and cardiorespiratory function in
children with congenital heart disease on high-caloric formulas. J Pediatr Gastroenterol Nutr.
1985;4:778-785. Abstract available at: http://www.ncbi.nlm.nih.gov/pubmed/4045636

Copyright 2009 by the American Academy of Pediatrics

page 201

2009 PREP SA on CD-ROM


Question: 95

An 18-month-old girl has been having an intermittent nonproductive cough for the past 6 months.
Her parents state that the cough awakens the toddler at night a few times a month and occurs
when playing vigorously. During a recent upper respiratory tract illness, her cough worsened
and occurred daily for 3 weeks. On physical examination, there is no nasal discharge, and the
toddler appears healthy.
Of the following, the MOST likely diagnosis is

A. asthma
B. atypical pneumonia
C. gastroesophageal reflux
D. sinusitis
E. upper airway cough syndrome

Copyright 2009 by the American Academy of Pediatrics

page 202

2009 PREP SA on CD-ROM


Critique: 95

Preferred Response: A

The chronic cough that is exacerbated during the night, with activity, and during an upper
respiratory tract infection described for the child in the vignette most likely represents asthma.
Chronic cough typically is defined as one that persists for more than 8 weeks. When the
patient's chest radiograph appears normal, three causes account for 95% of chronic coughs:
asthma, gastroesophageal reflux (GER), and upper airway cough syndrome (UACS)
(previously termed postnasal drip syndrome) (Item C95).
Asthma usually develops in early childhood, with 80% of patients reporting symptoms prior
to age 6 years. Symptoms may include cough, wheezing, shortness of breath, and chest
tightness. The most common trigger for infants and toddlers is a viral upper respiratory tract
infection (URI). Fortunately, URI-induced wheezing resolves in most infants by age 6 years (socalled "transient wheezers"). Those who continue to have asthma symptoms after age 6 are at
greater risk for persistent asthma.
UACS encompasses allergic rhinitis, nonallergic rhinitis, and sinusitis. Allergic rhinitis
typically occurs in children older than 3 years of age and is associated with other ocular and
nasal symptoms, such as pruritus, sneezing, and rhinorrhea. Sinusitis also is characterized by
rhinorrhea and postnasal symptoms.
Atypical pneumonia caused by Mycoplasma pneumoniae and Chlamydophila pneumoniae
(previously termed Chlamydia pneumoniae ) may present at any age, although it is unusual prior
to age 3 years. Characteristic constitutional symptoms include fever, malaise, and headache.
Cough can represent the sole manifestation of GER, but GER usually becomes
symptomatic during the first few postnatal months, improving by 12 months of age. GER may
worsen at night during supine positioning, but exercise and URIs are uncommon precipitating
factors for GER symptoms.
References:
Liu AH, Covar RA, Spahn JD, Leung DYM. Childhood asthma. In: Kleigman RM, Behrman RE,
Jenson HB, Stanton BF, eds. Nelson Textbook of Pediatrics. 18th ed. Philadelphia, Pa:
Saunders Elsevier; 2007:953-969
Weinberger M, Abu-Hasan M. Pseudo-asthma: when cough, wheezing, and dyspnea are not
asthma. Pediatrics. 2007;120:855-864. Available at:
http://pediatrics.aappublications.org/cgi/content/full/120/4/855

Copyright 2009 by the American Academy of Pediatrics

page 203

2009 PREP SA on CD-ROM


Question: 96

A 10-year-old boy comes to the office 2 days after falling off of his bicycle and injuring his
forehead. He denies vomiting or headache but complains of a runny nose. Physical examination
reveals a well-appearing boy who has a large ecchymotic swelling over the central portion of his
forehead with an overlying abrasion. The area is diffusely tender to palpation, and there is a
depression over the right lateral aspect of the swelling. Erythema around the abrasion is minimal,
and no purulent drainage is present. Clear fluid is draining from his right naris. The remainder of
his physical examination findings are normal. You order a computed tomography scan (Item
Q96).
Of the following, the MOST appropriate treatment of this boys injury should include

A. decongestants
B. nasal packing
C. no specific treatment
D. prophylactic antibiotics
E. surgical repair

Copyright 2009 by the American Academy of Pediatrics

page 204

2009 PREP SA on CD-ROM


Critique: 96

Preferred Response: E

The history of trauma and the physical findings of forehead ecchymosis with clear, persistent
rhinorrhea described for the boy in the vignette should raise concern for a cerebrospinal fluid
(CSF) leak. Prompt radiologic imaging can aid in determining the cause. In most cases, a frontal
sinus fracture is the cause, as exhibited by the boy in the vignette, and head computed
tomography (CT) scan usually is adequate to identify the injury (Item C96). If an obvious
fracture is not seen on CT scan, further imaging with magnetic resonance imaging or
radionucleotide cisternography may be necessary to localize the source. Testing of the fluid for
glucose or protein is not diagnostically sensitive and should not be used to determine further
evaluation and treatment. The incidence of CSF leak following frontal sinus fracture is nearly
20% in children, twice the rate seen in adults. The primary concern in patients who have CSF
rhinorrhea is the development of central nervous system (CNS) infections such as meningitis or
brain abscess. Such complications are seen most commonly after trauma and in patients whose
leaks do not resolve spontaneously within 7 days.
Anterior wall frontal sinus fractures are repaired primarily for cosmetic reasons, but those
involving the posterior wall (which occurs in 70% of these injuries in children) require open
reconstruction to ensure that the communication between the sinus and brain is eliminated.
Decongestants and nasal packing are not indicated in the treatment of CSF rhinorrhea. The
use of prophylactic antibiotics for the prevention of CNS infection is controversial, although most
studies suggest that the risk of developing antibiotic resistance is greater than the potential
prevention of meningitis.
References:
Kellman RM. Maxillofacial trauma. In: Cummings CW, Flint PW, Haughey BH, Robbins KT,
Thomas JR eds. Cummings Otolaryngology: Head & Neck Surgery. 4th ed. Philadelphia, Pa:
Mosby Elsevier; 2005:chap 26
Kerr JT, Chu FW, Bayles SW. Cerebrospinal fluid rhinorrhea: diagnosis and management.
Otolaryngol Clin North Am. 2005;38:597-611. Abstract available at:
http://www.ncbi.nlm.nih.gov/pubmed/16005720
Kravitz PR, Koltai PJ. Pediatric facial fractures. In: Cummings CW, Flint PW, Haughey BH,
Robbins KT, Thomas JR eds. Cummings Otolaryngology: Head & Neck Surgery. 4th ed.
Philadelphia, Pa: Mosby Elsevier; 2005:chap 202

Copyright 2009 by the American Academy of Pediatrics

page 205

2009 PREP SA on CD-ROM


Question: 97

A 16-year-old boy in your practice has cystic fibrosis. As a complication of his illness, he has
developed cirrhosis and cholestasis. He now complains of shaky hands. Neurologic examination
demonstrates hyporeflexia and tremor with hands outstretched.
Of the following, the patients symptoms are MOST consistent with deficiency of

A. vitamin A
B. vitamin B1 (thiamine)
C. vitamin C
D. vitamin D
E. vitamin E

Copyright 2009 by the American Academy of Pediatrics

page 206

2009 PREP SA on CD-ROM


Critique: 97

Preferred Response: E

Because the young man described in the vignette has chronic cholestasis, he is at risk for
developing deficiency of any of the fat-soluble vitamins, including vitamins A, D, E, and K. His
neurologic symptoms of tremor and hyporeflexia most strongly suggest vitamin E deficiency.
Vitamin E (tocopherol) is an important factor in stabilizing the lipid membrane of the red blood
cell and the lipids in the myelin sheath of neurons. Therefore, the most common presenting
features of hypovitaminosis E are hemolysis (primarily reported in preterm infants) and
peripheral neuropathy (identified in infants and children who have chronic cholestasis,
pancreatic insufficiency, or malabsorption).
Supplementation of formulas and parenteral nutrition with vitamin E has reduced
substantially the incidence of hemolysis in the vitamin E-deficient preterm infant. However,
patients who have cystic fibrosis or cholestatic liver disease require both monitoring of vitamin E
concentrations and supplementation with vitamin E. Because vitamin E is a fat-soluble vitamin,
those who have cholestasis may have difficulty absorbing alpha-tocopherol, the form of vitamin
E available in most dietary supplements. For this reason, d-alpha-tocopheryl polyethylene glycol
1,000 succinate, a water-soluble form of vitamin E, should be given to patients who have
significant cholestatic liver disease. The recommended dose for a patient who has cholestatic
liver disease is 15 to 25 IU/kg per day.
Deficiency of vitamin A, B1, C, or D would not be expected to cause such a clinical
presentation. Vitamin A deficiency causes impaired vision ("night blindness") and corneal ulcers;
vitamin B1 deficiency can cause myopathy and heart failure ("beriberi"); vitamin C deficiency
causes irritability, bone lesions, and bruising (scurvy); and vitamin D deficiency causes
osteopenia or rickets.
References:
Harmatz P, Burensky E, Lubin B. Nutritional anemias. In: Walker WA, Watkins JB, Duggan C,
eds. Nutrition in Pediatrics. 3rd ed. Hamilton, Ontario, Canada: BC Decker; 2003:830-847
Spinozzi NS. Hepatobiliary diseases. In: Hendricks KM, Duggan C. Manual of Pediatric Nutrition.
4th ed. Hamilton, Ontario, Canada: BC Decker; 2005:586-592

Copyright 2009 by the American Academy of Pediatrics

page 207

2009 PREP SA on CD-ROM


Question: 98

You are making rounds with medical students in the neonatal intensive care unit and examining a
2-kg, 34 weeks gestation newborn whose mother had gestational diabetes mellitus. The infant
has no respiratory distress. A medical student asks how to test for fetal lung maturity to predict
the risk of neonatal respiratory distress syndrome in the offspring of a pregnant woman who has
diabetes mellitus.
Of the following, the MOST appropriate test is

A. disaturated lecithin presence


B. lecithin:sphingomyelin ratio
C. phosphatidylglycerol presence
D. phosphatidylinositol presence
E. total surfactant activity

Copyright 2009 by the American Academy of Pediatrics

page 208

2009 PREP SA on CD-ROM


Critique: 98

Preferred Response: E

The human fetus continues to develop lung maturity up until a term gestation, but the lungs
generally are sufficiently mature to maintain extrauterine respiration by 36 weeks' gestation.
Infants delivered prior to this time have variable degrees of pulmonary maturity as surfactant
composition, synthesis, and storage change with advancing fetal development. Tests for
determining fetal lung maturation depend on amniotic fluid analysis for surfactant presence and
composition because fetal surfactant leaves the lung as an effluent that ebbs and flows with fetal
breathing movement and is balanced by fetal swallowing.
In general, in the healthy fetus, growth, size, and maturation typically are linked with
gestational age. Fetal lung maturity has been assessed using the lecithin-to-sphingomyelin ratio
(L:S) for more than 30 years. This test is time-consuming and requires thin-layer
chromatography. It depends on fetal lung fluid flowing into the amniotic fluid and altering the
amniotic fluid phospholipid composition, with the results expressed as the ratio of a lecithin
(phosphatidylcholine) to sphingomyelin per milliliter of amniotic fluid. The sphingomyelin content
of amniotic fluid decreases after 32 weeks' gestational age, while the lecithin content, including
that portion that is disaturated, increases. The L:S ratio is designed to account for varying
amniotic fluid volumes that generally cannot be measured precisely. An L:S ratio of 2.0 typically
is achieved by 35 weeks' gestation. Clinically, respiratory distress syndrome (RDS) due to
surfactant deficiency is very unlikely if the L:S ratio is 2.0 or greater, it is indeterminate if the L:S
ratio is 1.5 to 2.0, and the incidence of RDS is high if the L:S ratio is less than 1.0. Of note, the
L:S ratio can increase over a period of several days or can be induced by the administration of
glucocorticoids to the mother.
Phosphatidylinositol (PI) is a pulmonary phospholipid that increases throughout gestation, as
does disaturated lecithin. PI concentrations decrease after 35 weeks' gestation. The mere
presence of PI or disaturated lecithin, therefore, does not indicate lung maturity.
Phosphatidylglycerol (PG) generally can be detected in the amniotic fluid at 36 weeks' gestation.
Due to its appearance relatively late in gestation, it has been used as an indicator of pulmonary
surfactant maturity.
In pregnancies complicated by maternal diabetes, a mature L:S ratio (>2.0) or the presence
of PG may not indicate pulmonary surfactant maturity. In this setting, fetal lung maturity is
determined best using a quantitative analysis of total surfactant activity. This rapidly determined,
automated fluorescence polarization assay measures all surfactant phospholipids in amniotic
fluid referenced as milligrams of surfactant per gram of albumin present. Recent data suggest
that a total surfactant activity of greater than 45 mg surfactant phospholipids per gram of albumin
is as good as, or better than, an L:S ratio of 2.0 in predicting fetal lung maturity, especially in the
presence of maternal diabetes.
References:
Jobe AH. Lung development and maturation. In: Martin RJ, Fanaroff AA, Walsh MC, eds.
Fanaroff and Martin's Neonatal-Perinatal Medicine. 8th ed. Philadelphia, Pa: Mosby Elsevier;
2006:1069-1086
Grenache DG, Gronowski AM. Fetal lung maturity. Clin Biochem. 2006;39:1-10. Abstract
available at: http://www.ncbi.nlm.nih.gov/pubmed/16303123
Winn-McMillan T, Karon BS. Comparison of the TDx-FLM II and lecithin to sphingomyelin ratio
assays in predicting fetal lung maturity. Am J Obstet Gynecol. 2005;193:778-782. Abstract
available at: http://www.ncbi.nlm.nih.gov/pubmed/16150274

Copyright 2009 by the American Academy of Pediatrics

page 209

2009 PREP SA on CD-ROM


Question: 99

At the end of the summer, you notice an increase in the number of preparticipation sports
examinations you are performing. You are pleased at the number of your patients who are
involved in sports activities but are reminded that many medical conditions preclude sports
participation and must be screened for during the preparticipation visit.
Of the following, the medical condition that is considered a CONTRAINDICATION for sports
participation is

A. a boy who has chronic leukemia and splenomegaly wishing to play golf
B. a boy who has insulin-dependent diabetes wishing to play tennis
C. a boy who has well-controlled seizures wishing to participate in basketball
D. a girl who has Marfan syndrome wishing to participate in gymnastics
E. a girl who has one ovary wishing to participate in softball

Copyright 2009 by the American Academy of Pediatrics

page 210

2009 PREP SA on CD-ROM


Critique: 99

Preferred Response: D

Sports participation can play an important role in the social development of children and
adolescents by teaching leadership and team-building skills and encouraging physical fitness.
Most children can be cleared easily for such participation, but not all sports are safe for children
and adolescents, and some medical conditions warrant special consideration with regard to
participation in some sports. In all cases, the clinician should perform a complete preparticipation
sports physical examination and discuss the implications of the sports the patient plans to
pursue.
Sports can be classified by contact level (Item C99A). Children who have splenomegaly,
acute hepatomegaly, and contagious skin lesions should avoid contact sports but can
participate in noncontact sports. Children who have a single kidney should be advised to avoid
high-contact sports, but those who have other single organs, such as ovaries and testes, can
be cleared because the risk of injury is low or protective gear can be worn. Protective eyewear
can reduce the risk of injury in the case of a single functional eye. It is prudent for the clinician to
advise such patients of the risk to the remaining organ during sports participation, and some
advocate having the parents and the athlete sign a document that indicates their understanding
and acceptance of such risk.
Sports also may be classified by intensity (dynamic and static demand) (Item C99B). This
classification is especially important for children and adolescents who have cardiovascular
diseases. For example, children who have ventricular dysfunction should avoid high dynamic
sports; those who have significant essential hypertension, left heart obstructive disease, or an
increased risk of aortic dissection should avoid high static sports. Accordingly, a girl who has
Marfan syndrome should avoid gymnastics, a sport that has potentially high static intensity.
Several cardiac diseases, such as acute pericarditis or myocarditis, cardiomyopathy, severe
pulmonary hypertension, and right-to-left shunting, are absolute contraindications to participation
in sports. A full listing of these conditions is outlined in the 36th Bethesda Conference guidelines.
Patients who have well-controlled epilepsy can be cleared to participate in most sports, but
those whose seizures are poorly controlled should be advised to avoid sports in which they
could sustain significant injury should a seizure occur during participation, such as swimming,
diving, and riflery. Patients who have diabetes should be encouraged to participate in sports, but
they should pay attention to hydration and insulin therapy, especially with sports that last longer
than 30 minutes.
References:
36th Bethesda conference: eligibility recommendations for competitive athletes with
cardiovascular abnormalities. J Am Coll Cardiol. 2005;45:1312-1375. Available at:
http://content.onlinejacc.org/cgi/reprint/45/8/1312
American Academy of Pediatrics Committee on Sports Medicine and Fitness. Medical conditions
affecting sports participation. Pediatrics. 2001;107:1205-1209. Available at:
http://pediatrics.aappublications.org/cgi/content/full/107/5/1205
Metzl JD. Preparticipation examination of the adolescent athlete: part 1. Pediatr Rev.
2001;22:199-204. Available at: http://pedsinreview.aappublications.org/cgi/content/full/22/6/199
Metzl JD. Preparticipation examination of the adolescent athlete: part 2. Pediatr Rev.
2001;22:227-239. Available at: http://pedsinreview.aappublications.org/cgi/content/full/22/7/227
Singh A, Silberbach M. Consultation with the specialist: cardiovascular preparticipation sports
screening. Pediatr Rev. 2006;27:418-424. Available at:
http://pedsinreview.aappublications.org/cgi/content/full/27/11/418

Copyright 2009 by the American Academy of Pediatrics

page 211

2009 PREP SA on CD-ROM


Question: 100

A child presents to a clinic associated with a disaster relief shelter after a hurricane destroys the
community. The mother states that they have run out of the creams prescribed for her
daughters eczema before the storm, and the child is itchy. She is concerned because there are
some blisters and crusting in the antecubital fossae and popliteal fossa where the itching is
worst. Physical examination reveals erosions (Item Q100) and erythema surrounding areas of
lichenification, with a few vesicles both in clusters and scattered.
Of the following, the MOST likely pathogen involved in this pattern of infection is

A. herpes simplex virus


B. human papillomavirus
C. Sporothrix sp
D. varicella-zoster virus
E. viridans streptococci

Copyright 2009 by the American Academy of Pediatrics

page 212

2009 PREP SA on CD-ROM


Critique: 100

Preferred Response: A

Children who have atopic dermatitis are prone to recurrent skin infections, particularly with
Staphylococcus aureus and herpes simplex virus (HSV), for several reasons. Exacerbations of
eczema disrupt the skin's protective barrier. The failure to produce endogenous antimicrobial
peptides has been offered as a reason for an increase risk for infection with S aureus.
Dissemination of cutaneous viral infections (eg, HSV and molluscum contagiosum) is believed to
be due to defects in cytokine production and T-cell function.
Secondary infection with S aureus may result in "weepy" or crusted erosions (Item C100A).
Use of topical and systemic antistaphylococcal antibiotics, with attention to the emergence of
community-acquired antibiotic-resistant strains, is indicated. Use of topical immune modulators
(eg, tacrolimus) should be interrupted during the infection because of the potential for inhibiting
local immune response and the development of irritation, including burning and stinging.
Eczema herpeticum results when areas of active dermatitis are infected by HSV. Initially,
there may be an increase in complaints of itching and scratching, with a noticeable increase in
erythema; fever may be present. As described for the child in the vignette, clustered, often
umbilicated vesicles appear; these rupture, forming ulcers that have a "punched-out"
appearance (Item C100B). Infection may become disseminated and sometimes is fatal. For
those who have extensive involvement, recommended treatment is parenteral acyclovir.
Sporotrichosis is a dimorphic fungus that can be isolated from soil and plants such as roses;
it also may be found in hay, straw, and decaying vegetation. It may cause infection at a site of
minor trauma that appears as a solitary violaceous nodule (Item C100C). Although infections
caused by streptococci (viridans and enterococci) may be observed in immunocompromised
hosts and in neonates, they do not cause the type of disease described for the child in the
vignette. Papules, not vesicles, are characteristic of human papillomavirus infection of the skin.
Primary infection with varicella-zoster virus results in varicella, with individual, not grouped,
vesicles that rupture, leaving shallow erosions, not "punched-out" ulcers.
References:
American Academy of Pediatrics. Non-group A or B streptococcal and enterococcal infections.
In: Pickering LK, Baker CJ, Long SS, McMillan JA, eds. Red Book: 2006 Report of the
Committee on Infectious Diseases. 27th ed. Elk Grove Village, Ill: American Academy of
Pediatrics; 2006:627-629
American Academy of Pediatrics. Sporotrichosis. In: Pickering LK, Baker CJ, Long SS, McMillan
JA, eds. Red Book: 2006 Report of the Committee on Infectious Diseases. 27th ed. Elk Grove
Village, Ill: American Academy of Pediatrics; 2006:595-597
Bunikowski R, Mielke M, Skarabis H, et al. Prevalence and role of serum IgE antibodies to the
Staphylococcus aureus-derived superantigens SEA and SEB in children with atopic dermatitis. J
Allergy Clin Immunol. 1999;103:119-124. Abstract available at:
http://www.ncbi.nlm.nih.gov/pubmed/9893195
Horii KA, Simon SD, Liu DY, Sharma V. Atopic dermatitis in children in the United States, 19972004: visit trends, patient and provider characteristics, and prescribing patterns. Pediatrics.
2007;120:e527-e534. Available at:
http://pediatrics.aappublications.org/cgi/content/full/120/3/e527
Knoell KA, Greer KE. Atopic dermatitis. Pediatr Rev. 1999;20:46-52. Available at:
http://pedsinreview.aappublications.org/cgi/content/full/20/2/46
Stanbury LR. Herpes simplex virus. In: Kliegman RM, Behrman RE, Jenson HB, Stanton BF,
eds. Nelson's Textbook of Pediatrics. Philadelphia, Pa: Saunders Elsevier; 2007:1360-1365
Waggoner-Fountain LA, Grossman LB. Herpes simplex virus. Pediatr Rev. 2004;25:86-93.

Copyright 2009 by the American Academy of Pediatrics

page 213

2009 PREP SA on CD-ROM

Available at: http://pedsinreview.aappublications.org/cgi/content/full/25/3/86

Copyright 2009 by the American Academy of Pediatrics

page 214

2009 PREP SA on CD-ROM


Question: 101

A 4-week-old infant who was born at term without any complications ate well and gained weight
for the first 3 weeks after birth. Over the last week, however, his mother reports that he appears
hungry but fatigues with feeding and now takes twice as long to complete his feeding as he did 1
week ago. He also breathes fast during his feedings and stops frequently to "catch his breath."
Of the following, the MOST likely explanation for the symptoms in this infant is

A. aspiration syndrome
B. congestive heart failure
C. gastroesophageal reflux disease
D. inborn error of metabolism
E. pneumonia

Copyright 2009 by the American Academy of Pediatrics

page 215

2009 PREP SA on CD-ROM


Critique: 101

Preferred Response: B

The symptoms described for the child in the vignette are typical of progressive congestive heart
failure (CHF). The "hunger" described by the mother suggests that the problem results from the
infant's inability to take enough calories for satiation and growth. Such so-called poor feeding is
due to his inability to generate a prolonged suck while maintaining nasal breathing because of the
tachypnea that is caused by pulmonary congestion. Thus, the infant seems to stop sucking and
"catch his breath." Decreased caloric intake coupled with increased caloric expenditure caused
by tachypnea and tachycardia makes it difficult for the infant to gain weight. Indeed, weight loss
is common in infants who have CHF.
CHF is a clinical syndrome that reflects the inability of the myocardium to meet the metabolic
requirements of the body, including those for growth. Congenital heart defects are the most
common reason for pediatric heart failure, and the failure develops most frequently during early
infancy. CHF results from excessive workload imposed on the cardiac muscle that usually is
caused by the structural defects. The cardiac defects may impose an excessive volume load on
the left ventricle (eg, large ventricular septal defect, atrioventricular septal defect), an excessive
pressure load on the ventricle (eg, aortic stenosis), or a combination of volume and pressure
load (eg, ventricular septal defect with coarctation). Less commonly, CHF may be caused by an
intrinsic alteration in myocardial performance, which could result from an inflammatory or an
infectious process that directly affects the myocardium and depresses its contractile function.
Aspiration syndrome typically is an acute event resulting from the passage of
gastrointestinal contents (including food) into the lungs. The symptoms usually are acute and not
associated with poor feeding in spite of hunger. Similarly, gastrointestinal reflux presents more
acutely with signs of discomfort that might include arching during feedings. Pneumonia is
associated with other signs of infection, including tachypnea when at rest, fever, or change in
behavior. Inborn errors of metabolism, although less common, are important to consider in the
neonate and infant who is not feeding well. Most commonly, they are associated with symptoms
that are present both at rest and during feeding as well as other systemic signs.
References:
Balfour I. Management of chronic congestive heart failure in children. Curr Treat Options
Cardiovasc Med. 2004;6:407-416. Abstract available at:
http://www.ncbi.nlm.nih.gov/pubmed/15324616
Dreyer WJ, Fisher DJ. Clinical recognition and management of chronic congestive cardiac
failure. In: Garson A Jr, Bricker JT, Fisher DJ, Neish SR, eds. The Science and Practice of
Pediatric Cardiology. 2nd ed. Baltimore, Md: Williams & Wilkins, 1998:2309-2325
Silberbach M, Hannon D. Presentation of congenital heart disease in the neonate and young
infant. Pediatr Rev. 2007;28:123-131. Available at:
http://pedsinreview.aappublications.org/cgi/content/full/28/4/123
Talner NS, McGovern JJ, Carboni MP. Congestive heart failure. In: Moller JH, Hoffman JIE, eds.
Pediatric Cardiovascular Medicine. Philadelphia, Pa: Churchill Livingstone; 2000:817-829

Copyright 2009 by the American Academy of Pediatrics

page 216

2009 PREP SA on CD-ROM


Question: 102

A 10-year-old boy has double vision and drooping eyelids. On physical examination, he is
afebrile and has normal mentation. Pupillary responses are normal, but he has bilateral ptosis.
He cannot fully adduct his right eye. You note that his ptosis increases with sustained upward
gaze (Item Q102). Bedside forced vital capacity is normal.
Of the following, the test MOST likely to confirm the diagnosis is

A. brainstem auditory evoked potentials


B. cold caloric testing
C. edrophonium test
D. lumbar puncture
E. visual evoked potentials

Copyright 2009 by the American Academy of Pediatrics

page 217

2009 PREP SA on CD-ROM


Critique: 102

Preferred Response: C

The child described in the vignette presents with progressive fatigable weakness affecting his
ocular muscles and bilateral eyelids. Fatigable weakness means that at the onset of a voluntary
movement, or after a period of rest, muscle strength is normal. However, after a period of
sustained use, muscle strength diminishes. Such a pattern is characteristic of weakness due to
pathology at the neuromuscular junction.
New-onset weakness mandates urgent evaluation and consideration of problems involving
airway protection and respiratory insufficiency, particularly if there is evidence on physical
examination of generalized weakness. If the pharyngeal muscles are affected, speech sounds
hyponasal. Negative inspiratory force or forced vital capacity should be documented promptly
because these findings determine whether the child should be transferred to an intensive care
unit for ventilatory monitoring and support.
The first step in the diagnostic process is to localize the problem to the proper level of the
nervous system: brain, brainstem/cerebellum, spinal cord, nerve, junction, or muscle. Such a
determination not only allows for urgent appropriate diagnosis and treatment but can reduce
unnecessary discomfort, risks, and costs of inappropriate diagnostic testing. Consultation with a
neurologist can be useful before ordering testing. In this case, bilateral weakness in the eyelids
and normal mentation make a cerebral or midbrain cause unlikely. Lateral gaze palsy from a
pontine lesion also should affect the full face (as occurs in Bell palsy), which is not present. A
multifocal cranial neuropathy is rare and unlikely. The ocular fatigability suggests a
neuromuscular junction problem such as myasthenia gravis.
The diagnosis of myasthenia gravis can be confirmed with the edrophonium test ("Tensilon
test"), a specialized assessment using the acetylcholinesterase inhibitor edrophonium. Because
the test can pose a risk of life-threatening bradycardia, it should be undertaken by an
experienced physician with atropine at the bedside. Neurologic examination before and after
administration ideally should be documented on video (Item C102).
Lumbar puncture is the appropriate test for suspected Guillain-Barr syndrome (GBS), an
acute inflammatory demyelinating polyneuropathy. Typically, affected children present with
weakness beginning in the proximal legs, pain, and absent reflexes. A variant of GBS can
involve predominantly the brainstem and cerebellum, but usually weakness is more extensive
than described for this child, and the weakness is not fatigable. Further, at this early stage, the
characteristic cerebrospinal fluid findings in GBS, normal cells and high protein, often have not
emerged. Therefore, lumbar puncture is not the preferred study in this setting. Cold caloric
testing evaluates vestibular function. This patient has no nystagmus and does not require this
uncomfortable test. Neurophysiologic testing with auditory or visual evoked potentials is not
needed when no clinical evidence suggests that these systems are affected.
Treatment of myasthenia gravis is complex and is managed best by an experienced
neurologist. Treatment may include immunomodulation (eg, intravenous immunoglobulin or
plasmapheresis) and acetylcholinesterase inhibitors. Imaging to assess for the presence of a
thymoma also is standard practice in a child. The patient and family also should be educated
about disease management and early recognition of impending myasthenic crisis.
References:
Andrews PI. Autoimmune myasthenia gravis in childhood. Semin Neurol. 2004;24:101-110.
Abstract available at: http://www.ncbi.nlm.nih.gov/pubmed/15229797
Mehta S. Neuromuscular disease causing acute respiratory failure. Respir Care. 2006;51:10161023. Available at: http://www.rcjournal.com/contents/09.06/09.06.1016.pdf
Parr JR, Jayawant S. Childhood myasthenia: clinical subtypes and practical management. Dev
Med Child Neurol. 2007;49:629-635. Available at: http://www.blackwellsynergy.com/doi/abs/10.1111/j.1469-8749.2007.00629.x
Sarnat HB. Disorders of neuromuscular transmission and of motor neurons. In: Kliegman RM,
Copyright 2009 by the American Academy of Pediatrics

page 218

2009 PREP SA on CD-ROM

Behrman RE, Jenson HB, Stanton BF, eds. Nelson Textbook of Pediatrics. 18th ed. Philadelphia,
Pa: Saunders Elsevier; 2007:2554-2558

Copyright 2009 by the American Academy of Pediatrics

page 219

2009 PREP SA on CD-ROM


Question: 103

You are called to the newborn nursery to evaluate an infant who has a limb anomaly. The infant
is normally grown and vigorous. On physical examination, you note a terminal transverse limb
defect at the distal aspect of the right forearm, resulting in absence of the hand on that side (Item
Q103).
Of the following, these findings are MOST likely related to prenatal exposure to

A. alcohol
B. cocaine
C. marijuana
D. methamphetamine
E. tobacco

Copyright 2009 by the American Academy of Pediatrics

page 220

2009 PREP SA on CD-ROM


Critique: 103

Preferred Response: B

Despite extensive medical literature on the teratogenic effects of cocaine, the degree to which
maternal cocaine use affects the developing embryo/fetus remains largely uncertain. Many
studies on this subject are fraught with design flaws, and a bias toward publishing adverse
outcomes has been suggested. Adding to this confusion is the fact that women who abuse
cocaine during pregnancy typically do not use the drug in isolation; they often drink alcohol,
smoke cigarettes, and use other recreational drugs as well. Even so, prenatal cocaine exposure
generally is believed to be associated with vascular disruptive events in the embryo/fetus due to
its potent vasoconstrictive and hypertensive effects. Such events include cerebral infarction,
urogenital anomalies, and limb reduction defects, as described for the infant in the vignette (Item
C103A). In addition, the incidence of placental abruption appears to be increased in cocaineexposed pregnancies. Although neurobehavioral alterations have been reported in neonates
who were exposed to cocaine prenatally, it is not clear that these are due solely to cocaine
exposure.
Prenatal alcohol exposure is associated with a spectrum of abnormalities termed fetal
alcohol spectrum disorder. Individuals who have fetal alcohol syndrome, which is the most
severe end of the spectrum, typically exhibit intrauterine growth restriction, microcephaly,
neurodevelopmental impairment, and a characteristic pattern of dysmorphic features (Item
C103B). Although they may have hypoplasia of the distal fingers and toes, they typically do not
have more severe limb deficiencies.
The effects of cigarette smoking on pregnancy outcome have been, and continue to be,
studied extensively. Cigarette smoking is associated with an increased risk for miscarriage,
reduced fetal growth, and abnormal placentation. Generally, cigarette smoking is not associated
with major congenital anomalies, although there may be an increase in facial clefting. An
association with congenital limb deficiencies has been reported in some studies, but it has not
been borne out in others.
No increased incidence of birth defects appears to be associated with marijuana or
methamphetamine use during pregnancy.
References:
Cigarette smoking, methamphetamine, alcohol, cocaine, marijuana. Reprotox. Available for
subscription at: http://www.reprotox.org
Cigarette smoking, methamphetamine, alcohol, cocaine, marijuana. Teris. Available for
subscription at http://depts.washington.edu/terisweb/teris/
Hoyme HE, May PA, Kalberg WO, et al. A practical clinical approach to diagnosis of fetal alcohol
spectrum disorders: clarification of the 1996 Institute of Medicine criteria. Pediatrics.
2005;115:39-47. Available at: http://pediatrics.aappublications.org/cgi/content/full/115/1/39

Copyright 2009 by the American Academy of Pediatrics

page 221

2009 PREP SA on CD-ROM


Question: 104

A 16-year-old girl who attends boarding school in your community comes to your office because
she is feeling depressed. You see her alone for the visit, and she relates that she feels suicidal
at this time and has a plan to kill herself.
Of the following, the BEST description of your obligation to alert her parents to her situation is
that

A. no parental notification is necessary because she is a mature minor


B. no parental notification is necessary because she is an emancipated minor
C. parental notification is necessary due to billing issues
D. parental notification is necessary due to her serious threats of self-harm
E. parental notification is prohibited by the Health Insurance Portability and Accountability Act

Copyright 2009 by the American Academy of Pediatrics

page 222

2009 PREP SA on CD-ROM


Critique: 104

Preferred Response: D

Confidentiality and consent issues related to adolescents are often complex and problematic.
Confidentiality is defined as an agreement between the patient and clinician that medical
information will not be shared without the patient's explicit permission. For the girl described in
the vignette, confidentiality must be breached with parental notification due to the serious nature
of her threat to herself. Such breaching of confidentiality also would be necessary if she posed a
threat to others.
Parental notification is not necessary for billing purposes. However, confidentiality may be
breached inadvertently by billing procedures, such as parents receiving an explanation of
benefits document. The physician should explain such possible breaches and work with the
adolescent to avoid them.
Informed consent has the vital components of being given voluntarily, the patient having all
of the information necessary to make an informed decision, and the individual being capable of
making a decision based on the information provided. All states require parental consent for
medical care for minors, with some notable exceptions, including emergency care; care for the
"emancipated minor"; and care related to certain conditions such as pregnancy, sexually
transmitted infections, contraception, substance abuse, and mental illness. The exceptions vary
by state. An emancipated minor is a legal status existing for minors who are living apart from
parents and are financially independent, but this designation varies among states, and some
states do not have this legal designation. Marriage, parenthood, military service, and living
independently are common criteria for the emancipated status. Generally, this status allows a
minor to consent for all health care. A "mature minor" is one who is deemed by a physician as
capable of giving the same degree of consent for treatment as an adult, provided the treatment
is low-risk. The mature minor rule may be used in states where no state law regarding minor
consent exists. The girl described in the vignette is not financially independent from her parents
and, as a result, cannot be considered emancipated. Because her condition and treatment are
not low-risk, the mature minor principle would not apply.
The Health Insurance Portability and Accountability Act (HIPAA) of 1996 protects the
confidentiality of adolescents who are considered minors under some circumstances. Parents
and guardians have control over health information access for nonemancipated minors, except
in situations in which minors are able to consent to their own health care. HIPAA defers to state
laws that allow or prohibit disclosure of confidential information to parents. HIPAA allows a
physician to disclose information to a parent if a state law requires the physician to do so. If the
state law permits, but does not require, disclosure to a parent, HIPAA allows the physician
discretion to disclose. If state law prohibits the disclosure of information to a parent, disclosure
must not be made without the minor's permission. If there is no state law in place, the physician
has discretion to disclose or not to a parent. For the girl described in the vignette, disclosure is
necessary due to the health threat, not by any HIPAA regulation.
The Center for Adolescent Health and the Law has published a compendium of state laws
that addresses confidentiality and consent and allows clinicians to determine the specifics of the
law in their own state of practice.
References:
English A, Kenney KE. State Minor Consent Laws: A Summary. 2nd ed. Chapel Hill, NC: Center
for Adolescent Health & the Law; 2003
Joffe A. Legal and ethical issues in adolescent health care. In: Osborn LM, DeWitt TG, First LR,
Zenel JA eds. Pediatrics. Philadelphia, Pa: Elsevier Mosby; 2005:1428-1430.
Weddle M, Kokotailo P. Adolescent substance abuse: confidentiality and consent. Pediatr Clin
North Am. 2002;49:301-315. Abstract available at:
http://www.ncbi.nlm.nih.gov/pubmed/11993284
Weddle M, Kokotailo PK. Confidentiality and consent in adolescent substance abuse: an update.

Copyright 2009 by the American Academy of Pediatrics

page 223

2009 PREP SA on CD-ROM

Virtual Mentor: American Medical Association Journal of Ethics. 2005;7(3). Available at:
http://virtualmentor.ama-assn.org/2005/03/pfor1-0503.html

Copyright 2009 by the American Academy of Pediatrics

page 224

2009 PREP SA on CD-ROM


Question: 105

You are assisting a pediatric resident in evaluating a 12-year-old girl who has type 1 diabetes
and has been vomiting for the past 12 hours. Initial laboratory results include:
Blood glucose, 630.0 mg/dL (35.0 mmol/L)
Serum sodium, 150.0 mEg/L (150.0 mmol/L)
Serum potassium, 6.0 mEq/L (6.0 mmol/L)
Serum chloride, 90.0 mEq/L (90.0 mmol/L)
Serum bicarbonate, 10.0 mEq/L (10.0 mmol/L)
The anion gap for this child is CLOSEST to

A. 4
B. 10
C. 50
D. 80
E. 323

Copyright 2009 by the American Academy of Pediatrics

page 225

2009 PREP SA on CD-ROM


Critique: 105

Preferred Response: C

The anion gap is the difference between routinely measured cations and anions in the blood.
Clinically, this is estimated by using the following equation: [Na+]-[Cl-]-[HCO3-]. Thus, the anion
gap for the child described in the vignette is 150-90-10=50. A normal anion gap is 8 to 16 and
reflects unmeasured anions such as proteins, sulfates, phosphates, and organic acids.
Calculation of the anion gap assists the clinician in both classifying metabolic acidosis as normal
anion gap and elevated anion gap and directing further evaluation and treatment.
Metabolic acidosis with an increased anion gap occurs when bicarbonate is lost and
replaced by unmeasured anions such as ketones, lactate, phosphate, or urate. Common
causes of increased anion gap acidosis in children include ketoacidosis, as seen in diabetes
mellitus or starvation; lactic acidosis, as seen in shock or with inborn errors of metabolism; and
renal failure. Increased anion gap acidosis also is seen with the accumulation of toxic
metabolites from ingestions such as methanol, ethylene glycol, and salicylate.
Normal anion gap metabolic acidosis is characterized by loss of bicarbonate with an equal
increase in chloride that preserves the normal anion difference. Common causes of normal
anion gap acidosis in children include diarrhea, renal tubular acidosis, administration of large
volumes of normal saline, and hypoaldosteronism.
References:
Greenbaum LA. Electrolyte and acid-base disorders. In; Kliegman RM, Behrman RE, Jenson
HB, Stanton BF, eds. Nelson Textbook of Pediatrics. 18th ed. Philadelphia, Pa: Saunders
Elsevier; 2007:267-308
Schwaderer AL, Schwartz GJ. Back to basics: acidosis and alkalosis. Pediatr Rev. 2004;25:350357. Available at: http://pedsinreview.aappublications.org/cgi/content/full/25/10/350

Copyright 2009 by the American Academy of Pediatrics

page 226

2009 PREP SA on CD-ROM


Question: 106

The parents of a 12-year-old girl in whom you recently diagnosed type 1 diabetes mellitus ask
you about potential long-term complications. In your discussion, you stress the importance of
blood glucose control to prevent complications and review risk factors for diabetes
complications, including hyperglycemia and tobacco smoking.
Of the following, the MOST important additional risk factor for diabetes complications is

A. celiac disease
B. hypertension
C. hypothyroidism
D. lack of regular exercise
E. undernutrition

Copyright 2009 by the American Academy of Pediatrics

page 227

2009 PREP SA on CD-ROM


Critique: 106

Preferred Response: B

The Diabetes Control and Complications Trial (DCCT) results, published in 1994, demonstrated
unambiguously that glycemic control directly correlates with the long-term prevention of
complications of diabetes mellitus type 1 (DM1). However, additional risk factors, such as
hypertension and cigarette smoking, are almost as important as hyperglycemia in the
development of diabetes complications.
Celiac disease occurs in approximately 6% of individuals who have DM1 in North America
and may hamper diabetes control because of malabsorption of nutrients. It is also a risk factor
for poor bone mineralization in individuals who have DM1. However, it does not alter the risk of
long-term complications directly. Hypothyroidism due to chronic lymphocytic thyroiditis may
develop in 5% or more of people who have DM1, but it is not an independent risk factor for
cardiovascular or other DM1 complications unless it is chronically untreated and affects lipid
metabolism. Lack of regular exercise has been associated with an increase in cardiovascular
risk factors for children who have DM1, but it has not yet been correlated directly with the
development of long-term complications. Undernutrition, unless it is the result of poor diabetes
control, has not been correlated with long-term complications of diabetes.
References:
Freemark M, Levitsky LL. Screening for celiac disease in children with type 1 diabetes: two
views of the controversy. Diabetes Care. 2003;26:1932-1939. Available at:
http://care.diabetesjournals.org/cgi/content/full/26/6/1932
Gallego PH, Wiltshire E, Donaghue KC. Identifying children at particular risk of long-term
diabetes complications. Pediatr Diabetes. 2007;8(suppl 6):40-48. Abstract available at:
http://www.ncbi.nlm.nih.gov/pubmed/17727384
Glastras SJ, Mohsin F, Donaghue KC. Complications of diabetes mellitus in childhood. Pediatr
Clin North Am. 2005;52:1735-1753. Abstract available at:
http://www.ncbi.nlm.nih.gov/pubmed/16301091
Herbst A, Kordonouri O, Schwab KO, Schmidt F, Holl RW, on behalf of the DPV Initiative of the
German Working Group for Pediatric Diabetology Germany. Impact of physical activity on
cardiovascular risk factors in children with type 1 diabetes. Diabetes Care. 2007;30:2098-2100.
Available at: http://care.diabetesjournals.org/cgi/content/full/30/8/2098
Levitsky LL, Misra M. Complications and screening in children and adolescents with type 1
diabetes mellitus. UpToDate Online 15.3. 2008. Available for subscription at:
http://www.uptodateonline.com/utd/content/topic.do?topicKey=pediendo/17677
Raile K, Galler A, Hofer S, et al. Diabetic nephropathy in 27,805 children, adolescents, and adults
with type 1 diabetes: effect of diabetes duration, A1C, hypertension, dyslipidemia, diabetes
onset, and sex. Diabetes Care. 2007;30:2523-2528. Available at:
http://care.diabetesjournals.org/cgi/content/full/30/10/2523

Copyright 2009 by the American Academy of Pediatrics

page 228

2009 PREP SA on CD-ROM


Question: 107

A 12-year-old boy recently took a standardized achievement test at school. His score dropped
from 105 on last years achievement test to 95 on the most recent test. Last season the boy
played hockey and fell down, hitting his head, although he did not lose consciousness. He had
no previous head injury. He was evaluated in the emergency department and had normal
findings on computed tomography scan. The mother asks whether the boy had suffered brain
injury due to his fall that caused him to lose academic skills.
Of the following, the MOST appropriate response is to

A. explain that it is normal to have a small variation between scores


B. recommend neurorehabilitation due to loss of his academic skills
C. restrict the childs contact sports activity for the next season
D. send the child for comprehensive neuropsychological testing
E. send the child for head magnetic resonance imaging

Copyright 2009 by the American Academy of Pediatrics

page 229

2009 PREP SA on CD-ROM


Critique: 107

Preferred Response: A

The small difference in scores on standardized achievement tests described for the boy in the
vignette is normal. To be significant, the change would need to be greater than 1 standard
deviation (15 points). Because the values reported for the boy are within that expected for testretest variation, brain magnetic resonance imaging, contact sports restriction,
neuropsychological testing, and neurorehabilitation are not needed.
Standardized scores measure performance for a group of students, such as all the students
in one school, at a certain grade, or in a single school district. Standardized scores employ a
procedure in which scores are reported in terms of standard deviation units from the mean.
When scores are standardized to the normal bell curve, two thirds of the students in the sample
fall between -1 and +1 standard deviations from the mean. Scores also may be expressed as
percentiles or, in some cases, grade equivalents. Standardized scores are less useful for
evaluating change in an individual student. A single student may be learning more every day and
growing in ability or skill, but still be at the same relative rank compared with his or her peers.
Achievement tests measure what a child has learned in specific subject areas such as reading
or mathematics. Criterion-based testing evaluates how much a student knows or can do using a
specific set of standards, not by comparison with other students, which probably makes it more
useful for identifying whether a student is improving or meeting objectives. A child who has a
significant discrepancy (determined by each state) between an expected achievement score
based on an intelligence quotient test and the actual score may have a specific learning disability
and be eligible for specialized educational services.
References:
Ewen JB, Shapiro BK. Specific learning disabilities. In: Accardo PJ. Capute & Accardo's
Neurodevelopmental Disabilities in Infancy and Childhood. Volume II: The Spectrum of
Neurodevelomental Disabilities. 3rd ed. Baltimore, Md: Paul H. Brookes Publishing Co; 2008:553577
Mahone EM. Psychological assessment. In: Accardo PJ, ed. Capute & Accardo's
Neurodevelopmental Disabilities in Infancy and Childhood. Volume II: The Spectrum of
Neurodevelopmental Disabilities. 3rd ed. Baltimore, MD: Paul H. Brookes Publishing Co;
2008:261-281

Copyright 2009 by the American Academy of Pediatrics

page 230

2009 PREP SA on CD-ROM


Question: 108

The microbiology laboratory called your junior partner today to tell her that the blood culture from
a patient she admitted 2 days ago is growing Haemophilus influenzae type b. Because she has
never treated an infection caused by this organism, she wants to know what antimicrobial agent
would be best to use for her patient.
Of the following, the MOST appropriate antimicrobial agent to treat this infection is

A. ampicillin
B. cefotaxime
C. clindamyin
D. gentamicin
E. vancomycin

Copyright 2009 by the American Academy of Pediatrics

page 231

2009 PREP SA on CD-ROM


Critique: 108

Preferred Response: B

Haemophilus influenzae type b (Hib) is a pleomorphic gram-negative coccobacillus. These types


of organisms are not susceptible to antimicrobial agents used against gram-positive pathogens,
such as clindamycin or vancomycin, or some agents that are effective against other gramnegative organisms, such as the aminoglycosides (eg, gentamicin). Because approximately
30% to 40% of Hib isolates produce beta-lactamase, ampicillin should not be used empirically
until susceptibility testing supports its potential efficacy. For patients who have potentially lifethreatening illness (eg, bacteremia) from Hib, cefotaxime or ceftriaxone has become the
antimicrobial agent of choice, with meropenem or chloramphenicol serving as alternative agents.
References:
American Academy of Pediatrics. Haemophilus influenzae infections. In: Pickering LK, Baker CJ,
Long SS, McMillan JA, eds. Red Book: 2006 Report of the Committee on Infectious Diseases.
27th ed. Elk Grove Village, Ill: American Academy of Pediatrics; 2006:310-318
Tzanakaki G, Mastrantonio P. Aetiology of bacterial meningitis and resistance to antibiotics of
causative pathogens in Europe and in the Mediterranean region. Int J Antimicrob Agents.
2007;29:621-629. Abstract available at: http://www.ncbi.nlm.nih.gov/pubmed/17368858

Copyright 2009 by the American Academy of Pediatrics

page 232

2009 PREP SA on CD-ROM


Question: 109

A 9-year-old previously healthy boy presents for evaluation of a progressively worsening


cellulitis of his left leg. Two days ago, he sustained an abrasion to his shin after falling off his
bicycle onto a gravel road. Over the last 12 hours, he has developed a temperature of 102.0F
(38.9C), and the wound has become very erythematous, swollen, and tender, with some red
streaking. On physical examination, the boy has a temperature of 101.5F (38.6C) and a 5x6cm abrasion of the anterior lateral surface of his left shin that is draining a serosanguineous
discharge. The abrasion is surrounded by an 8-cm area of erythema, swelling, and induration,
with a red streak extending up toward his knee. The area is tender to palpation, and he limps
when walking. There is some shotty left inguinal adenopathy. A complete blood count shows a
peripheral white blood cell count of 16.0x103/mcL (16.0x109/L) with a differential count of 65%
neutrophils, 5% band forms, 25% lymphocytes, and 5% monocytes.
Of the following, the MOST likely pathogen causing this patients condition is

A. Staphylococcus aureus (methicillin-resistant)


B. Staphylococcus aureus (methicillin-sensitive)
C. Staphylococcus epidermidis
D. Streptococcus pyogenes
E. Streptococcus pneumoniae

Copyright 2009 by the American Academy of Pediatrics

page 233

2009 PREP SA on CD-ROM


Critique: 109

Preferred Response: D

Streptococcus pyogenes (group A Streptococcus) is a ubiquitous organism that is the most


common bacterial cause of pharyngitis and can be the source of a variety of other cutaneous
and systemic infections. Streptococcal cellulitis is an acute, spreading inflammation of the skin
and subcutaneous tissues. Clinical findings include local pain, tenderness, swelling, and
erythema that progresses rapidly and may involve large areas of skin. Systemic manifestations
include fever, chills, and malaise with associated lymphangitis (red, tender, linear streaks
directed toward enlarged, tender regional lymph nodes) or bacteremia. The patient described in
the vignette has the classic clinical picture for a cellulitis caused by group A Streptococcus.
Streptococcal pharyngitis primarily occurs among children 5 to 15 years of age, with the
peak incidence seen during the first few years of school. However, all age groups are
susceptible to infection. Symptoms consist of the abrupt onset of severe sore throat, malaise,
fever, headache, nausea, vomiting, and abdominal pain. Coryza, cough, and conjunctivitis are
absent. Prominent physical findings include: erythema and edema of the posterior pharynx, with
enlarged hyperemic tonsils; patchy discrete tonsillopharyngeal exudates (Item C109A); and
tender, enlarged, submandibular lymph nodes.
The cutaneous clinical manifestations of a group A streptococcal infection include scarlet
fever, impetigo, erysipelas, streptoccocal cellulitis, and necrotizing fasciitis. Scarlet fever results
from infection with a streptococcal strain that produces streptococcal pyrogenic exotoxins. This
disease usually is associated with pharyngeal infections but also may follow streptococcal
infections at other sites. Clinical findings include a diffuse erythematous blush that blanches with
pressure and spares the face, palms, and soles. Skin folds in the neck, axillae, groin, elbows,
and knees appear as lines of deeper red (Pastia lines) (Item C109B). The skin has a sandpaper
texture (Item C109C), and there is usually circumoral pallor.
Impetigo (pyoderma) consists of discrete purulent lesions representing primary infection of
the skin. The lesion begins as a papule that rapidly evolves into a vesicle surrounded by an area
of erythema. The vesicle gives rise to pustules that enlarge and subsequently break down to
form thick crusts (Item C109D). The lesions heal slowly and may leave areas of
hypopigmentation.
Erysipelas is a superficial cutaneous process that has prominent lymphatic involvement. Its
distinguishing features are lesions that are raised and have a clear line of demarcation between
involved and uninvolved tissue. The cutaneous lesion begins as a localized area of erythema
and swelling and subsequently spreads rapidly, with advancing red margins that are raised and
well-demarcated from adjacent normal tissue. There is marked edema, often with bleb formation
(Item C109E).
Necrotizing fasciitis is an infection of the deeper subcutaneous tissues and fascia
characterized by extensive and rapidly spreading necrosis and gangrene of the skin and
underlying tissue. Typically, necrotizing fasciitis begins at a site of minor or inapparent trauma.
The initial lesion is an area of mild erythema that rapidly evolves to severe extensive
inflammation; the skin becomes dusky and purplish, and bullae containing yellow or hemorrhagic
fluid appear (Item C109F). Systemically, patients are ill and have a high temperature and toxic
appearance.
The systemic clinical manifestations of group A streptococcal infection include toxic shock
syndrome (TSS), pneumonia, septic arthritis and osteomyelitis, and meningitis. TSS is defined
as any streptococcal infection associated with the sudden onset of shock and organ failure. The
first phase of TSS begins with an influenza-like prodrome characterized by fever, chills,
myalgias, nausea, vomiting, and diarrhea that precedes the development of hypotension by 24
to 48 hours. A defined or superficial portal of entry or evidence of infection also may be present
during this stage. Phase 2 of TSS is characterized by tachycardia, tachypnea, persistent fever,
and if present, increasingly severe pain at a site of infection. Phase 3 of TSS is characterized by
the previously noted symptoms accompanied by the sudden onset of shock and organ failure.
Pneumonia due to group A Streptococcus frequently is associated with a preceding viral
infection. The onset of disease usually is abrupt and is characterized by fever, dyspnea, cough,
and pleuritic chest pain. Chest radiography shows a consolidative lobar pneumonia that has
moderate-to-large pleural effusion or empyema. Meningitis due to group A Streptococcus is

Copyright 2009 by the American Academy of Pediatrics

page 234

2009 PREP SA on CD-ROM

relatively rare and typically follows an upper respiratory tract infection (otitis and sinusitis) or
neurosurgical conditions.
Staphylococcus epidermidis does not cause cellulitis. S pneumoniae is a cause of preseptal
cellulitis but is not associated with other forms of cellulitis. The cellulitis caused by S aureus
(methicillin-sensitive or -resistant) is not associated with lymphangitis, and progression is not as
rapid as with group A streptococcal cellulitis.
References:
Bisno AL, Stevens DL. Streptococcal infections of skin and soft tissues. N Engl J Med.
1996;334:240-246. Extract available at: http://content.nejm.org/cgi/content/extract/334/4/240
Bisno AL, Stevens DL. Streptococcus pyogenes. In: Mandell GL, Bennett JE, Dolin R, eds.
Mandell, Douglas, and Bennett's Principles and Practice of Infectious Diseases. 6th ed.
Philadelphia, Pa: Elsevier Churchill Livingstone; 2005:2362-2379
Sellers BJ, Woods ML, Morris SE, Saffle JR. Necrotizing group A streptococcal infections
associated with streptococcal toxic shock syndrome. Am J Surg. 1996;172:523-528. Abstract
available at: http://www.ncbi.nlm.nih.gov/pubmed/8942557
Stevens DL. Dilemmas in the treatment of invasive Streptococcus pyogenes infections. Clin
Infect Dis. 2003;37:341-343. Available at:
http://www.journals.uchicago.edu/doi/full/10.1086/376652
Stevens DL. Streptococcal toxic-shock syndrome: spectrum of disease, pathogenesis, and new
concepts in treatment. Emerg Infect Dis. 1995;1:69-78. Abstract available at:
http://www.ncbi.nlm.nih.gov/pubmed/8903167

Copyright 2009 by the American Academy of Pediatrics

page 235

2009 PREP SA on CD-ROM


Question: 110

A 6-year-old boy presents to an urgent care center with the complaint of bright red blood and
clots in the urine. There is no history of trauma, and the boy has no dysuria, frequency,
urgency, abdominal pain, or back pain. On physical examination, his temperature is 98.6F
(37C), heart rate is 76 beats/min, respiratory rate is 14 breaths/min, and blood pressure is
110/68 mm Hg. He has no abdominal tenderness, flank tenderness, or edema. Urinalysis
reveals a specific gravity of 1.025, pH of 6.5, 3+ blood, trace protein, and negative leukocyte
esterase and nitrite. Microscopy shows more than 100 red blood cells/high-power field (HPF),
less than 5 white blood cells/HPF, and no casts. Electrolyte values are normal, blood urea
nitrogen is 14.0 mg/dL (5.0 mmol/L), and creatinine is 0.5 mg/dL (44.2 mcmol/L).
Of the following, the MOST appropriate next test is

A. antinuclear antibody titer


B. complement component 3 (C3) measurement
C. computed tomography scan of the abdomen
D. renal biopsy
E. renal/bladder ultrasonography

Copyright 2009 by the American Academy of Pediatrics

page 236

2009 PREP SA on CD-ROM


Critique: 110

Preferred Response: E

A good first step in the evaluation of gross hematuria in a child who is otherwise well is to
characterize the urine color to aid in localizing the lesion to a particular anatomic region. Patients
who have cola- or tea-colored urine often have painless hematuria without clots, as is seen
classically in glomerulonephritis. Bright red urine (often with clots), as described for the boy in
the vignette, may have an underlying nonglomerular renal cause, a lower urinary tract cause
(ureter or bladder), or rarely be due to a hematologic disorder.
Patients who have bright red urine with clots need to be evaluated for possible structural
causes for hematuria. Such nonglomerular renal causes include a ruptured cyst in cases of
cystic kidney disease (as in autosomal dominant and recessive forms of polycystic kidney
disease), a renal mass (Wilms tumor) (Item C110), or a renal stone (nephrolithiasis or
urolithiasis). Additional causes include renal vein thrombosis, papillary necrosis, or
hypercalciuria. Hematologic disorders such as sickle trait/disease or a bleeding disorder (von
Willebrand disease) can result in a similar clinical picture and should be considered in the
investigation. Lower urinary tract causes include hemorrhagic cystitis, bladder calculi, or rarely,
a tumor of the lower urinary tract (hemangioma of the bladder or rhabdomyosarcoma of the
bladder). The evaluation for possible cysts, stones, or tumors in all children who have gross
hematuria involves renal/bladder ultrasonography to look for a structural abnormality. In rare
cases, additional testing is required, such as an abdominal computed tomography (CT) scan to
look for renal calculi if the clinical suspicion is high and the ultrasonography result is negative.
However, routine use of the CT scan is strongly discouraged due to long-term concerns about
the ionizing radiation exposure (estimated to be equivalent to the radiation dose of 250 two-view
chest radiographs).
Patients who have cola-colored urine require monitoring of blood pressure and renal function
while looking for an underlying cause for the suspected glomerulonephritis. The best single
diagnostic test in the evaluation of acute glomerulonephritis is measurement of complement
component 3 (C3). Additional testing usually performed at the outset includes measurement of
C4, antinuclear antibody, antidouble-stranded DNA antibody, and serum albumin. These tests
help the clinician to categorize the glomerulonephritis into hypocomplementemic or
normocomplementemic types and narrow the differential diagnosis accordingly. A renal biopsy
usually is considered in patients who have acute glomerulonephritis accompanied by a rising
serum creatinine value or in cases of normocomplementemic glomerulonephritis or suspected
lupus nephritis. Because the patient in the vignette is unlikely to have glomerulonephritis, he
does not require testing for C3 or ANA or a renal biopsy.
References:
Brody AS, Frush DP, Huda W, Brent RL, and the Section on Radiology. Radiation risk to children
from computed tomography. Pediatrics. 2007;120:677-682. Available at:
http://pediatrics.aappublications.org/cgi/content/full/120/3/677
Moxey-Mims M. Hematuria and proteinuria. In: Kher KK, Schnaper HW, Makker SP, eds. Clinical
Pediatric Nephrology. 2nd ed. London, England: Informa Healthcare; 2007:129-141
Pan CG. Evaluation of gross hematuria. Pediatr Clin North Am. 2006;53:401-412. Abstract
available at: http://www.ncbi.nlm.nih.gov/pubmed/16716787

Copyright 2009 by the American Academy of Pediatrics

page 237

2009 PREP SA on CD-ROM


Question: 111

A 10-year-old boy presents with a 2-month history of chronic cough. His parents are unsure of a
specific preceding trigger. They are concerned because the school nurse has called on multiple
occasions requesting that the boy be taken home due to his persistent cough. The boy denies
any chest pain, dyspnea, or syncope. Use of a sedating antihistamine and over-the-counter cold
and cough liquid has not alleviated his symptoms. On physical examination, the boy has vital
signs within the normal range and appears healthy. A thorough examination reveals no
abnormalities. During the encounter, the boy repeatedly exhibits a harsh, "barky" cough that
resolves when you leave the examination room, only to recur when you return. You suspect he
has a psychogenic cough.
Of the following, the MOST accurate statement regarding psychogenic cough is that

A. most cases are associated with underlying psychological illness


B. most cases are preceded by an upper respiratory tract infection
C. resolution of the cough often is followed by recurrent wheezing
D. symptoms persist during the day and while the child is asleep
E. the cough noise often is dramatically different from the postnasal drip syndrome cough

Copyright 2009 by the American Academy of Pediatrics

page 238

2009 PREP SA on CD-ROM


Critique: 111

Preferred Response: B

Psychogenic cough, sometimes called habit-cough syndrome, is an uncommon cause of


chronic cough in children and adolescents. Many experts discourage using the label
"psychogenic" because underlying psychological or psychosomatic concerns are uncommon in
affected children. Most cases of psychogenic cough begin with an uncomplicated upper
respiratory tract viral illness, but the cough lingers for months to years. The classic triad of
symptoms includes a repetitive cough, cessation during sleep, and the characteristic hacking,
barky cough, as described for the boy in the vignette. Although most affected children undergo
multiple therapeutic interventions and diagnostic tests, the characteristic presentation may allow
clinicians to avoid expensive or invasive tests to rule out other causes.
Successful therapies for habit-cough include behavioral therapy, suggestion therapy, and
self-hypnosis. Once resolved, most cases of habit-cough do not recur. Unlike some cases of
viral upper respiratory tract infections, habit-cough resolution is not followed by recurrent
wheezing.
References:
Linz AJ. The relationship between psychogenic cough and the diagnosis and misdiagnosis of
asthma: a review. J Asthma. 2007;44:347-355. Abstract available at:
http://www.ncbi.nlm.nih.gov/pubmed/17613629
Weinberger M, Abu-Hasan M. Pseudo-asthma: when cough, wheezing, and dyspnea are not
asthma. Pediatrics. 2007;120:855-864. Available at:
http://pediatrics.aappublications.org/cgi/content/full/120/4/855

Copyright 2009 by the American Academy of Pediatrics

page 239

2009 PREP SA on CD-ROM


Question: 112

A 6-year-old girl fell onto her outstretched right arm while roller skating yesterday. She continued
to skate, but on returning home, she noticed that her right forearm was swollen and painful. Her
mother applied ice and gave her ibuprofen, but the swelling is worse today. On physical
examination, the girl has moderate swelling over the right distal radius with minimal pain on
palpation. She has full range of motion of her wrist and hand. Radiographs are obtained (Item
Q112).
Of the following, a TRUE statement regarding this patients fracture is that

A. median nerve injury often is seen in association with this fracture


B. reduction of the fracture is not necessary if there is less than 15 degrees of angulation
C. the fracture frequently is associated with growth arrest
D. the fracture is likely to produce permanent deformity
E. the patient should be immobilized in a cast for 8 to 10 weeks

Copyright 2009 by the American Academy of Pediatrics

page 240

2009 PREP SA on CD-ROM


Critique: 112

Preferred Response: B

Pediatric fracture patterns differ from those in adults for a variety of biomechanical, physiologic,
and anatomic reasons. Compared to adult bone, pediatric bone is highly porous; contains lower
amounts of mineral and higher amounts of cartilage; has thicker, stronger, and more
metabolically active periosteum; and has relatively stronger ligaments and open growth plates.
These differences lead to fracture types such as greenstick fractures, plastic deformation, torus
fractures, and physeal or growth plate fractures that are unique to children.
Greenstick fractures are incomplete fractures characterized by a cortical break on one side
of the bone with intact periosteum on the opposing side (Item C112A), as seen in the radiograph
obtained for the girl in the vignette. In some cases, the bone may bend without breaking (Item
C112B). The strong periosteum and decreased resistance to bending forces of pediatric bones
are responsible for this phenomenon. Torus fractures result from compressive forces on the
bone that lead to a periosteal "buckle" (Item C112C). Children who have open growth plates are
at risk for physeal fractures that involve the relatively weak, cartilage-rich growth plates.
Treatment for the greenstick fracture with less than 15 degrees of angulation, as described
for the child in the vignette, involves cast immobilization and close orthopedic follow-up, but no
reduction of the fracture is necessary. In older children who have fractures with greater than 15
degrees of angulation, reduction often is necessary to decrease the risk of permanent deformity.
Greenstick fractures typically are not associated with nerve entrapment or growth arrest (if the
physis is not involved) and heal without residual sequelae following cast immobilization for 4 to 6
weeks. Patients should be cautioned to avoid reinjury following cast removal. Refracture is seen
in 7% to 17% of forearm fractures and occurs most commonly following greenstick fractures.
References:
Carson S, Woolridge DP, Colletti J, Kilgore K. Pediatric upper extremity injuries. Pediatr Clin
North Am. 2006;53:41-67. Abstract available at: http://www.ncbi.nlm.nih.gov/pubmed/16487784
Gholve PA, Hosalkar HS, Wells L. Common fractures. In: Kleigman RM, Behrman RE, Jenson
HB, Stanton BF, eds. Nelson Textbook of Pediatrics. 18th ed. Philadelphia, Pa: Saunders
Elsevier; 2007:2834-2841
Greenstick fracture of forearm. In: Wheeless' Textbook of Orthopaedics. Available at:
http://wheelessonline.com/ortho/green_stick_frx_of_forearm

Copyright 2009 by the American Academy of Pediatrics

page 241

2009 PREP SA on CD-ROM


Question: 113

A 5-year-old child is admitted to the hospital with epigastric pain and vomiting. On physical
examination, she has a tender epigastrium, but no peritoneal signs. Her amylase is 400 U/L and
lipase is 670 U/L. Abdominal ultrasonography demonstrates a prominent pancreatic head, but no
gallstones or biliary tract dilation. Review of her chart demonstrates two prior hospitalizations
over the past 3 years due to pancreatitis. She has no other significant findings in her medical
history and no history of trauma preceding any of these episodes.
Of the following, the condition that BEST explains the patients history is

A. alpha-1-antitrypsin deficiency
B. colipase deficiency
C. hereditary pancreatitis
D. primary sclerosing cholangitis
E. systemic lupus erythematosus

Copyright 2009 by the American Academy of Pediatrics

page 242

2009 PREP SA on CD-ROM


Critique: 113

Preferred Response: C

The child described in the vignette has a history of at least three episodes of pancreatitis before
age 5 years, and gallstones have been excluded by ultrasonography. Children who have
hereditary pancreatitis (HP) can present with recurrent episodes of pancreatitis and often are
well between episodes. In adulthood, affected individuals are at risk for developing pancreatic
insufficiency and cancer.
Systemic lupus erythematosus (SLE) may be associated with pancreatitis in the context of
severe vasculitis, but SLE is rare before 5 years of age, and the child has no other signs of the
condition. Alpha-1-antitrypsin deficiency and primary sclerosing cholangitis may cause chronic
liver disease, but they rarely affect the pancreas. Colipase deficiency may result in pancreatic
insufficiency and fat malabsorption, but it usually does not cause pancreatic inflammation.
Pancreatitis generally is less severe in children than in adults. A child who has acute
pancreatitis typically presents with epigastric pain and vomiting, and epigastric tenderness can
be appreciated on palpation. More severe pancreatitis can cause intravascular volume
depletion, third spacing, oliguria, pulmonary edema, and pleural effusions. The most common
laboratory abnormalities in mild pancreatitis are elevated amylase and lipase values. Lipase is a
more specific enzyme for pancreatitis; hyperamylasemia also can be seen with salivary gland
inflammation, ovarian tumors, renal insufficiency, diabetic ketoacidosis, and ruptured ectopic
pregnancy. Laboratory abnormalities associated with more severe pancreatitis include
hyperglycemia, hypocalcemia, high blood urea nitrogen and creatinine, low hematocrit, and
coagulopathy. In severe cases of pancreatitis, abdominal computed tomography scan can
identify the extent and severity of pancreatic injury.
Some children experience recurrent episodes of pancreatitis without an obvious cause (ie,
no history of medications that can cause pancreatitis, gallstones, trauma, or systemic
vasculitis). Anatomic and genetic causes should be considered in such children. Anatomic
causes such as choledochocele, pancreas divisum, or a long common channel connecting the
pancreas and bile ducts can be identified by either magnetic resonance imaging or endoscopic
retrograde cholangiopancreatography.
The most common genetic causes of recurrent pancreatitis are cystic fibrosis (CF) and HP.
Patients who have CF and present with pancreatitis often have minimal pulmonary disease and
no clinical signs of pancreatic insufficiency or malabsorption. In some cases, a single mutation in
the CFTR gene may cause a heterozygous carrier for CF to present with recurrent pancreatitis.
HP is caused most commonly by a mutation in the pancreatic cationic trypsinogen gene and is
inherited in an autosomal dominant pattern. Although there often is a family history, some
patients have no family history because a de novo gene mutation has developed.
References:
Pietzak MM, Thomas DW. Pancreatitis in childhood. Pediatr Rev. 2000;21: 406-412. Available at:
http://pedsinreview.aappublications.org/cgi/content/full/21/12/406
The National Pancreas Foundation web site. Available at: www.pancreasfoundation.org

Copyright 2009 by the American Academy of Pediatrics

page 243

2009 PREP SA on CD-ROM


Question: 114

You admit a term newborn to the neonatal intensive care unit because of noisy breathing.
Findings on physical examination include mild micrognathia, an intact palate, and inspiratory
stridor with suprasternal retractions when the infant is in the supine position that diminish but do
not disappear when the infant is prone. Stridor becomes more audible when the infant cries.
When the infant is asleep and prone, the breath sounds are clear and equal bilaterally, with no
stridor or wheezing. There is no heart murmur. Pulse oximetry is 94% on room air.
Of the following, the MOST likely cause of this infants stridor is

A. cleft lip
B. laryngomalacia
C. tracheal hemangioma
D. tracheomalacia
E. vocal cord polyp

Copyright 2009 by the American Academy of Pediatrics

page 244

2009 PREP SA on CD-ROM


Critique: 114

Preferred Response: B

The infant described in the vignette has congenital stridor (noisy inspiratory breathing) and
exhibits only mild increased work of breathing when supine and when distressed. Although no
immediate intervention is required to maintain airway patency, the clinician examining the patient
needs to determine the cause of the stridor. Some have reported a characteristic coarseness of
the stridor when due to a supraglottic (laryngeal) disorder, a musical quality associated with
glottic (vocal cord) disorders, and more of an expiratory noise or prolonged expiratory phase
with a wheeze in subglottic disorders of airway obstruction (Item C114A).
The most common cause of congenital stridor is laryngomalacia, a congenital weakness, or
floppiness, in airway laryngeal cartilages (epiglottis, arytenoid cartilages, larynx) (Item C114B). A
cleft lip does not cause stridor. A tracheal hemangioma (Item C114C) may cause airway
obstruction, but usually this is associated with expiratory wheezing. Tracheomalacia may
complicate prolonged intubation and positive pressure ventilation in preterm infants and typically
is associated with expiratory wheezing or inspiratory cessation of airflow with airway collapse
unless positive distending pressure is applied (Item C114D). A vocal cord polyp is a rare
congenital condition that may be associated with human papillomavirus infection that can cause
stridor, which would not be expected to change with position.
References:
Aly H. Respiratory disorders in the newborn: identification and diagnosis. Pediatr Rev.
2004;25:201-208. Available at: http://pedsinreview.aappublications.org/cgi/content/full/25/6/201
Brodsky L. Consultation with the specialist: congenital stridor. Pediatr Rev. 1996;17:408-411.
Available at: http://pedsinreview.aappublications.org/cgi/reprint/17/11/408
Sprecher RC, Arnold JE. Upper airway lesions. In: Martin RJ, Fanaroff AA, Walsh MC, eds.
Fanaroff and Martin's Neonatal-Perinatal Medicine. 8th ed. Philadelphia, Pa: Mosby Elsevier;
2006:1146-1154
Vicencio AG, Parikh S, Adam HM. In brief: laryngomalacia and tracheomalacia: common
dynamic airway lesions. Pediatr Rev. 2006;27:e33-e35. Available at:
http://pedsinreview.aappublications.org/cgi/content/full/27/4/e33

Copyright 2009 by the American Academy of Pediatrics

page 245

2009 PREP SA on CD-ROM


Question: 115

A 13-year-old boy who plays baseball comes to your office for a preparticipation sports physical
examination. He always has been an average player and is interested in a preseason
conditioning program to improve his strength and agility because he wants to play on his school
team. His mother is concerned about the program because it involves weight training, and she
asks for your advice.
Of the following, a TRUE statement about conditioning programs for young athletes is that these
programs

A. have been shown to decrease ultimate linear growth if begun before puberty
B. prevent future catastrophic sports-related injuries
C. should begin with low-resistance exercise, with weight added in small increments as tolerated
D. should involve high-weight, low-repetition activities
E. should involve weight training only without aerobic activities

Copyright 2009 by the American Academy of Pediatrics

page 246

2009 PREP SA on CD-ROM


Critique: 115

Preferred Response: C

Preventive conditioning is an important part of sports programs for preadolescents and


adolescents because participation can help to improve strength and overall performance.
Programs should include both aerobic conditioning and strength training. The most significant
risks to the athlete involve strength, or resistance, training, but such risks can be minimized by
adhering to a few guidelines.
Weight training programs that consist of high-repetition activities with low resistance are safe
for both preadolescents and adolescents. Training should begin by teaching proper technique
with no weight, adding small increments of weight as strength increases. Sessions lasting 20 to
30 minutes undertaken 2 to 3 days a week are sufficient to improve strength. Power lifting
programs, which involve low-repetition, high-resistance exercises, should not be undertaken by
preadolescents because of the risk of injury. In general, such programs should be avoided until
skeletal maturity is reached. Injuries reported in studies evaluating strength training programs
include muscle strain, epiphyseal injuries in the wrist, and apophyseal injuries, but most of these
can be avoided by using proper technique. Participation in these programs does not decrease
linear growth.
Although preventive conditioning has been shown to improve strength and performance,
there is no evidence that participation in such programs can prevent catastrophic injury.
However, athletes who have particular recurrent injuries, such as to the shoulder or ankle, can
reduce their risk of further injury by participating in specific strengthening exercises focusing on
those areas.
References:
American Academy of Pediatrics Committee on Sports Medicine and Fitness. Strength training
by children and adolescents. Pediatrics. 2001;107:1470-1472. Available at:
http://pediatrics.aappublications.org/cgi/content/full/107/6/1470
Metzl JD. Preparticipation examination of the adolescent athlete: part 2. Pediatr Rev.
2001;22:227-239. Available at: http://pedsinreview.aappublications.org/cgi/content/full/22/7/227

Copyright 2009 by the American Academy of Pediatrics

page 247

2009 PREP SA on CD-ROM


Question: 116

A 10-year-old boy who recently emigrated from Central America is referred by the school nurse
for evaluation of obesity.Physical examination reveals an obese but generally healthy boy who
has acanthosis nigricans (Item Q116).He has had limited access to medical care in the past.
Of the following, the physical finding MOST likely to suggest an underlying cause for the child's
obesity is

A. a normal blood pressure


B. abdominal striae
C. penile length at 1 standard deviation below the mean
D. small hands and feet
E. stature greater than the 95th percentile

Copyright 2009 by the American Academy of Pediatrics

page 248

2009 PREP SA on CD-ROM


Critique: 116

Preferred Response: D

The incidence of obesity continues to increase among children in industrialized nations. In


general, the causes in most children relate to increased caloric intake and decreased physical
activity. However, the pediatrician must exclude genetic, metabolic, or other underlying causes
of obesity in children for two reasons: 1) some underlying causes may require treatment and 2)
the focus on changing the child's lifestyle must involve parents who are reassured that hormonal
or other abnormalities are not the cause. A child who exhibits normal linear growth, has normal
developmental milestones, and has normal findings on physical examination is unlikely to have
an underlying cause for his or her obesity.
The most common metabolic cause of obesity is hypothyroidism, but routine testing for
thyroid function in the overweight child who has normal linear growth velocity for age and no
clinical signs of hypothyroidism is unwarranted. A stature greater than the 95th percentile should
be interpreted with caution because one measurement is not sufficient to determine growth
velocity. However, this finding is reassuring because normal to above-average height is unlikely
in a child who has hypothyroidism and common in children who have exogenous obesity.
Children who have small hands and feet, hypogonadism, learning disabilities, or mental
retardation should be evaluated for Prader- Willi and Bardet-Biedl syndromes. These syndromes
often present in infancy or early childhood with hypotonia and developmental delay. Small hands
and feet are common and strongly suggest an underlying anomaly for the child described in the
vignette (Item C116A). Genetic testing for these conditions is available.
The normal blood pressure reported for the boy in the vignette is reassuring because
hypertension often complicates obesity. Penile length is normal at 1 standard deviation below the
mean, but it is important to remember that measurement of penile length can be a challenge in
obese children due to interference from the pubic fat pad.
Striae may be seen in many overweight children simply due to rapid weight gain. Although
this finding also occurs with cortisol excess (Item C116B), conditions associated with
hypercortisolism usually are associated with other signs and symptoms. Thus, the finding of
striae in an obese child is not an indication that additional testing is necessary.
Treatment of childhood obesity is difficult, although there is some evidence for a variety of
interventions that may prove successful in some populations. Numerous diets, including lowcarbohydrate diets, appear to have some impact on obesity in children. For morbidly obese
adolescents who have comorbidities unresponsive to diet and exercise, some centers are
moving toward bariatric surgery.
Barriers to healthy lifestyles (lack of availability in some communities of safe outdoor play
areas, specific exercise programs geared to very obese children, and school lunch programs)
remain topics of scientific and political investigation.
References:
Arterburn DE. Obesity in children. BMJ Clinical Evidence. 2007. Available for subscription at:
http://clinicalevidence.bmj.com/ceweb/conditions/chd/0325/0325.jsp
Rodearmel SJ Wyatt HR, Stroebele N, Smith SM, Ogden LG, Hill JO. Small changes in dietary
sugar and physical activity as an approach to preventing excessive weight gain: the America on
the Move Family Study. Pediatrics. 2007;120:e869-e879 Available at:
http://pediatrics.aappublications.org/cgi/content/full/120/4/e869
Schneider MB, Brill SR. Obesity in children and adolescents. Pediatr Rev. 2005;26:155-162.
Available at: http://pedsinreview.aappublications.org/cgi/content/full/26/5/155
Shaw K, Gennat H, O'Rourke P, Del Mar C. Exercise for overweight or obesity. Cochrane
Database Syst Rev. 2006;4:CD003817. Available at:
http://www.mrw.interscience.wiley.com/cochrane/clsysrev/articles/CD003817/frame.html
Summerbell CD, Waters E, Edmunds LD, Kelly S, Brown T, Campbell KJ. Interventions for

Copyright 2009 by the American Academy of Pediatrics

page 249

2009 PREP SA on CD-ROM

preventing obesity in children. Cochrane Database Syst Rev. 2005;3:CD001871. Available at:
http://www.mrw.interscience.wiley.com/cochrane/clsysrev/articles/CD001871/frame.html
Thomas DE, Elliott EJ, Baur L. Low glycaemic index or low glycaemic load diets for overweight
and obesity. Cochrane Database Syst Rev. 2007;3:CD005105. Available at:
http://www.mrw.interscience.wiley.com/cochrane/clsysrev/articles/CD005105/frame.html

Copyright 2009 by the American Academy of Pediatrics

page 250

2009 PREP SA on CD-ROM


Question: 117

You are evaluating a 14-year-old girl in the clinic. She has had a fever for nearly 2 weeks, which
she has attributed to a "cold," although she has not had cough or upper respiratory tract
symptoms. She is concerned about some "spots" that she has noticed on her palms and soles.
On physical examination, you note splenomegaly and erythematous, nontender macules on her
fingers, palms (Item Q117), and soles of her feet. Additionally, she has lost 8 lb since her visit 6
months ago.
Of the following, the MOST appropriate next study for evaluation of this patient is

A. antinuclear antibody
B. echocardiography
C. Lyme titers
D. ophthalmologic examination
E. tuberculin skin test

Copyright 2009 by the American Academy of Pediatrics

page 251

2009 PREP SA on CD-ROM


Critique: 117

Preferred Response: B

The fever, Janeway lesions (erythematous, nontender macular lesions on the fingers and
soles), splenomegaly, and weight loss reported for the girl in the vignette strongly suggest
infective endocarditis. Infective endocarditis describes an infection involving the endocardial
surface of the heart that can occur in individuals who have structurally normal hearts, although it
is more common in children who have congenital heart disease. The most common sites are the
cardiac valves, but infection also may occur on the margins of a ventricular septal defect, along
the chordae supporting the atrioventricular valves, or along vascular grafts or foreign material
such as a prosthetic valve. Although a relatively rare occurrence in the pediatric population,
infective endocarditis continues to be a major source of morbidity and even mortality. Viridans
streptococci (eg, S mitis, S bovis) as well as Staphylococcus aureus are the most common
bacterial pathogens causing endocarditis in children.
The clinical manifestations of infective endocarditis are many and can present variably in
affected individuals. The most common is fever, which may be associated with shaking chills.
Constitutional nonspecific manifestations include anorexia, weight loss, malaise, night sweats,
arthralgias, myalgias, and splenomegaly. In addition, a number of extracardiac manifestations
are associated with infective endocarditis, including petechiae and splinter hemorrhages seen
under the nails, hematuria, and glomerulonephritis. Roth spots are retinal hemorrhages that have
a clear center and can be seen on ophthalmologic examination (Item C117A). Janeway lesions
may be evident on the fingers, palms, or soles (Item C117B). Osler nodes are small, raised
erythematous or purple nodules on the pads of the digits that typically are very painful.
Among the cardiac manifestations of infective endocarditis is a murmur, which is heard in
nearly 50% of affected patients. Such murmurs typically result from valvular insufficiency, and
the left heart valves are affected far more commonly than the right heart valves. Regurgitation of
the mitral valve produces a holosystolic murmur typically heard best at the cardiac apex, with
radiation to the left axilla (Item C117C). Regurgitation of the aortic valve produces a diastolic
murmur that generally is heard best at the mid-left or right sternal border with the patient in the
sitting position, leaning forward (Item C117D). If the regurgitation is severe, congestive heart
failure also can be present.
The causative pathogen of infective endocarditis can be identified with blood cultures.
Imaging of cardiac valves and evaluation of myocardial function by echocardiography are
important parts of an assessment that can help guide management of valvular regurgitation and
congestive heart failure, if present. An ophthalmologic examination is an important component of
the evaluation, but it is not diagnostic and will not affect treatment. With a presentation such as
that of the girl described in the vignette, it is important to rule out other possible causes of
systemic disease such as lupus erythematosus, Lyme disease, and tuberculosis, but such
diagnostic tests do not have the same urgency or impact on management as echocardiography
in a patient who clearly has infective endocarditis.
References:
Baltimore RS. Infective endocarditis in children. Pediatr Infect Dis J. 1992;11:907-912. Abstract
available at: http://www.ncbi.nlm.nih.gov/pubmed/1454430
Hoyer A, Silberbach M. Infective endocarditis. Pediatr Rev. 2005;26:394-400. Available at:
http://pedsinreview.aappublications.org/cgi/content/full/26/11/394
Taubert KA, Dajani AS. Infective endocarditis. In: Moller JH, Hoffman JIE, eds. Pediatric
Cardiovascular Medicine. Philadelphia, Pa: Churchill Livingstone; 2000:768-779

Copyright 2009 by the American Academy of Pediatrics

page 252

2009 PREP SA on CD-ROM


Question: 118

A mother brings her 8-year-old daughter to your office after the girl experiences a first
unprovoked generalized tonic-clonic seizure at school. The child had been seen in an
emergency department, and results of a head computed tomography scan performed there
were normal. Her development, school performance, and results of physical examination are
normal. You review safety concerns (no unsupervised time in bathtub or pools, wearing a
bicycle helmet) and seizure first aid with the mother. Following published guidelines, you obtain
routine electroencephalography (EEG), which a neurologist interprets as normal. The mother
asks you about anticonvulsant therapy to prevent further seizures.
Of the following, you are MOST likely to advise her that

A. repeat sleep-deprived EEG is needed to select medication


B. treatment with carbamazepine should be initiated
C. treatment with phenobarbital should be initiated
D. treatment with phenytoin should be initiated
E. treatment with seizure medication should be deferred

Copyright 2009 by the American Academy of Pediatrics

page 253

2009 PREP SA on CD-ROM


Critique: 118

Preferred Response: E

For a child who has a first unprovoked seizure, as described in the vignette, safety concerns
(no unsupervised time in bathtub or pools, wearing a helmet while on a bike or scooter) and
seizure first aid should be reviewed with the parents and documented in the chart. Treatment
after a single seizure in childhood is not recommended, no matter what EEG shows.
In some cases, retrospective evaluation of children suggests that they have had subtle
complex partial seizures or previously have been found after an unwitnessed seizure.
Therefore, it is critical for the physician seeing the child after an apparent first seizure to obtain a
very careful history to be certain that the seizure is, indeed, the first.
The most important diagnostic decision after a first unprovoked seizure is whether to obtain
neuroimaging to diagnose a focal, treatable cause for seizures. Neuroimaging should be
obtained after a first unprovoked seizure when the seizure occurs in the first year after birth, is
focal/asymmetric, or begins with a stare and subsequently generalizes. Neuroimaging also
should be obtained after a first unprovoked seizure when the history or physical examination
results indicate probable focal central nervous system pathology, such as if the child has
unilateral weakness or when postictal confusion is prolonged, particularly if the child is febrile (to
rule out herpes encephalitis).
Obtaining an EEG is recommended by published practice guidelines. There are two primary
problems to keep in mind when deciding whether to order an EEG:
1) Inter-reader agreement in the interpretation of EEGs is low. The level of training of
neurologists reading pediatric EEGs may be low in some communities, which is a particular
problem for neonatal EEGs, and even experienced EEG readers frequently differ in their visual
interpretations.
2) Positive and negative predictive values are not very informative. The sensitivity of EEG
varies widely, depending on who reads it, but at best is approximately 50%; that is, about 50% of
children who have epilepsy and 50% of children who have a first unprovoked seizure that
eventually will recur have epileptiform EEGs, but the rest have normal EEGs. Accordingly, a
normal EEG reading does not rule out recurrence or epilepsy and only means that a recurrent
seizure is somewhat less likely. The specificity after first seizure is, at best, about 70%; that is,
about 30% of children who do not have a seizure recurrence still have epileptiform discharges
on the EEG. Thus, a positive EEG also does not predict epilepsy accurately.
Repeat EEGs with sleep deprivation have been advocated by neurology practice
parameters for many years in cases where the first EEG is read as normal and sleep was not
present. The best available evidence suggests, at most, modest benefit to this practice if used
routinely in children. Any benefits of the higher yield should be weighed against the potential for
additional false-positive results, the time for the additional trip to the EEG laboratory, missed
school and work, cost, and the burden of sleep deprivation. After a first seizure, the repeat EEG
will not result in any additional information that changes acute management.
Regardless of EEG results, two or more unprovoked seizures in a child are diagnostic of
epilepsy and warrant treatment and consultation with a neurologist for selection of medication.
Because phenobarbital causes sedation and cognitive and behavioral changes in young
children, it would not be used, except in countries where resources for medical care are very
limited. Phenytoin has somewhat erratic absorption and can cause gingival hyperplasia in
children, so it is not first-line therapy for a new diagnosis of epilepsy. Carbamazepine is a good
choice for a child diagnosed with partial epilepsy, but not for generalized epilepsy. Valproic acid
is a good choice for both partial and generalized epilepsy in children older than age 2 years.
Each of these generic choices, along with the plethora of new medications, has unique adverse
event profiles, which physicians must understand.
References:
Gilbert DL, Buncher CR. An EEG should not be obtained routinely after first unprovoked seizure
in childhood. Neurology. 2000;54:635-641. Abstract available at:
http://www.ncbi.nlm.nih.gov/pubmed/10680796

Copyright 2009 by the American Academy of Pediatrics

page 254

2009 PREP SA on CD-ROM

Gilbert DL, DeRoos S, Bare MA. Does sleep or sleep deprivation increase epileptiform
discharges in pediatric electroencephalograms? Pediatrics. 2004;114:658-662. Available at:
http://pediatrics.aappublications.org/cgi/content/full/114/3/658
Gilbert DL, Sethuraman G, Kotagal U, Buncher CR. Meta-analysis of EEG test performance
shows wide variation among studies. Neurology. 2003;60:564-570. Abstract available at:
http://www.ncbi.nlm.nih.gov/pubmed/12601093
Hirtz D, Ashwal S, Berg A, et al. Practice parameter: evaluating a first nonfebrile seizure in
children: report of the Quality Standards Subcommittee of the American Academy of Neurology,
the Child Neurology Society, and the American Epilepsy Society. Neurology. 2000;55:616-623.
Available at: http://www.neurology.org/cgi/content/full/55/5/616
Hirtz D, Berg A, Bettis D, et al. Practice parameter: treatment of the child with a first unprovoked
seizure: report of the Quality Standards Subcommittee of the American Academy of Neurology
and the Practice Committee of the Child Neurology Society. Neurology. 2003;60:166-175.
Available at: http://www.neurology.org/cgi/content/full/60/2/166
Stroink H, van Donselaar CA, Geerts AT, Peters AC, Brouwer OF, Arts WF. The accuracy of
the diagnosis of paroxysmal events in children. Neurology. 2003;60:979-982. Abstract available
at: http://www.ncbi.nlm.nih.gov/pubmed/12654963

Copyright 2009 by the American Academy of Pediatrics

page 255

2009 PREP SA on CD-ROM


Question: 119

You receive a call from a local neonatologist because the mother of a newly admitted baby has
identified you as her pediatrician. The baby was born at term and had Apgar scores of 1, 2, and
4 at 1, 5, and 10 minutes, respectively. She is requiring significant ventilatory and fluid support,
and she has multiple congenital anomalies, including bilateral microtia, depressed nasal bridge,
ocular hypertelorism, cleft palate, and macrocephaly. Echocardiography shows truncus
arteriosus, and head ultrasonography reveals dilated ventricles consistent with hydrocephalus.
Of the following, the teratogenic agent that is MOST likely responsible for this infants features is

A. alcohol
B. isotretinoin
C. phenobarbital
D. phenytoin
E. thalidomide

Copyright 2009 by the American Academy of Pediatrics

page 256

2009 PREP SA on CD-ROM


Critique: 119

Preferred Response: B

The newborn described in the vignette has features consistent with retinoic acid embryopathy,
also called "Accutane embryopathy" (Accutane is one of the brand names for isotretinoin [13cis-retinoic acid]). Soon after the drug was licensed for use in the United States, Lammer and
associates published an article describing 21 infants who had histories of prenatal isotretinoin
exposure and negative outcomes. It now is known that many retinoids are teratogenic in a
number of species, with malformations largely due to inhibited migration of cranial neural crest
cells during embryogenesis. The craniofacial, cardiovascular, and central nervous systems
sustain the greatest deleterious effects.
Currently, a 35% risk for retinoic acid embryopathy is believed to exist in the infants of
women who take isotretinoin beyond 15 days after conception. Maternal use of isotretinoin prior
to conception does not appear to be associated with an increased risk for fetal anomalies.
Paternal use of isotretinoin does not cause birth defects.
Prenatal alcohol exposure is associated with fetal alcohol spectrum disorder. The most
severe end of the spectrum is fetal alcohol syndrome (FAS); affected infants typically exhibit
intrauterine growth restriction; microcephaly; and dysmorphisms that include short palpebral
fissures, smooth philtrum, and a narrow upper vermilion (Item C119A). Congenital heart defects
are seen in some infants who have FAS.
Phenobarbital and phenytoin use during pregnancy can result in a fetal anticonvulsant
syndrome in the exposed infant. Affected neonates also may have intrauterine growth
restriction, ocular hypertelorism, midface hypoplasia (Item C119B), cleft lip+/-cleft palate, and
heart defects.
Thalidomide is a sedative that was marketed in the 1950s in Europe and was prescribed to
pregnant women to ameliorate morning sickness. It is probably the most notorious of all
teratogens and was taken off the market for many years but was reapproved by the United
States Food and Drug Administration in 1998 for use in the treatment of erythema nodosum
leprosum and multiple myeloma. The prescribing of thalidomide is strictly controlled, and
physicians who do so must be registered in a special safety program. Thalidomide exposure
during pregnancy is associated with a variety of anomalies, the most striking of which is severe
limb reduction defects.
References:
Isotretinoin, alcohol, phenytoin, phenobarbital, thalidomide. Reprotox. Available for subscription
at: http://www.reprotox.org
Isotretinoin, alcohol, phenytoin, phenobarbital, thalidomide. Teris. Available for subscription at:
http://depts.washington.edu/terisweb/teris/
Jones KL. Retinoic acid embryopathy. In: Smith's Recognizable Patterns of Human
Malformation. 6th ed. Philadelphia, Pa; Elsevier Saunders; 2006:660-661
Lammer EJ, Chen DT, Hoar RM, et al. Retinoic acid embryopathy. N Engl J Med. 1985;313:837841. Abstract available at: http://www.ncbi.nlm.nih.gov/pubmed/3162101

Copyright 2009 by the American Academy of Pediatrics

page 257

2009 PREP SA on CD-ROM


Question: 120

You are evaluating a 14-year-old boy for his preparticipation sports physical examination before
he tries out for the freshman football team. He has no chronic health problems and no previous
history of head injuries. His mother expresses concern about recent reports of professional
football players sustaining cognitive damage due to repeated concussions.
Of the following, the MOST appropriate statement to include in your counseling regarding head
injuries in contact sports is that

A. baseline, detailed neuropsychological testing has been well established as a tool to use in the
management of head injuries in pediatric athletes

B. most athletic participants who sustain concussions lose consciousness


C. return-to-play guidelines for pediatric athletes are well established and evidence-based
D. seasonal and lifetime sports exclusion guidelines for pediatric athletes after head injury are
well established and evidence-based

E. the preparticipation visit history of an athlete should include the number of prior concussions,
timing and severity of each, and description of resulting symptoms

Copyright 2009 by the American Academy of Pediatrics

page 258

2009 PREP SA on CD-ROM


Critique: 120

Preferred Response: E

Although sports participation can have many physical and social benefits for children and
adolescents, there are risks, including athletic concussion, which is described as mild traumatic
brain injury that is caused by an impact to the head. Numerous guidelines and literature reviews
have examined athletic concussion, but few have focused specifically on the pediatric athlete.
Medical examination frequently is mandated by schools and organizations before children
can participate in sports, and preparticipation assessment of an athlete is an opportune time to
focus on how to prevent head trauma as well recognition of concussions. Loss of
consciousness is not the only indication that an athlete has suffered a concussion; in fact, most
sports participants who sustain concussions do not lose consciousness. Documenting the
history of the number of prior concussions, timing and severity of each, and the description of
resulting symptoms can help establish a baseline for medical care. The initial "grade" of the
concussion generally is accepted as less important than the systematic tracking of each
athlete's recovery course over time.
Although conventional neuropsychological assessment is a well-established and
recommended methodology for the evaluation of children and adults who have sustained
neurologic injury or who have neurodevelopmental problems, the use of comprehensive
neuropsychological evaluations at a preparticipation visit is impractical and expensive because
of the extensive time needed to complete an evaluation. Abbreviated "baseline"
neuropsychological testing for athletic purposes has been introduced, but sufficient data
regarding the reliability, validity, and clinical utility of neuropsychological instruments used in this
model are lacking for pediatric athletes. Similarly, the value of such baseline testing in making
individual decisions about return to play after a head injury has not been demonstrated.
However, neuropsychological testing still can play a valuable role in evaluating and managing
certain athletes after injury, especially those who experience poor recovery and multiple injuries,
and in developing appropriate posttraumatic educational management plans if cognitive
difficulties exist.
Return to play after concussion is a controversial area, and none of the current guidelines
have garnered sufficient scientific support to be recognized as the "gold standard." Experts
agree that the earliest an athlete should return to play is when no signs or symptoms of any type
are apparent at rest or exertion; neurologic examination results are normal; and neuroimaging,
when performed, provides unremarkable results. Some advocate for even more conservative
standards with pediatric athletes, including always removing them from the day's competition
after a concussion and not allowing return to play unless cleared medically. Seasonal and
lifetime sports exclusion guidelines are not well-established or evidence-based for pediatric
athletes, but a conservative approach is inarguably appropriate.
References:
Guskiewicz KM, Weaver NL, Padua DA, Garrett WE Jr. Epidemiology of concussion in collegiate
and high school football players. Am J Sports Med. 2000;28:643-650. Abstract available at:
http://www.ncbi.nlm.nih.gov/pubmed/11032218
Kirkwood MW, Yeates KO, Wilson PE. Pediatrics sport-related concussion: a review of the
clinical management of an oft-neglected population. Pediatrics. 2007;117:1359-1371. Available
at: http://pediatrics.aappublications.org/cgi/content/full/117/4/1359

Copyright 2009 by the American Academy of Pediatrics

page 259

2009 PREP SA on CD-ROM


Question: 121

You are evaluating a 1-year-old girl who was admitted to the pediatric intensive care unit
following 3 days of diarrhea and decreased oral intake. Her heart rate is 160 beats/min,
respiratory rate is 30 breaths/min, blood pressure is 70/40 mm Hg, and she has weak peripheral
pulses. An arterial blood gas evaluation on room air reveals pH of 7.08, Paco2 of 25 mm Hg,
Pao2 of 100 mm Hg, and HCO3 of 5.0 mEq/L (5.0 mmol/L). Initial electrolyte values are: sodium
of 130.0 mEq/L (130.0 mmol/l), potassium of 4.0 mEq/L (4.0 mmol/L), chloride of 95.0 mEq/L
(95.0 mmol/L), bicarbonate of 6.0 mEq/L (6.0 mmol/L), and glucose of 100.0 mg/dL (5.6 mmol/L).
Of the following, the MOST appropriate initial therapy is

A. 5 mcg/kg per minute dopamine infusion


B. 1 mEq/kg intravenous potassium chloride
C. 1 mEq/kg intravenous sodium bicarbonate
D. 20 mL/kg intravenous 0.9% sodium chloride
E. 1 unit/kg intravenous insulin

Copyright 2009 by the American Academy of Pediatrics

page 260

2009 PREP SA on CD-ROM


Critique: 121

Preferred Response: D

The child described in the vignette demonstrates severe metabolic acidosis, normally defined as
a pH of less than 7.10 or serum bicarbonate concentration of less than 8.0 mEq/L (8.0 mmol/L),
with partial respiratory compensation. Severe metabolic acidosis is a medical emergency that
requires urgent intervention via an organized approach. Because this patient's respiratory drive,
oxygenation, and ventilation are adequate, therapy to correct the metabolic acidosis and its
underlying cause should be the first priority.
Volume repletion using isotonic fluids (such as normal saline) at an initial dose of 20 mL/kg is
the first-line therapy to correct metabolic acidosis. Administration of 1 mEq/kg sodium
bicarbonate to raise the pH to above 7.2 may be indicated to correct the adverse effects of
severe acidemia, such as decreased myocardial contractility, increased pulmonary vascular
resistance, sensitization to cardiac arrhythmias, and decreased responsiveness to
catecholamine administration, but volume administration remains the initial therapy of choice.
Adequate ventilation is imperative before administering sodium bicarbonate to ensure that the
additionally generated carbon dioxide is eliminated and not retained, which would decrease
cellular pH further. Children who have severe shock may require vasopressor (eg, dopamine)
administration if they do not respond to initial volume resuscitation.
Because metabolic acidosis produces an extracellular shift of potassium and subsequent
increase in serum potassium concentration, administration of potassium should be deferred until
adequate urine output is assured and correction of the underlying acidosis has begun. Insulin is
the cornerstone of treatment of the metabolic acidosis seen in diabetic ketoacidosis, but the child
in the vignette has no evidence of diabetes mellitus.
References:
Greenbaum LA. Electrolyte and acid-base disorders. In: Kliegman RM, Behrman RE, Jenson
HB, Stanton BF, eds. Nelson Textbook of Pediatrics. 18th ed. Philadelphia, Pa: Saunders
Elsevier; 2007:267-308
Schwaderer AL, Schwartz GJ. Back to basics: acidosis and alkalosis. Pediatr Rev. 2004;25:350357. Available at: http://pedsinreview.aappublications.org/cgi/content/full/25/10/350

Copyright 2009 by the American Academy of Pediatrics

page 261

2009 PREP SA on CD-ROM


Question: 122

A 3-year-old girl presents to the emergency department in an almost unresponsive state. Her
parents say that she has become increasingly ill over the past 5 days and has been very thirsty,
with increased urination. This morning she began to vomit and could not keep down fluids.
Findings on physical examination in addition to unresponsiveness include rapid, sighing
respirations and flushed cheeks. You estimate that she is 10% dehydrated. Initial laboratory
studies reveal a blood glucose concentration of 700.0 mg/dL (38.9 mmol/L), sodium of 130.0
mEq/L (130.0 mmol/L), potassium of 4.6 mEq/L (4.6 mmol/L), chloride of 96.0 mEq/L (96.0
mmol/L), bicarbonate of 8.0 mEq/L (8.0 mmol/L), and a venous pH of 7.0.
Of the following, the MOST appropriate action to decrease this childs risk for cerebral edema
during treatment is to

A. avoid potassium replacement until the serum potassium value is less than 4.0 mEq/L (4.0
mmol/L)

B. correct acidosis rapidly with sodium bicarbonate


C. rehydrate initially with 3% saline
D. rehydrate slowly using 0.45% to 0.9% saline
E. replace continuing urinary fluid losses with 0.2% saline

Copyright 2009 by the American Academy of Pediatrics

page 262

2009 PREP SA on CD-ROM


Critique: 122

Preferred Response: D

Treatment of diabetic ketoacidosis (DKA) requires supplementation with fluid, electrolytes,


insulin, and carbohydrate to replenish losses of fluid and electrolytes and supply insulin for
proper carbohydrate, protein, and lipid metabolism and to repair acidosis. The major lifethreatening complication of the treatment of DKA in children is cerebral edema, which occurs in 1
in 100 to 1 in 400 episodes of DKA. Slow rehydration with fluid containing adequate electrolytes
may decrease the risk of cerebral edema. The rate of replacement still is argued, but calculated
steady replacement of fluid losses as 0.45% to 0.9% saline, with glucose and potassium added
as necessary over 36 to 48 hours, generally is considered appropriate.
Large urinary potassium losses occur during the development of DKA. Even children who
present with hyperkalemia have had potassium losses. Insulin and glucose drive potassium into
cells and lower circulating potassium concentrations relatively rapidly during treatment.
Therefore, potassium replacement should be started as soon as the child is urinating and there
is no worrisome hyperkalemia. Avoiding potassium replacement until the potassium is less than
4.0 mEq/L (4.0 mmol/L) is unnecessary.
Peripheral acidosis is corrected rapidly by sodium bicarbonate, but bicarbonate dissociates
in blood into bicarbonate ion and carbon dioxide, and the carbon dioxide crosses cellular and
blood-brain barriers more rapidly than bicarbonate ion. Therefore, the net biologic intracellular
effect of bicarbonate administration is an increase in intracellular and central nervous system
(as measured in cerebrospinal fluid) acidosis. Several studies suggest that administration of
bicarbonate may increase the chance for an adverse outcome, including the increased risk of
cerebral edema. On the other hand, it is possible that only those who have the most severe
acidosis receive bicarbonate and that it is disease severity rather than treatment that worsens
outcome.
Three percent saline is hypertonic and would worsen renal fluid losses. This is not a good
maintenance fluid for treatment of DKA, although some evidence suggests that hypertonic saline
is as effective as mannitol in reducing symptomatic cerebral swelling, should it occur. Urine
sodium losses in DKA usually are about 75.0 mEq/L (75.0 mmol/L). Therefore, 0.2% saline is a
hypotonic replacement solution whose use is ill advised.
References:
Dunger DB, Sperling MA, Acerini CL, et al. European Society for Paediatric
Endocrinology/Lawson Wilkins Pediatric Endocrine Society consensus statement on diabetic
ketoacidosis in children and adolescents. Pediatrics. 2004;113:e133-e140. Available at:
http://pediatrics.aappublications.org/cgi/content/full/113/2/e133
Edge JA, Jakes RW, Roy Y, et al. The UK case-control study of cerebral oedema complicating
diabetic ketoacidosis in children. Diabetologia. 2006;49:2002-2009. Abstract available at:
http://www.ncbi.nlm.nih.gov/pubmed/16847700
Wolfsdorf J, Craig ME, Daneman D, et al; International Society for Pediatriac and Adolescent
Diabetes. Diabetic ketoacidosis. Pediatr Diabetes. 2007;8:28-43

Copyright 2009 by the American Academy of Pediatrics

page 263

2009 PREP SA on CD-ROM


Question: 123

A 14-year-old boy has been receiving occupational therapy due to weakness in his graphomotor
(eg, handwriting) skills. During the school annual Individualized Education Plan (IEP) meeting, his
mother asks about alternative strategies that could be used to help him compensate for his area
of weakness.
Of the following, the BEST alternative strategy is to

A. allow him to use print rather than cursive writing for his notes
B. have a class scribe write notes for him
C. have him use a word processor/laptop computer
D. have him use audio books
E. provide preferential seating near the blackboard

Copyright 2009 by the American Academy of Pediatrics

page 264

2009 PREP SA on CD-ROM


Critique: 123

Preferred Response: C

As students who have learning difficulties progress in their education, the emphasis changes
from remediation to accommodation. Accommodation encompasses modifications in how tasks
are given to students so that affected children may complete the same school work as other
students. Such accommodations allow students who have learning issues to present their
knowledge without being affected adversely by their disabilities.
The boy in the vignette, who has weakness in his graphomotor skills, should be allowed
access to electronic devices such as a laptop computer to help accommodate his area of
weakness. The use of a class scribe will not foster his independence in taking his own notes.
Allowing him to print instead of writing in script may make his notes neater to read, but it will slow
his ability to take notes efficiently. Preferential seating near the blackboard may be helpful for a
student who has a visual acuity problem, but it will not help this student's handwriting.
Audio texts are useful for students who read slowly. Other potential accommodations allow a
change in timing, formatting, setting, scheduling, response, or presentation of the assignment or
test. Such accommodations do not alter what the test or assignment measures in any significant
manner. Examples of such accommodations include a Braille version of a test for a student who
is blind or taking a test alone in a quiet room for one who has attention-deficit/hyperactivity
disorder.
References:
Thorne G. Graphomotor Skills: Why Some Kids Hate to Write. Center for Development and
Learning. Covington, La. 2006. Available at: http://www.cdl.org/resourcelibrary/articles/graphomotor.php
Shaywitz SE, Shaywitz BA. Dyslexia (specific reading disability) Pediatr Rev. 2003;24:147-153.
Available at: http://pedsinreview.aappublications.org/cgi/content/full/24/5/147

Copyright 2009 by the American Academy of Pediatrics

page 265

2009 PREP SA on CD-ROM


Question: 124

During hospital rounds, you are evaluating a 7-year-old boy who has been hospitalized for 1
month after developing a perforated appendix. He recently started eating solid foods, but today
he developed loose stools and a mild amount of abdominal discomfort. He has had four bowel
movements within the past 6 hours that are described as watery and nonbloody. His parents are
very concerned that this is another complication of his perforated appendix. On your physical
examination, the boy is afebrile and has active bowel sounds and only minor discomfort on
palpation to his abdomen.
Of the following, the MOST likely organism to be causing this patients diarrhea is

A. Campylobacter jejuni
B. Clostridium difficile
C. Salmonella enteritidis
D. Shigella sonnei
E. Yersinia enterocolitica

Copyright 2009 by the American Academy of Pediatrics

page 266

2009 PREP SA on CD-ROM


Critique: 124

Preferred Response: B

Although any gastrointestinal pathogen, including Campylobacter jejuni, Salmonella enteritidis,


Shigella sonnei, or Yersinia enterocolitica, can be acquired in a nosocomial outbreak,
hospitalized patients are most likely to have gastrointestinal problems due to Clostridium difficile.
Infection with this organism usually occurs in the setting of altered bowel flora, such as in
children who have a history of receiving antimicrobial therapy or other medications (eg,
chemotherapeutic agents), those who have required repeated enemas, and those who have had
prolonged nasal gastric tube insertions. Although penicillins, clindamycin, and cephalosporins
commonly are associated with C difficile infection, almost every antimicrobial agent has been
described as a potential cause. In fact, many clinical microbiology laboratories will not accept a
stool culture for the other pathogens from a patient hospitalized for longer than 3 days unless
testing for C difficile is negative.
Clinical symptoms of C difficile infection vary widely, and asymptomatic colonization is
common in infants and young children. Infants become colonized during the first postnatal week,
with up to 50% of infants younger than 1 year of age possibly being colonized. Carriage rates
decrease to 1% to 5% by 2 years of age. Risk factors for becoming colonized in the hospital
include having an infected roommate and having symptomatic patients on the same ward.
Symptomatic disease is due to the action of the toxins (A and B) that are produced by the
organism.
The clinical manifestations of a C difficile infection include diarrhea; abdominal cramps;
abdominal tenderness; and in some instances, fever, systemic toxicity, and bloody stools.
Severe or even fatal cases have been described, especially in patients who have problems with
neutropenia or those who have other gastrointestinal problems (eg, Hirschsprung disease).
The diagnosis of disease due to C difficile is made by documenting the presence of toxin A
or B with a commercially available kit or by endoscopic findings of pseudomembranous
enterocolitis (Item C124). Symptomatic infants younger than 1 year of age should be
investigated for other causes of diarrhea because the carriage rate is so high, and the presence
of toxin may not be responsible for the patient's clinical signs or symptoms.
References:
American Academy of Pediatrics. Clostridium difficile. In: Pickering LK, Baker CJ, Long SS,
McMillan JA, eds. Red Book: 2006 Report of the Committee on Infectious Diseases. 27th ed. Elk
Grove Village, Ill: American Academy of Pediatrics; 2006:261-263
Brook I. Pseudomembranous colitis in children. J Gastroenterol Hepatol. 2005;20:182-186.
Abstract available at: http://www.ncbi.nlm.nih.gov/pubmed/15683418

Copyright 2009 by the American Academy of Pediatrics

page 267

2009 PREP SA on CD-ROM


Question: 125

You are evaluating a 7-year-old boy who has human immunodeficiency virus (HIV) infection and
failure to thrive. Over the last 2 months, he has had intermittent temperatures to 102.5F
(39.2C), anorexia, abdominal pain, diarrhea, and a 6-lb weight loss. His HIV infection has been
poorly controlled because of noncompliance with medications. His viral load is 150,000 copies,
and his CD4 count is 40 cells/mm3. Physical examination shows a thin, small-for-age boy who is
playing quietly. He weighs 15.8 kg (<5th percentile). He has some oral thrush, cervical and
inguinal adenopathy, and mild diffuse abdominal tenderness to palpation.
Of the following, the MOST likely pathogen causing this patients illness is

A. Cryptosporidium
B. cytomegalovirus
C. Mycobacterium avium-intracellulare complex
D. Mycobacterium tuberculosis complex
E. Salmonella sp

Copyright 2009 by the American Academy of Pediatrics

page 268

2009 PREP SA on CD-ROM


Critique: 125

Preferred Response: C

The child described in the vignette displays clinical signs and symptoms that are classic for
disseminated Mycobacterium avium-intracellulare complex (MAC) disease. This infection
occurs in patients who have impaired cell-mediated immunity (eg, congenital immune defects or
human immunodeficiency virus [HIV] infection). The greatest risk factor for MAC in patients who
have HIV infection is a severe depression of the CD4 cell count. Patients whose CD4 counts are
less than 100 cells/mm3 are at the highest risk for disease. Clinical manifestations of
disseminated nontuberculous mycobacterial infection may include high fever, night sweats,
weight loss, lymphadenopathy, abdominal pain, fatigue, hepatosplenomegaly, diarrhea, and
anemia. Diagnosis is established by isolation of the organism from blood or infected tissues.
The nontuberculous mycobacteria that most commonly cause disease in children are: MAC
(eg, M avium and M intracellulare), M fortuitum, M abscessus, M marinum, M chelonae, and M
kansasii. These organisms may cause a variety of clinical syndromes, including cervical
lymphadenitis (MAC), cutaneous infections (M fortuitum, M abscessus, M marinum, M
chelonae), otitis media (M abscessus), osteomyelitis (MAC, M fortuitum, M kansasii), central
catheter infections (M chelonae, M fortuitum), pulmonary disease (MAC, M kansasii, M
fortuitum), and disseminated infection (MAC). In the immunocompromised host, the most
common manifestations of cytomegalovirus disease are pneumonia, colitis, and retinitis, none of
which are manifested in this patient. Disease caused by Cryptosporidium is associated with a
chronic diarrhea; fever usually is not seen. M tuberculosis generally is associated with
pulmonary manifestations. Salmonella sp is associated with acute diarrhea.
References:
American Academy of Pediatrics. Diseases caused by nontuberculous mycobacteria (atypical
mycobacteria, mycobacteria other than Mycobacterium tuberculous). In: Pickering LK, Baker
CJ, Long SS, McMillan JA, eds. Red Book: 2006 Report of the Committee on Infectious
Diseases. 27th ed. Elk Grove Village, Ill: American Academy of Pediatrics; 2006:698-704
Gordin FM, Horsburgh CR Jr. Mycobacterium avium complex. In: Mandell GL, Bennett JE, Dolin
R, eds. Mandell, Douglas, and Bennett's Principles and Practice of Infectious Diseases. 6th ed.
Philadelphia, Pa: Elsevier Churchill Livingstone; 2005:2897-2909
Horsburgh CR Jr, Caldwell MB, Simonds RJ. Epidemiology of disseminated nontuberculous
mycobacterial disease in children with acquired immunodeficiency syndrome. Pediatr Infect Dis
J. 1993;12:219-222. Abstract available at: http://www.ncbi.nlm.nih.gov/pubmed/8095716

Copyright 2009 by the American Academy of Pediatrics

page 269

2009 PREP SA on CD-ROM


Question: 126

You are seeing a 7-year-old boy for occasional nocturnal enuresis. His weight and height are at
the 50th percentile for age, his blood pressure is 110/66 mm Hg, and there are no unusual
findings on physical examination. Urinalysis shows a specific gravity of 1.030, pH of 6.5, 2+
blood, and no protein. Urine microscopy reveals 10 to 20 red blood cells/high-power field and no
casts or crystals. Results of a repeat urine sample 3 weeks later are unchanged. Laboratory
findings include:
Blood urea nitrogen, 12.0 mg/dL (4.3 mmol/L)
Creatinine, 0.4 mg/dL (35.4 mcmol/L)
Complement component 3 (C3), 110.0 mg/dL (normal, 86.0 to 166.0 mg/dL)
Complement component 4 (C4), 22.0 mg/dL (normal, 13.0 to 32.0 mg/dL)
Antinuclear antibody, negative
Erythrocyte sedimentation rate, 6 mm/hr
Of the following, the MOST appropriate next step is

A. abdominal computed tomography scan


B. referral for cystoscopy
C. referral for renal biopsy
D. renal/bladder ultrasonography
E. repeat urinalysis in 1 month

Copyright 2009 by the American Academy of Pediatrics

page 270

2009 PREP SA on CD-ROM


Critique: 126

Preferred Response: D

Microscopic hematuria is defined as a positive dipstick test for blood and more than 5 red blood
cells/high-power field on microscopy (Item C126). Persistent microscopic hematuria is defined
as blood on urinalysis detected on repeat samples over a 1-month period. Children who have
persistent microscopic hematuria, such as the boy described in the vignette, require
investigation for an underlying cause. The urgency of such an evaluation in the absence of
symptoms is predicated on whether the patient has accompanying proteinuria (>1+ on a
dipstick). Those who have proteinuria require urgent evaluation to look for an underlying
glomerulopathy in which disruption of the glomerular capillary barrier (as occurs in
glomerulonephritis) results in red blood cells and albumin gaining access to the urinary space.
Up to 4% of all children may have microscopic hematuria on a routine screening urinalysis.
Based on this high incidence, the initial recommendation for a child who has isolated,
asymptomatic microscopic hematuria is to undergo repeat urinalysis in 2 to 3 weeks time. If the
hematuria persists, as occurred in the patient in the vignette, the clinician should evaluate the
patient for an underlying genitourinary problem. This evaluation typically includes renal function
tests (serum creatinine) and serologic testing for an underlying immune complex-mediated
glomerulonephritis (complement components 3 and 4, antinuclear antibody). In addition, renal
imaging (renal/bladder ultrasonography) to screen for cysts, stones, and tumors is indicated.
Abdominal computed tomography scan is not indicated prior to a renal/bladder ultrasonography
in this clinical setting. More invasive testing, such as cystoscopy, should not be undertaken
unless there is a strong suspicion of bladder pathology, especially prior to screening with
ultrasonography. Similarly, renal biopsy is not indicated in the absence of proteinuria,
hypertension, azotemia, or gross hematuria. Repeat urinalysis is redundant because persistent
hematuria already has been substantiated.
References:
Moxey-Mims M. Hematuria and proteinuria. In: Kher KK, Schnaper HW, Makker SP, eds. Clinical
Pediatric Nephrology. 2nd ed. London, England: Informa Healthcare; 2007:129-141
Patel HP, Bissler JJ. Hematuria in children. Pediatr Clin North Am. 2001;48:1519-1537. Abstract
available at: http://www.ncbi.nlm.nih.gov/pubmed/11732128
Vehaskari VM, Rapola J, Koskimies O, Savilahti E, Vilska J, Hallman N. Microscopic hematuria
in school children: epidemiology and clinicopathologic evaluation. J Pediatr. 1979;95:676-684.
Abstract available at: http://www.ncbi.nlm.nih.gov/pubmed/490233

Copyright 2009 by the American Academy of Pediatrics

page 271

2009 PREP SA on CD-ROM


Question: 127

An 8-year-old girl presents with multiple episodes of "bronchitis." For the past 2 years, she has
had problems with coughing, wheezing, and difficulty catching her breath during vigorous
exercise. Treatment with a metered dose beta2 agonist inhaler has improved her symptoms. In
your office, you discuss the different tests to assess lung function.
Of the following, the BEST test to measure lung function for this girl is

A. arterial blood gas


B. exhaled breath condensate
C. exhaled nitric oxide
D. pulse oximetry
E. spirometry

Copyright 2009 by the American Academy of Pediatrics

page 272

2009 PREP SA on CD-ROM


Critique: 127

Preferred Response: E

Spirometry, also referred to as a pulmonary function test (PFT), measures inspiratory and
expiratory respiratory effort. Established pretest norms are based on the patient's height and
ethnicity. Measurements of lung function that can be obtained with spirometry include the
forced vital capacity (FVC), volume of air exhaled during the first second (FEV1), FEV1/FVC
ratio, peak expiratory flow (PEF), and airflow during the middle half of the effort (forced
expiratory flow) (FEF25-75). Because ideal test conditions require a patient to exhale for 6
seconds, children younger than 6 to 7 years of age often are unable to complete the test. Also,
spirometry does not measure total lung capacity (TLC) or residual volume (RV).
An arterial blood gas is a gas diffusion measurement that provides insight into oxygenation
(PO2) and ventilation (PCO2), but it does not measure lung function. Pulse oximetry is a simple,
noninvasive method to measure oxygenation, but it also does not measure ventilation or lung
function.
Exhaled nitric oxide (eNO) measurement was approved by the United States Food and
Drug Administration in 2003 for children ages 4 years and older. Although not a measure of lung
function, eNO is a useful noninvasive tool to measure nitric oxide, a marker of airway
inflammation.
Exhaled breath condensate is a noninvasive method to measure the pH of the airway,
another marker of inflammation. Similar to eNO, exhaled breath condensate does not measure
lung function but is being developed as another tool to assess airway inflammation and assist
with asthma management.
References:
Guill MF. Asthma update: clinical aspects and management. Pediatr Rev. 2004;10:335-344.
Available at: http://pedsinreview.aappublications.org/cgi/content/full/25/10/335
Liu AH, Covar RA, Spahn JD, Leung DYM. Childhood asthma. In: Kliegman RM, Behrman RE,
Jenson HB, Stanton BF, eds. Nelson Textbook of Pediatrics. 18th ed. Philadelphia, Pa:
Saunders Elsevier; 2007:953-969
Silkoff PE, Carlson M, Bourke T, Katial R, Ogren E, Szefler SJ. The Aerocrine exhaled nitric
oxide monitoring system NIOX is cleared by the US Food and Drug Administration for monitoring
therapy in asthma. J Allergy Clin Immunol. 2004;114:1241-1256. Abstract available at:
http://www.ncbi.nlm.nih.gov/pubmed/15536442

Copyright 2009 by the American Academy of Pediatrics

page 273

2009 PREP SA on CD-ROM


Question: 128

A 4-year-old girl is brought to the emergency department after sticking a hair pin in a household
electrical outlet. The mother reports that she heard the child scream, and when she investigated,
smoke was coming from the outlet and the child was crying, holding her right hand. There was a
black imprint on her fingers in the shape of the hair pin. She washed the fingers with soap and
water and drove the child to the emergency department for further evaluation. On physical
examination, the child is tearful but awake and alert. Her right index finger and thumb have
erythematous burn imprints with small blisters surrounded by soot. She has no other burns or
other findings of note on the remainder of her examination.
Of the following, it is MOST important to evaluate this patient for

A. arrhythmias
B. compartment syndrome
C. immunization status
D. myoglobinuria
E. skin grafting

Copyright 2009 by the American Academy of Pediatrics

page 274

2009 PREP SA on CD-ROM


Critique: 128

Preferred Response: C

Electrical injuries account for approximately 5% of admissions to burn units in the United States
annually. Most injuries occur in adults and are related to high-voltage sources in the workplace.
Pediatric injuries typically are related to contact with household, low-voltage sources such as
electric cords or wall outlets. The primary determinant of tissue damage following electrical
exposure is current strength, which is directly proportional to voltage and inversely related to
tissue resistance. Because voltage is the only variable commonly known, electrical exposures
are classified as low- (<1,000 V) and high- (>1,000 V) voltage. Standard household electricity in
the United States is 110 V; utility power lines carry in excess of 14,000 V.
The most common injury following contact with electricity is burns. Excluding lightening
injuries, electrical burns may occur from any of the following mechanisms: electrothermal
heating from direct contact with the electrical source, arc exposure in which the body becomes
part of the electrical circuit, flash contact in which the current arc strikes the skin but does not
enter the body, and flame burns that result when clothing or other objects combust in the
presence of electrical current. Oral burns due to chewing on an electrical cord (Item C128)
usually are electrothermal. Arc exposure can be associated with deep tissue burns and internal
organ involvement as the current flows through body; the extent of injury may be
underestimated by the appearance of the skin wounds. Flash contact is characterized by
surface burns accompanied by soot, as described for the girl in the vignette.
Although both high- and low-voltage exposures can lead to injuries in a variety of organ
systems, high-voltage exposures are responsible for most significant injuries. The most
commonly affected organ is the skin, with burns resulting as described previously. Arrhythmias,
including asystole and ventricular fibrillation, may occur, usually at the time of contact. Deep
thermal injuries to bone and muscle may occur after arc exposure and lead to compartment
syndrome or rhabdomyolysis with subsequent renal damage. Risk factors for these more
serious injuries include extensive full-thickness burns, cardiac arrest, or high-voltage exposure.
It is rare for children exposed to household current to sustain injuries other than burns.
The girl in the vignette has a surface thermal burn related to contact with the heated hair pin,
and the presence of soot on her finger is evidence of flash contact. She has no clinical features
or risk factors for rhabdomyolysis, myoglobinuria, compartment syndrome, or arrhythmias. The
wound is small and should not require skin grafting. Therefore, other than ascertaining her
tetanus immunization status and providing wound care, no further evaluation is indicated.
References:
Chen EH, Sareen A. Do children require ECG evaluation and inpatient telemetry after household
electrical exposures? Ann Emerg Med. 2007;49:64-67
Pinto DS, Clardy PF. Environmental electrical injuries. UpTo Date Online 15.3. 2008 Available for
subscription at:
http://www.utdol.com/utd/content/topic.do?topicKey=ad_emerg/2283&selectedTitle=1~150&sour
ce=search_result
Price TG, Cooper MA. Electrical and lightning injuries. In Marx JA, ed. Rosen's Emergency
Medicine: Concepts and Clinical Practice. 6th ed. Philadelphia, Pa: Mosby Elsevier; 2006:chap
140
Rosen CL, Adler JN, Rabban JT, et al. Early predictors of myoglobinuria and acute renal failure
following electrical injury. J Emerg Med. 1999;17:783-789. Abstract available at:
http://www.ncbi.nlm.nih.gov/pubmed/10499690

Copyright 2009 by the American Academy of Pediatrics

page 275

2009 PREP SA on CD-ROM


Question: 129

A 12-year-old girl has had intermittent periumbilical abdominal pain for the past 4 years.
Sometimes the pain worsens when she drinks a glass of milk. A lactose breath hydrogen test
demonstrates a breath hydrogen of 40 ppm after 1 hour (normal, <20 ppm).
Of the following, the food that this girl is MOST likely to tolerate is

A. buttermilk
B. cheddar cheese
C. ice cream
D. skim milk
E. whole milk

Copyright 2009 by the American Academy of Pediatrics

page 276

2009 PREP SA on CD-ROM


Critique: 129

Preferred Response: B

Lactose intolerance (lactose maldigestion) is a common cause of pediatric recurrent abdominal


pain. Lactase is a digestive enzyme located on the intestinal villi that converts the disaccharide
lactose to the monosaccharides glucose and galactose. The monosaccharides can be
absorbed across the intestinal epithelial cells. In an individual who has low intestinal lactase
(lactose intolerance), the lactose cannot be broken down and passes into the lower intestine and
colon. The malabsorbed lactose can cause osmotic diarrhea or be fermented by bacteria,
resulting in pain, gas, and bloating. Lactose intolerance can either be primary (lactase activity
that gradually declines with aging) or secondary (an infection or enteropathy damages the villi,
resulting in lactose intolerance). Primary lactose intolerance (adult-type hypolactasia) is
extremely common, occurring in approximately 20% of white adults, 80% of African American
adults, and 90% of Asian adults. It is uncommon in children younger 5 years of age, but as
children grow, those who are genetically predisposed to lactose intolerance may develop the
condition gradually during their school age years.
Lactose intolerance may be diagnosed clinically by elimination diet and rechallenge or
through lactose breath hydrogen testing. Breath testing offers the advantage of providing a more
definitive diagnosis, so milk is not withdrawn from the diet unnecessarily. In a breath hydrogen
test, an adult is given approximately 50 g of lactose (the equivalent of 4 cups of milk) as a single
dose. Patients who cannot digest and absorb this amount of lactose have the lactose fermented
to hydrogen by intestinal bacteria, which can be measured in the breath. Because intestinal
bacterial fermentation is essential for an accurate test, patients should not receive antibiotics for
at least 2 weeks prior to the test.
Many patients who have lactose intolerance diagnosed on breath test can tolerate small
amounts of lactose in their diet. Specifically, hard cheeses, such as cheddar, which often have
very little lactose (<0.5 g/serving), can be eaten by many patients who have lactose intolerance.
Other products, such as whole or skim milk, ice cream, or buttermilk, have more lactose
(approximately 10 g/serving) and are less likely to be tolerated (Item C129).
References:
Heyman MB for American Academy of Pediatrics Committee on Nutrition. Lactose intolerance in
infants, children, and adolescents. Pediatrics. 2006;118:1279-1286. Available at:
http://pediatrics.aappublications.org/cgi/content/full/118/3/1279
Thiessen PN. Recurrent abdominal pain. Pediatr Rev. 2002;23:39-46. Available at:
http://pedsinreview.aappublications.org/cgi/content/full/23/2/39

Copyright 2009 by the American Academy of Pediatrics

page 277

2009 PREP SA on CD-ROM


Question: 130

You are treating a former extremely low-gestational age newborn (ELGAN) who was born at 26
weeks gestation weighing 700 g. She is now 4 weeks old. Her nurse asks when the eye
examination for retinopathy of prematurity (ROP) will be performed and what risk for significant
visual impairment exists in this infant.
Of the following, the BEST time to obtain the first ROP screening eye examination in this infant is

A. 4 weeks after discharge from the hospital


B. 4 weeks after weaning from oxygen
C. 5 weeks after birth
D. 5 weeks after the expected delivery date
E. 5 weeks after weaning from the ventilator

Copyright 2009 by the American Academy of Pediatrics

page 278

2009 PREP SA on CD-ROM


Critique: 130

Preferred Response: C

Retinopathy of prematurity (ROP) is a multifactorial disorder that has been linked historically and
epidemiologically to the survival of low-birthweight preterm infants. As the survival of smaller and
more immature infants has increased throughout the past 5 decades, the incidence of ROP has
shifted toward the smallest, most immature, and sickest babies in neonatal intensive care units
(NICUs) in the United States and worldwide.
Prospective studies and critical reviews and analyses of extensive data confirm that the
greatest known risks for ROP among NICU patients include: very low birthweight (<1,500 g),
severity of illness, gestational immaturity (especially <28 weeks' gestational age), intrauterine
growth restriction, male sex, and systemic fungal infection. Among the additional possible risks
are antenatal dexamethasone exposure (compared to betamethasone) and late treatment with
dexamethasone (treatment of infants more than 3 weeks of age for 2 or more weeks).
The American Academy of Pediatrics recommends a specific timetable for testing at-risk
preterm infants (Item C130). In general, infants should be tested between 31 and 34 weeks
gestational age. Accordingly, the infant described in the vignette should be tested 5 weeks after
birth.
Because most 26 weeks' gestation infants are discharged from the hospital at a gestational
equivalent of 34 to 36 weeks, or 8 to 10 postnatal weeks of age, this would be too late for an
initial examination, as would one occurring 4 weeks later. Many 26 weeks' gestation infants may
have chronic lung disease and be receiving assisted ventilation for days to weeks or be
discharged from the hospital on oxygen, so screening is not based on the time of weaning from
assisted ventilation or oxygen. Five weeks after the expected delivery date is too late for initial
screening.
References:
Bharwani SK, Dhanireddy R. Systemic fungal infection is associated with the development of
retinopathy of prematurity in very low birth weight infants: a meta-review. J Perinatol. 2007;28:6166. Abstract available at: http://www.ncbi.nlm.nih.gov/pubmed/18046338
Darlow BA, Hutchinson JL, Henderson-Smart DJ, Donoghue DA, Simpson JM, Evans NJ on
behalf of the Australian and New Zealand Neonatal Network. Prenatal risk factors for severe
retinopathy of prematurity among very preterm infants of the Australian and New Zealand
Neonatal Network. Pediatrics. 2005;115:990-996. Available at:
http://pediatrics.aappublications.org/cgi/content/full/115/4/990
Hagadorn JI, Richardson DK, Schmid CH, Cole CH. Cumulative illness severity and progression
from moderate to severe retinopathy of prematurity. J Perinatol. 2007;27:502-509. Abstract
available at: http://www.ncbi.nlm.nih.gov/pubmed/17568754
Karlowicz MG, Giannone PJ, Pestian J, Morrow AL, Shults J. Does candidemia predict threshold
retinopathy of prematurity in extremely low birth weight (<1000 g) neonates? Pediatrics.
2000;105:1036-1040. Available at:
http://pediatrics.aappublications.org/cgi/content/full/105/5/1036
Lee BH, Stoll BJ, McDonald SA, Higgins RD for the National Institute of Child Health and Human
Development Neonatal Research Network. Adverse neonatal outcomes associated with
antenatal dexamethasone versus antenatal betamethasone. Pediatrics. 2006;117:1503-1510.
Available at: http://pediatrics.aappublications.org/cgi/content/full/117/5/1503
Markestad T, Kaaresen PI, Rnnestad A, et al on behalf of the Norwegian Extreme Prematurity
Study Group. Early death, morbidity, and need of treatment among extremely premature infants.
Pediatrics. 2005;115:1289-1298. Available at:
http://pediatrics.aappublications.org/cgi/content/full/115/5/1289

Copyright 2009 by the American Academy of Pediatrics

page 279

2009 PREP SA on CD-ROM

Section on Ophthalmology, American Academy of Pediatrics, American Academy of


Ophthalmology, American Association for Pediatric Ophthalmology and Strabismus. Screening
examination of premature infants for retinopathy of prematurity. Pediatrics. 2006;117:572-576.
Available at: http://pediatrics.aappublications.org/cgi/content/full/117/2/572

Copyright 2009 by the American Academy of Pediatrics

page 280

2009 PREP SA on CD-ROM


Question: 131

A 15-year-old boy who has mild persistent asthma is brought to the emergency department
because of increased work of breathing of 1 days duration. He reports a low-grade fever and
nonproductive cough for the past 4 days, but this morning he developed difficulty breathing and
a cough that produced a small amount of yellowish sputum. His respiratory rate is 24
breaths/min, heart rate is 80 beats/min, and temperature is 99.0F (37.3C). He appears in no
respiratory distress, but his lung examination reveals bilateral rales and occasional wheezes. A
chest radiograph shows bilateral diffuse infiltrates with no effusions.
Of the following, the MOST likely etiologic agent causing his symptoms is

A. Haemophilus influenzae
B. Mycobacterium tuberculosis
C. Mycoplasma pneumoniae
D. Staphylococcus aureus
E. Streptococcus pneumoniae

Copyright 2009 by the American Academy of Pediatrics

page 281

2009 PREP SA on CD-ROM


Critique: 131

Preferred Response: C

Mycoplasma pneumoniae is a common cause of community-acquired pneumonia in school-age


children and adolescents, but it is rare in infants and toddlers. Other clinical manifestations of M
pneumoniae infection include upper respiratory tract infection, pharyngitis, bullous myringitis, and
bronchitis. Occasionally, systemic symptoms such as malaise and fever are present. Among
the rarer clinical presentations are hemolytic anemia, aseptic meningitis, and Stevens-Johnson
syndrome. M pneumoniae is responsible for many cases of pneumonia and acute chest
syndrome in children who have sickle cell disease. Infection also may precipitate an
exacerbation of asthma, as described for the boy in the vignette. A recent study showed that
children of different ages may respond differently to infection with M pneumoniae. Children
younger than 5 years of age were more likely to experience coryza, vomiting and diarrhea, and
tachypnea than were older children.
Typical radiographic findings are bilateral, diffuse infiltrates (Item C131A), but occasionally a
lobar infiltrate or pleural effusion may be seen. The diagnosis frequently can be made clinically,
but serologic testing is available. The complement fixation test and, more recently,
immunofluorescence and enzyme immunoassay can be used to measure levels of
immunoglobulin M (IgM) and IgG antibodies to M pneumoniae. High concentrations of IgM may
be present for months after acute infection. Comparing acute and convalescent IgG titers allows
for more accurate diagnosis, although this may be impractical. Serum cold hemagglutinin titers
also may be positive in patients who have acute infection but have limited sensitivity and
specificity. Direct polymerase chain reaction is both sensitive and specific for the diagnosis but
is not widely available. Macrolide antibiotics generally are used for treatment, although many
infections are self-limited, and a recent review found no conclusive evidence that antibiotics are
effective in reducing symptoms.
Haemophilus influenzae and Streptococcus pneumoniae also cause community-acquired
pneumonia, but fever generally is present with these infections, and radiographs typically reveal
lobar infiltrates (Item C131B) with or without pleural effusions. Lobar infiltrates also are typical of
pneumonia caused by Staphylococcus aureus, and pneumatoceles (Item C131C) or lung
abscesses are common with this infection. The radiologic features of M tuberculosis infection
are varied and include lobar infiltrates, hilar adenopathy, and cavitary lesions (Item C131D).
References:
American Academy of Pediatrics. Mycoplasma pneumoniae infections. In: Pickering LK, Baker
CJ, Long SS, McMillan JA, eds. Red Book: 2006 Report of the Committee on Infectious
Diseases. 27th ed. Elk Grove Village, Ill: American Academy of Pediatrics; 2006:468-470
Gavranich JB, Chang AB. Antibiotics for community acquiredlower respiratory tract infections
(LRTI) secondary to Mycoplasma pneumoniae in children. Cochrane Database Syst Rev.
2005;3:CD004875. Available at:
http://www.mrw.interscience.wiley.com/cochrane/clsysrev/articles/CD004875/frame.html
Othman N, Isaacs D, Kesson A. Mycoplasma pneumoniae infections in Australian children. J
Paediatr Child Health. 2005;41:671-676. Abstract available at:
http://www.ncbi.nlm.nih.gov/pubmed/16398873

Copyright 2009 by the American Academy of Pediatrics

page 282

2009 PREP SA on CD-ROM


Question: 132

A resident in continuity clinic approaches you to review the laboratory values obtained at a
patients 12-month health supervision visit. The fingerstick hemoglobin measurement was 10.5
g/dL (105.0 g/L), and the lead concentration was 11.0 mcg/dL (0.53 mcmol/L).
Of the following, the next BEST step for this boy is to

A. admit him to the hospital for chelation therapy


B. call child protective services to move him to a shelter
C. call the health department to arrange for an environmental investigation
D. measure the venous lead concentration
E. refer him for formal developmental evaluation and neuropsychometric testing

Copyright 2009 by the American Academy of Pediatrics

page 283

2009 PREP SA on CD-ROM


Critique: 132

Preferred Response: D

Routine screening of children for exposure to environmental lead combined with environmental
abatement has resulted in an overall decrease in lead toxicity among children living in the United
States. However, the laboratory findings of mild anemia and a capillary blood lead concentration
that is marginally elevated, as described for the child in the vignette, are still common. Repeat
measurement of the blood lead by venipuncture to avoid skin contamination as well as further
evaluation of the child's mild anemia are appropriate next steps.
In 1997, as a result of the decrease in lead exposure and prevalence of lead poisoning, the
Centers for Disease Control and Prevention (CDC) updated its screening guidelines and
incorporated, for the first time, screening policies that use local blood lead concentration data or
housing data collected by the United States Bureau of Census to determine risk. In geographic
areas where limited exposure to lead and limited prevalence of elevated blood lead
concentrations are documented, targeted (selective) screening may be more appropriate than
universal screening. However, one of the goals of Healthy People 2010 is the elimination of
childhood lead poisoning as a public health problem.
Current guidelines state that health-care professionals should use blood lead tests to screen
children at ages 1 and 2 years and children 36 to 72 months of age who previously had not been
screened if they meet specific criteria. The CDC and state and local health departments
continue to develop surveillance and intervention tools to determine the best practices for
targeted screening. Pediatricians may consult the CDC Web site and their local health
departments for zip code-related risk levels as well as questionnaires to assess individual
children's risk factors based on history of housing or other exposures. Screening of toddlers for
lead exposure continues to be recommended and even mandated by health-care organizations.
The American Academy of Pediatrics policy statement recommends the use of venous
samples for initial screening whenever possible. If capillary testing is performed and the lead
concentration is greater than 10.0 mcg/dL (0.5 mcmol/L), the lead concentration must be
confirmed by a venous sample because microlead sampling is more likely to yield false-positive
results due to contamination from environmental lead.
Lead ingestion may cause a microcytic anemia by interfering with iron absorption and
utilization in heme production and can inhibit enzymes required for heme synthesis directly.
Children who have lead poisoning may have pica either as a cause or symptom of lead
poisoning. In these children, iron supplementation should be initiated until the presence or
absence of iron deficiency is determined.
In many communities, formal environmental evaluation of the child's home or removal of the
child and family from that home is not conducted until the blood lead value has been confirmed
and is higher than 20.0 mg/dL (1.0 mcmol/L). Although the child's developmental status should
be monitored carefully in the future as a part of recommended surveillance and screening,
formal neurodiagnostic testing is not indicated at this time unless there are other risk factors for
developmental delay.
Counseling the family about potential sources of lead in the environment (eg, dirt, cosmetics,
or household items) and lead solder in old pipes may be beneficial.
Most asymptomatic children who have mildly elevated blood lead concentrations are not
candidates for chelation therapy with the currently available drugs because the toxicity of these
drugs outweighs the potential benefit of treatment, and chelation is unlikely to increase lead
excretion significantly. Chelation therapy should be considered, however, if lead concentrations
are higher than 44.0 mcg/dL (2.12 mcmol/L). The role of chelation is not clearly defined for
children whose blood lead concentrations range from 20.0 to 45.0 mcg/dL (0.97 to 2.17
mcmol/L). In this range, the clinician may choose to pursue further environmental screening,
attempt to eradicate lead from the child's environment, and measure blood lead concentrations
monthly. If the concentration remains in this range, despite successful eradication of the lead
source, the physician should institute behavior modification, nutritional sufficiency, or chelation
treatment.
Succimer is the drug of choice for children whose blood lead concentrations are 45.0 to
100.0 mcg/dL (2.17 to 4.8 mcmol/L). At values higher than 69.0 mcg/dL (3.3 mcmol/L), a second
drug, CaNa2EDTA, is added. For children who require two-drug treatment, the first dose always
Copyright 2009 by the American Academy of Pediatrics

page 284

2009 PREP SA on CD-ROM

is succimer, followed 4 hours later by CaNa2EDTA, because children who present with lead
encephalopathy may deteriorate when treated with CaNa2EDTA alone. D-penicillamine is not
recommended as first-line therapy because adverse effects and allergy are common (33%).
Dimercaprol is not a first-line drug because the rate of adverse effects approaches 50%.
Serious adverse effects of chelation therapy may occur due to chelation of other electrolytes.
NaEDTA has been associated with fatal hypocalcemia and, therefore, is contraindicated. The
clinician should be careful not to confuse CaNa2EDTA with NaEDTA. Chelation therapy should
be conducted in conjunction with a pediatric toxicologist or pharmacist under very close
supervision. Oral agents may be used, but succimer is not palatable and must be emptied from
a capsule onto food.
Once lead has been ingested, the percentage absorbed may be modified by essential
nutrients. A healthy diet can be recommended for both lead-exposed and nonexposed children.
Particular attention must be given to calcium and iron intake. If iron deficiency is diagnosed,
supplemental iron should be prescribed. For children who are not iron-deficient, a multivitamin
with iron can be recommended, but its efficacy is unproven. Similarly, no published data support
a role for therapeutic administration of calcium or iron as treatment for lead poisoning in the
absence of deficiency. Such studies are being conducted.
Blood lead concentrations fall precipitously after completion of chelation, but rebound within
weeks, even if there is no further exposure to lead, due to release of lead from bone stores. In
general, the concentrations do not return to the high values seen before chelation, and a second
course of chelation rarely is indicated.
References:
Binns,HJ, Campbell,C, Brown,MJ for the Advisory Committee on Childhood Lead Poisoning
Prevention. Interpreting and managing blood lead levels of less than 10 mcg/dL in children and
reducing childhood exposure to lead: recommendations of the Centers for Disease Control and
Prevention Advisory Committee on Childhood Lead Poisoning Prevention. Pediatrics.
2007;120:e1285-e1298. Available at:
http://pediatrics.aappublications.org/cgi/content/full/120/5/e1285
Centers for Disease Control and Prevention. Lead program: state and local programs. Available
at: http://www.cdc.gov/nceh/lead/grants/contacts/CLPPP%20Map.htm
Laraque D, Trasande L. Lead poisoning: successes and 21st century challenges. Pediatr Rev.
2005;26:435-443. Available at: http://pedsinreview.aappublications.org/cgi/content/full/26/12/435
Markowitz M. Lead poisoning. Pediatr Rev. 2000;21:327-335. Available at:
http://pedsinreview.aappublications.org/cgi/content/full/21/10/327
Richardson M. Microcytic anemia. Pediatr Rev. 2007;28:5-14.
Rischitelli G, Nygren P, Bougatsos C, Freeman M, Helfand M. Screening for Elevated Lead
Levels in Childhood and Pregnancy: An Update of a 1996 U.S. Preventive Services Task Force
Review. Rockville, Md: Agency for Healthcare Research and Quality, U.S. Department of Health
and Human Services; 2006. Available at: http://www.ahrq.gov/clinic/uspstf06/lead/leadsum.htm
Yuan W, Holland SK, Cecil KM, et al. The impact of early childhood lead exposure on brain
organization: a functional magnetic resonance imaging study of language function. Pediatrics.
2006;118:971-977. Available at: http://pediatrics.aappublications.org/cgi/content/full/118/3/971

Copyright 2009 by the American Academy of Pediatrics

page 285

2009 PREP SA on CD-ROM


Question: 133

You are evaluating a 15-year-old boy in the emergency department who presents with fever,
chills, malaise, and blood in his urine. On physical examination, he appears comfortable and alert
and has a temperature of 102.7F (39.3C), a blood pressure of 110/40 mm Hg, no rashes, and
clear breath sounds. He has a diastolic murmur heard best in the sitting position (Item Q133).
You elicit no abdominal or flank tenderness.
Of the following, the BEST next step in the management of this patient is

A. administration of broad-spectrum antibiotics


B. blood cultures
C. renal ultrasonography
D. transesophageal echocardiography
E. urine culture

Copyright 2009 by the American Academy of Pediatrics

page 286

2009 PREP SA on CD-ROM


Critique: 133

Preferred Response: B

The patient described in the vignette has history and physical examination findings that are
highly suggestive of infective endocarditis. These include symptoms of chills and malaise; a
history of fever; and the findings of hematuria, a new murmur, and fever. Typically, the diagnosis
is confirmed by isolation of the offending organism from blood cultures. Blood cultures from three
to five sites should be obtained prior to initiation of antibiotic therapy. Because the bacterial
shedding is constant, the practitioner should not wait until the patient is febrile to obtain blood
cultures. Viridans streptococci (eg, S bovis, S mitis) as well as Staphylococcus aureus are the
most common bacterial pathogens causing endocarditis in children. However, clinicians must be
concerned about organisms such as Enterococcus, coagulase-negative Staphylococcus, fungi,
and a group of bacteria referred to as the HACEK organisms (Haemophilus sp, Actinobacillus
actinomycetemcomitans, Cardiobacterium hominis, Eikenella corrodens, and Kingella kingae).
The HACEK organisms are gram-negative oral and pharyngeal flora that are fastidious and slowgrowing, often requiring growth factors and carbon dioxide to be isolated in cultures.
Treatment of endocarditis depends on the isolated organism. In general, long-term antibiotic
treatment (4 to 6 weeks) is undertaken in an effort to eradicate completely the bacteria that have
been sequestered in a nonvascular vegetation. Surgery is reserved for patients who develop
severe congestive heart failure from severe valve regurgitation or deterioration.
The boy in the vignette requires intravenous antibiotic treatment, but blood cultures should
be obtained before therapy is begun. He also should undergo echocardiography, which may be
performed from the transesophageal approach to improve the sensitivity, but similar to renal
ultrasonography, such a study is performed after blood cultures have been obtained. The
absence of vegetation at the time of echocardiography does not rule out a diagnosis of infective
endocarditis. Patients who have infective endocarditis may exhibit hematuria from the deposition
of immune complexes resulting in glomerulonephritis. Although fever and hematuria may be
associated with urinary tract infection, the presence of a diastolic murmur and absence of
urinary symptoms make such a diagnosis unlikely.
References:
Baltimore RS. Infective endocarditis in children. Pediatr Infect Dis J. 1992;11:907-912. Abstract
available at: http://www.ncbi.nlm.nih.gov/pubmed/1454430
Hoyer A, Silberbach M. Infective endocarditis. Pediatr Rev. 2005;26:394-400. Available at:
http://pedsinreview.aappublications.org/cgi/content/full/26/11/394
Taubert KA, Dajani AS. Infective endocarditis. In: Moller JH, Hoffman JIE, eds. Pediatric
Cardiovascular Medicine. Philadelphia, Pa: Churchill Livingstone; 2000:768-779

Copyright 2009 by the American Academy of Pediatrics

page 287

2009 PREP SA on CD-ROM


Question: 134

A 6-year-old boy has had difficulty walking and lower leg pain for 2 days. Five days ago, he had
fever and cough that had lasted for 3 days. On physical examination, the child has no fever, and
vital signs are normal, as are cranial nerves, speech, and language. Muscle bulk, tone, and
reflexes are normal and symmetric, but his lower legs are painful to palpation. Serum creatine
kinase is 2,000 U/L, and urine is negative for myoglobin.
Of the following, the MOST likely diagnosis is

A. dermatomyositis
B. Duchenne muscular dystrophy
C. Guillain-Barr syndrome
D. metabolic myopathy
E. viral myositis

Copyright 2009 by the American Academy of Pediatrics

page 288

2009 PREP SA on CD-ROM


Critique: 134

Preferred Response: E

A gait disturbance, such as described for the boy in the vignette, can result from a variety of
potentially serious disease processes and, therefore, requires urgent evaluation. The muscle
pain and otherwise normal neurologic findings prompted measurement of serum creatine kinase,
which can help to localize the problem rapidly. The prodrome of an upper respiratory tract illness
and rapid onset of symptoms is suggestive of viral myositis.
Dermatomyositis is a more indolent, chronic process and should not present acutely with
muscle pain. Moreover, dermatomyositis is characterized by specific skin findings such as the
heliotrope rash over the eyelids (Item C134A) and Gottron papules (Item C134B). Duchenne
muscular dystrophy presents with more chronic weakness. Although Guillain-Barr syndrome
can present with pain and weakness, the preserved reflexes, focal pain over the leg muscles,
and elevated creatine kinase value are not consistent with that diagnosis. Metabolic myopathies
due to mitochondrial dysfunction can present with acute pain, weakness, tender muscles, and
rhabdomyolysis. However, the prevalence is much lower than viral myositis.
Influenza A and B and enteroviruses may cause viral myositis. Other causes are in Item
C134C.
References:
Moughan B. Musculoskeletal symptom complexes. In: Long SS, Pickering LK, Prober CG, eds.
Principles and Practice of Pediatric Infectious Diseases. New York, NY: Churchill Livingstone;
2003:150-159
Pasternack MS, Swartz MN. Myositis. In: Mandell GL, Bennett JE, Dolin R, eds. Mandell,
Douglas, and Bennett's Principals and Practice of Infectious Diseases. 6th ed. Philadelphia, Pa:
Elsevier Churchill Livingstone; 2005:1194-1203
Roos KL. Viral infections. In: Goetz CG, ed. Textbook of Clinical Neurology. 3rd ed. Saunders
Elsevier; 2007:chapt 41

Copyright 2009 by the American Academy of Pediatrics

page 289

2009 PREP SA on CD-ROM


Question: 135

The pregnant mother of a child in your practice recently learned that her grandmother had a
child who died of "probable metabolic disease" at 2 days of age. She does not know details, and
medical records on that child no longer are available. The mother asks if her pregnancy can be
tested to see if the fetus could be affected with the same disorder.
Of the following, the MOST accurate statement regarding metabolic disease in the prenatal
setting is that

A. fetuses affected with metabolic diseases are unlikely to come to term


B. knowing the parents ethnic backgrounds aids in determining which tests should be offered
C. level 2 ultrasonography during the second trimester is likely to be helpful in detecting metabolic
disease

D. poor fetal growth is common in metabolic diseases


E. prenatal metabolic screening panels are widely available

Copyright 2009 by the American Academy of Pediatrics

page 290

2009 PREP SA on CD-ROM


Critique: 135

Preferred Response: B

Most inborn errors of metabolism do not become evident until after birth. Typically, affected
fetuses are normally formed and normally grown, and no abnormalities are detectable on
prenatal ultrasonography. With rare exception, maternal serum screening does not detect
metabolic abnormalities in the fetus. Usually, affected individuals are born at term. Although most
of the United States has extended newborn screening panels to detect a variety of inborn errors,
no such panel exists for routine screening during pregnancy.
The vignette highlights the importance of accessing as much information as possible
regarding family members who die from metabolic causes. Even if a diagnosis cannot be made
in a particular instance, medical records may be helpful in offering the expectant couple
guidance or in making a diagnosis posthumously. Taking a careful family history, with attention
to familial ethnic, religious, and geographic origins, may bring to light conditions that are likely to
run in families. For example, because of the high carrier rate for some metabolic diseases,
parents of Ashkenazi Jewish descent, whose ancestors originated from Eastern Europe, are at
increased risk for having children affected by Tay-Sachs disease (Item C135) and people of
French-Canadian heritage are at increased risk for having children affected by tyrosinemia.
More than 100 metabolic disorders can be diagnosed prenatally if clinicians have sufficient
information available to know which disorder may occur in a family.
References:
Driscoll DA, Sehdev HM, Marchiano DA. Prenatal carrier screening for genetic conditions.
NeoReviews. 2004;5:e290-e295. Available for subscription at:
http://neoreviews.aappublications.org/cgi/content/full/5/7/e290
Prenatal diagnosis: emerging technologies for prenatal diagnosis. In: Nussbaum RL, McInnes
RR, Willard HF, eds. Thompson & Thompson Genetics in Medicine. 7th ed. Philadelphia, Pa:
Elsevier Saunders; 2007:456

Copyright 2009 by the American Academy of Pediatrics

page 291

2009 PREP SA on CD-ROM


Question: 136

A 17-year-old young man comes to your office for a preparticipation sports physical examination
for high school wrestling. He reports that his coach would like him to wrestle in a weight category
that is 10 lb less that his current weight. After you determine that his desired weight is in the
range of healthy weight for his height, you counsel him regarding safe weight loss.
Of the following, the MOST appropriate statement regarding healthy weight control practices for
young athletes is that

A. an appropriate diet for most athletes consists of a minimum of 1,500 kcal/day


B. dehydration causes greater body heat storage, reduces blood volume, and results in
increased exercise tolerance

C. most high school boys who participate in "weight-sensitive" sports practice unhealthy weight
loss behaviors

D. optimal values for body composition have been established for all sports
E. weight loss beyond 1.5% of body weight per week results in muscle weakness

Copyright 2009 by the American Academy of Pediatrics

page 292

2009 PREP SA on CD-ROM


Critique: 136

Preferred Response: E

Sports in which a lean body mass is desirable or competing at the lowest possible weight is seen
as an advantage include distance running, cross country skiing, swimming and diving, dance,
figure skating, gymnastics, and weight-class football and wrestling. Other sports, such as
football, rugby, basketball, and power lifting, emphasize weight gain in the form of lean body
mass. Whether attempting to lose weight and body fat or gain weight and muscle mass, some
youth may resort to unhealthy dieting and exercise practices, supplement use, and drug
ingestion.
Some practices, including rapid weight loss with voluntary dehydration, the use of anabolic
steroids, and inappropriate use of stimulants or insulin, may be fatal. Inappropriate weight loss
also may result in medical complications such as delayed physical maturation and potential
growth impairment; oligomenorrhea and amenorrhea; development of eating disorders;
depression; an increased incidence of infectious diseases; and renal, endocrine,
gastrointestinal, and thermoregulatory disturbances. Disordered eating behaviors are prevalent
in both female and male athletes. It is reported that 10% to 15% of high school boys in "weightsensitive sports" practice unhealthy weight loss behaviors, with numerous studies reporting
such behaviors in wrestlers.
Voluntary dehydration frequently is used to lose weight, and techniques can include fluid
restriction, spitting, use of laxatives and diuretics, wearing of rubber suits during activities to
cause sweating, and excessive sweating in saunas and steam baths. Dehydration results in
greater body heat storage, causing an excessive increase in the core body temperature and a
decreased blood volume, and decreased exercise tolerance. It also increases the risk of heatrelated illness.
No optimal values for body composition have been established for any sport. Rather than a
specific percentage of body fat for an individual athlete, a range that is realistic and appropriate
should be recommended. If it is appropriate that an adolescent loses weight, healthy weight loss
should not exceed 1.5% of the total body weight per week or 1 to 2 lb/wk. Weight loss beyond
these guidelines results in the breakdown and metabolism of muscle, causing muscular
weakness. An appropriate diet for most athletes consists of a minimum of 2,000 kcal/day. The
ideal method of losing weight is to consume 1,750 fewer kcal per week and expend 1,750 kcal
more per week by exercising. A well-balanced diet that contains approximately 55% to 65% of
calories from carbohydrates, 15% to 20% of calories from protein, and 20% to 30% of calories
from fat is recommended.
References:
American Academy of Pediatrics Council on Sports Medicine and Fitness and Council on School
Health. Active healthy living: prevention of childhood obesity through increased physical activity.
Pediatrics. 2006;117:1834-1842. http://pediatrics.aappublications.org/cgi/content/full/117/5/1834
American Academy of Pediatrics Committee on Sports Medicine and Fitness. Promotion of
healthy weight-control practices in young athletes. Pediatrics. 2005;116:1557-1564. Available at:
http://pediatrics.aappublications.org/cgi/content/full/116/6/1557
McCrory P, Johnston K, Meeuwisse W, et al. Summary and agreement statement of the 2nd
International Conference on Concussion in Sport, Prague 2004. Br J Sports Med. 2005;39:196204. Abstract available at: http://www.ncbi.nlm.nih.gov/pubmed/15793085

Copyright 2009 by the American Academy of Pediatrics

page 293

2009 PREP SA on CD-ROM


Question: 137

You are evaluating a 2-week-old breastfed infant who is 15% below his birthweight and has been
lethargic and fed poorly over the past 4 days. You administer a normal saline fluid bolus.
Laboratory results include:
Blood glucose of 126.0 mg/dl (7.0 mmol/L)
Serum sodium of 170.0 mEq/L (170.0 mmol/L)
Serum potassium of 5.0 mEq/L (5.0 mmol/L)
Blood urea nitrogen of 31.0 mg/dL (11.1 mmol/L)
Serum creatinine of 2.9 mg/dL (256.4 mcmol/L)
Of the following, the MOST appropriate initial fluid for correction is a solution containing 5%
dextrose and

A. NaCl (%): 0.20


KCl (mEq/L): 40
Duration of Infusion(hr): 12 to 24

B. NaCl (%): 0.45


KCl (mEq/L): 0
Duration of Infusion(hr): 48 to 72

C. NaCl (%): 0.45


KCl (mEq/L): 40
Duration of Infusion(hr): 12 to 24

D. NaCl (%): 0.9


KCl (mEq/L): 0
Duration of Infusion(hr): 12 to 24

E. NaCl (%): 0.9


KCl (mEq/L): 40
Duration of Infusion(hr): 48 to 72

Copyright 2009 by the American Academy of Pediatrics

page 294

2009 PREP SA on CD-ROM


Critique: 137

Preferred Response: B

Hypernatremia (serum sodium concentration greater than 145.0 mEq/L [145.0 mmol/L]) is
common in hospitalized children. Hypernatremia results from excessive sodium administration
(incorrectly mixed formula, sodium bicarbonate, hypertonic saline) or a deficit of water in relation
to sodium. Such a deficit may occur from decreased water intake (inadequate breastfeeding,
fluid restriction, lack of access to fluids), fluid losses (nephrogenic and central diabetes
insipidus, increased insensible losses), or fluid losses in greater proportion than sodium losses
(diarrhea, vomiting, diuretic use, burns).
Treatment of hypernatremia is directed at correction of both the serum sodium
concentration and the circulatory volume. Initially, some children may require isotonic fluid
boluses to restore circulation. It is important to note that children who have hypernatremia often
appear less dehydrated than they actually are due to preservation of the extracellular volume.
The following equation can be used to estimate the free water deficit:
Water deficit (mL)= 4 mL x ideal body weight (kg) x desired change in serum sodium
concentration
Hypernatremia, especially if chronic, should be corrected slowly, with a desired goal of
decreasing the serum sodium by 0.5 mEq/L per hour to avoid cerebral edema. Severe
hypernatremia (serum sodium >170.0 mEq/L [170.0 mmol/L]), as described for the child in the
vignette, should be corrected over 48 to 72 hours. Fluid administration generally consists of 1/4
to 1/2 normal saline solutions. Symptoms of overcorrection, such as changes in mental status or
onset of seizures, suggest the development of cerebral edema and should be treated with
hypertonic saline and slowing of the sodium correction. In general, potassium administration
should be withheld in cases of severe hypernatremic dehydration until adequate urine output is
assured.
References:
Greenbaum LA. Electrolyte and acid-base disorders. In: Kliegman RM, Behrman RE, Jenson
HB, Stanton BF, eds. Nelson Textbook of Pediatrics. 18th ed. Philadelphia, Pa: Saunders
Elsevier; 2007:267-308
Moritz ML, Ayus JC. Disorders of water metabolism in children: hyponatremia and
hypernatremia. Pediatr Rev. 2002:23:371-380. Available at:
http://pedsinreview.aappublications.org/cgi/content/full/23/11/371

Copyright 2009 by the American Academy of Pediatrics

page 295

2009 PREP SA on CD-ROM


Question: 138

You are asked to see a 7-year-boy in whom medulloblastoma (primitive neuroectodermal tumor)
was diagnosed at age 3 years. Treatment at that time consisted of chemotherapy and
craniospinal irradiation. During the past year, he grew 2 cm, although he is eating normally, and
his weight is appropriate for height. Despite spinal irradiation, the upper-to-lower segment ratio is
normal for his age.
Of the following, the MOST likely diagnosis is

A. acquired growth hormone deficiency


B. chemotherapy-induced renal failure
C. Cushing syndrome
D. irradiation-induced epiphyseal fusion
E. tumor recurrence

Copyright 2009 by the American Academy of Pediatrics

page 296

2009 PREP SA on CD-ROM


Critique: 138

Preferred Response: A

Cranial irradiation has gradual deleterious effects on pituitary hormone secretion due to damage
to hypothalamic releasing factors. Growth hormone, thyroid-stimulating hormone (TSH), and the
gonadotropins tend to be affected most, with adrenocorticotropic hormone (ACTH) relatively
protected, although all neuroendocrine function can be disturbed by cranial irradiation. The boy
described in the vignette has a normal upper-to-lower segment ratio, indicating that his spine still
is growing despite radiation. This suggests that his growth attenuation is due to an acquired
endocrine deficiency of either growth hormone or thyroid hormone. Early in the deficiency
process, if this child were tested with standard stimuli to growth hormone release, he might be
able to release growth hormone, but if continuous nocturnal growth hormone sampling were
performed, diminished and disorderly spontaneous growth hormone release might be
documented. With time, secretory function to stimuli also would be lost.
Renal failure from chemotherapy would be expected to have manifested earlier and been
recognized by this time. Cushing syndrome is a possible but rare cause of growth attenuation
caused by adrenal or pituitary oversecretion of cortisol or ACTH, respectively, and is not
associated with cranial irradiation. Irradiation-induced epiphyseal fusion at the level of the spine
is common following spinal irradiation. The epiphyseal fusion of the spine results in an increased
lower segment (measured from the pubic symphysis to the heel) compared with the upper
segment because the legs continue to grow while the spine does not. This results in an
increased lower segment-to-upper segment ratio. Tumor recurrence has more obvious
manifestations than statural growth attenuation.
References:
Ballonoff A, Kavanagh B. Complications of cranial irradiation. UpToDate Online 15.3. 2008.
Available for subscription at:
http://www.uptodateonline.com/utd/content/topic.do?topicKey=rad_ther/2462
Darzy KH, Pezzoli SS, Thorner MO, Shalet SM. The dynamics of growth hormone (GH)
secretion in adult cancer survivors with severe GH deficiency acquired after brain irradiation in
childhood for nonpituitary brain tumors: evidence for preserved pulsatility and diurnal variation
with increased secretory disorderliness. J Clin Endocrinol Metab. 2005;90:2794-2803. Available
at: http://jcem.endojournals.org/cgi/content/full/90/5/2794
Rose SR. Growth failure after childhood cancer: role of growth hormone deficiency. UpToDate
online 15.3. 2008. Available for subscription at:
http://www.uptodateonline.com/utd/content/topic.do?topicKey=ped_onco/2817

Copyright 2009 by the American Academy of Pediatrics

page 297

2009 PREP SA on CD-ROM


Question: 139

A 24-month-old child has been evaluated and found eligible for early intervention services
because of language delay. His mother is reluctant to pursue therapy because she feels that his
language will improve without intervention.
Of the following, the MOST appropriate response is to

A. agree with the mother that his speech probably will improve without therapy
B. explain that therapy at this age is parent-based training to promote appropriate development in
the home setting

C. explain that therapy is critical for the child or the child will have regression in her language
development

D. postpone therapy pending re-evaluation in 6 months


E. warn the mother that if she does not enroll the child in therapy, she can be reported for child
neglect

Copyright 2009 by the American Academy of Pediatrics

page 298

2009 PREP SA on CD-ROM


Critique: 139

Preferred Response: B

Federal legislation for the provision of services to infants, toddlers, and preschool children who
have disabilities has evolved since 1986. From its inception as part of PL99-457 (the Education
of the Handicapped Amendments) through the most recent changes defined in PL 105-17
(Individuals with Disabilities Education Act, Part C), legislative efforts uphold the rights of
students and parents to the key components of a free and appropriate public education. Federal
regulations define early intervention services as services that "are designed to meet the
developmental needs of each child eligible under this part and the needs of the family related to
enhancing the child's development."
The child described in the vignette will benefit from early intervention therapy, the goal of
which is to enhance the development of infants and toddlers who have disabilities and minimize
their potential for developmental delays. The services are designed to meet the needs of the
child and family and promote the child's development in natural environments. Therapy at this
stage may be selected by the parents, and parent-based training in the home is most
appropriate for this child. Because language delays have been identified for this boy, his parents
should be encouraged to initiate services without delay. His speech probably will improve more
slowly without therapy, but it will not regress without therapy.
Participation in the Part C early intervention system for infants and toddlers is voluntary for
the family; they have the right to accept or decline specific early intervention services. Thus,
declining such services does not constitute child neglect.
Early intervention services were established to allow children who have developmental
problems to reach their potential. Research studies have identified specific times in which a
child's brain is especially efficient at learning specific information. Coordinated, community-based
multidisciplinary programs for early intervention represent effective public policy because they
not only help to improve some children's cognitive outcome, but they also aid in family
functioning.
References:
American Academy of Pediatrics Committee on Children With Disabilities. Role of the pediatrician
in family-centered early intervention services. Pediatrics. 2001;107:1155-1157. Available at:
http://pediatrics.aappublications.org/cgi/content/full/107/5/1155
American Academy of Pediatrics Committee on Children With Disabilities. The pediatrician's role
in development and implementation of an Individual Education Plan and/or and Individual Family
Service Plan. Pediatrics. 1999;104:124-127. Available at:
http://pediatrics.aappublications.org/cgi/content/full/104/1/124 Policy reaffirmed. Pediatrics.
2006;117:1846-1847. Available at:
http://pediatrics.aappublications.org/cgi/content/full/117/5/1846

Copyright 2009 by the American Academy of Pediatrics

page 299

2009 PREP SA on CD-ROM


Question: 140

A 10-year-old boy presents with a 1-day history of fever and a swollen leg. According to his
mother, the boy developed a small abrasion on his leg while playing outside 3 days ago. Last
night he began to complain of pain in the area and had a low-grade fever. This morning his
temperature was 102.4F (39.1C) and the area around the abrasion looked very red and was
tender to palpation. About 2 hours later, the swelling had increased. Physical examination
reveals a boy in no apparent distress who has a temperature of 101.4F (38.6C), a heart rate of
93 beats/min, a respiratory rate of 23 breaths/min, and a blood pressure of 95/65 mm Hg. All
other findings are normal, except for a small erythematous abrasion just above the medial
malleolus that has no discharge. Erythema from this area extends to a well-demarcated region
of the mid-calf and is tender to touch (Item Q140).
Of the following, the MOST likely pathogen is

A. Pseudomonas aeruginosa
B. Staphylococcus aureus
C. Streptococcus pneumoniae
D. Streptococcus pyogenes
E. Vibrio vulnificans

Copyright 2009 by the American Academy of Pediatrics

page 300

2009 PREP SA on CD-ROM


Critique: 140

Preferred Response: D

Cellulitis is an infection and inflammation of the connective tissue that involves the dermis and
often is preceded by a break in the skin. Cellulitis also can be seen in patients who have an
underlying condition that predisposes them to such an illness (eg, lymphatic stasis, diabetes
mellitus). Streptococcus pyogenes and Staphylococcus aureus are the two most common
bacterial agents responsible for the development of this infection. Although distinction between
the two pathogens is difficult, the rapidly spreading erythema without purulence described for
patient in the vignette is most consistent with S pyogenes infection (Item C140A). Infections
caused by S aureus tend to be more localized and produce purulent material. Confirmation of the
etiologic agent requires recovery of the organism from an aspirate of the most erythematous
area or a culture from purulent exudates or from the blood.
For most patients, initial empiric therapy is with an antimicrobial agent that has activity
against both of these gram-positive cocci. Empiric treatment courses have changed recently,
with the increased prevalence of methicillin-resistant S aureus (MRSA). Mild forms of cellulitis
may be treated with a topical antimicrobial ointment (eg, mupirocin). More advanced cases
require the addition of an oral antimicrobial agent such as clindamycin, trimethoprimsulfamethoxazole, or doxycycline in geographic areas that are highly endemic for MRSA.
Cephalexin or amoxicillin-clavulanate is an adequate initial empiric agent in a community that has
less than a 5% prevalence of MRSA. Severe forms of cellulitis require intravenous antimicrobial
agents and possibly even surgical debridement.
Pseudomonas aeruginosa can cause cellulitis but is more common when the patient has a
history of a penetrating injury to the skin that involves water (eg, cut on the leg from a stick in a
creek), is immunocompromised, or has diabetes mellitus. S pneumoniae occasionally causes
cellulitis but is not a common cause of this infection. Vibrio vulnificans can cause a lifethreatening form of cellulitis termed "necrotizing fasciitis," but such infections are associated with
injuries from fish or objects in salt water in an immunocompromised patient (Item C140B).
References:
Jaggi P, Shulman ST. Group A streptococcal infections. Pediatr Rev. 2006;27:99-105. Available
at: http://pedsinreview.aappublications.org/cgi/content/full/27/3/99
Todd JK. Staphylococcal infections. Pediatr Rev. 2005;26:444-450. Available at:
http://pedsinreview.aappublications.org/cgi/content/full/26/12/444

Copyright 2009 by the American Academy of Pediatrics

page 301

2009 PREP SA on CD-ROM


Question: 141

You are evaluating a previously healthy 3-year-old boy for a 3-day history of nausea; vomiting;
and profuse watery, nonbloody diarrhea that has worsened over the last 24 hours. He can keep
down water and an oral electrolyte maintenance solution but has no interest in eating solid food.
The family returned 5 days ago from a 1-week vacation at a resort in Acapulco, Mexico. On
physical examination, the tired-appearing little boy has a temperature of 100.8F (38.3C), moist
mucous membranes, and a soft abdomen with mild diffuse tenderness to palpation. Laboratory
tests document a peripheral white blood cell count of 6.7x103/mcL (6.7x109/L); hemoglobin of
12.0 g/dL (120.0 g/L); platelet count of 230.0x103/mcL (230.0x109/L); and a differential count of
50% neutrophils, 40% lymphocytes, and 10% monocytes.
Of the following, the MOST likely pathogen causing this patients condition is

A. Campylobacter jejuni
B. Escherichia coli
C. Giardia lamblia
D. Salmonella sp
E. Shigella sp

Copyright 2009 by the American Academy of Pediatrics

page 302

2009 PREP SA on CD-ROM


Critique: 141

Preferred Response: B

The profuse watery, nonbloody diarrhea and history of vacationing in Mexico described for the
child in the vignette are consistent with traveler's diarrhea. By far, the most common cause of
traveler's diarrhea is enterotoxigenic Escherichia coli (ETEC), accounting for up to 75% of
cases.
There are at least five types of diarrhea-producing E coli strains, which differ in
epidemiologic features and diarrhea type (Item C141).
The primary treatment of all diarrheal disease is to provide adequate fluid replacement by
the oral or parenteral routes. The use of additional antibiotic therapy depends on the type of E
coli that is suspected of causing the illness. Treatment of EHEC infections is entirely supportive.
Antibiotics are contraindicated because they can induce the expression and release of Shiga
toxins, and their use has been associated with a higher risk of hemolytic-uremic syndrome in
children.
If a patient who has E coli diarrhea does not improve after several days of supportive care
and assays for Shiga toxin are negative, antimicrobial therapy can be considered. Antimicrobial
agents that may be used in the treatment of severe watery ETEC diarrhea in a traveler or for
dysentery caused by EIEC strains include trimethoprim-sulfamethoxazole, azithromycin, or
ciprofloxacin. Selection of the agent is based on patient age and antimicrobial susceptibility of the
organism. Data regarding the treatment of EAEC infections with antibiotics are sparse, and
optimal therapy is unknown.
Shigella, Salmonella, and Campylobacter sp are much less common causes of traveler's
diarrhea, but may be responsible for up to 30% of cases, 15% of cases, and 15% of cases,
respectively. Giardia lamblia is not recognized as a cause of acute traveler's diarrhea.
References:
American Academy of Pediatrics. Escherichia coli diarrhea (including hemolytic-uremic
syndrome). In: Pickering LK, Baker CJ, Long SS, McMillan JA, eds. Red Book: 2006 Report of
the Committee on Infectious Diseases. 27th ed. Elk Grove Village, Ill: American Academy of
Pediatrics; 2006:291-296
Donnenberg MS. Enterobacteriaceae. In: Mandell GL, Bennett JE, Dolin R, eds. Mandell,
Douglas, and Bennett's Principles and Practice of Infectious Diseases. 6th ed. Philadelphia, Pa:
Elsevier Churchill Livingstone; 2005:2567-2586
Ericsson CD, DuPont HL. Travelers' diarrhea: approaches to prevention and treatment. Clin
Infect Dis. 1993;16:616-626
Guerrant RL, Bobak DA. Nausea, vomiting, and noninflammatory diarrhea. In: Mandell GL,
Bennett JE, Dolin R, eds. Mandell, Douglas, and Bennett's Principles and Practice of Infectious
Diseases. 6th ed. Philadelphia, Pa: Elsevier Churchill Livingstone; 2005:1236-1249
Nataro JP, Kaper JB. Diarrheogenic Escherichia coli. Clin Microbiol Rev. 1998;11:142-201.
Available at: http://cmr.asm.org/cgi/content/full/11/1/142?view=long&pmid=9457432

Copyright 2009 by the American Academy of Pediatrics

page 303

2009 PREP SA on CD-ROM


Question: 142

A 5-year-old boy presents with dark red urine, fever, and rhinorrhea. He was well until 2 days
ago, when he developed rhinorrhea and mild cough. He denies urgency, frequency, dysuria,
back pain, or musculoskeletal complaints. On physical examination, the slightly ill-appearing boy
has a temperature of 99.5F (37.5C), heart rate of 130 beats/min, respiratory rate of 18
breaths/min, and blood pressure of 104/58 mm Hg. He has pale conjunctivae, mild scleral
icterus, a hyperdynamic precordium, and a I/VI systolic murmur at the left upper sternal border.
There is no edema, and musculoskeletal and neurologic examination results are normal.
Urinalysis results include: red appearance, a specific gravity of 1.030, pH of 6.5, 3+ blood, and
no protein. Microscopy reveals fewer than 5 red blood cells/high-power field (HPF), fewer than 5
white blood cells/HPF, and no casts.
Of the following, the BEST initial test to obtain is

A. complete blood count with manual differential count


B. creatine phosphokinase measurement
C. renal/bladder ultrasonography
D. urine culture
E. viral culture of urine for adenovirus

Copyright 2009 by the American Academy of Pediatrics

page 304

2009 PREP SA on CD-ROM


Critique: 142

Preferred Response: A

The differential diagnosis for causes of red urine includes hematuria, hemoglobinuria,
myoglobinuria, and porphyrinuria. Some urinary tract pathogens (eg. Serratia marcescens) also
can be associated with red urine. In addition, red urine can result from ingestion of certain foods
(beets, blackberries, or food dyes) and medications (deferoxamine, rifampin, and
phenolphthalein).
The initial test for evaluating a child who has red urine is the urinalysis to look for the
presence or absence of blood. The 3+ blood on a dipstick described for the child in the vignette
narrows the differential diagnosis to hematuria, hemoglobinuria, or myoglobinuria. The dipstick
test incorporates an indicator impregnated into the paper strip that, upon contact with free
hemoglobin, myoglobin, or intact red blood cells, oxidizes the indicator to result in a blue color
change. This dipstick test is exquisitely sensitive to blood to the level of 5 red blood cells/highpower field. The fewer than 5 red blood cells/high power field on microscopy described for the
patient in the vignette eliminates hematuria. The presence of tachycardia, pale conjunctivae, and
a systolic murmur is suggestive of anemia, and scleral icterus supports a possible hemolytic
anemia. Accordingly, a complete blood count to assess for anemia is the most appropriate next
laboratory test to obtain for this boy.
The absence of musculoskeletal complaints and normal musculoskeletal and neurologic
examination results make myoglobinuria (usually related to rhabdomyolyis) unlikely. If
myoglobinuria was suspected, measurement of creatine phosphokinase would be useful.
Renal/bladder ultrasonography is helpful in evaluating patients who have bright red urine,
with clots and red blood cells noted on urinalysis. Such findings are suggestive of a structural
cause for the red urine, such as renal or bladder mass or stones.
Red urine caused by bacterial or viral infections is due to hematuria. Thus, culture of the
urine for bacteria or viral culture for adenovirus is inappropriate because this boy does not have
hematuria.
References:
Kalia A, Travis LB. Hematuria, leukocyturia, and cylindruria. In: Edelmann CM Jr, Bernstein J,
Meadow SR, Spitzer A, Travis LB, eds. Pediatric Kidney Disease. 2nd ed. Boston, Ma: Little,
Brown and Company; 1992:553-563
Moxey-Mims M. Hematuria and proteinuria. In: Kher KK, Schnaper HW, Makker SP, eds. Clinical
Pediatric Nephrology. 2nd ed. London, England: Informa Healthcare; 2007:129-141
Schwartz G. Clinical assessment of renal function. In: Kher KK, Schnaper HW, Makker SP, eds.
Clinical Pediatric Nephrology. 2nd ed. London, England: Informa Healthcare; 2007:71-93

Copyright 2009 by the American Academy of Pediatrics

page 305

2009 PREP SA on CD-ROM


Question: 143

You are evaluating a 16-year-old girl during her biannual asthma follow-up visit. She has a
history of mild persistent asthma that is well-controlled on a low-dose inhaled corticosteroid. You
review the asthma guidelines and recommend that she receive the influenza vaccine. Her
mother immediately replies, "Oh no, my daughter has a severe egg allergy and was told to never
get the influenza vaccine."
Of the following, the vaccine that is contraindicated in a patient who has a severe
immunoglobulin E-mediated egg allergy is

A. human papillomavirus vaccine


B. measles-mumps-rubella
C. tetanus diphtheria
D. varicella
E. yellow fever

Copyright 2009 by the American Academy of Pediatrics

page 306

2009 PREP SA on CD-ROM


Critique: 143

Preferred Response: E

Of currently available vaccines, the measles-mumps-rubella (MMR) vaccine, trivalent influenza


vaccine (TIV), live attenuated intranasal influenza vaccine (LAIV), and yellow fever vaccine all
contain small amounts of egg protein. All of these vaccines are contraindicated in patients who
have egg allergies except the MMR. Despite being developed in chick embryos, the MMR
vaccine contains only 0.5 to 1 ng of ovalbumin per dose, a quantity far lower than that required
to cause an immunoglobulin (Ig) E-mediated reaction. Patients who have experienced severe
anaphylaxis to egg in the past generally should not receive the TIV, LAIV, or yellow fever
vaccine.
Children who have mild egg allergies (eg, atopic dermatitis, cutaneous-only symptoms) and
who are at high risk for complications from influenza should be offered the TIV. Protocols for
skin testing, two-step dose challenge, single-dose challenge, and desensitization are available
for such patients. Clinicians should remember that many patients who have experienced
adverse food reactions in the past are not truly allergic to that food. Pursuing skin or serum
allergy testing to egg, particularly if negative, will aid in counseling the families on the risks of
future influenza vaccination and therapeutic options.
Although diphtheria and tetanus toxoids and acellular pertussis, human papillomavirus, and
varicella vaccines do not contain egg protein, other vaccine components may result in a
reaction.
References:
American Academy of Pediatrics. Active immunization. In: Pickering LK, Baker CJ, Long SS,
McMillan JA, eds. Red Book: 2006 Report of the Committee on Infectious Diseases. 27th ed. Elk
Grove Village, Ill: American Academy of Pediatrics; 2006:9-10
American Academy of Pediatrics. Reporting of adverse events. Reporting of vaccinepreventable diseases. In: Pickering LK, Baker CJ, Long SS, McMillan JA, eds. Red Book: 2006
Report of the Committee on Infectious Diseases. 27th ed. Elk Grove Village, Ill: American
Academy of Pediatrics; 2006:41-50
Cerecedo Carballo I, Dieguez Pastor MC, Bartolom Zavala B, Snchez Cano M, de la Hoz
Caballer B. Safety of measles-mumps-rubella vaccine (MMR) in patients allergic to eggs. Allergol
Immunopathol (Madr). 2007;35:105-109. Available at: http://db.doyma.es/cgibin/wdbcgi.exe/doyma/mrevista.pubmed_full?inctrl=05ZI0102&rev=105&vol=35&num=3&pag=10
5
Cox JE, Cheng TL. In brief: egg-based vaccines. Pediatr Rev. 2006;27:118-119. Available at:
http://pedsinreview.aappublications.org/cgi/content/full/27/3/118
Piquer-Gibert M, Plaza-Martn A, Martorell-Aragons A, et al. Recommendations for
administering the triple viral vaccine and antiinfluenza vaccine in patients with egg allergy.
Allergol Immunopathol (Madr). 2007;35:209-212. Available at: http://db.doyma.es/cgibin/wdbcgi.exe/doyma/mrevista.pubmed_full?inctrl=05ZI0102&rev=105&vol=35&num=5&pag=20
9

Copyright 2009 by the American Academy of Pediatrics

page 307

2009 PREP SA on CD-ROM


Question: 144

A 3-month-old infant is brought to the office for fussiness, increased sleeping, and poor feeding.
According to his mother, he was doing well until 4 days ago, when his formula intake decreased
from 6 oz every 3 to 4 hours to 1 to 2 oz every 4 hours and she had to awaken him to feed. He
has had no vomiting, diarrhea, or fever. He was born at term, and the mother had no antenatal
infections. On physical examination, the infant is difficult to console and has a high-pitched cry.
His temperature is 98.2F (36.8C), heart rate is 160 beats/min, and respiratory rate is 30
breaths/min. His anterior fontanelle is flat, pupils are 4 mm and equally reactive, and there is no
evidence of corneal abrasions. His lungs are clear, heart sounds are normal, and abdominal
evaluation findings are benign. His extremities are warm, well-perfused, and have normal tone.
Results of the initial laboratory evaluation, including a complete blood count with differential
count, electrolytes, and urinalysis, are normal. The fecal occult blood test result is negative.
Of the following, the MOST appropriate next study is

A. abdominal ultrasonography
B. chest radiography
C. computed tomography scan of the brain
D. serum ammonia determination
E. urine organic acid screen

Copyright 2009 by the American Academy of Pediatrics

page 308

2009 PREP SA on CD-ROM


Critique: 144

Preferred Response: C

The differential diagnosis of the irritable infant is extensive and includes conditions that affect all
organ systems (Item C144A). The evaluation should be based on a complete history and
physical examination as well as a high index of suspicion for serious occult causes. For the
patient described in the vignette, the concern should be high for nonaccidental head injury,
despite the lack of trauma history or external or cutaneous findings on physical examination, and
should prompt the physician to obtain neuroimaging (eg, computed tomography scan or
magnetic resonance imaging). Of note, recent studies have reconfirmed the incidence of occult
head injuries and the importance of neuroimaging in the evaluation of suspected child abuse.
Almost 30% of children undergoing child abuse evaluation in one study had occult brain injury
despite the absence of neurologic symptoms, and as many as 10% of brain injuries may be
missed if only skeletal surveys and ophthalmologic examinations are performed.
The presenting signs and symptoms of nonaccidental head trauma due to inflicted traumatic
brain injury (also known as "shaken baby syndrome") often are nonspecific. It is estimated that
as many of 30% of cases are not diagnosed initially, in part because the findings may be
attributed to other conditions, such as viral syndrome, colic, or formula intolerance. The history
also may be misleading, with most caretakers reporting no trauma. In some instances, the sole
finding may be a disproportionately large head circumference.
The absence of external findings is related largely to the biomechanics of the injurious event.
Vigorous shaking, with or without impact, leads to traction on the dural bridging veins. Shearing
of these veins causes bleeding into the subdural space (Item C144B). Especially in those cases
where there was no impact of the infant's head during the shaking episode, bruising or swelling
is likely to be absent.
Abdominal ultrasonography to evaluate for intussusception or hydronephrosis, serum
ammonia and urine organic acid determinations to evaluate for metabolic errors, or chest
radiography to look for cardiomegaly or pulmonary infiltrates may be indicated in the evaluation
of the irritable infant. However, in contrast to abusive head trauma, it is likely that signs,
symptoms, or abnormalities on screening laboratory evaluations would provide clues to these
other diagnoses.
References:
Herman M, Le A. The crying infant. Emerg Med Clin North Am. 2007;25:1137-1159. Abstract
available at: http://www.ncbi.nlm.nih.gov/pubmed/17950139
Keenan HT, Runyan DK, Marshall SW, Nocerna MA, Merten DF. A population-based
comparison of clinical and outcome characteristics of young children with serious inflicted and
noninflicted traumatic brain injury. Pediatrics. 2004;114:633-639. Available at:
http://pediatrics.aappublications.org/cgi/content/full/114/3/633
Laskey AL, Holsti M, Runyan DK, Socolar RRS. Occult head trauma in young suspected victims
of physical abuse. J Pediatr. 2004;144:719-722. Abstract available at:
http://www.ncbi.nlm.nih.gov/pubmed/15192615
Newton AW, Vandeven AM. Update on child maltreatment with a special focus on shaken baby
syndrome. Curr Opin Pediatr. 2005;17:246-251. Abstract available at:
http://www.ncbi.nlm.nih.gov/pubmed/15800421
Sirotnak AP, Grigsby T, Krugman R. Physical abuse of children. Pediatr Rev. 2004;25:264-277.
Available at: http://pedsinreview.aappublications.org/cgi/content/full/25/8/264

Copyright 2009 by the American Academy of Pediatrics

page 309

2009 PREP SA on CD-ROM


Question: 145

A 13-year-old boy who has a 1-year history of abdominal pain in the epigastric and periumbilical
regions presents for further evaluation. According to his history, the pain occurs one to three
times per week and sometimes interferes with school attendance and physical activity. Findings
on physical examination are normal. You review the diagnostic studies that have been
performed in the past year.
Of the following, the finding that MOST warrants referral for upper endoscopy is

A. abnormal lactose breath hydrogen test result


B. elevated serum alkaline phosphatase value
C. elevated serum amylase value
D. mild anemia (hematocrit, 33% [0.33])
E. positive tissue transglutaminase antibody

Copyright 2009 by the American Academy of Pediatrics

page 310

2009 PREP SA on CD-ROM


Critique: 145

Preferred Response: E

Most children who have recurrent abdominal pain (RAP) have functional abdominal pain.
Therefore, most affected children do not require gastrointestinal endoscopy. However, if certain
conditions are strongly suspected, endoscopy can be an essential part of the diagnostic
evaluation. Such conditions include peptic ulcer disease, bleeding esophageal varices, reflux
esophagitis, eosinophilic esophagitis, inflammatory bowel disease, and celiac disease. Most of
these conditions present with symptoms considered "red flags," including chronic diarrhea,
weight loss, unexplained fever, hematemesis, or lower gastrointestinal tract bleeding. The choice
of either upper gastrointestinal endoscopy or lower gastrointestinal endoscopy (colonoscopy) is
based on the patient's clinical symptoms. Tissue forceps biopsies can be taken through the
endoscope and tissue sent to the pathologist for identification of subtle lesions (eg, esophagitis,
celiac disease, giardiasis, Helicobacter pylori gastritis).
The strongest indication for upper endoscopy for the patient described in the vignette is a
positive tissue transglutaminase antibody test. This antibody is a sensitive and specific test for
celiac disease, although some false-positive results can be seen. Because a lifelong gluten-free
diet is the only treatment for celiac disease, it is still recommended that patients undergo
endoscopy with duodenal biopsies to establish the diagnosis. The endoscopic visual
appearance may be normal in patients who have celiac disease, but in severe cases,
"scalloped" duodenal folds may be seen. Generally, three to four biopsies of the duodenum are
needed to evaluate for celiac disease; histologic features include intraepithelial lymphocytosis
and villous atrophy.
Elevated alkaline phosphatase values suggest normal adolescent bone growth, bone
inflammation, osteopenia, or hepatobiliary disease. Elevated serum amylase values suggest
pancreatitis, and routine upper endoscopy is rarely helpful in affected patients (although
endoscopic retrograde cholangiopancreatography may be useful in evaluating recurrent
pancreatitis). Mild anemia is a nonspecific finding that can be seen for many different reasons.
An abnormal lactose breath test result suggests lactose intolerance, which does not require
endoscopic confirmation.
References:
Fox VL. Gastrointestinal endoscopy: patient preparation and surgical considerations. In: Walker
WA, Goulet O, Kleinman RE, Sherman PM, Shneider BL, Sanderson IR, eds. Pediatric
Gastrointestinal Disease. 4th ed. Hamilton, Ontario, Canada: BC Decker; 2004:1666-1673
Thakkar K, Gilger MA, Shulman RJ, El Serag H. Esophagogastroduodenoscopy in children with
abdominal pain: a systematic review. Am J Gastroenterol. 2007;102:654-661. Abstract available
at: http://www.ncbi.nlm.nih.gov/pubmed/17222318

Copyright 2009 by the American Academy of Pediatrics

page 311

2009 PREP SA on CD-ROM


Question: 146

You are called to the newborn nursery to see a 2.1-kg term infant whose bedside glucose
screening test value is 30 mg/dL (1.7 mmol/L). The nurse describes the baby as being generally
lethargic, jittery with stimulation, and intolerant of oral feeding attempts at 4 hours of age (poor
oral suckling and emesis of the small volumes of formula taken). He was born at 41 weeks
gestation to a mother who had poor weight gain, smoked cigarettes, and had hypertension. The
Apgar scores following a vaginal delivery were 6 and 8 at 1 and 5 minutes, respectively. There is
no history of maternal diabetes, illicit drug use, or intrapartum difficulties. On physical
examination, the babys vital signs are normal except for tachypnea (respiratory rate of 80
breaths/min), with pulse oximetry of 90% on room air. The infant has plethora, acrocyanosis,
and generalized low tone. He exhibits rapid, shallow tachypnea, with clear lungs bilaterally on
auscultation. There is a soft I/VI systolic murmur along the lower left sternal border and no
gallop. Upon stimulation, he has jittery hand movements. Laboratory findings include:
Serum glucose, 45.0 mg/dL (2.5 mmol/L)
White blood cell count, 7.0x10 3/mcL (7.0x109/L) with a normal differential count
Platelet count, 150.0x10 3/mcL (150.0x109/L)
Hematocrit, 70% (0.70)
An arterial blood gas reveals a pH of 7.40, Pao2 of 75 mm Hg, Paco2 of 30 mm Hg, and base
excess of -7 mEq/L.
Of the following, the MOST appropriate treatment for this infants underlying problem is

A. administration of amphotericin B
B. double-volume exchange transfusion
C. intubation and assisted ventilation
D. partial exchange transfusion
E. phototherapy

Copyright 2009 by the American Academy of Pediatrics

page 312

2009 PREP SA on CD-ROM


Critique: 146

Preferred Response: D

By definition, polycythemia exists when the hematocrit (HCT) is greater than 65% (0.65). This
condition occurs in newborns who are small for gestational age, infants of diabetic mothers, the
recipient twin in a twin-twin transfusion syndrome-affected pregnancy, or infants who have
delayed clamping of the umbilical cord after delivery. The decision to treat polycythemia is
contingent upon the presence of symptoms associated with the state of hyperviscosity of the
circulating blood conferred upon it by the increased HCT or an HCT of greater than 70% (0.70).
Such symptoms include those described for the infant in the vignette as well as those described
in the Table (Item C146). Although blood hyperviscosity also may occur due to markedly
elevated white blood cell numbers (generally, >100.0x103/mcL [100.0x109/L]) or with elevations
of certain plasma protein fractions, in the newborn, it is almost exclusively related to an increase
in the red blood cell mass, as reflected in the HCT.
The treatment of choice for symptomatic polycythemia, as seen in this infant, is a partial
exchange transfusion, which reduces the red blood cell mass and maintains a euvolemic state
by the administration of crystalloid (eg, normal saline). The partial exchange transfusion is partial
in that it removes only a portion of the circulating volume of blood (as opposed to a complete or
double-volume exchange transfusion) and replaces (exchanges) it with crystalloid. The volume
of such an exchange is based on the following formula:
Volume of exchange (mL) =
[Infant's blood volume] x [Observed HCT-Desired HCT]/Observed HCT
The blood volume of an infant who has polycythemia is 100 mL/kg.
For a 3-kg infant who has an observed HCT of 70% (0.70), the volume of exchange is:
= [3 kg x 100 mL/kg] x [0.70-0.55]/0.70
= 300 mL x 0.15/0.70
= 300 mL x 0.214
= 64 mL
Polycythemia cannot be treated solely with intravenous crystalloid because this fluid leaves
the circulatory compartment easily. Because the patient does not have evidence of systemic
fungal infection, amphotericin B is not indicated and would not treat polycythemia. The infant in
the vignette does not have hypoxemia or hypercarbia that warrants intubation and assisted
ventilation. Phototherapy does not treat polycythemia, only the hyperbilirubinemia that follows. A
double-volume exchange transfusion is used to treat severe hyperbilirubinemia.
References:
Ceriani Cernadas JM, Carroli G, Pellegrini L, et al. The effect of timing of cord clamping on
neonatal venous hematocrit values and clinical outcome at term: a randomized, controlled trial.
Pediatrics. 2006;117:e779-e786. Available at:
http://pediatrics.aappublications.org/cgi/content/full/117/4/e779
Linderkamp O. Blood viscosity of the neonate. NeoReviews. 2004;5:e406-e416. Available for
subscription at: http://neoreviews.aappublications.org/cgi/content/full/5/10/e406
Luchtman-Jones L, Schwartz AL, Wilson DB. Blood component therapy for the neonate. In:
Martin RJ, Fanaroff AA, Walsh MC, eds. Fanaroff and Martin's Neonatal-Perinatal Medicine. 8th
ed. Philadelphia, Pa: Mosby Elsevier; 2006:1344-1356
Philip AGS, Saigal S. When should we clamp the umbilical cord? NeoReviews. 2004;5:e142e154. Available for subscription at: http://neoreviews.aappublications.org/cgi/content/full/5/4/e142

Copyright 2009 by the American Academy of Pediatrics

page 313

2009 PREP SA on CD-ROM


Question: 147

A worried mother brings her 4-year-old son to your office because his right eye has been red for
3 days. She assumed it was pink eye that he contracted at child care, but she now is concerned
because he has developed swelling in front of his right ear, and his eye has become redder.
They live in a wooded area and got a new kitten 6 weeks ago, but there is no history of the kitten
scratching the child. Physical examination reveals a well-appearing child who has obvious
conjunctival injection (Item Q147A) of the right eye but no discharge or pain. You palpate a 2x2cm tender, mobile preauricular lymph node (Item Q147B) and a 2x3-cm anterior cervical lymph
node on the right. The remainder of the physical examination findings are normal.
Of the following, the MOST likely pathogen causing this boys symptoms is

A. Bartonella henselae
B. Francisella tularensis
C. Haemophilus influenzae
D. Pasteurella multocida
E. Staphylococcus aureus

Copyright 2009 by the American Academy of Pediatrics

page 314

2009 PREP SA on CD-ROM


Critique: 147

Preferred Response: A

The boy described in the vignette has signs of Parinaud oculoglandular syndrome, with painless,
nonpurulent conjunctivitis and ipsilateral preauricular lymphadenopathy (Item C147A). These
findings, combined with the cervical lymphadenopathy and his exposure to a kitten, make catscratch disease, or Bartonella henselae infection, the most likely possibility. Although most
patients who have cat-scratch disease report some contact with kittens or cats, many do not
recall being scratched. Cats younger than 1 year of age are most likely to transmit the organism
to humans; human-to-human transmission does not occur.
The most common clinical manifestation of cat-scratch disease is regional
lymphadenopathy, with cervical and axillary areas affected most (Item C147B). A papule (Item
C147C) at the site of the scratch often precedes the development of lymphadenopathy. Lymph
nodes may enlarge considerably and may be tender, warm, and erythematous; suppuration can
occur in up to 30% of patients. Nodes may remain enlarged for several months. Other clinical
manifestations of cat-scratch disease include oculoglandular syndrome, fever of unknown origin,
painful osteolytic lesions, and hepatic or splenic microabscesses. Among the less common
manifestations are encephalopathy, thrombocytopenic purpura, and neuroretinitis.
Cat-scratch disease can be diagnosed by obtaining an indirect immunofluorescent antibody
assay for B henselae. Biopsy of an affected node also may show the bacilli or granulomas but
should be reserved for atypical cases. The course of cat-scratch disease is self-limited. The
efficacy of antibiotic therapy is controversial, but some studies have shown improvement in
those who have severe systemic symptoms and those who are immunocompromised. Several
oral agents have shown efficacy: azithromycin, erythromycin, ciprofloxacin, trimethoprimsulfamethoxazole, and rifampin. Needle aspiration of suppurative lymph nodes may hasten
recovery and reduce pain, but incision and drainage is not recommended because of the risk of
sinus tract formation and persistent drainage.
Tularemia, infection with Francisella tularensis, also can cause an oculoglandular syndrome,
but the conjunctivitis typically is painful. A history of exposure to ticks or wild rabbits would make
this diagnosis more likely. Haemophilus influenzae can cause conjunctivitis with ipsilateral
preauricular lymphadenopathy, but the conjunctivitis typically is purulent, and ipsilateral otitis
media might be expected. Pasteurella multocida is the most common organism causing infection
after a cat bite. Typical findings include cellulitis and suppuration at the area of the bite;
conjunctivitis is rare. Staphylococcus aureus also can cause purulent conjunctivitis, but the
history of kitten exposure for the patient in the vignette makes this a less likely causative
pathogen.
References:
American Academy of Pediatrics. Cat-scratch disease. In: Pickering LK, Baker CJ, Long SS,
McMillan JA, eds. Red Book: 2006 Report of the Committee on Infectious Diseases. 27th ed. Elk
Grove Village, Ill: American Academy of Pediatrics; 2006:246-248.
English R. Cat-scratch disease. Pediatr Rev. 2006;27:123-128. Available at:
http://pedsinreview.aappublications.org/cgi/content/full/27/4/123

Copyright 2009 by the American Academy of Pediatrics

page 315

2009 PREP SA on CD-ROM


Question: 148

A mother brings in her 3-month-old boy because he has had a worsening cough over the past 2
days. She reports that he has been afebrile but not feeding as well as he normally does. You
note a few coarse breath sounds, rare wheezing, and intermittent subcostal retractions on
physical examination. His respiratory rate is 56 breaths/min and temperature is 101.5F
(38.6C). There is a family history of asthma. A trial of albuterol results in minimal improvement in
his chest findings. His oxygen saturation is 89% on room air.
Of the following, the next BEST course of action is to

A. arrange for home administration of albuterol


B. hospitalize the boy for intravenous steroid and antibiotic administration
C. hospitalize the boy for observation and supportive care
D. order a complete blood count
E. refer the boy to a pulmonologist

Copyright 2009 by the American Academy of Pediatrics

page 316

2009 PREP SA on CD-ROM


Critique: 148

Preferred Response: C

The child described in the vignette clearly exhibits tachypnea for age and has clinical
examination findings consistent with respiratory distress. The indications for hospital admission
for a child who has bronchiolitis are similar to those for any child who has any lower respiratory
tract disorder. Respiratory distress that is indicated by tachypnea or hypoxemia; signs of
accessory muscle use; poor feeding or inability to maintain the airway; signs of respiratory
failure such as cyanosis, listlessness, or fatigue; and dehydration necessitate immediate
hospitalization.
At the age of 3 months, the boy is at risk for dehydration and poor feeding due to his inability
to suck and swallow from breast and bottle in conjunction with the reported signs and
symptoms.
A trial of bronchodilator did not improve his condition, which is not unusual in acute
bronchiolitis. For the minority of children (<35%) who do respond to albuterol, a trial in the home
with very close follow-up and revisit might be considered. Although nebulized racemic
epinephrine has been found to be effective in some children who have bronchiolitis, home use
never has been shown to be safe or effective.
Viral bronchiolitis has a natural course of worsening over the first 3 to 5 days, which
represents an additional concern for the child in the vignette.
The use of antibiotics has not been proven to be helpful in viral bronchiolitis, and the use of
steroids is controversial and not clearly efficacious. For children who have classic signs and
symptoms of bronchiolitis, such as described for this child, a complete blood count is not
indicated.
Most cases of viral bronchiolitis resolve without sequelae, although there is some
suggestion of an increased risk for future development of recurrent wheezing in affected
children. Bronchiolitis alone is not an indication for a pediatric pulmonary consultation unless the
child fails to improve or exhibits other signs of underlying pulmonary disease. Once a decision
has been made to admit a child who has bronchiolitis, whether the child should be admitted to an
observation, inpatient, or intensive care unit depends on local or practice preference. There is
evidence that the sequential use of respiratory scoring systems may allow for early discharge or
the use of observation units.
References:
Gadomski AM, Bhasale AL. Bronchodilators for bronchiolitis. Cochrane Database Syst Rev.
2006;3:CD001266. Available at:
http://www.mrw.interscience.wiley.com/cochrane/clsysrev/articles/CD001266/frame.html
Lozan JM. Bronchiolitis (updated). BMJ Clinical Evidence. 2007. Available for subscription at:
http://clinicalevidence.bmj.com/ceweb/conditions/chd/0308/0308.jsp
Pelletier AJ, Mansbach JM, Camargo CA Jr. Direct medical costs of bronchiolitis hospitalizations
in the United States.Pediatrics 2006;118:2418-2423. Available at:
http://pediatrics.aappublications.org/cgi/content/full/118/6/2418
Subcommittee on Diagnosis and Management of Bronchiolitis. Clinical practice
guideline:diagnosis and management of bronchiolitis. Pediatrics. 2006;118:1774-1793. Available
at: http://pediatrics.aappublications.org/cgi/content/full/118/4/1774
Willson DF, Horn SD, Hendley JO, Smout R, Gassaway J. Effect of practice variation on
resource utilization in infants hospitalized for viral lower respiratory illness. Pediatrics.
2001;108;851-855. Available at: http://pediatrics.aappublications.org/cgi/content/full/108/4/851

Copyright 2009 by the American Academy of Pediatrics

page 317

2009 PREP SA on CD-ROM


Question: 149

A 15-year-old patient is brought to your office with a complaint of chest pain. She had been
healthy until 3 days ago, when she developed a fever. The pain is precordial, referred to the
epigastrium, and exacerbated by deep breathing and coughing. She refuses to lie down and
prefers to sit leaning forward.
Of the following, the MOST likely expected finding on electrocardiography is

A. elevation of the S-T segments


B. first-degree heart block
C. pre-excitation with a delta wave
D. tall peaked T waves
E. T-wave flattening

Copyright 2009 by the American Academy of Pediatrics

page 318

2009 PREP SA on CD-ROM


Critique: 149

Preferred Response: A

Chest pain in children and adolescents is a common problem for which patients and parents
frequently seek medical care. The causes of chest pain in the pediatric population are varied
and can be considered by organ systems: musculoskeletal, respiratory, gastrointestinal,
psychological, and cardiac. Among the musculoskeletal causes are chest wall strain, trauma,
costochondritis, and the precordial catch syndrome. Respiratory causes include asthma,
pneumonia, pneumothorax, pneumomediastinum, and chronic cough. Chest pain may result
from gastritis, esophagitis, or indigestion. Psychogenic processes, including anxiety, fear, and
attention-seeking behaviors, may cause or exacerbate chest pain. Perhaps the most common
causes of chest pain in pediatrics are those referred to as idiopathic. Such a diagnosis often is
given to the patient who presents with a 1- to 2-week history of intermittent, brief, sharp, or
stabbing pain that is not associated with exercise or exertion.
The cardiac causes of chest pain are important to recognize because they can be
associated with significant morbidity and mortality. Pericardial pain (resulting from inflammation
and often associated with pericarditis), angina and myocardial ischemia, arrhythmias, or aortic
dissection may cause chest pain. The common causes of pericarditis are viral, inflammatory,
and rheumatologic. The typical pain of pericarditis frequently is substernal, positional, and can be
severe. Patients often prefer to sit leaning forward, as described for the girl in the vignette, and
may refuse to lie down. The pain worsens with deep inspiration, coughing, or movement of the
upper torso. Because the pericardium is inflamed, pericardial effusion may occur in affected
patients, which may lead to pericardial tamponade. Some, but not all, pericardial effusions in
pericarditis have the associated finding of a friction rub noted on auscultation. The absence of a
rub does not exclude pericardial effusion or pericarditis.
Electrocardiographic findings can include S-T segment elevation, low voltage, or in cases of
large pericardial effusion, electrical alternans. The latter finding is a beat-to-beat variation in
voltage that likely results from the pendular motion of the heart in the effusion.
First-degree heart block (Item C149A) is not likely in pericarditis. Pre-excitation is seen in
patients who have Wolff-Parkinson-White syndrome (Item C149B) rather than pericarditis.
Abnormalities of the T waves usually implicate an electrolyte abnormality. Tall, peaked T waves
are seen with hyperkalemia (Item C149C), and flattened T waves are a nonspecific finding that
may be seen with hypokalemia (Item C149D) or certain ventricular strain patterns.
References:
Cava J, Sayger PL. Chest pain in children and adolescents. Pediatr Clin North Am.
2004;51:1553-1568. Abstract available at: http://www.ncbi.nlm.nih.gov/pubmed/15561173
Demmler GJ. Infectious pericarditis in children. Pediatr Infect Dis J. 2006;25:165-166
Fahey J. Chest pain. In: Rudolph C, Rudolph A, eds. Rudolph's Pediatrics. 21st ed. New York,
NY: McGraw Hill Medical Publishing Division; 2003:1894-1897
Nowlen TT, Bricker JT. Pericardial disease. In: Moller JH, Hoffman JIE, eds. Pediatric
Cardiovascular Medicine. Philadelphia, Pa: Churchill Livingstone; 2000:780-792

Copyright 2009 by the American Academy of Pediatrics

page 319

2009 PREP SA on CD-ROM


Question: 150

A 14-year-old boy who has epilepsy presents to the emergency department after a generalized
tonic-clonic seizure that began on the playground at school. He continued to convulse en route
in the ambulance, where he received 15 mg diazepam rectally and intravenous access was
achieved. In the emergency department, he continues to be unresponsive, exhibiting
tachycardia and nonsuppressable bilateral synchronous rhythmic clonic jerks.
Of the following, the MOST appropriate medication to administer next is

A. fosphenytoin 20 mg/kg intravenously


B. pentobarbital 5 mg/kg intravenously
C. phenobarbital 20 mg/kg intravenously
D. phenytoin 7 mg/kg orally
E. valproic acid 15 mg/kg intravenously

Copyright 2009 by the American Academy of Pediatrics

page 320

2009 PREP SA on CD-ROM


Critique: 150

Preferred Response: A

Standard recommendations for pharmacologic treatment of status epilepticus and refractory


status epilepticus in adults were established in the 1990s. They also represent the standard of
care for children and remain valid at the time of this writing (Item C150).
Because the boy described in the vignette continues to have seizures after administration of
a reasonable dose of diazepam (step 2), he now should receive either phenytoin 20 mg/kg
intravenously or fosphenytoin 20 mg phenytoin equivalents/kg intravenously. Use of
phenobarbital before phenytoin is not standard. Pentobarbital is reserved for use when other
treatments have failed to induce a medical coma, a state of complete unresponsiveness
manifested by a characteristic electroencephalographic pattern of "burst suppression" (bursts
interspersed with flat or nearly flat tracing). Intravenous valproic acid and levetiracetam are new
options for use in selected cases, but they are not standard at this time.
References:
Johnston MV. Seizures in childhood. In: Kliegman RM, Behrman RE, Jenson HB, Stanton BF,
eds. Nelson Textbook of Pediatrics. 18th ed. Philadelphia, Pa: Saunders Elsevier; 2007:24572475
Riviello JJ Jr, Ashwal S, Hirtz D, et al. Practice parameter: diagnostic assessment of the child
with status epilepticus (an evidence-based review): report of the Quality Standards
Subcommittee of the American Academy of Neurology and the Practice Committee of the Child
Neurology Society. Neurology. 2006;67:1542-1550. Available at:
http://www.neurology.org/cgi/content/full/67/9/1542
Riviello JJ Jr, Holmes GL. The treatment of status epilepticus. Semin Pediatr Neurol.
2004;11:129-138. Abstract available at: http://www.ncbi.nlm.nih.gov/pubmed/15259866

Copyright 2009 by the American Academy of Pediatrics

page 321

2009 PREP SA on CD-ROM


Question: 151

A 2-day-old male is approaching hospital discharge from the regular nursery. You receive an
urgent call from the nurse caring for him, who says that he would not awaken for his last feeding
and is now difficult to arouse. A blood glucose determination is normal. You arrange for
laboratory tests and call the neonatologist to evaluate the baby while you make plans to leave
your office. When you arrive at the hospital 45 minutes later, the baby has been transferred to
the neonatal intensive care unit, is now comatose, and has irregular breathing. Results of a
complete blood count with differential count, platelets, and a chemistry panel, including renal and
liver function tests, are normal.
Of the following, the MOST appropriate laboratory test for diagnosis and formulating a treatment
plan for this baby is

A. acylcarnitine profile
B. serum ammonia
C. total and free carnitine
D. urine organic acids
E. very long-chain fatty acids

Copyright 2009 by the American Academy of Pediatrics

page 322

2009 PREP SA on CD-ROM


Critique: 151

Preferred Response: B

The differential diagnosis for any infant who has reduced arousability or coma must include
inborn errors of metabolism, particularly those that cause elevations of serum ammonia. This is
especially true when the infant has previously been alert and feeding well because the change in
mental status suggests the possible accumulation of a substance that impairs consciousness.
In addition to evaluation for sepsis, initial laboratory tests should include measurement of
serum ammonia, electrolytes, and urine ketones. The anion gap also must be calculated.
Elevated ammonia concentrations are associated with a variety of inborn errors, including urea
cycle defects, transient hyperammonemia, maple syrup urine disease, organic acidopathies,
fatty acid oxidation defects, and nonketotic hyperglycinemia (Item C151). Normally, ammonia
values are below 110.0 mcmol/L in newborns. Because elevated ammonia concentrations can
cause irreversible brain damage, the measurement of serum ammonia and initiation of treatment
for hyperammonemia must be the first steps in evaluating the infant in a coma. In a metabolic
emergency involving an unknown diagnosis and a serum ammonia concentration of 200.0
mcmol/L or greater, benzoate and phenylacetate (or one of the recently marketed preparations
for this purpose) should be administered intravenously to lower the concentration. If the
ammonia concentration is 600.0 mcmol/L or greater, hemodialysis is recommended.
The infant described in this vignette has laboratory values suggestive of a urea cycle defect,
such as ornithine transcarbamylase deficiency. If he had a low glucose value or acidosis,
evaluation for other causes of hyperammonemia would be appropriate and should include
measurement of total and free carnitine, urine organic acids, and very long-chain fatty acids as
well as acylcarnitine profile.
References:
Burton BK. Inborn errors of metabolism in infancy: a guide to diagnosis. Pediatrics. 1998;102:e69e78. Available at: http://pediatrics.aappublications.org/cgi/content/full/102/6/e69
Hoffmann GF, Nyhan WL, Zschocke J, Kahler SG, Mayatepek E. Approach to the patient with
metabolic disease. In: Inherited Metabolic Diseases. Philadelphia, Pa: Lippincott Williams &
Wilkins; 2002:19-94
Niemi A-K, Enns GM. Pharmacology review: sodium phenylacetate and sodium benzoate in the
treatment of neonatal hyperammonemia. NeoReviews. 2006;7:e486-e495. Available for
subscription at: http://neoreviews.aappublications.org/cgi/content/full/7/9/e486
Nyhan WL, Barshop NA, Ozand PT. Hyperammonemia and disorders of the urea cycle:
introduction to hyperammonemia and disorders. In: Atlas of Metabolic Diseases. 2nd ed.
London, England: Hodder Arnold; 2005:191-198

Copyright 2009 by the American Academy of Pediatrics

page 323

2009 PREP SA on CD-ROM


Question: 152

During the annual health supervision visit of a 16-year-old patient, he reports smoking a pack of
cigarettes daily. He plans to become a vocal music major in college and is concerned that
smoking may affect his voice, but he is uncertain if he wants to stop smoking at this time.
Of the following, the MOST appropriate statement to include in your counseling regarding
smoking cessation is that

A. chronic obstructive lung disease is the first pulmonary problem to arise in cigarette smokers
B. cigarettes contain few other toxic chemicals aside from nicotine
C. erectile dysfunction is not associated with cigarette smoking
D. initial symptoms of nicotine dependence occur in some teens after only a few cigarettes
E. lung cancer is the only cancer associated with cigarette smoking

Copyright 2009 by the American Academy of Pediatrics

page 324

2009 PREP SA on CD-ROM


Critique: 152

Preferred Response: D

Tobacco use has decreased for adolescents in the United States since peaks in the late 1990s,
but it still remains prevalent, with 23% of 9th through 12th graders reporting cigarette use on one
or more of the past 30 days preceding the Centers for Disease Control and Prevention (CDC)
National Youth Risk Behavior Survey. Although relatively few studies have examined smoking
cessation techniques specific to adolescents, brief counseling interventions modeled after
similar interventions demonstrated to be effective with adults are recommended. The "5 A's"
(Ask, Advise, Assess, Assist, and Arrange) are advocated for use by the United States Public
Health Service Task Force practice guidelines in counseling adolescents to stop tobacco use.
Among the physiologic consequences of tobacco use are ischemic heart disease;
numerous cancers, including lung, head and neck, esophageal, gastric, colorectal, bladder,
renal, prostate, and cervical; premature wrinkling of the skin; low-birthweight infants associated
with maternal smoking in pregnancy; adverse effects on natural killer cells and other immune
modulators; and chronic obstructive pulmonary disease, with small airway disease present
before evidence of chronic disease is demonstrable. Cigarettes usually contain thousands of
chemicals aside from nicotine, including cyanide, carbon monoxide, formaldehyde, and tar, that
may have long-term detrimental effects on health.
Emphasizing the immediate and short-term effects of tobacco use rather than the long-term
effects is believed to be a better counseling technique for teens. Discussing the effects on
athletic performance and on vocal and dramatic performance where voice is important, the
potential for erectile dysfunction, and the possibility of symptoms of nicotine dependence
occurring in some teens after only a few cigarettes may be more salient to a young person and
more important in making a decision to stop tobacco use.
References:
Ammerman SD. Tobacco. In: Neinstein LS, ed. Adolescent Health Care: A Practical Guide. 5th
ed. Philadelphia, Pa: Lippincott Williams & Wilkins;, 2008:888-907
Centers for Disease Control and Prevention. Youth Risk Behavior Survey: 2007. Available at:
http://www.cdc.gov/Features/RiskBehavior/
Centers for Disease Control and Prevention. Youth risk behavior surveillance-United States,
2005. MMWR Surv Summ. 2006;55(No.SS-5). Available at:
http://www.cdc.gov/mmwr/PDF/SS/SS5505.pdf
Klein JD, Camenga DR. Tobacco prevention and cessation in pediatric patients. Pediatr Rev.
2004;25:17-26. Available at: http://pedsinreview.aappublications.org/cgi/content/full/25/1/17

Copyright 2009 by the American Academy of Pediatrics

page 325

2009 PREP SA on CD-ROM


Question: 153

You are evaluating a 2-month-old girl who has suspected infantile botulism. On physical
examination, she has a weak cry, poor head control, dilated pupils, and a markedly decreased
gag reflex. Her respiratory rate is 30 breaths/min, and she has decreased breath sounds
bilaterally at her lung bases. Her oxygen saturation on room air is 85%, but has increased to
90% on 3 L/min of oxygen administered by nasal cannula. An arterial blood gas evaluation on 3
L/min oxygen shows a pH of 7.24, a Paco2 of 60 mm Hg, and a Pao2 of 70 mm Hg.
Of the following, the MOST appropriate immediate step is

A. intubation with mechanical ventilation


B. oxygen administration via a nonrebreathing face mask
C. oxygen administration via a high-flow nasal cannula
D. placement of a nasal trumpet
E. serial measurements of negative inspiratory force

Copyright 2009 by the American Academy of Pediatrics

page 326

2009 PREP SA on CD-ROM


Critique: 153

Preferred Response: A

The patient described in the vignette demonstrates respiratory failure, which is defined as the
inability to maintain adequate oxygenation (hypoxemia, as defined by a PaO2 less than 60 mm
Hg) and ventilation (hypercarbia, as defined by a PaCO2 greater than 50 mm Hg). Although her
oxygen saturation has improved with oxygen administration, findings of her neurologic
examination include decreased tone, impaired airway protection, and decreased aeration in the
setting of a progressive neurologic disease. In addition, her elevated PaCO2 reflects her
inadequate respiratory effort with subsequent impaired ventilation. Impending respiratory failure
in pediatric patients typically is characterized by tachypnea, increased use of accessory
muscles, or retractions. Hypoxemia, grunting, and fatigue are late signs and worrisome for
respiratory collapse.
Oxygen should be administered to patients who have evidence of impending respiratory
failure in their position of comfort with a goal of maintaining the oxygen saturation at greater than
90%. A variety of factors, including the cause of the respiratory failure, speed of progression,
patient age, associated morbidities, and anticipated procedures such as the need for imaging
studies or transport, can guide the clinician in managing respiratory failure and deciding whether
to proceed with intubation. The impending respiratory failure seen for the patient in the vignette
mandates placement of an endotracheal tube and assistance with ventilation.
Careful monitoring is imperative for patients experiencing respiratory failure, especially those
who may have chronic respiratory failure and an impaired respiratory drive, although the first
priority is correction of the hypoxemia. Administration of oxygen to patients who have chronic
respiratory failure should be at the lowest concentration needed to maintain an oxygen
saturation of 90%, with frequent arterial blood gas monitoring to ensure that their PaCO2 is not
rising.
Failure to respond to oxygen administration indicates the need for more aggressive
intervention. Bag-valve-mask ventilation can support the oxygenation and ventilation needs of
most pediatric patients while appropriate personnel trained in endotracheal intubation are
assembled. Patients who have underlying or acute neurologic or neuromuscular processes that
interfere with their ability to protect their airways or generate adequate tidal volumes are
candidates for early intubation.
Serial negative inspiratory force measurements are an excellent method of following
respiratory effort in patients who have underlying neuromuscular disorders, but they normally
require a cooperative patient of at least 5 years of age. Administration of higher concentrations
of oxygen (eg, by high-flow nasal cannula or nonrebreathing face mask) has the potential of
increasing the patient's oxygenation but would not improve the ventilation. Placement of a nasal
trumpet is indicated for patients who have an intact respiratory drive and evidence of upper
airway obstruction.
References:
Frankel LR. Respiratory distress and failure. In: Kliegman RM, Behrman RE, Jenson HB,
Stanton, BF, eds. Nelson Textbook of Pediatrics. 18th ed. Philadelphia, Pa: Saunders Elsevier;
2007:421-423
Pope J, McBride J. Consultation with the specialist: respiratory failure in children. Pediatr Rev.
2004;25:160-167. Available at: http://pedsinreview.aappublications.org/cgi/content/full/25/5/160

Copyright 2009 by the American Academy of Pediatrics

page 327

2009 PREP SA on CD-ROM


Question: 154

A 12-year-old girl who developed type 1 diabetes at age 3 years comes in with her parents for a
health supervision visit. Her diabetes control has been excellent. Physical examination reveals
Sexual Maturity Rating 2 pubic hair and breast development and a palpable and somewhat firm
thyroid gland.
Of the following, the MOST likely cause of the thyroid enlargement is

A. Graves disease
B. Hashimoto thyroiditis
C. iodine deficiency
D. multinodular goiter
E. pubertal thyroid enlargement

Copyright 2009 by the American Academy of Pediatrics

page 328

2009 PREP SA on CD-ROM


Critique: 154

Preferred Response: B

Approximately 10% of people who have type 1 diabetes develop autoantibodies against the
thyroid or chronic lymphocytic thyroiditis, sometimes referred to as Hashimoto thyroiditis. This is
more common in children in whom the onset of type 1 diabetes occurs before the age of 10
years, such as the girl described in the vignette. A smaller percentage, perhaps 50% or more of
individuals who have chronic lymphocytic thyroiditis, eventually develop hypothyroidism, and a
very small number may develop other autoimmune thyroid disorders, including Graves disease.
Iodine deficiency is not very common in the United States because iodine is freely available
from sources such as iodized salt, seafood, iodophors used to clean stainless steel milk storage
containers, and sodium alginate thickeners derived from seaweed that are found in many fast
foods. Multinodular goiter would be unusual in an iodine-sufficient child of this age without preexisting thyroid problems. On physical examination, a multinodular goiter has a bumpy or
irregular firm surface. Although the thyroid becomes somewhat more palpable during puberty,
perhaps because of increased need for thyroid hormone during rapid growth, it is soft and not
firm.
Thyroid autoimmunity may be associated with other autoimmune disorders. Approximately
3% to 4% of individuals who have chronic lymphocytic thyroiditis develop celiac disease. Vitiligo
also can be found. Less frequently, adrenal insufficiency, pernicious anemia, ovarian failure,
type 1 diabetes, or other autoimmune disorders can develop in an individual who initially has
chronic lymphocytic thyroiditis. Some have classified these disorders as polyglandular
autoimmune syndromes type 1, 2, or 3. Type 1 is caused by a known gene defect in AIRE, an
immune regulator. Individuals who have this rare severe autosomal recessive disorder can
exhibit candidiasis, hypoparathyroidism, Addison disease, and pernicious anemia in addition to
type 1 diabetes, vitiligo, and other autoimmunities. A specific underlying gene defect has not
been identified in those who have type 2 and type 3 autoimmune endocrinopathies, but these
conditions have linkages to human lymphocyte antigen types related to the autoimmune
response. Type 2 autoimmune endocrinopathy includes autoimmune thyroid disease or diabetes
with adrenal insufficiency. The type 3 disorder usually occurs in adults and is comprised of
variable combinations of autoimmune endocrine disorders, but not adrenal insufficiency.
References:
Aldasouqi SA, Akinsoto OPA, Jabbour SA. Polyglandular autoimmune syndrome type 1.
eMedicine Specialties, Endocrinology, Multiple Endocrine Disease and Miscellaneous Endocrine
Disease. 2006. Available at: http://www.emedicine.com/med/topic1867.htm
Kaloumenou I, Alevizaki M, Ladopoulos C, et al. Thyroid volume and echostructure in
schoolchildren living in an iodine-replete area: relation to age, pubertal stage, and body mass
index. Thyroid. 2007;17:875-881. Abstract available at:
http://www.ncbi.nlm.nih.gov/pubmed/17956161
Kordonouri O, Hartmann R, Deiss D, Wilms M, Grters-Kieslich A. Natural course of
autoimmune thyroiditis in type 1 diabetes: association with gender, age, diabetes duration, and
puberty. Arch Dis Child. 2005;90:411-414. Abstract available at:
http://www.ncbi.nlm.nih.gov/pubmed/15781936
LaFranchi S. Thyroiditis. In: Kliegman RM, Behrman RE, Jenson HB, Stanton BF, eds. Nelson
Textbook of Pediatrics. 18th ed. Philadelphia, Pa: Saunders Elsevier; 2007:2327-2329

Copyright 2009 by the American Academy of Pediatrics

page 329

2009 PREP SA on CD-ROM


Question: 155

A 9-year-old child has been struggling in his regular third-grade classroom and has not yet
received additional educational support. A comprehensive psychoeducational evaluation reveals
a significant discrepancy between cognitive testing scores and academic performance for
reading and writing.
Of the following, the BEST educational intervention for this boy is

A. after-school private tutoring for language arts


B. mainstream classroom with preferential seating
C. resource services for language arts and reading
D. self-contained classroom for all subjects
E. summer school so he can catch up

Copyright 2009 by the American Academy of Pediatrics

page 330

2009 PREP SA on CD-ROM


Critique: 155

Preferred Response: C

The Individuals with Disabilities Education Act (IDEA) of 1990 (PL 101-476) defines the
guidelines for education of children in the United States who have specific learning disabilities.
According to the "least restrictive clause" in IDEA, children who have learning disorders should
be integrated into the mainstream classroom as much as possible.
The child described in the vignette is having learning issues in the area of reading and
writing. The most appropriate setting for him is in the mainstream class for all of his subjects
except language arts and reading. For these skills, he should receive extra educational support.
Typically, this is accomplished by having the student go to a specialized classroom (resource
room) staffed by a special education teacher.
Preferential seating close to the teacher in a regular classroom will not address his areas of
academic weakness. Although he may benefit from additional tutoring and summer school, these
strategies could be offered in addition to resource services. Children who have more significant
learning issues (eg, autism spectrum disorder, cognitive impairment) may require a selfcontained classroom that provides more individualized and intensive educational support.
References:
American Academy of Pediatrics Committee on Children With Disabilities. The pediatrician's role
in development and implementation of an Individual Education Plan and/or an Individual Family
Service Plan. Pediatrics. 1999;104:124-127. Available at:
http://pediatrics.aappublications.org/cgi/content/reprint/104/1/124
Council on Children With Disabilities. Policy statement: provision of educationally related
services for children and adolescents with chronic diseases and disabling conditions. Pediatrics.
2007;119:1218-1223. Available at:
http://pediatrics.aappublications.org/cgi/content/full/119/6/1218
Ewen JB, Shapiro BK. Specific learning disabilities. In: Accardo PJ. Capute & Accardo's
Neurodevelopmental Disabilities in Infancy and Childhood. Volume II: The Spectrum of
Neurodevelomental Disabilities. 3rd ed. Baltimore, Md: Paul H. Brookes Publishing Co; 2008:553577
Fessler MA, Plourde PA. Psychoeducational assessment. In: Accardo PJ. Capute & Accardo's
Neurodevelopmental Disabilities in Infancy and Childhood. Volume II: The Spectrum of
Neurodevelomental Disabilities. 3rd ed. Baltimore, Md: Paul H. Brookes Publishing Co; 2008:591610

Copyright 2009 by the American Academy of Pediatrics

page 331

2009 PREP SA on CD-ROM


Question: 156

You are the physician for the child care center that your child attends. When an outbreak of
diarrhea occurs at the center, the director calls you. She knows infections due to Giardia lamblia
are common in child care centers, but asks if she needs to be worried about anything else.
Of the following, the organism MOST likely to cause a child care outbreak is

A. Aeromonas hydrophila
B. Campylobacter jejuni
C. Salmonella sp
D. Shigella sp
E. Yersinia enterocolitica

Copyright 2009 by the American Academy of Pediatrics

page 332

2009 PREP SA on CD-ROM


Critique: 156

Preferred Response: D

Outbreaks of diarrheal illness can occur in child care facilities when caretakers do not adhere to
hand hygiene policies and the pathogen requires a small inoculum for disease production.
Giardia lamblia and Shigella sp commonly cause child care center outbreaks. Shigella are gramnegative bacilli that are spread readily person-to-person through fecal-oral and oral-anal
contamination or by houseflies or other contaminated fomites. Among the common bacterial
causes of gastroenteritis, Shigella is unique because ingestion of as few as 10 organisms can
cause human disease. There are four primary Shigella pathogens: S sonnei, S flexneri, S boydii,
and S dysenteriae.
The typical incubation period is 2 to 4 days, and the illness caused by Shigella can range
from a mild diarrhea to life-threatening dysentery. Fever and abdominal cramps with highvolume, watery stools followed 24 to 48 hours later by small-volume, bloody mucoid stools with
tenesmus is a common presentation. Bloody mucoid stools are more common with diarrhea due
to S dysenteriae and S flexneri; S boydii and S sonnei usually cause only watery diarrhea.
Seizures can occur, with an incidence that is higher than would be expected from febrile
seizures alone. Other complications can include Reiter syndrome, hemolytic-uremic syndrome,
colonic perforation, and toxic encephalopathy. Stool microscopy revealing a large number of
neutrophils in a patient who has clinical findings suggestive of shigellosis supports the diagnosis
but is not specific for any species of Shigella.
Aeromonas hydrophila, Campylobacter jejuni, Salmonella sp, and Yersinia enterocolitica are
not commonly involved in outbreaks of gastrointestinal disease in child care centers due to their
large inoculum requirements.
References:
American Academy of Pediatrics. Children in out-of-home child care. In: Pickering LK, Baker CJ,
Long SS, McMillan JA, eds. Red Book: 2006 Report of the Committee on Infectious Diseases.
27th ed. Elk Grove Village, Ill: American Academy of Pediatrics; 2006:130-145.
American Academy of Pediatrics. Shigella infections. In: Pickering LK, Baker CJ, Long SS,
McMillan JA, eds. Red Book: 2006 Report of the Committee on Infectious Diseases. 27th ed. Elk
Grove Village, Ill: American Academy of Pediatrics; 2006:589-591
Spence JT, Cheng TL. In brief: Shigella species. Pediatr Rev. 2004;25:329-330. Available at:
http://pedsinreview.aappublications.org/cgi/content/full/25/9/329

Copyright 2009 by the American Academy of Pediatrics

page 333

2009 PREP SA on CD-ROM


Question: 157

A frantic mother brings in her three children, all of whom suddenly developed fevers, red and
sore eyes, headaches, and sore throats 2 days after attending a swimming party at a country
club. The mother states that five other children and 10 adults who attended the party have
similar symptoms. On physical examination, all of the children have temperatures higher than
102.0F (38.9C), bilateral conjunctivitis, nasal congestion, and exudative pharyngitis.
Of the following, the MOST likely cause of these findings is

A. adenovirus
B. Chlamydia trachomatis
C. Epstein-Barr virus
D. Leptospira sp
E. Mycoplasma pneumoniae

Copyright 2009 by the American Academy of Pediatrics

page 334

2009 PREP SA on CD-ROM


Critique: 157

Preferred Response: A

The respiratory tract is the most common site of adenoviral infections. Clinical presentations of
infection include symptoms of the common cold (cough, fever, sore throat, and rhinorrhea),
pharyngitis, tonsillitis, otitis media, pharyngoconjunctival fever, acute hemorrhagic conjunctivitis,
croup, bronchiolitis, and a pertussislike syndrome. Pharyngoconjunctival fever is characterized
by conjunctivitis (Item C157), pharyngitis, rhinitis, cervical adenitis, and fever. Community
outbreaks of pharyngoconjunctival fever have been attributed to exposure to water from
contaminated swimming pools and fomites from shared towels. The onset is acute, and fever
and symptoms last for 3 to 5 days.
The children described in the vignette have physical findings consistent with adenovirusassociated pharyngoconjunctival fever. Conjunctivitis is not seen with disease caused by
Mycoplasma pneumoniae. Conjunctivitis is one of the most distinct findings with Leptospira
infection, but pharyngitis usually is not seen, and patients are much more ill, exhibiting chills,
myalgias, uveitis, rash, and aseptic meningitis. Conjunctivitis is seen in a small percentage of
persons infected with Epstein-Barr virus, but community outbreaks associated with
contaminated swimming pools do not occur and do not represent a mode of disease
transmission. Chlamydia trachomatis is associated with neonatal conjunctivitis and trachoma,
but trachoma is rare in the United States.
References:
American Academy of Pediatrics. Adenovirus infections. In: Pickering LK, Baker CJ, Long SS,
McMillan JA, eds. Red Book: 2006 Report of the Committee on Infectious Diseases. 27th ed. Elk
Grove Village, Ill: American Academy of Pediatrics; 2006:202-204
American Academy of Pediatrics. Chlamydia trachomatis. In: Pickering LK, Baker CJ, Long SS,
McMillan JA, eds. Red Book: 2006 Report of the Committee on Infectious Diseases. 27th ed. Elk
Grove Village, Ill: American Academy of Pediatrics; 2006:252-257
American Academy of Pediatrics. Leptospirosis. In: Pickering LK, Baker CJ, Long SS, McMillan
JA, eds. Red Book: 2006 Report of the Committee on Infectious Diseases. 27th ed. Elk Grove
Village, Ill: American Academy of Pediatrics; 2006:424-426
American Academy of Pediatrics. Mycoplasma pneumoniae infections. In: Pickering LK, Baker
CJ, Long SS, McMillan JA, eds. Red Book: 2006 Report of the Committee on Infectious
Diseases. 27th ed. Elk Grove Village, Ill: American Academy of Pediatrics; 2006:468-470
Singh-Naz N, Rodriguez W. Adenoviral infections in children. Adv Pediatr Infect Dis. 1996;11:365388

Copyright 2009 by the American Academy of Pediatrics

page 335

2009 PREP SA on CD-ROM


Question: 158

A 16-year-old girl presents with symptoms of burning with micturition and back pain. Her
temperature is 101.3F (38.5C), heart rate is 88 beats/min, respiratory rate is 14 breaths/min,
and blood pressure is 108/64 mm Hg. You can elicit costovertebral angle tenderness on the left
side and suprapubic tenderness. Her urinalysis demonstrates a urine specific gravity of 1.025,
pH of 6.5, 2+ blood, 1+ protein, 3+ leukocyte esterase, and positive for nitrite. Microscopy
reveals 5 to 10 red blood cells/high-power field (HPF), 50 to 100 white blood cells/HPF, 4+
bacteria, and occasional squamous epithelial cells.
Of the following, the MOST likely diagnosis is

A. acute bacterial cystitis


B. acute glomerulonephritis
C. acute pyelonephritis
D. bacterial urethritis
E. nephrolithiasis

Copyright 2009 by the American Academy of Pediatrics

page 336

2009 PREP SA on CD-ROM


Critique: 158

Preferred Response: C

The adolescent described in the vignette presents with classic symptoms of acute
pyelonephritis (upper urinary tract infection [UTI]): fever, back pain, and dysuria associated with
physical findings of costovertebral angle and suprapubic tenderness.
UTIs typically are classified as lower urinary tract (involving the bladder and urethra) and
upper urinary tract (involving the ureters, renal pelvis, and kidneys). In an adolescent, a bladder
infection (cystitis) is not accompanied by flank pain or fever. Urethritis, which may be caused by
a sexually transmitted infection, is accompanied by symptoms of dysuria, but not symptoms of
flank pain or fever.
Distinguishing between upper and lower UTI may be more difficult in younger children. In
neonates and infants, the presence of fever in the setting of a UTI characterizes an upper tract
infection (pyelonephritis). In contrast, the absence of fever suggests lower tract infection. The
diagnosis often is made during the evaluation of an infant for fever and irritability. A study from
Taiwan revealed that a UTI was present in 13.6% of infants younger than 8 weeks of age who
presented with fever. Urine white blood cell counts greater than 10/high-power field (pyuria)
were the best indicators of UTI in this setting. Bacteremia accompanies UTI in 31% of pediatric
patients younger than 1 month of age and 21% of those 1 to 2 months of age. Febrile infants
who have no apparent source of fever had UTI in 7.5% of cases in another study. Older infants
present with fever and vomiting, and the diagnosis is based on a positive urine culture collected
by transurethral or suprapubic catheterization. The incidence of bacteremia falls to 3% in
children 2 to 36 months of age.
As children develop verbal skills, it becomes easier to distinguish between upper and lower
tract infection because they can express their symptoms of dysuria and frequency.
Acute glomerulonephritis does not present with dysuria and flank pain. Nephrolithiasis can
present with flank pain and possibly dysuria (if accompanied by crystalluria), but the presence of
suprapubic pain, fever, and disproportionate pyuria to hematuria in the setting of a positive nitrite
result make this diagnosis unlikely.
References:
Ginsburg CM, McCracken GH Jr. Urinary tract infections in young infants. Pediatrics.
1982;69;409-412. Available at: http://pediatrics.aappublications.org/cgi/content/abstract/69/4/409
Hoberman A, Chao HP, Keller DM, Hickey R, Davis HW, Ellis D. Prevalence of urinary tract
infection in febrile infants. J Pediatr. 1993;123:17-23. Abstract available at:
http://www.ncbi.nlm.nih.gov/pubmed/8320616
Johnson CE. New advances in childhood urinary tract infections. Pediatr Rev. 1999:20:335-342.
Available at: http://pedsinreview.aappublications.org/cgi/content/full/20/10/335
Raszka WV Jr, Khan O. Pyelonephritis. Pediatr Rev. 2005;26:364-370. Available at:
http://pedsinreview.aappublications.org/cgi/content/full/26/10/364

Copyright 2009 by the American Academy of Pediatrics

page 337

2009 PREP SA on CD-ROM


Question: 159

A 17-year-old boy presents to the emergency department with respiratory distress and hypoxia
(room air Po2, 86%). His parents called 911 after their son started "gasping for air." The boy
has a history of moderate persistent asthma that recently worsened after a viral infection. He
uses a daily inhaled corticosteroid and an as-needed beta2 agonist inhaler. During the past 2
days, he has not been able to go to school because of his breathing problems, and his parents
have used his nebulizer every 2 hours over the past 12 hours. On physical examination, the boy
is awake and responsive to questions, but his respiratory rate is 34 breaths/min, and he has
nasal flaring and intercostal retractions. Lung examination demonstrates equal breath sounds
bilaterally but obvious expiratory wheezing with a prolonged expiratory phase. The results of an
arterial blood gas on a nonrebreather with 100% oxygen are: pH of 7.35 (normal, 7.35 to 7.45),
Pco2 of 45 mm Hg (normal, 35 to 45 mm Hg), bicarbonate of 24.0 mEq/L (24.0 mmol/L)(normal,
22.0 to 26.0 mEq/L [22.0 to 26.0 mmol/L]), and Po2 of 90 mm Hg (normal, 75 to 100 mm Hg).
Of the following, a TRUE statement regarding the management of this boys condition is that

A. a normal carbon dioxide value on an arterial blood gas measurement makes an asthma
exacerbation unlikely

B. if assisted ventilation is required, the ventilator rate should be set at 34 breaths/min


C. initial corticosteroid therapy can be administered either orally or intravenously
D. the parents should have doubled his inhaled corticosteroid dose when his beta2 agonist was
not working

E. the patient should blow into a paper bag to help calm him

Copyright 2009 by the American Academy of Pediatrics

page 338

2009 PREP SA on CD-ROM


Critique: 159

Preferred Response: C

Regardless of the level of asthma severity, affected patients can experience exacerbations. The
National Heart, Lung, and Blood Institute 2007 Expert Panel 3 Asthma Guidelines have outlined
in detail the importance of recognition and treatment of asthma exacerbations both at home and
in the emergency department (available at
http://www.nhlbi.nih.gov/guidelines/asthma/asthgdln.pdf).
The teenager described in the vignette is having a severe asthma exacerbation that has not
improved with multiple short-acting beta2 agonist (SABA) nebulization treatments administered at
home. Although he is able to answer questions and is alert, he exhibits hypoxia, tachypnea, and
accessory muscle use. Additional signs of severe obstruction include peak expiratory flow rates
less than 50% of predicted, quiet breath sounds, an inability to speak whole phrases, and
cyanosis. He should receive supplemental oxygen, an arterial blood gas, a SABA nebulizer
treatment, and corticosteroids as soon as possible. Most patients, including this boy, should be
able to tolerate oral corticosteroids, which should be continued for 3 to 10 days. Intravenous
corticosteroids typically are reserved for life-threatening asthma exacerbations. Most experts
and published experience support oral and intravenous corticosteroids as being equally
effective. One commonly used technique during an asthma exacerbation is to double the inhaled
steroid dose at the onset of the asthma exacerbation. However, this recommendation is not
supported by results of a controlled trial or the 2007 Expert Panel 3 asthma guidelines.
One of the primary concerns during an acute exacerbation is whether the patient has
impending respiratory failure. A patient who is too dyspneic to speak, perspiring, or has a peak
expiratory flow of less than 25% may need to be intubated. An initial blood gas measurement can
be helpful in assessing the patient's ventilatory status. However, a normal or elevated PCO2 in
the context of significant tachypnea is worrisome and requires aggressive treatment. Indeed, a
patient who has a respiratory rate of 34 breaths/min should have a low PCO2 (respiratory
alkalosis) unless there is airway obstruction. Patients who have respiratory failure due to
asthma and require intubation should be treated differently from other patients who have
respiratory failure requiring intubation. Because the expiratory phase during an asthma
exacerbation is prolonged, setting the ventilatory rate to match the patient's tachypnea does not
allow sufficient time to expel carbon dioxide. An inappropriately high ventilator rate also can lead
to increased thoracic pressure, increased positive end-expiratory pressure, and possibly a
pneumothorax. A slower rate (8 to 12 breaths/min) is recommended if such a patient is
intubated. Blowing in a paper bag has no role in the treatment of asthma.
An asthma action plan should be developed for patients and their families to prevent asthma
exacerbations and emergency department visits. The action plan can help patients understand
the symptoms associated with asthma exacerbation, the role of peak expiratory flows in
determining the severity of an exacerbation, and specific action(s) to take during an
exacerbation.
References:
Harrison TW, Oborne J, Newton S, Tattersfield AE. Doubling the dose of inhaled corticosteroid
to prevent asthma exacerbations: randomised controlled trial. Lancet. 2004;363:271-275.
Abstract available at: http://www.ncbi.nlm.nih.gov/pubmed/14751699
Keeley D, McKean M. Asthma and other wheezing disorders in children. BMJ Clinical Evidence.
2006. Available for subscription at:
http://clinicalevidence.bmj.com/ceweb/conditions/chd/0302/0302.jsp538Q1
National Asthma Education and Prevention Program. Expert Panel Report 3 (EPR-3): Guidelines
for the Diagnosis and Management of Asthma-Summary Report 2007. J Allergy Clin Immunol.
2007;120:S94-S138. Abstract available at: http://www.ncbi.nlm.nih.gov/pubmed/17983880

Copyright 2009 by the American Academy of Pediatrics

page 339

2009 PREP SA on CD-ROM


Question: 160

You are camping with a group of boys at a rural campground in the southeastern United States
when one of the campers is bitten by a snake. His tent mates kill the snake (Item Q160). The
victim is crying and guarding his right hand. On examination of the boys hand, you note several
small, erythematous abrasions but no swelling or ecchymosis.
Of the following, the MOST appropriate course of action is to

A. apply a tourniquet above the bite


B. apply ice to the wound
C. incise and suction the wound
D. provide local wound care
E. transport the boy to the hospital for antivenom

Copyright 2009 by the American Academy of Pediatrics

page 340

2009 PREP SA on CD-ROM


Critique: 160

Preferred Response: D

Each year approximately 7,000 to 8,000 venomous snake bites occur in the United States that
result in five to six deaths and significant morbidity. Although most bites occur in young adult
males, children are disproportionately represented among the fatalities because their smaller
size leads to a higher venom dose per kilogram. In the United States, almost all venomous
snakes are pit vipers (cottonmouths, rattlesnakes [Item C160A], copperheads [Item C160B]). Of
these, eastern and western diamondback rattlesnakes are responsible for most fatal bites.
Besides pit vipers, the only other venomous snake in the United States is the coral snake, which
is found in the southern United States.
Management of a snake bite is predicated on whether the bite was caused by a venomous
or nonvenomous snake, which can be determined either by examining the (preferably dead)
snake or by the appearance of the wound. A pit viper can be identified by its triangular-shaped
head, elliptical pupils, and presence of fangs; a nonvenomous snake has a round head and
pupils and no fangs or rattle (Item C160C). Although the coral snake also has a round head, it is
recognizable by its unique striped coloration in which red and yellow bands are adjacent ("red on
yellow, kill a fellow") (Item C160D and Item C160E). Because the snake often is not available for
inspection, examination of the wound should yield findings that can guide further treatment. A pit
viper bite generally has visible fang marks; a nonvenomous snake typically leaves an abrasion
or scratches at the site, such as described in the vignette. If the wound has been envenomated
(approximately 25% of pit viper bites are "dry" or nonenvenomated), pain at the site is
immediate, followed by rapid development of erythema, swelling, and regional spread of the pain.
Over the next several hours, hemorrhagic bulla and lymphangitis may be noted (Item C160F).
Early systemic symptoms include nausea, vomiting, hypotension, and tachycardia. In contrast
to pit viper venom, which contains a mixture of hemotoxic, cardiotoxic, myotoxic, and proteolytic
enzymes, coral snake venom is primarily a neurotoxin. The bite typically is painless, but
neurologic and autonomic symptoms, including hypersalivation, weakness, altered mental
status, cranial nerve palsies, and respiratory paralysis, may develop within 12 hours.
Both the snake and wound appearance for the patient described in the vignette indicate that
he was bitten by a nonvenomous snake. Local wound care and tetanus immunization update are
indicated. Had the wound been envenomated, field treatment would include maintenance of the
victim's respiratory and cardiovascular functions; immobilization and dependent positioning of
the extremity; and removal of watches, rings, and tight clothing. Use of tourniquets, wound
incision and suction, or application of ice is not recommended and may lead to increased tissue
injury. Rapid transport to the nearest appropriate facility where antivenom can be given is
critical.
The currently available pit viper antivenom is the sheep-derived Fab antibody-segmentbased product (crotaline Fab). In six small studies performed to evaluate safety and efficacy,
crotaline Fab controlled both local and systemic symptoms effectively and caused fewer cases
of acute allergic reactions or serum sickness than the previously available but no longer
manufactured horse-derived crotaline antivenom. Although the studies included few children, the
limited data support its use.
References:
Gold BS, Dart RC, Barish RA. Bites of venomous snakes. N Engl J Med. 2002;347:347-356.
Extract available at: http://content.nejm.org/cgi/content/extract/347/5/347
Schmidt JM. Antivenom therapy for snakebites in children: is there evidence? Curr Opin Pediatr.
2005;17:234-238. Abstract available at: http://www.ncbi.nlm.nih.gov/pubmed/15800419
Singletary EM, Rochman AS, Bodmer JCA, Holstege CP. Envenomations. Med Clin North Am.
2005;89:1195-1224. Abstract available at: http://www.ncbi.nlm.nih.gov/pubmed/16227060

Copyright 2009 by the American Academy of Pediatrics

page 341

2009 PREP SA on CD-ROM


Question: 161

An 8-year-old girl presents to the emergency department with a history of recurrent severe
vomiting. According to the family, she has had four similar episodes in the past 6 months. Each
time, the child awakens from sleep, vomits every 20 minutes for 6 hours, and then goes back to
bed. Between episodes, which occur approximately every 4 to 6 weeks, the child is happy and
playful. Normal results have been found on prior head magnetic resonance imaging, upper
gastrointestinal radiograph series, and renal ultrasonography. In the emergency department, the
girl is quiet, somewhat listless, and prefers to be in a dark room with an emesis basin. Results of
physical examination are unremarkable, and optic discs are sharp. After 8 hours of intravenous
hydration, the symptoms resolve, and she is discharged.
Of the following, the MOST appropriate medication to treat her underlying condition is

A. amitriptyline
B. lubiprostone
C. omeprazole
D. ranitidine
E. sucralfate

Copyright 2009 by the American Academy of Pediatrics

page 342

2009 PREP SA on CD-ROM


Critique: 161

Preferred Response: A

The history described for the girl in the vignette is most consistent with cyclic vomiting syndrome
(CVS), a symptom complex usually considered a migraine variant. The hallmarks of this
syndrome are intense periods of vomiting that usually last between 8 and 48 hours. The typically
affected patient has an aura of intense nausea or malaise that is followed by vomiting of
undigested food, gastric fluid, and ultimately bile. During the peak of the attack, patients may
vomit as frequently as three to six times per hour. Physical examination findings usually are
benign, but patients characteristically are listless and in a state characterized as "conscious
coma." Abdominal pain, dehydration, headache, and electrolyte abnormalities can accompany
the episode. Most episodes are self-limited and resolve spontaneously. In between episodes,
the patient usually feels completely well and has no significant symptoms of nausea or vomiting.
Precipitating factors for episodes include infection, psychological stress, and menstrual periods
(catamenial migraine), but stressors cannot be identified in most episodes.
Although most patients who have CVS have a migraine variant, other conditions can present
with recurrent episodic vomiting, including pancreatitis, ureteropelvic junction obstruction,
malrotation, eosinophilic gastroenteritis, and even intracranial lesions. Most notably, certain
metabolic defects (urea cycle defects, fatty acid oxidation defects) can present with recurrent
vomiting. Therefore, a thorough evaluation should exclude other causes. Helpful diagnostic tests
include upper gastrointestinal radiographic series (to exclude malrotation), abdominal
ultrasonography (to exclude ureteropelvic junction obstruction), and magnetic resonance
imaging of the head (to exclude intracranial lesions). In addition, in the middle of a CVS episode,
electrolytes should be measured and metabolic studies (ammonia, urinary organic acids and
acylcarnitine, plasma amino acids, pyruvate, and lactate) obtained.
Supportive care during a vomiting episode of CVS includes intravenous fluids, antiemetics
(ondansetron), and antianxiety medications (lorazepam). If the episodes are frequent,
prophylactic antimigraine therapy is effective in reducing their frequency and severity. This
therapy should be instituted in consultation with a specialist (pediatric neurologist or
gastroenterologist). Daily amitriptyline is used commonly as first-line prophylactic therapy for
children older than age 5 years; cyproheptadine often is used in younger children. Lubiprostone
is used in the treatment of refractory constipation. Omeprazole, ranitidine, and sucralfate are
used to treat reflux or gastritis and have not been shown to be useful in treating CVS.
References:
Cyclic Vomiting Syndrome Association of America web site. Available at:
http://www.cvsaonline.org
Li BU, Misiewicz L. Cyclic vomiting syndrome: a brain-gut disorder. Gastroenterol Clin North Am.
2003;32:997-1019. Abstract available at: http://www.ncbi.nlm.nih.gov/pubmed/14562585

Copyright 2009 by the American Academy of Pediatrics

page 343

2009 PREP SA on CD-ROM


Question: 162

You are examining a preterm infant who was delivered at 34 weeks gestation due to premature
labor and a maternal urinary tract infection. His mother is 23 years old and has had insulindependent diabetes mellitus for many years. Her diabetes reportedly was well managed during
the pregnancy, and a recent hemoglobin A1c measurement was 7.0%. On physical examination,
the infant is appropriately grown for gestational age, weighs 2 kg, and has a gestational age
assessment that equates to 34 weeks. He requires supplemental oxygen with an FiO2 of 0.40
administered by continuous nasal positive airway pressure.
Of the following, the GREATEST concerns for this infant related to his mothers diabetes are

A. anemia, hypotension, and hypercalcemia


B. apnea, bradycardia, and hypercalcemia
C. hyperglycemia, hyperacusis, and aortic stenosis
D. hyperglycemia, hypocalcemia, and sepsis
E. hypoglycemia, hypocalcemia, and polycythemia

Copyright 2009 by the American Academy of Pediatrics

page 344

2009 PREP SA on CD-ROM


Critique: 162

Preferred Response: E

The infant described in the vignette is an appropriately grown infant of a diabetic mother (IDM).
In this instance, the mother has longstanding insulin insufficiency (type 1 diabetes mellitus).
Other classifications of diabetes in pregnancy include previously existing insulin resistance (type
2 diabetes mellitus) and gestational diabetes (insulin resistance occurring only in pregnancy).
Common sequelae of maternal diabetes seen in the fetus include fetal macrosomia,
polyhydramnios, and congenital anomalies. Neonatal problems requiring evaluation and
management in the IDM include:
* Birth defects (congenital anomalies)
* Septal hypertrophy of the myocardium and possible heart failure
* Hypoglycemia (not hyperglycemia)
* Hypocalcemia (not hypercalcemia)
* Hyperbilirubinemia
* Macrosomia with or without visceromegaly
* Polycythemia (not anemia)
* Polyhydramnios
* Renal vein thrombosis
* Neonatal small left colon
Sepsis is not more common in IDMs, nor is hypotension, except in the presence of septal
hypertrophy. Apnea and bradycardia may occur in a preterm 34-week gestation infant, but it is
not related to maternal diabetes. Aortic stenosis is not a common cardiac sequela in IDMs.
Hyperacusis is not related to maternal diabetes.
References:
Cowett RM. Neonatal care of the infant of the diabetic mother. NeoReviews. 2002;3:e190-e196.
Available for subscription at: http://neoreviews.aappublications.org/cgi/content/full/3/9/e190
Cowett RM. The infant of the diabetic mother. NeoReviews. 2002;3:e173-e189. Available for
subscription at: http://neoreviews.aappublications.org/cgi/content/full/3/9/e173
Kalhan SC, Parimi PS. Disorders of carbohydrate metabolism. In: Martin RJ, Fanaroff AA, Walsh
MC, eds. Fanaroff and Martin's Neonatal-Perinatal Medicine. 8th ed. Philadelphia, Pa: Mosby
Elsevier; 2006:1467-1490

Copyright 2009 by the American Academy of Pediatrics

page 345

2009 PREP SA on CD-ROM


Question: 163

You are discussing the pharmacokinetics and potential interactions of drugs used in the pediatric
population with a group of medical students. One of them asks you if medications should be
taken with food, and you respond that interactions between food and drugs may either reduce or
increase the drug absorption, depending on the type of medication.
Of the following, the MOST accurate advice regarding taking medications with food is that

A. amoxicillin should not be given with meals


B. ciprofloxacin should be taken with dairy products
C. griseofulvin should be taken with high-protein meals
D. isoniazid should be taken with food
E. tetracycline should not be taken with dairy products

Copyright 2009 by the American Academy of Pediatrics

page 346

2009 PREP SA on CD-ROM


Critique: 163

Preferred Response: E

It is important for the clinician to recognize that the concomitant use of foods and medications
may result in an undesired response to the drug, either through decreased efficacy or increased
risk of toxicity. Such interactions may result from a feature of the drug or food. For example,
some medications, such as isotretinoin or griseofulvin, are more bioavailable when taken with
fatty foods because they are lipophilic agents. Others, such as digoxin, have lower bioavailability
with high-fiber meals because the fiber binds the drug. In fact, most food-drug interactions occur
because some characteristic of the food affects the bioavailability of the drug.
Components in some foods and dairy products chelate a portion of the drug and reduce
bioavailability, which occurs with many antibiotics. Other antibiotics are acid-labile agents, so the
increased gastric acid secretion that occurs with food intake may reduce their bioavailability. For
some medications, the effect is minimal unless the dose is inadequate, but for others, the risk is
higher and warrants careful attention to avoid treatment failure. Item C163 lists some antibiotics
commonly used in the pediatric population that have potential food-drug interactions. In general,
for medications that should not be taken with food, the drug should be given either 1 to 2 hours
before or 2 hours after the ingestion of the food in question.
Many other medications should be administered with caution to ensure proper bioavailability
and efficacy. Patients should be advised to pay close attention to any instructions offered by
their pharmacist to avoid adverse reactions or treatment failures.
References:
Gal P, Reed M. Principles of drug therapy. In: Kliegman RM, Behrman RE, Jenson HB, Stanton
BF, eds. Nelson Textbook of Pediatrics. 18th ed. Philadelphia, Pa: Saunders Elsevier; 2007:331338
Schmidt LE, Dalhoff K. Food-drug interactions. Drugs. 2002;62:1481-1502. Abstract available at:
http://www.ncbi.nlm.nih.gov/pubmed/12093316

Copyright 2009 by the American Academy of Pediatrics

page 347

2009 PREP SA on CD-ROM


Question: 164

During the health supervision visit of a term newborn boy, his mother relates that a cousin's child
died at age 4 months from sudden infant death syndrome. She asks what she can do to prevent
such an occurrence in her son.
Of the following, the single MOST important preventive measure is to

A. avoid use of a pacifier


B. cosleep in the parental bed for close observation
C. ensure side sleep positioning to prevent aspiration
D. ensure supine sleep positioning
E. use home apnea and bradycardia monitoring

Copyright 2009 by the American Academy of Pediatrics

page 348

2009 PREP SA on CD-ROM


Critique: 164

Preferred Response: D

Sudden infant death syndrome (SIDS) remains a significant cause of death in the first postnatal
year. However, the rate of death has diminished by more than 50% since the recommendation
was made more than a decade ago for supine rather than prone sleep position for children
younger than 1 year of age.
In the United States, infant cosleeping with parents increases the risk of SIDS, possibly
related to frank suffocation. However, the recommendation to avoid cosleeping remains
controversial when providing support for breastfeeding and considering the prevalence of
cosleeping worldwide. Further risk factors associated with cosleeping include parental obesity
and sleeping on a very soft surface, such as a waterbed. Tobacco smoke exposure increases
the risk of SIDS, especially in the setting of cosleeping.
Although initial recommendations suggested placing the child either supine or on the side to
prevent SIDS, subsequent research has shown that the side position also can increase the risk
of SIDS, possibly due to the propensity for a bundled infant placed on his or her side to roll
forward into the prone position. Therefore, parents should be urged to place the infant supine
instead of on the side for sleep.
Home apnea and bradycardia monitoring has not been shown to reduce the risk of SIDS,
even in families where a previous child has died from SIDS. Its use is limited to preterm infants
who have apnea of prematurity and infants in whom central apnea, known cardiac arrhythmia, or
other identifiable cause of events that may respond to monitoring and cardiorespiratory
resuscitation is a distinct possibility.
Pacifier use may reduce the incidence of SIDS but is not without controversy due to
possible interference with breastfeeding in the early weeks after birth. However, the peak
incidence of SIDS occurs between 2 and 6 months, which is primarily after the establishment of
successful breastfeeding during the first 2 postnatal weeks.
Modeling of appropriate infant sleep position and good patient education in the hospital may
be contributing factors to adoption of these practices.
References:
Colson ER, Levenson S, Rybin D, et al. Barriers to following the supine sleep recommendation
among mothers at four centers for the Women, Infants, and Children Program. Pediatrics.
2006;118:e243-e250. Available at:
http://pediatrics.aappublications.org/cgi/content/full/118/2/e243
Creery D, Mikrogianakis A. Sudden infant death syndrome. BMJ Clinical Evidence. 2006.
Available for subscription at:
http://clinicalevidence.bmj.com/ceweb/conditions/chd/0315/0315.jsp
Farrell PA, Weiner GM, Lemons JA. SIDS, ALTE, apnea, and the use of home monitors. Pediatr
Rev. 2002;23:3-9. Available at: http://pedsinreview.aappublications.org/cgi/content/full/23/1/3
Fu LY, Moon RY. Apparent life-threatening events (ALTES) and the role of home monitors.
Pediatr Rev. 2007;28:203-208. Available at:
http://pedsinreview.aappublications.org/cgi/content/full/28/6/203
Hauck FR, Omojokun OO, Siadaty MS. Do pacifiers reduce the risk of sudden infant death
syndrome? A meta-analysis. Pediatrics. 2005;116:e716-e723. Available at:
http://pediatrics.aappublications.org/cgi/content/full/116/5/e716
Hein HA, Pettit SF. Back to Sleep: good advice for parents but not for hospitals? Pediatrics.
2001;107:537-539. Available at: http://pediatrics.aappublications.org/cgi/content/full/107/3/537
Moon RY, Fu LY. Sudden infant death syndrome. Pediatr Rev. 2007;28:209-214. Available at:
http://pedsinreview.aappublications.org/cgi/content/full/28/6/209

Copyright 2009 by the American Academy of Pediatrics

page 349

2009 PREP SA on CD-ROM


Question: 165

You are leading teaching rounds with the residents at the hospital. They present an 18-monthold boy who has had 6 days of a temperature to at least 102.3F (39.1C). He also has
nonexudative conjunctivitis, a polymorphous rash, erythema of his lips, and swelling of his hands
and feet. The residents ask you to comment on the use of echocardiography in this condition.
Of the following, the MOST accurate statement about echocardiography in this disease is that

A. abnormal results at diagnosis suggest a poor outcome


B. it should be performed only if C-reactive protein concentrations are elevated
C. it should be performed to confirm the diagnosis
D. normal results at diagnosis obviate the need to repeat the study
E. the study may be useful in confirming atypical cases

Copyright 2009 by the American Academy of Pediatrics

page 350

2009 PREP SA on CD-ROM


Critique: 165

Preferred Response: E

The diagnostic criteria for Kawasaki disease (KD) are fever for at least 5 days and at least four
of the following five features:
* Changes in the extremities (edema, erythema, desquamation)
* Polymorphous exanthema
* Conjunctival injection
* Erythema and/or fissuring of the lips and oral cavity
* Cervical lymphadenopathy
In addition, to meet the diagnosis of KD, such findings should not be explainable by another
disease process. The patient described in the vignette has had fever for more than 5 days and
has four of the criteria of KD. He requires echocardiography for assessment and to obtain
baseline measurements of the coronary arteries. Echocardiography is an important component
of the evaluation of the patient who is suspected to have KD because it is a noninvasive,
portable, and virtually risk-free method of assessing cardiac structure and function. The
complete echocardiographic assessment of the patient who has acute KD includes an
evaluation of cardiac function, determination of the presence of valvular dysfunction,
assessment for pericardial effusion, and baseline measurement of the coronary arteries.
Myocarditis, with or without valvular dysfunction and with or without pericardial effusion, can
occur in those who have KD and usually is not significant. However, myocarditis must be
suspected in the patient who has abnormal hemodynamic findings, and echocardiography
should be pursued. By the end of the first week of the illness, as many as 50% of children who
have KD exhibit echocardiographic evidence of coronary arteritis. This usually manifests as a
"brightness" of the perivascular component of the coronary artery and is the result of edema
and thickening of the vascular wall. In addition, there may be dilation, also referred to as ectasia,
of the coronary arteries during this phase of the illness. Such dilation, typically transient and
asymptomatic, is distinct from the coronary artery aneurysms that can develop in the second
stage (2 to 4 weeks) of the illness (Item C165).
Echocardiography also can play a role diagnosing KD that presents in an incomplete or
atypical pattern (with fewer of the findings described previously at presentation). For example, a
child who has had a fever of unknown origin for 5 or more days but only two or three of the
clinical criteria of KD may have atypical KD. Newburger and colleagues have published an
extensive protocol to facilitate the evaluation of such patients. For those who have suspected
atypical KD, further laboratory evaluation, including measurement of C-reactive protein (CRP)
and erythrocyte sedimentation rate (ESR), can be helpful. If the CRP is 3.0 mg/dL or greater and
the ESR is 40 mm/hr or greater, supplemental laboratory tests should include echocardiography
and assessment of serum albumin, hemoglobin, alanine aminotransferase, white blood cell
count, and platelet count. If the CRP is less than 3.0 mg/dL and the ESR is less than 40 mm/hr,
the child should be followed daily; if the fever persists for at least 7 days, echocardiography
should be performed. Echocardiography also should be obtained, even if the fever resolves, if
typical skin peeling occurs under the nail beds of the fingers and toes.
Abnormal findings, such as coronary arteritis, in patients who have KD do not portend a
poor outcome; rather, they are consistent with the diagnosis of KD. For this patient, who meets
the criteria for the diagnosis, echocardiography is performed regardless of the findings on
laboratory tests. Echocardiography neither confirms nor dispels the diagnosis for patients who
meet the clinical criteria of KD, and normal results do not obviate the need to repeat the study to
assess disease progression.
References:
American Academy of Pediatrics. Kawasaki disease. In: Pickering LK, Baker CJ, Long SS,
McMillan JA, eds. Red Book: 2006 Report of the Committee on Infectious Disease. 27th ed. Elk
Grove Village, Ill: American Academy of Pediatrics; 2006:412-415
Miyashita M, Karasawa K, Taniguchi K, et al. Usefulness of real-time 3-dimensional
echocardiography for the evaluation of coronary artery morphology in patients with Kawasaki

Copyright 2009 by the American Academy of Pediatrics

page 351

2009 PREP SA on CD-ROM

disease. J Am Soc Echocardiogr. 2007;20:930-933. Abstract available at:


http://www.ncbi.nlm.nih.gov/pubmed/17555938
Newburger JW, Takahashi M, Gerber MA, et al. AHA scientific statement. Diagnosis, treatment,
and long-term management of Kawasaki disease. A statement for health professionals from the
Committee on Rheumatic Fever, Endocarditis and Kawasaki Disease, Council on
Cardiovascular Disease in the Young, American Heart Association. Circulation. 2004;110:27472771. Available at: http://circ.ahajournals.org/cgi/content/full/110/17/2747
Satou GM, Giamelli J, Gewitz MH. Kawasaki disease: diagnosis, management, and long-term
implications. Cardiol Rev. 2007;15:163-169. Abstract available at:
http://www.ncbi.nlm.nih.gov/pubmed/17575479

Copyright 2009 by the American Academy of Pediatrics

page 352

2009 PREP SA on CD-ROM


Question: 166

A 6-year-old boy presents in late summer to the emergency department with a severe headache
and muscle pains. He recently returned from a camping trip. On physical examination, he is
febrile and has no focal weakness, but he suffers a prolonged tonic-clonic seizure and becomes
unresponsive. Head computed tomography scan reveals no abnormalities. Acyclovir and
fosphenytoin are administered. Magnetic resonance imaging shows subtle, diffuse signal change
and thickening in the cerebral cortex, no signal changes in temporal lobes, and no meningeal
enhancement.
Of the following, the MOST likely cause of the boys symptoms is

A. arbovirus
B. Borrelia burgdorferi
C. herpes simplex virus 1
D. Listeria monocytogenes
E. Taenia solium

Copyright 2009 by the American Academy of Pediatrics

page 353

2009 PREP SA on CD-ROM


Critique: 166

Preferred Response: A

The child described in the vignette has symptoms of an acute viral infection, with fever, muscle
pains, and a prolonged seizure. Involvement of the central nervous system should be suspected
because of the seizure. The magnetic resonance imaging (MRI) findings suggest focal
encephalitis that could be due to an arbovirus or herpes simplex infection. Arbovirus infection is
more likely to occur in late summer because arbovirus is transmitted to humans by mosquitoes.
In addition, herpes simplex infection results in necrotizing focal encephalitis in the temporal lobes
(Item C166A) rather than the subtle thickening described in the vignette. In both cases, focal
infection and inflammation in the brain's cortex can lead to seizures. In the case of herpes,
outside the neonatal period, the temporal lobe infection occurs because of reactivation of the
herpes simplex virus (HSV) in the trigeminal nerve and spread directly into adjacent brain tissue.
Virus also can enter the brain via the blood, in which case generalized encephalitis is more
likely. Examples include HSV in the neonatal period or measles, although this is much rarer
because of the measles-mumps-rubella vaccine. Other viruses prone to cause encephalitis
include human immunodeficiency virus, cytomegalovirus, and rabies.
The other infectious etiologies are less likely, given the details of this case. Ingestion of the
eggs of Taenia solium (pork tapeworm) can lead to neurocysticercosis. This can present with
focal-onset seizures, but affected patients usually do not appear ill, and computed tomography
scan or MRI typically reveal isolated or multiple cysts (Item C166B). Borrelia burgdorferi is
transmitted by ticks, and early infection presents with the characteristic rash (erythema
migrans). Late-stage disease can present with central nervous system findings, the most
common of which is facial nerve palsies. Meningitis is less likely but can occur. Focal
encephalitis is extremely unlikely. Listeria monocytogenes is not likely to present in a child of this
age, and the characteristic central nervous system infection is brainstem encephalitis.
References:
Halstead S. Arborvirus encephalitis in North America. In: Kliegman RM, Behrman RE, Jenson
HB, Stanton BF, eds. Nelson Textbook of Pediatrics. 17th ed. Philadelphia, Pa: Saunders
Elsevier; 2007:1405-1408
Mazzulli T. Laboratory diagnosis of infection due to viruses, Chlamydia, and Mycoplasma. In:
Long SS, Pickering LK, Prober CG, eds. Principles and Practice of Pediatric Infectious
Diseases. 2nd ed. New York, NY: Churchill Livingstone; 2003:1392-1408
Willoughby RE Jr. Encephalitis, meningoencephalitis, and postinfectious encephalomyelitis. In:
Long SS, Pickering LK, Prober CG, eds. Principles and Practice of Pediatric Infectious
Diseases. 2nd ed. New York, NY: Churchill Livingstone; 2003:291-295

Copyright 2009 by the American Academy of Pediatrics

page 354

2009 PREP SA on CD-ROM


Question: 167

A 7-month-old boy presents to the emergency department with vomiting and diarrhea. Findings
on physical examination are normal except for dehydration and lethargy. Laboratory tests reveal
a serum glucose concentration of 30.0 mg/dL (1.7 mmol/L). The mother tells you that she
recently had the flu. Family history is negative for any serious or chronic illnesses. You are
considering an inborn error of metabolism.
Of the following, the MOST helpful next laboratory test is measurement of

A. serum calcium
B. serum lipids
C. serum sodium
D. urine ketones
E. urine reducing substances

Copyright 2009 by the American Academy of Pediatrics

page 355

2009 PREP SA on CD-ROM


Critique: 167

Preferred Response: D

In the absence of sepsis, hypoglycemia in infancy most commonly is associated with disorders
of carbohydrate metabolism, such as glycogen storage diseases (GSDs) or fatty acid oxidation
disorders. A disorder of glycogen storage should be suspected for the infant who presents with
hypoglycemia, massive hepatomegaly without splenomegaly, and lactic acidosis; in these
conditions, glycogen stores in the liver cannot be broken down to supply necessary glucose.
Symptoms of tremulousness and irritability with fasting may be present for some time prior to
diagnosis.
The infant described in the vignette is unlikely to have GSD because he has normal findings
on physical examination. His hypoglycemia is in association with symptoms of vomiting and
diarrhea and a recent exposure to influenza, raising the question of whether he is unable to
create energy from fat stores during this hypermetabolic state. The next step in making a
diagnosis is to measure urine ketones. If he is able to break down fatty acids for energy, he
should have large ketones in his urine; if this is not the case, he will have little to no ketones in
his urine. Should the latter be the case, total and free carnitine concentrations should be
measured and an acylcarnitine profile be obtained to determine which type of fatty acid oxidation
defect is present.
Individuals who have GSD often have marked hyperlipidemia with apparent hyponatremia
(correction must be made for the increased serum solids), but the absence of these findings is
not very helpful in making a diagnosis for the child described in the vignette.
Serum calcium values typically are normal in both fatty acid oxidation and glycogen storage
disorders. Urine reducing substances characteristically are elevated in infants who have
galactosemia, which presents in the newborn period with hepatomegaly and jaundice.
References:
Burton BK. Inborn errors of metabolism in infancy: a guide to diagnosis. Pediatrics. 1998;102:e69e78. Available at: http://pediatrics.aappublications.org/cgi/content/full/102/6/e69
Hoffmann GF, Nyhan WL, Zschocke J, Kahler SG, Mayatepek E. Approach to the patient with
metabolic disease. In: Inherited Metabolic Diseases. Philadelphia, Pa. Lippincott Williams &
Wilkins; 2002:19-94
Nyhan WL, Barshop BA, Ozand PT. Disorders of carbohydrate metabolism. In: Atlas of
Metabolic Diseases. 2nd ed. London, England: Hodder Arnold; 2005:371-372
Nyhan WL, Barshop BA, Ozand PT. Hyperammonemia and disorders of the urea cycle:
introduction to hyperammonemia and disorders of the urea cycle. In: Atlas of Metabolic
Diseases. 2nd ed. London, England: Hodder Arnold; 2005:191-192
Nyhan WL, Barshop BA, Ozand PT. Organic acidemia: introduction. In: Atlas of Metabolic
Diseases. 2nd ed. London, England: Hodder Arnold; 2005:1-3, 191-192,371-372

Copyright 2009 by the American Academy of Pediatrics

page 356

2009 PREP SA on CD-ROM


Question: 168

A 15-year-old young woman has had joint pain for the past 3 days. She developed fever, chills,
and fatigue 4 days ago, but the fever has resolved. In addition, she explains that her left elbow,
right knee, and right wrist are all painful, red, and swollen, and she has a rash on her hands and
feet that looks like pus-filled bumps. She is sexually active, with inconsistent condom use for
contraception. Physical examination reveals an afebrile young woman who has swelling,
tenderness, and mild erythema of the left elbow, right knee, and right wrist. She has a few
pustules and vesicles on the right palm and bilateral soles (Item Q168). The abdomen is not
tender and is without masses.
Of the following, the MOST likely pathogen causing this patients symptoms is

A. Chlamydia trachomatis
B. group A beta-hemolytic streptococci
C. Neisseria gonorrhoeae
D. parvovirus B19
E. Treponema pallidum

Copyright 2009 by the American Academy of Pediatrics

page 357

2009 PREP SA on CD-ROM


Critique: 168

Preferred Response: C

Although probably most recognizable as causing urethritis or cervicitis, Neisseria gonorrhoeae


can be implicated in a number of other infections, including neonatal conjunctivitis, salpingitis,
proctitis, pelvic inflammatory disease, and disseminated gonococcal infection (DGI). DGI
produces gonococcal bacteremia and may result in petechial or pustular acral skin lesions,
asymmetric arthralgia, tenosynovitis, and septic arthritis, as described for the young woman in
the vignette. Less frequently, perihepatitis, endocarditis, and meningitis can occur. Patients who
have DGI typically present with a triad of tenosynovitis, dermatitis, and polyarthralgias without
purulent arthritis or with purulent arthritis without associated skin lesions. There may be
crossover between these two syndromes, however. The dermatitis is described as painless
lesions that often are few in number and usually pustular or vesiculopustular (Item C168A). Such
lesions frequently are transient, lasting for 3 to 4 days.
The differential diagnosis of DGI may include group A beta-hemolytic streptococcal infection
with acute rheumatic fever, but the rash of this infection is very transient and almost never
pustular or vesicopustular. Parvovirus B19 also may cause a rash (fifth disease or erythema
infectiosum) and arthritis, but the rash usually is reticular on the extremities (Item C168B) and
has a "slapped cheeks" appearance on the face (Item C168C). Chlamydia trachomatis infection
is not associated with a rash or arthralgias. Secondary syphilis, caused by Treponema pallidum,
is characterized by rash, mucocutaneous lesions, and lymphadenopathy. The rash is described
as polymorphic, generalized maculopapular (Item C168D) and typically involves the palms and
soles (Item C168E).
References:
American Academy of Pediatrics. Syphilis. In: Pickering LK, Baker CJ, Long SS, McMillan JA,
eds. Red Book: 2006 Report of the Committee on Infectious Diseases. 27th ed. Elk Grove
Village, Ill: American Academy of Pediatrics; 2006:631-644
Goldenberg DL, Sexton DJ. Disseminated gonococcal infection. UpToDate Online 15.3. 2008.
Available for subscription at: http://www.utdol.com/utd/content/topic.do?topicKey=stds/9841
Workowski KA, Berman SM, Centers for Disease Control and Prevention. Sexually transmitted
diseases treatment guidelines, 2006. MMWR Recomm Rep. 2006;55(RR11):1-94. Available at:
http://www.cdc.gov/mmwr/preview/mmwrhtml/rr5511a1.htm

Copyright 2009 by the American Academy of Pediatrics

page 358

2009 PREP SA on CD-ROM


Question: 169

A 5-year-old boy has been receiving mechanical ventilation in the pediatric intensive care unit for
1 week due to complicated adenoviral pneumonia. Over the past several days, he has
developed markedly increased oxygen requirements and progressive opacification of his
bilateral lung fields on chest radiography (Item Q169). He now meets the clinical criteria for
acute respiratory distress syndrome (ARDS). His parents ask about the prognosis.
Of the following, the MOST accurate statement regarding the natural history of ARDS is that

A. complete resolution usually occurs by 1 month


B. development of other organ failure significantly increases the mortality rate
C. mortality is unrelated to the underlying cause
D. the major cause of death is unremitting respiratory failure
E. the overall mortality rate in children is 80%

Copyright 2009 by the American Academy of Pediatrics

page 359

2009 PREP SA on CD-ROM


Critique: 169

Preferred Response: B

Acute respiratory distress syndrome (ARDS) represents the severe end of the acute lung injury
spectrum. Classically, ARDS is defined as a syndrome of lung inflammation with increased
vascular permeability characterized by an acute onset, bilateral infiltrates on chest radiography
(Item C169), absence of left atrial hypertension, and a PaO2/FiO2 of less than 200 mm Hg. The
exact incidence of ARDS in the pediatric population has been difficult to quantify because of
changes in disease definition and limitations of epidemiologic studies (both in methodology and
age ranges studied). Most experts agree that ARDS accounts for 2% to 4% of pediatric
intensive care unit admissions. It has been estimated that affected patients account for 8% of
total patient days and 33% of deaths in the pediatric intensive care unit.
Historically, risk factors for development of ARDS included sepsis, trauma, aspiration,
pneumonia, near-drowning, and multiorgan dysfunction. ARDS normally progresses through
several phases. The initial exudative phase is characterized by pulmonary edema, profound
hypoxemia, and atelectasis. Some patients recover in 3 to 7 days, but others progress to the
fibroproliferative stage, which is characterized by persistent hypoxemia, decreased lung
compliance, and development of pneumothoraces. The final recovery phase consists of
resolution of hypoxemia and improved lung compliance. Complete resolution may take 6 to 12
months, and some patients develop lifelong restrictive lung disease, lung cysts, or decreased
exercise tolerance.
Overall mortality in ARDS has been reported to be 30% to 40%, although exact rates in
pediatric patients remain uncertain. Patients rarely die solely from lung failure; multiorgan system
failure or withdrawal of life support is the most common cause of death. Mortality rates vary,
depending on the underlying disease process, even when corrected for severity of illness.
Patients who develop ARDS following trauma, for example, tend to have much better survival
rates than those who develop ARDS following bone marrow transplantation or gram-negative
sepsis.
References:
Frankel LR. Respiratory distress and failure. In: Kliegman RM, Behrman RE, Jenson HB,
Stanton BF, eds. Nelson Textbook of Pediatrics. 18th ed. Philadelphia, Pa: Saunders Elsevier;
2007:421-423
Vish M, Shanley TP. Acute lung injury and acute respiratory distress syndrome. In: Wheeler DS,
Wong HR, Shanley TP, eds. Pediatric Critical Care Medicine: Basic Science and Clinical
Evidence. New York, NY: Springer-Verlag; 2007:395-411

Copyright 2009 by the American Academy of Pediatrics

page 360

2009 PREP SA on CD-ROM


Question: 170

A 6-month-old baby whose family has just emigrated from Ecuador is brought to your office by
the maternal grandmother. Physical examination reveals length at the 5th percentile, weight at
the 10th percentile, head circumference at the 25th percentile, a sallow complexion with
jaundice, hoarse cry, dry skin, and large tongue. The anterior fontanelle measures 3x4 cm. You
diagnose primary hypothyroidism and start appropriate thyroid hormone replacement therapy.
Of the following, the MOST likely long-term outcome in this baby is

A. adrenal insufficiency
B. microcephaly
C. normal adult height
D. normal intellectual function
E. precocious puberty

Copyright 2009 by the American Academy of Pediatrics

page 361

2009 PREP SA on CD-ROM


Critique: 170

Preferred Response: C

The baby described in the vignette has congenital hypothyroidism and has not received early
treatment with thyroid hormone. Fortunately, congenital hypothyroidism now usually is
diagnosed soon after birth by newborn screening programs in the United States, but this
screening may not be available in resource-limited countries. Children who have severe
hypothyroidism at birth tend to have poorer intellectual outcomes than their siblings, even if
therapy is started in the first few days after birth. However, earlier therapy with larger doses of
thyroid hormone leads to improved intellectual outcomes, even in the most high-risk athyreotic
group of infants. Despite the guarded prognosis for intellectual outcome, with continued and
appropriate thyroid hormone therapy, this child should grow normally in height, and adult height
should be appropriate for the family.
Congenital hypothyroidism is not associated with adrenal insufficiency, although acquired
hypothyroidism because of chronic lymphocytic thyroiditis may be associated with the
development of autoimmune adrenal insufficiency (Addison disease). Microcephaly is an
unusual outcome for children treated appropriately with thyroid hormone. Indeed, brain catch-up
growth in the first postnatal year and a somewhat more flattened basal skull can give affected
children larger head circumferences, split sutures, and the appearance of macrocephaly with
increased intracranial pressure. On the other hand, overtreatment with thyroid hormone can
lead to more rapid fusion of the cranial sutures and microcephaly, so treatment must be
monitored with careful and frequent laboratory measurements of thyroid-stimulating hormone
and thyroid hormone (free thyroxine [T4] or total T4 if free T4 is not available). The potential for
normal intellectual function in this child is guarded. At the least, he is likely to have learning
disabilities and some cerebellar problems, including clumsiness and decreased fine motor skills.
Precocious puberty is not an outcome of congenital hypothyroidism unless a child is overtreated
with thyroid hormone and develops premature maturation, which is very unusual. Some children
who have severe acquired hypothyroidism have manifested signs of sexual precocity that
disappear after treatment of the hypothyroidism (Van Wyk-Grumbach syndrome).
References:
American Academy of Pediatrics, Rose SR and the Section on Endocrinology and Committee on
Genetics, American Thyroid Association, Brown RS and the Public Health Committee, Lawson
Wilkins Pediatric Endocrine Society. Update of newborn screening and therapy for congenital
hypothyroidism. Pediatrics. 2006;117:2290-2303. Available at:
http://pediatrics.aappublications.org/cgi/content/full/117/6/2290
Kempers MJ, van der Sluijs Veer L, Nijhuis-van der Sanden RW, et al. Neonatal screening for
congenital hypothyroidism in the Netherlands: cognitive and motor outcome at 10 years of age. J
Clin Endocrinol Metab. 2007;92:919-924. Abstract available at:
http://www.ncbi.nlm.nih.gov/pubmed/17164300
Postellon DJ, Bourgeois MJ, Varma S. Congenital hypothyroidism. eMedicine Specialties,
Pediatrics: General Medicine, Endocrinology. 2007. Available at:
www.emedicine.com/ped/topic501.htm

Copyright 2009 by the American Academy of Pediatrics

page 362

2009 PREP SA on CD-ROM


Question: 171

You are meeting with a family that recently moved to the United States. The two children, ages
11 months and 24 months, show evidence of global developmental delays. The parents ask
what services are available for their children. You explain that United States federal law provides
children with early intervention services.
Of the following, the BEST explanation about the provisions of the law is that

A. eligibility depends on the socioeconomic level of the family


B. family participation is mandated by law
C. the provision of services requires referral by a physician
D. the services are provided only to children who have autism
E. services for infants and toddlers must be family-based and culturally competent

Copyright 2009 by the American Academy of Pediatrics

page 363

2009 PREP SA on CD-ROM


Critique: 171

Preferred Response: E

Public Law 94-142, the Education of the Handicapped Act, passed in 1975, gave children
between the ages of 5 and 18 years access to a free, appropriate public education. Services to
children between the ages of 3 and 5 years of age were optional. In 1986, the law was amended
as 99-457, which established early intervention programs for children from birth to 3 years who
had developmental delay. The law also mandated that preschool children receive services. The
law was amended again in 1990 with 101-476, the "Individuals with Disabilities Education Act"
(IDEA). The portion entitled Part H, the Program for Infants and Toddlers with Disabilities,
required states to develop and implement community-based systems of care that are
coordinated, family-based, and culturally competent, involving greater interagency collaboration.
It mandated early identification and provision of services to infants and toddlers who have
developmental delays and established conditions. In 1997, the IDEA amendment, PL 105-17
(Part C, formerly Part H), encouraged the states that did not serve the at-risk population to track
and monitor such children so that they could be referred when needed.
Eligibility for such programs is dependent on whether a child has any developmental
disability, not just autism. Families do not need a physician's referral for participation, they are
not required to participate, and such participation is not dependent on income.
References:
American Academy of Pediatrics Committee on Children With Disabilities. Role of the pediatrician
in family-centered Early Intervention Services. Pediatrics. 2001;107:1155-1157. Available at:
http://pediatrics.aappublications.org/cgi/content/full/107/5/1155
Davidson L. In brief: law and the child. Pediatr Rev. 2003;24:213-214. Available at:
http://pedsinreview.aappublications.org/cgi/content/full/24/6/213
Individuals with Disabilities Education Act 1997. Office of Special Education and Rehabilitative
Services. U.S. Department of Education. Available at:
http://www.ed.gov/about/offices/list/osers/policy.html

Copyright 2009 by the American Academy of Pediatrics

page 364

2009 PREP SA on CD-ROM


Question: 172

Results of a stool culture from a 2-year-old boy who has been hospitalized with bloody diarrhea
indicate that the causative agent is Shigella sp. The boy is allergic to trimethoprimsulfamethoxazole.
Of the following, the MOST appropriate antimicrobial agent to use for this patient is

A. amoxicillin
B. azithromycin
C. cefdinir
D. ciprofloxacin
E. linezolid

Copyright 2009 by the American Academy of Pediatrics

page 365

2009 PREP SA on CD-ROM


Critique: 172

Preferred Response: B

Shigella are gram-negative bacilli that are readily spread person-to-person through fecal-oral
contamination. They usually are recovered from bacterial stool cultures using MacConkey and
Hektoen-Enteric agars. The four primary Shigella species (S sonnei, S flexneri, S boydii, and S
dysenteriae) cannot be distinguished clinically; they must be identified in the microbiology
laboratory. Shigella sp primarily infect the large intestine and may cause a range of
manifestations from mild diarrhea to life-threatening dysentery.
Unlike the nontyphoidal forms of Salmonella for which the indications for antimicrobial
therapy are limited to young (<3 months of age) or immunocompromised patients who have
gastroenteritis, all patients who have shigellosis benefit from therapy. Antimicrobial therapy can
shorten the duration of diarrhea and eradicate organisms from the stool. Although trimethoprimsulfamethoxazole is a good agent for empiric therapy of shigellosis, antimicrobial susceptibility
testing always should be performed due to increasing problems with resistance. If ampicillin and
trimethoprim-sulfamethoxazole resistance is verified, parenteral ceftriaxone, a fluoroquinolone
(eg, ciprofloxacin), or azithromycin can be used.
Choosing an appropriate antimicrobial agent to treat the boy described in the vignette, who is
allergic to trimethoprim-sulfamethoxazole and is 2 years of age, is a challenge. Although
ciprofloxacin is effective, it should not be used routinely in children younger than 18 years of age
unless no other options are available. Surprisingly, the clinical usefulness of the oral
cephalosporins (eg, cefdinir), with the exception of cefixime, have not been demonstrated.
Amoxicillin is less effective than ampicillin because of its rapid absorption from the
gastrointestinal tract, and linezolid is an antimicrobial agent from a class of drugs called the
oxazolidinones that are indicated for the treatment of resistant gram-positive organisms (eg,
methicillin-resistant Staphylococcus aureus). Therefore, the agent of choice for this boy is
azithromycin. The duration of antibiotic therapy for shigellosis typically is 5 days.
References:
American Academy of Pediatrics. Shigella infections. In: Pickering LK, Baker CJ, Long SS,
McMillan JA, eds. Red Book: 2006 Report of the Committee on Infectious Diseases. 27th ed. Elk
Grove Village, Ill: American Academy of Pediatrics; 2006:589-591
Niyogi SK. Increasing antimicrobial resistance-an emerging problem in the treatment of
shigellosis. Clin Microbiol Infect. 2007;13:1141-1143. Abstract available at:
http://www.ncbi.nlm.nih.gov/pubmed/17953700
Spence JT, Cheng TL. In brief: Shigella species. Pediatr Rev. 2004;25:329-330. Available at:
http://pedsinreview.aappublications.org/cgi/content/full/25/9/329

Copyright 2009 by the American Academy of Pediatrics

page 366

2009 PREP SA on CD-ROM


Question: 173

You are called to the delivery room to evaluate a term female infant born by precipitous normal
spontaneous vaginal delivery to an 18-year-old young woman who received no prenatal care.
The mother reports using marijuana and alcohol early in her pregnancy and was seen in the
emergency room on two occasions for urinary tract infections. She had several "colds" late in
her pregnancy. She lives with her boyfriend and has two dogs, a cat, and a turtle as pets.
Physical examination of the infant reveals a 2-kg lethargic, jaundiced infant who has a weak cry,
microcephaly, and a distended abdomen. Her liver is palpable 6 cm below the right costal
margin, and her spleen is palpable 4 cm below the left costal margin. She has a diffuse petechial
rash with areas of purpura on her extremities (Item Q173). Laboratory tests show a peripheral
white blood cell count of 10.6x103/mcL (10.6x109/L), hemoglobin of 12.0 mg/dL (120.0 g/L), and
platelet count of 60.0x103/mcL (60.0x109/L). The alanine aminotransferase measurement is 300
U/L, and the aspartate aminotransferase value is 420 U/L. Head ultrasonography shows
scattered intracerebral calcifications.
Of the following, the MOST rapid test for making the diagnosis in this infant is

A. blood culture
B. cerebrospinal fluid polymerase chain reaction
C. nasopharyngeal viral culture
D. serology
E. urine culture

Copyright 2009 by the American Academy of Pediatrics

page 367

2009 PREP SA on CD-ROM


Critique: 173

Preferred Response: E

The newborn described in the vignette has signs and symptoms suggestive of congenital
cytomegalovirus (CMV) infection. CMV is a ubiquitous DNA virus that may be transmitted
vertically from mother to infant in utero by transplacental passage of maternal bloodborne virus,
at birth by passage through an infected maternal genital tract, or postnatally by ingestion of CMVpositive human milk. Approximately 1% of all liveborn infants are infected in utero and excrete
CMV at birth. The risk for infection is greatest during the first half of gestation. In utero fetal
infection can occur after maternal primary infection or after reactivation of infection during
pregnancy, but sequelae are much more common in infants exposed to maternal primary
infection, with 10% having manifestations evident at birth.
Symptomatic congenital CMV disease is characterized by intrauterine growth restriction;
jaundice; hepatosplenomegaly; hepatitis; thrombocytopenia with petechiae and purpura (Item
C173A); and severe central nervous system involvement that can be characterized by
microcephaly, intracerebral calcifications, chorioretinitis (Item C173B), or sensorineural hearing
loss.
Viral culture is the test of choice for confirming the diagnosis of congenital CMV infection.
The diagnosis is established by isolation of the virus from urine, stool, cerebrospinal fluid (CSF),
or saliva in the first 1 to 2 postnatal weeks. CSF polymerase chain reaction testing can be used
to detect CMV DNA, but it is less sensitive than viral isolation by culture and is not used
routinely. Routine blood culture and nasopharyngeal viral culture are not helpful for detecting
CMV. Standard serologic testing is a cumbersome approach to diagnosing congenital CMV
disease; serial samples need to be obtained to make the diagnosis clearly. The presence of
CMV immunoglobulin M antibodies at birth is highly suggestive of a congenital CMV infection, but
a confirmatory urine culture for CMV is recommended to establish the diagnosis definitively.
References:
American Academy of Pediatrics. Cytomegalovirus infection. In: Pickering LK, Baker CJ, Long
SS, McMillan JA, eds. Red Book: 2006 Report of the Committee on Infectious Diseases. 27th ed.
Elk Grove Village, Ill: American Academy of Pediatrics; 2006:273-277
Demmler GJ. Congenital cytomegalovirus infection and disease. Adv Pediatr Infect Dis.
1996;11:135-162
Demmler GJ. Cytomegalovirus. In: Feigin RD, Cherry JD, Demmler GJ, Kaplan SL, eds.
Textbook of Pediatric Infectious Diseases. 5th ed. Philadelphia, Pa: Saunders; 2004:1912-1932
Modlin JF, Grant PE, Makar RS, Roberts DJ, Krishnamoorthy KS. Case records of the
Massachusetts General Hospital. Weekly clinicopathological exercises. Case 25-2003: a
newborn boy with petechiae and thrombocytopenia. N Engl J Med. 2003;349:691-700

Copyright 2009 by the American Academy of Pediatrics

page 368

2009 PREP SA on CD-ROM


Question: 174

A 6-year-old girl is experiencing daytime and nighttime enuresis of 1 months duration. She
achieved daytime continence at age 3 and has been dry at night since age 4. She has no history
of fever, but does have some dysuria. The physical examination is remarkable only for
suprapubic tenderness. Urinalysis demonstrates a specific gravity of 1.015, pH of 6.5, 1+ blood,
trace protein, 3+ leukocyte esterase, and positive for nitrite. Microscopy reveals 2 to 5 red blood
cells/high-power field (HPF), 20 to 50 white blood cells/HPF, and 3+ bacteria. Results of a urine
culture are pending.
Of the following, the MOST appropriate empiric treatment for this patient is

A. amoxicillin
B. cefixime
C. cephalexin
D. ciprofloxacin
E. trimethoprim-sulfamethoxazole

Copyright 2009 by the American Academy of Pediatrics

page 369

2009 PREP SA on CD-ROM


Critique: 174

Preferred Response: E

The child described in the vignette has dysuria, incontinence, and suprapubic tenderness, but
she is otherwise well and has no fever or systemic symptoms. Based on the urinary findings of
positive leukocyte esterase and nitrite with pyuria on microscopy, she most likely has a lower
urinary tract infection (UTI) or cystitis.
Cystitis is treated with empiric outpatient oral antibiotic therapy directed at the most likely
urinary pathogens until the urine culture and antibiotic susceptibilities are available.
Approximately 90% of UTIs are caused by Escherichia coli. Because E coli are usually sensitive
to trimethoprim-sulfamethoxazole (TMP-SMX), it is an excellent agent for initial treatment of
cystitis. In some regions of the country, bacterial resistance to this agent may be increasing, but
it still is considered the best first-line agent because it may be more effective than beta-lactam
antibiotics (eg, amoxicillin) in treating UTI, is inexpensive and readily available, and allows the
clinician to reserve more broad-spectrum antibiotics for treatment failures. Due to high
resistance rates, amoxicillin should not be used as first-line empiric treatment for cystitis.
Broader-spectrum antibiotics such as first-generation (cephalexin) and third-generation
(cefixime) cephalosporins or fluoroquinolones (ciprofloxacin) should be reserved for second-line
treatment unless the child has recurrent UTI and a history of resistance or allergy to TMP-SMX.
The standard duration of treatment is 7 to 14 days. A recent meta-analysis supported the
use of a short course (2 to 4 days) of antibiotics, which compared favorably with a standard
course (7 to 14 days) in terms of treatment failures and recurrence of lower UTI. Single-dose
antibiotic regimens have been used in adults, but pediatric studies demonstrate an increased
number of treatment failures and UTI recurrences compared with standard 7- to 14-day
treatment regimens.
References:
Jantausch B, Kher K. Urinary tract infection. In: Kher KK, Schnaper HW, Makker SP, eds.
Clinical Pediatric Nephrology. 2nd ed. London, England: Informa Healthcare; 2007:553-573
Keren R, Chan E. A meta-analysis of randomized, controlled trials comparing short- and longcourse antibiotic therapy for urinary tract infections in children. Pediatrics. 2002;109:e70.
Available at: http://pediatrics.aappublications.org/cgi/content/full/109/5/e70
Larcombe J. Urinary tract infection in children. BMJ Clinical Evidence. 2007. Available for
subscription at: http://clinicalevidence.bmj.com/ceweb/conditions/chd/0306/0306.jsp
Michael M, Hodson EM, Craig JC, Martin S, Moyer VA. Short versus standard duration oral
antibiotic therapy for acute urinary tract infection in children. Cochrane Database Syst Rev.
2003;1:CD003966. Available at:
http://www.mrw.interscience.wiley.com/cochrane/clsysrev/articles/CD003966/frame.html
Raszka WV Jr, Khan O. Pyelonephritis. Pediatr Rev. 2005;26:364-370. Available at:
http://pedsinreview.aappublications.org/cgi/content/full/26/10/364

Copyright 2009 by the American Academy of Pediatrics

page 370

2009 PREP SA on CD-ROM


Question: 175

An 8-year-old boy presents with wheezing, coughing, and difficulty breathing of 6 months
duration. Findings on his history and pulmonary function tests are suggestive of moderate
persistent asthma. In preparation for asthma management, you have reviewed the current
asthma guidelines, educated the patient on peak flow monitoring, and discussed possible
therapeutic options. You decide to start him on a daily inhaled corticosteroid.
Of the following, the MOST likely adverse event he may experience from inhaled corticosteroids
is

A. acne
B. decreased adult height
C. mood swings
D. oral candidiasis
E. weight gain

Copyright 2009 by the American Academy of Pediatrics

page 371

2009 PREP SA on CD-ROM


Critique: 175

Preferred Response: D

Despite remaining the cornerstone therapy for persistent asthma, many parents are reluctant to
have their children begin using inhaled corticosteroids. Addressing parents' concerns and
outlining the potential adverse effects may improve understanding and compliance.
Local oropharyngeal symptoms, including dysphonia, oral candidiasis, and cough, are
commonly encountered adverse effects of inhaled corticosteroids. Local symptoms appear to
be dose-related and can be lessened by using a "spacer" and rinsing out the mouth after use. A
number of studies have documented a transient decrease in growth velocity during the first year
of inhaled corticosteroid therapy, but a daily cumulative dose up to 800 mcg of inhaled
budesonide has been shown not to affect predicted adult height.
Less common adverse effects of inhaled steroids include acne, mood swings, weight gain,
decreased serum immunoglobulin G concentrations, and rarely, posterior subcapsular cataracts
and adrenal suppression.
Inhaled steroids are used in persistent asthma to help halt lung remodeling by decreasing
bronchial inflammation, reducing inflammatory mediators, and decreasing bronchial
hyperresponsiveness.
References:
Allen DB, Bielory L, Derendorf H, Dluhy R, Colice GL, Szefler SJ. Inhaled corticosteroids: past
lessons and future issues. J Allergy Clin Immunol. 2003;112(3 suppl):S1-S40. Abstract available
at: http://www.ncbi.nlm.nih.gov/pubmed/14515117
Keeley D, McKean M. Asthma and other wheezing disorders in children. BMJ Clinical Evidence.
2006. Available for subscription at:
http://clinicalevidence.bmj.com/ceweb/conditions/chd/0302/0302.jsp538Q1
Schielmer RP, Spahn JD, Covar R, Szefler SJ. Glucocorticoids. In: Adkinson NF, Jr, Yunginger
JW, Busse WW, Bochner BS, Holgate ST, Simons FE, eds. Middleton's Allergy Principles and
Practice. 6th ed. Philadelphia, Pa: Mosby Inc; 2003:870-913

Copyright 2009 by the American Academy of Pediatrics

page 372

2009 PREP SA on CD-ROM


Question: 176

Your first patient of the day is a 2-year-old girl who is brought in by her mother after a brown
spider was found in the childs bed. The mother has brought the spider for you to inspect (Item
Q176A). On physical examination, there is a 2-cm bulla with 4 cm of surrounding erythema on
the medial aspect of the girls calf (Item Q176B). The child otherwise appears well and
occasionally scratches at the lesion.
Of the following, the MOST appropriate course of action for this patient is to

A. begin dapsone therapy


B. begin local wound care
C. prescribe a 5-day course of prednisone
D. refer the child to a surgeon for excision of the bite area
E. transfer the patient to the emergency department for antivenom

Copyright 2009 by the American Academy of Pediatrics

page 373

2009 PREP SA on CD-ROM


Critique: 176

Preferred Response: B

In the United States, two spider species are responsible for most spider bite-related illness and
injury: Lactrodectus sp (black widow spiders) (Item C176A) and Loxosceles sp. Black widow
spider bites cause a systemic syndrome characterized by autonomic dysfunction, muscle
cramping, and rigidity due to neurotoxins in the venom. Spiders of the genus Loxosceles, of
which the brown recluse spider (Loxosceles recluse) is best known, are recognized primarily as
a cause of necrotic skin lesions, although systemic symptoms, including a flulike illness,
hemolytic anemia, and renal failure, may occur.
The spider responsible for the skin lesion described for the child in the vignette is a brown
recluse, which can be recognized by its brown color, violin-shaped marking on the thorax (Item
C176B), and three sets of eyes. Found throughout the central and southern United States, they
live in dark, protected environments such as wood piles or storage sheds but can be found
indoors in bedsheets and clothing hampers. They are nocturnal, and human bites typically occur
at night. Brown recluse spider venom contains a variety of proteolytic enzymes that can cause
extensive tissue damage. The bite itself usually is painless and may go unnoticed until pain,
erythema, and pruritus develop at the site. The initial erythematous maculopapular lesion
becomes bullous (Item C176C) and increases in size over the subsequent 48 hours. After that
time period, an eschar develops (Item C176D) and subsequently separates, leaving a deep
ulcer at the site (Item C176E). Systemic symptoms include fever, chills, vomiting, and
arthralgias. Because children are smaller and receive a larger per kilogram venom dose, they
are affected more commonly by the systemic syndrome, known as loxoscelism.
Black widow spider bites also are painless and leave little more than two small red, pinpoint
marks on the skin. The venom causes severe muscular cramping, tremors, and autonomic
symptoms such as drooling and sweating. The symptoms seen after black widow spider bites
frequently are mistaken for a variety of other conditions, including cholinergic crisis and acute
abdominal processes.
Treatment of brown recluse spider bites, black widow spider bites, and bites of all other
spiders indigenous to North America is largely supportive and includes conscientious wound
care and tetanus immunization, if indicated. The muscular cramping caused by a black widow
spider bite is controlled readily with benzodiazepines, narcotics, and calcium gluconate. An
antivenom is available, although its use is indicated only when autonomic symptoms or pain
cannot be managed with usual measures. Skin grafting may be necessary if tissue necrosis
occurs following a brown recluse spider bite. Many therapeutic modalities have been advocated
to limit the extent of the brown recluse bite wound, including dapsone, early wound excision,
hyperbaric oxygen, and steroids, but none have proven effective. Loxosceles antivenom is not
available in the United States.
Skin findings caused by a wide variety of other pathologic conditions often are attributed to
brown recluse spider bites. The differential diagnosis is extensive and includes conditions such
as staphylococcal and streptococcal skin lesions, cutaneous anthrax, atypical mycobacterial
infection, sporotrichosis, ecthyma gangrenosum, herpes simplex and zoster, vasculitic lesions,
erythema chronicum migrans, and erythema nodosum. Because necrotic skin lesions can be
caused by a number of serious conditions, and documented spider bites are rare, clinicians
should consider alternative diagnoses when faced with a necrotic skin lesion, if the spider is not
available for inspection.
References:
Singletary EM, Rochman AS, Bodmer JCA, Holstege CP. Envenomations. Med Clin North Am.
2005;89:1195-1224. Abstract available at: http://www.ncbi.nlm.nih.gov/pubmed/16227060
Sjogren R, MacGregor RS, Zenel J. Visual diagnosis: an infant who has a red papule on a
swollen, tender arm. Pediatr Rev. 2004;25:182-185. Available at:
http://pedsinreview.aappublications.org/cgi/content/full/25/5/182
Swanson DL, Vetter RS. Bites of brown recluse spiders and suspected necrotic arachnidism. N

Copyright 2009 by the American Academy of Pediatrics

page 374

2009 PREP SA on CD-ROM

Engl J Med. 2005;352:700-707. Extract available at:


http://content.nejm.org/cgi/content/extract/352/7/700

Copyright 2009 by the American Academy of Pediatrics

page 375

2009 PREP SA on CD-ROM


Question: 177

A 12-year-old boy has had intermittent heartburn for the past several years. Results of an upper
gastrointestinal radiographic series performed at age 11 years were normal. Over the past year,
he has had several episodes of "food getting stuck in his chest." The most common foods that
cause him difficulty are hot dogs, steak tips, and chicken strips. Physical examination findings
are unremarkable. He has been treated with omeprazole for 3 months, but symptoms persist.
Of the following, the MOST appropriate next diagnostic test is

A. esophageal manometry study


B. esophagogastroduodenoscopy with biopsy
C. radionuclide gastric emptying scan
D. serum immunoglobulin E measurement
E. 24-hour esophageal pH probe study

Copyright 2009 by the American Academy of Pediatrics

page 376

2009 PREP SA on CD-ROM


Critique: 177

Preferred Response: B

The chronic heartburn and dysphagia to solid foods, primarily meats, described for the boy in
the vignette strongly suggest either reflux esophagitis (possibly with an esophageal stricture) or
eosinophilic esophagitis (EE) (Item C177A). The most appropriate next test to evaluate the
dysphagia is esophagogastroduodenoscopy with biopsies. The endoscopy can characterize the
severity of inflammation, exclude Barrett esophagus (a premalignant complication of
gastroesophageal reflux [GER]), and potentially differentiate between GER and EE. Esophageal
manometry, radionuclide gastric emptying scan, measurement of serum immunoglobulin E, and
24-hour esophageal pH probe study are potentially useful in the evaluation of GER or allergy, but
they are not the appropriate initial tests in evaluating solid food dysphagia.
GER is defined as excessive regurgitation of gastric contents (both acid and nonacid) into
the esophagus. The most common gastrointestinal symptoms are heartburn (pyrosis),
dysphagia (difficulty swallowing), and odynophagia (painful swallowing). GER also may result in
extraintestinal symptoms, including hoarseness, chronic cough, exacerbation of asthma, apnea,
and aspiration pneumonia. GER is extremely common in childhood. According to one crosssectional study, 3% to 5% of healthy children report reflux symptoms at any given time.
Because GER frequently is mild and self-limited, empiric therapy with an antacid, histamine
receptor blocker, or proton pump inhibitor is appropriate and can alleviate symptoms. Prolonged,
persistent reflux (ie, more than 3 months) or complicated reflux (ie, associated with
hematemesis or respiratory symptoms) warrants further evaluation. Among the useful tests to
evaluate GER are upper gastrointestinal radiographic series (which excludes malrotation and
hiatal hernia), pH probe study or esophageal impedance (which determine the extent and
duration of reflux over a 24-hour period), gastric emptying scan (which rules out gastroparesis),
and esophageal motility evaluation (which assesses peristalsis and esophageal sphincter
pressure). Upper endoscopy with biopsy generally is the most valuable test because it can
determine if chronic GER has caused any esophageal injury (Item C177B).
In the past 10 years, researchers have recognized that not all heartburn or dysphagia is
caused by GER. EE presents similarly to reflux, but is believed to be an atopic condition.
Affected patients tend to have more dysphagia symptoms than heartburn. Often, there is a
history of meat impaction in the esophagus. As noted previously, upper endoscopy with biopsies
frequently can differentiate between the two conditions. GER usually is characterized by
ulceration and injury limited to the distal 5 cm of the esophagus. In contrast, EE is characterized
by inflammation of the proximal, middle, and distal esophagus, with large numbers of eosinophils
seen on biopsy. Differentiating between these two conditions is important because treatment of
GER involves acid blockade. In contrast, the therapies for EE include anti-inflammatory therapy
(eg, fluticasone, budesonide), acid blockade, or elimination diets.
References:
Furuta GT, Straumann A. Review article: the pathogenesis and management of eosinophilic
oesophagitis. Aliment Pharmacol Ther. 2006;24:173-182. Available at: http://www.blackwellsynergy.com/doi/full/10.1111/j.1365-2036.2006.02984.x
Kumar Y, Sarvananthan R. Gastro-oesophageal reflux in children. BMJ Clinical Evidence. 2007.
Available for subscription at: http://clinicalevidence.bmj.com/ceweb/conditions/chd/0310/0310.jsp
Nelson SP, Chen EH, Syniar GM, Kaufer Christoffer K; for the Pediatric Practice Research
Group. Prevalence of symptoms of gastroesophageal reflux during childhood. Arch Pediatr
Adolesc Med. 2000;154:150-154.

Copyright 2009 by the American Academy of Pediatrics

page 377

2009 PREP SA on CD-ROM


Question: 178

You are seeing a 12-month-old infant who was born at 26 weeks gestation. He receives daily
diuretics and nasal cannula oxygen with a baseline flow of 0.1 L/min, but his mother called this
morning reporting that he had a temperature of 100.5F (37.8C), nasal congestion, increased
work of breathing with a rapid respiratory rate, and a "wheezing" cough. You instructed her to
increase the oxygen flow rate to 0.5 L/min and come directly to the clinic. Physical examination
reveals intercostal and subcostal retractions, a respiratory rate of 80 breaths/min, and a
prolonged expiratory phase with audible wheezing. A copious, cloudy, green nasal discharge is
present. No heart murmur is audible. Pulse oximetry while receiving 0.5 L/min oxygen reveals an
oxygen saturation of 85% at rest.
Of the following, the BEST explanation for this childs presenting signs of respiratory distress is

A. acute cor pulmonale


B. foreign body aspiration
C. gastroesophageal reflux
D. lower respiratory tract infection
E. upper respiratory tract infection

Copyright 2009 by the American Academy of Pediatrics

page 378

2009 PREP SA on CD-ROM


Critique: 178

Preferred Response: D

The infant described in the vignette has symptoms of a lower respiratory tract infection. The
presence of nasal discharge suggests a concomitant or preceding upper respiratory tract
infection, but an upper respiratory tract infection alone would not result in tachypnea, retractions,
or hypoxemia. It is unlikely that the infant has aspirated a foreign body because pulmonary
findings are bilateral. There is no history of emesis or prolonged concerns for gastroesophageal
reflux. Although chronic hypoxemia may contribute to cor pulmonale, this is not an acute
condition and is inconsistent with the findings described in the vignette.
Chronic lung disease of prematurity (CLD), also referred to as bronchopulmonary dysplasia,
remains a continuing concern for graduates of the neonatal intensive care unit (NICU) who have
required intubation, assisted ventilation, and oxygen therapy for extended periods of time.
Primarily affecting very low-birthweight (VLBW) infants (<1,500 g), CLD is characterized by a
sustained oxygen requirement beyond 36 weeks' gestation corrected age; obstructive small
airways disease that may be reversible with bronchodilators; and inflammatory characteristics
that include increased airway secretions, smooth muscle bronchoreactivity, and increased
interstitial lung water. The infant who has CLD generally has an increased work of breathing,
with an associated increased caloric expenditure, and a decreased pulmonary reserve capacity.
Hence, he or she has little ability to meet increased metabolic and pulmonary work associated
with exertion (eg, physical exercise) or increased respiratory work (associated with lower
respiratory tract infection). As a result, up to 50% of VLBW infants who have CLD are
rehospitalized for pulmonary decompensation in the first year after discharge from the NICU.
The infant who has CLD requires increased calories and protein to grow compared with
unaffected age- and birthweight-matched preterm infants. Good nutrition can help the at-risk
growing preterm infant who has CLD in attaining good overall physical growth, pulmonary
development, and neurodevelopment as well as optimizing immune function.
References:
American Academy of Pediatrics Committee on Fetus and Newborn. Postnatal corticosteroids to
treat or prevent chronic lung disease in preterm infants. Pediatrics. 2002;109;330-338. Available
at: http://pediatrics.aappublications.org/cgi/content/full/109/2/330
Bancalari EH. Bronchopulmonary dysplasia and neonatal chronic lung disease. In: Martin RJ,
Fanaroff AA, Walsh MC, eds. Fanaroff and Martin's Neonatal-Perinatal Medicine. 8th ed.
Philadelphia, Pa: Mosby Elsevier; 2006:1155-1167
Baraldi E, Filippone M. Chronic lung disease after premature birth. N Engl J Med. 2007;357:19461955. Extract available at: http://content.nejm.org/cgi/content/extract/357/19/1946
Bhandari A, Panitch HB. Pulmonary outcomes in bronchopulmonary dysplasia. Semin Perinatol.
2006;30:219-226. Abstract available at: http://www.ncbi.nlm.nih.gov/pubmed/16860162
Biniwale MA, Ehrenkranz RA. The role of nutrition in the prevention and management of
bronchopulmonary dysplasia. Semin Perinatol. 2006;30:200-208. Abstract available at:
http://www.ncbi.nlm.nih.gov/pubmed/16860160
Shaw NJ, Kotecha S. Management of infants with chronic lung disease of prematurity in the
United Kingdom. Early Hum Dev. 2005;81:165-170. Abstract available at:
http://www.ncbi.nlm.nih.gov/pubmed/15748971
Vaucher YE. Bronchopulmonary dysplasia: an enduring challenge. Pediatr Rev. 2002;23:349358. Available at: http://pedsinreview.aappublications.org/cgi/content/full/23/10/349

Copyright 2009 by the American Academy of Pediatrics

page 379

2009 PREP SA on CD-ROM


Question: 179

You are evaluating a 10-month-old boy brought to the emergency department because of
fussiness for 1 day. His mother reports that she was carrying him while answering the phone
yesterday and that he fell from her arms onto the linoleum floor. Physical examination reveals a
thin boy who is crying. He resists weight-bearing on the left leg, but you cannot elicit specific
tenderness. He has bruises on the left temporal region, upper arm, and thighs. You suspect
nonaccidental trauma and order a skeletal survey.
Of the following, the skeletal survey finding that is MOST specific for nonaccidental trauma is

A. linear nondisplaced skull fracture


B. long bone nondisplaced shaft fracture
C. metaphyseal chip fracture
D. spiral tibial fracture
E. subperiosteal new bone formation

Copyright 2009 by the American Academy of Pediatrics

page 380

2009 PREP SA on CD-ROM


Critique: 179

Preferred Response: C

Whenever child abuse is suspected, it is critical for the clinician to evaluate the child fully for any
and all potential manifestations of such abuse. The evaluation begins with a thorough trauma
history and social history, paying close attention to the child-parent interaction, and a complete
physical examination. When bruises or other lesions are present, or the history is inconsistent
with the mechanism of action for the lesions, child abuse should be strongly suspected, and
further evaluation is required. Such evaluation includes a complete skeletal survey to look for
fractures.
A skeletal survey consists of radiographs of the skull, lateral cervical spine, ribs, pelvis,
thoracolumbar spine, arms, hands, legs, and feet. Anteroposterior and lateral views should be
obtained to examine all aspects of the bones fully. A skeletal survey can provide information
about the age of a fracture, although some fractures, such as those of the ribs, may not be
recognized early. A repeat survey 2 weeks later can be helpful to identify healing fractures that
exhibit new bone formation.
Certain types of fractures have a high specificity for nonaccidental trauma. Among the most
specific are metaphyseal chip fractures (Item C179A) and metaphyseal bucket handle fractures
(Item C179B), both of which are seen in children younger than 1 year of age. Other concerning
findings include fractures of the posterior ribs (Item C179C), scapula, spinous process, and
sternum as well as multiple fractures, fractures of different ages, femur fractures in children
younger than 1 year of age (Item C179D), and complex skull fractures.
Common skeletal survey findings that have a low specificity for child abuse include simple
linear skull fractures, nondisplaced long bone fractures, and subperiosteal new bone formation.
A nondisplaced spiral fracture of the tibia, also known as a "toddler's fracture" (Item C179E), is
the result of a twisting motion of the tibia and can occur in young children who have just learned
to walk. Even in these types of fractures, however, it is important to obtain a complete trauma
history. If the history is inconsistent with the injury, a full evaluation for nonaccidental trauma is
indicated. Further evaluation includes imaging of the head (either computed tomography scan or
magnetic resonance imaging) to look for hematomas, ophthalmologic examination to look for
retinal hemorrhages (Item C179F), and urinalysis and liver and pancreatic enzyme
measurement to look for evidence of blunt abdominal trauma. Any suspicious findings warrant
mandatory reporting to child protective services.
References:
Jenny C; Committee on Child Abuse and Neglect. Evaluating infants and young children with
multiple fractures. Pediatrics. 2006;118:1299-1303. Available at:
http://pediatrics.aappublications.org/cgi/content/full/118/3/1299
Kellogg ND and the Committee on Child Abuse and Neglect. Evaluation of suspected child
physical abuse. Pediatrics. 2007;119:1232-1241. Available at:
http://pediatrics.aappublications.org/cgi/content/full/119/6/1232
Sirotnak AP, Grigsby T, Krugman RD. Physical abuse of children. Pediatr Rev. 2004;25:264277. Available at: http://pedsinreview.aappublications.org/cgi/content/full/25/8/264

Copyright 2009 by the American Academy of Pediatrics

page 381

2009 PREP SA on CD-ROM


Question: 180

A medical student notes on rounds that a 2-year-old girl admitted for pneumonia has a complete
blood count (CBC) that includes a hematocrit of 35% (0.35), hemoglobin of 11.5 g/dL (115.0 g/L),
mean corpuscular volume of 68.0 fL, and platelet and white blood cell counts that are normal for
age. During the bedside encounter with the childs mother, you advise her to start the child on a
multivitamin with iron and have her primary care physician obtain another CBC in a month or so.
The medical student asks why you recommended iron supplementation when the child has a
normal hematocrit.
Of the following, the BEST reason to prescribe supplemental iron therapy for this child at this
time is to prevent

A. diminished cognitive abilities


B. fatigue
C. rapid progression to anemia
D. recurrent infections
E. short stature

Copyright 2009 by the American Academy of Pediatrics

page 382

2009 PREP SA on CD-ROM


Critique: 180

Preferred Response: A

The child described in the vignette likely has iron deficiency, as evidenced by her low mean
corpuscular volume. Providing iron supplementation may improve her cognition.
It is not clear whether effects on cognition and behavior caused by iron deficiency are
completely reversible with iron therapy. Of the two studies that treated children for 2 months or
longer, one reported dramatic benefits for development and the other did not. Evidence suggests
that children who had iron deficiency as toddlers may have slightly impaired cognition in
elementary school, even if they were treated with iron and the anemia resolved. Finally,
according to some evidence, iron deficiency may cause or be associated with symptoms of
attention-deficit/hyperactivity disorder that may be improved with iron therapy.
For many children, such as the girl described in the vignette, iron deficiency is revealed on a
complete blood count obtained because of a febrile illness. Fever may cause transient anemia
and microcytosis due to hemolysis, and repeat screening is recommended when the child
recovers. Routine iron supplementation with a multivitamin with iron should be prescribed until it
is determined whether iron therapy is required.
Iron deficiency has not been proven to cause fatigue in the absence of anemia, and there is
no definitive association between isolated iron deficiency or anemia and short stature. Iron
deficiency does not cause recurrent infection. Iron deficiency does not lead rapidly to anemia;
rather, anemia may develop over weeks to months.
References:
Konofal E, Lecendreux M, Arnulf I, Mouren MC. Iron deficiency in children with attentiondeficit/hyperactivity disorder. Arch Pediatr Adolesc Med. 2004;158:1113-1115.
Lozoff B, De Andraca I, Castillo M, Smith JB, Walter T, Pino P. Behavioral and developmental
effects of preventing iron-deficiency anemia in healthy full-term infants. Pediatrics. 2003;112:846854. Available at: http://pediatrics.aappublications.org/cgi/content/full/112/4/846
Lozoff B, Jimenez E, Smith JB. Double burden of iron deficiency in infancy and low
socioeconomic status: a longitudinal analysis of cognitive test scores to age 19 years. Arch
Pediatr Adolesc Med. 2006;160:1108-1113.
Martins S, Logan S, Gilbert R. Iron therapy for improving psychomotor development and
cognitive function in children under the age of three with iron deficiency anaemia. Cochrane
Database Syst Rev. 2001;2;CD001444. Available at:
http://www.mrw.interscience.wiley.com/cochrane/clsysrev/articles/CD001444/frame.html
Wu AC, Lesperance L, Bernstein H. Screening for iron deficiency. Pediatr Rev. 2002;23:171178. Available at: http://pedsinreview.aappublications.org/cgi/content/full/23/5/171

Copyright 2009 by the American Academy of Pediatrics

page 383

2009 PREP SA on CD-ROM


Question: 181

The youngest child in a family affected by neurofibromatosis 1, who is 5 years old, has just had
the diagnosis confirmed. You begin the process of counseling the family.
Of the following, the MOST accurate statement about potential medical complications in affected
children is that they

A. are at increased risk of developing cataracts


B. have an increased incidence of acoustic neuroma
C. have an increased incidence of childhood hypertension
D. should be restricted from contact sports
E. should be screened annually for optic gliomas using computed tomography scan

Copyright 2009 by the American Academy of Pediatrics

page 384

2009 PREP SA on CD-ROM


Critique: 181

Preferred Response: C

Neurofibromatosis (NF) is an autosomal dominant neurocutaneous disorder of which there are


at least two types. Type 1 has an incidence of approximately 1 in 3,500 people, and its
characteristic features are the cutaneous caf au lait macules and benign neurofibromas (Item
C181). Type 2 has an incidence of approximately 1 in 40,000, and a characteristic feature is
vestibular schwannoma. Individuals affected by NF-1 are at risk for the development of a
number of complications, including optic pathway tumors, skeletal deformations, speech
impediments, and learning disabilities. In addition, there is variable association with hypertension,
short stature, constipation, and chronic headache. Patients who have NF-2 are not at increased
risk for such complications.
The child described in the vignette is at risk for the development of hypertension, particularly
during ages 2 through 10 years. Although most patients have idiopathic hypertension, NF-1 can
be associated with renal artery stenosis and pheochromocytoma. This child's health supervision
visits should include a detailed history and accurate blood pressure measurements to ensure
early identification of this complication. The child is not at an increased risk for the development
of cataracts, and there is no reason to restrict him from contact sports if he has no other
complications. Because approximately 15% of individuals who have NF-1 develop optic pathway
tumors, periodic ophthalmologic evaluation is important. However, the potential high cumulative
radiation dose of annual computed tomography scanning likely outweighs the benefits; the
necessity and frequency of such imaging should be determined in consultation with the
ophthalmologist. Acoustic neuroma, now referred to as vestibular schwannoma, is seen in NF-2,
not NF-1.
References:
Hersh JH, Committee on Genetics. Health supervision for children with neurofibromatosis.
Pediatrics. 2008;121:633-642. Available at:
http://pediatrics.aappublications.org/cgi/content/full/121/3/633
Lama G, Graziano L, Calabrese E, et al. Blood pressure and cardiovascular involvement in
children with neurofibromatosis type 1. Pediatr Nephrol. 2004;19:413-418. Abstract available at:
http://www.ncbi.nlm.nih.gov/pubmed/14991390
Viskochil DH. Neurocutaneous disorders. In: Rudolph C, Rudolph A, eds. Rudolph's Pediatrics.
21st ed. New York, NY: McGraw Hill Medical Publishing Division; 2003:769-774

Copyright 2009 by the American Academy of Pediatrics

page 385

2009 PREP SA on CD-ROM


Question: 182

During the routine health supervision visit for a 4-month-old infant, you note low tone and poor
visual interaction. His head shape is symmetric and his head circumference is 36 cm (<2nd
percentile). He had been born at term with a head circumference of 32 cm (2nd percentile).
Of the following, the MOST helpful initial diagnostic test to explain the cause of the infants
abnormal examination findings is

A. brain magnetic resonance imaging


B. electroencephalography
C. head ultrasonography
D. high-resolution karyotyping
E. three-dimensional head computed tomography scan

Copyright 2009 by the American Academy of Pediatrics

page 386

2009 PREP SA on CD-ROM


Critique: 182

Preferred Response: A

The child described in the vignette had microcephaly at birth and has continued to develop below
the standard head growth curve, increasing by just 4 cm, instead of 7 cm, during the first 4
months. Such slow growth most likely is due to a primary brain problem rather than any skull
anomaly (Item C182). Microcephaly in the presence of abnormal neurodevelopment, as
described for this infant, indicates a need for a thorough evaluation to determine the neurologic
cause of symptoms. Magnetic resonance imaging (MRI) allows detailed assessment of all brain
structures, including the brainstem and gray and white matter integrity, features that can guide
additional evaluation.
Head circumference should be measured and plotted at birth and at all subsequent health
supervision visits, particularly for the first 3 postnatal years, because head growth normally
corresponds with rapid brain growth in early childhood. Suboptimal head growth during the first
year and microcephaly at 1 year of age are associated with subnormal developmental outcomes
in children who had hypoxic-ischemic injuries at term birth.
Head ultrasonography is a reasonable alternative to MRI because it does not require
sedation and, therefore, is easier on the child (and family). However, it has lower resolution for
peripheral gray matter and deep brainstem structures. This is a better choice for evaluation of a
large head (macrocephaly) because it can diagnose hydrocephalus accurately.
Electroencephalography findings are too nonspecific to guide diagnosis or treatment.
High-resolution karyotyping may be reasonable, particularly if the infant displays other
dysmorphic features or congenital brain structural anomalies, because chromosomal
abnormalities can result in microcephaly. However, this evaluation can be deferred until after
additional information about brain structure has been obtained. For example, if there is evidence
of a stroke, hypoxic-ischemic injury, or a congenital infection, karyotyping may not be needed.
Three-dimensional computed tomography scan is used to diagnose craniosynostosis.
Craniosynostosis usually presents with an unusual head shape due to closure of one or more
skull sutures. This infant's symmetric microcephaly plus an abnormal neurodevelopmental
history make a bony abnormality in the skull unlikely.
References:
Kinsman SL, Johnston MV. Congenital anomalies of the central nervous system. In: Kliegman
RM, Behrman RE, Jenson HB, Stanton BF, eds. Nelson Textbook of Pediatrics. 18th ed.
Philadelphia, Pa: Saunders Elsevier; 2007:2443-2456
Shevell M, Ashwal S, Donley D, et al. Practice parameter: evaluation of the child with global
developmental delay. Report of the Quality Standards Subcommittee of the American Academy
of Neurology and The Practice Committee of the Child Neurology Society. Neurology.
2003;60:367-380. Available at: http://www.neurology.org/cgi/content/full/60/3/367
Shevell M, Majnemer A, Platt RW, Webster R, Birnbaum R. Developmental and functional
outcomes in children with global developmental delay or developmental language impairment.
Dev Med Child Neurol. 2005;47:678-683. Available at: http://www.blackwellsynergy.com/doi/abs/10.1111/j.1469-8749.2005.tb01053.x

Copyright 2009 by the American Academy of Pediatrics

page 387

2009 PREP SA on CD-ROM


Question: 183

A 2-day-old infant is transferred from the regular nursery to the neonatal intensive care unit for
evaluation and management of poor feeding and lethargy. A serum metabolic panel reveals a
carbon dioxide concentration of 12.0 mEq/L (12.0 mmol/L) and a borderline low white blood cell
count. After stopping all feedings, a septic evaluation is performed, and intravenous antibiotics
are started. During this time, she becomes alert and vigorous, and her carbon dioxide value
normalizes. Three days later, results of the septic evaluation are negative, and the infant
resumes human milk feedings. Initially she does well, but after 2 days, she begins to vomit and
becomes less active. Serum metabolic panel shows a glucose concentration of 35.0 mg/dL (1.9
mmol/L), a carbon dioxide concentration of 8.0 mEq/L (8.0 mmol/L), and an anion gap of 25; the
serum ammonia value is twice the upper limit of normal. The baby is again made NPO and given
intravenous fluids.
Of the following, the MOST critical diagnostic test for this baby is

A. lysosomal enzyme analysis


B. peroxisomal enzyme analysis
C. serum galactose-1-phosphate measurement
D. serum toxicology screen
E. urine organic acids measurement

Copyright 2009 by the American Academy of Pediatrics

page 388

2009 PREP SA on CD-ROM


Critique: 183

Preferred Response: E

The finding of metabolic acidosis with an increased anion gap in an infant or child who is acutely
ill, such as the infant described in the vignette, is an important clue to the possible diagnosis of
an inborn error of metabolism. Even so, it is important to remember that any condition that
produces a metabolic acidosis, such as infection and necrosis, can present similarly. Of the
inborn metabolic errors, those characteristically associated with overwhelming metabolic
acidosis in the young infant are the organic acid disorders, especially propionic, methylmalonic,
and isovaleric acidemias. The infant in the vignette has propionic acidemia.
Evaluation of the infant or child who has acidosis and an elevated anion gap should include
measurement of plasma lactate, plasma pyruvate, complete blood and platelet counts, serum
ammonia, urine organic and amino acids, plasma amino acids, and total and free carnitine as
well as an acylcarnitine profile. Metabolic acidosis can interfere with normal urea cycle function,
leading to elevated serum ammonia concentrations; it also can depress bone marrow
production.
Lysosomal storage disorders, such as Hurler and Hunter syndromes, typically do not
present in the newborn period with metabolic imbalance; they become apparent in toddlers who
exhibit decreased acquisition of skills and progressive deformation.
Peroxisomal enzyme disorders that present in the newborn period, such as Zellweger
syndrome, are associated with altered sensorium, hypotonia, and hepatomegaly and are not
responsive to the withholding of feedings.
Serum galactose-1-phosphate concentrations are elevated in the most common form of
galactosemia. Galactosemia presents after days to weeks of human or cow milk feedings and is
associated with jaundice, hepatomegaly, and Escherichia coli sepsis.
A serum toxicology screen in this neonate would not be expected to reveal the source of her
illness, and the infant's presentation is not characteristic of maternal substance abuse.
References:
Burton BK. Inborn errors of metabolism in infancy: a guide to diagnosis. Pediatrics. 1998;102:e69e78. Available at: http://pediatrics.aappublications.org/cgi/content/full/102/6/e69
Enns GM. Inborn errors of metabolism masquerading as hypoxic-ischemic encephalopathy.
NeoReviews. 2005;6:e549-e558. Available for subscription at:
http://neoreviews.aappublications.org/cgi/content/full/6/12/e549
Hoffmann GF, Nyhan WL, Zschocke J, Kahler SG, Mayatepek E. Approach to the patient with
metabolic disease. In: Inherited Metabolic Diseases. Philadelphia, Pa: Lippincott Williams &
Wilkins; 2002:19-94
Nyhan WL, Barshop BA, Ozand PT. Organic acidemias. In: Atlas of Metabolic Diseases. 2nd ed.
London, England: Hodder Arnold; 2005:1-108

Copyright 2009 by the American Academy of Pediatrics

page 389

2009 PREP SA on CD-ROM


Question: 184

A 17-year-old young man comes to the clinic in the juvenile detention center with a penile
discharge. He has no other symptoms. He was tested 1 week ago at a sexually transmitted
infections clinic, and results of the rapid urine testing by nucleic acid amplification are positive for
Neisseria gonorrhoeae and negative for Chlamydia trachomatis.
Of the following, the MOST appropriate treatment regimen is

A. benzathine penicillin G 2.4 million units intramuscularly in a single dose


B. ceftriaxone 125 mg intramuscularly in a single dose
C. ciprofloxacin 500 mg orally in a single dose
D. ofloxacin 400 mg orally in a single dose
E. spectinomycin 2 g intramuscularly in a single dose

Copyright 2009 by the American Academy of Pediatrics

page 390

2009 PREP SA on CD-ROM


Critique: 184

Preferred Response: B

Gonorrhea is the second most commonly reported notifiable disease in the United States.
Although the Centers for Disease Control and Prevention (CDC) Sexually Transmitted Diseases
Treatment Guidelines, 2006, recommended treating uncomplicated gonococcal infections of the
cervix, urethra, and rectum with either ceftriaxone, cefixime, ciprofloxacin, ofloxacin, or
levofloxacin as single-dose therapy, in April 2007, the CDC issued an update to the guidelines,
indicating that fluoroquinolones no longer are recommended for the treatment of gonococcal
infections. Current recommendations for uncomplicated gonococcal infections of the cervix,
urethra, and rectum include ceftriaxone 125 mg intramuscularly or cefixime 400 mg orally, both
in a single dose, plus treatment for chlamydia if chlamydial infection has not been ruled out.
These recommendations apply to all adult and adolescent patients, regardless of travel history
or sexual behavior.
Alternative regimens include spectinomycin 2 g in a single intramuscular dose or parenteral
cephalosporin single-dose regimens, including ceftizoxime 500 mg intramuscularly, cefoxitin 2 g
intramuscularly with probenecid 1 g orally, or cefotaxime 500 mg intramuscularly. Spectinomycin
is recommended for persons who have penicillin or cephalosporin allergies. Benzathine penicillin
is not recommended due to increased resistance to this drug.
References:
Centers for Disease Control and Prevention. Update to CDC's sexually transmitted diseases
treatment guidelines, 2006: fluoroquinolones no longer recommended for treatment of
gonococcal infections. MMWR Morbid Mortal Wkly Rep. 2007;56:332-336. Available at:
http://www.cdc.gov/mmwr/preview/mmwrhtml/mm5614a3.htm
Workowski KA, Berman SM, Centers for Disease Control and Prevention. Sexually transmitted
diseases treatment guidelines, 2006. MMWR Recomm Rep. 2006;55(RR11):1-94. Available at:
http://www.cdc.gov/mmwr/preview/mmwrhtml/rr5511a1.htm

Copyright 2009 by the American Academy of Pediatrics

page 391

2009 PREP SA on CD-ROM


Question: 185

A 3-year-old boy is admitted to the hospital for fever, cough, and increasing respiratory
insufficiency of 2 days duration. Chest radiography demonstrates a right middle lobe and lower
lobe pneumonia with a significant pleural effusion (Item Q185). You aspirate a sample of pleural
fluid and send it to the laboratory for analysis.
Of the following, the MOST likely expected laboratory findings are

A. White Blood Cells: Moderate to high


Lactate Dehydrogenase (U/L): 2 to 3 times the serum concentration
Protein (g/dL): <3
Gram Stain: Gram-positive cocci

B. White Blood Cells: Moderate to high


Lactate Dehydrogenase (U/L): Equal to the serum concentration
Protein (g/dL): <3
Gram Stain: Negative

C. White Blood Cells: Moderate to high


Lactate Dehydrogenase (U/L): 2 to 3 times the serum concentration
Protein (g/dL): >3
Gram Stain: Gram-positive cocci

D. White Blood Cells: Rare


Lactate Dehydrogenase (U/L): Equal to the serum concentration
Protein (g/dL): >3
Gram Stain: Negative

E. White Blood Cells: Rare


Lactate Dehydrogenase (U/L): Equal to the serum concentration
Protein (g/dL): <3
Gram Stain: Gram-positive cocci

Copyright 2009 by the American Academy of Pediatrics

page 392

2009 PREP SA on CD-ROM


Critique: 185

Preferred Response: C

The child described in the vignette has a pleural effusion and lobar pneumonia. Pleural effusions
in pediatric patients have a variety of causes, including pneumonia, trauma, malignancy, renal
disease, liver failure, and congestive heart disease. Parapneumonic effusions (those associated
with an underlying pneumonia) account for up to 70% of pediatric pleural effusions. Pleural
effusions may be seen in up to 40% of bacterial pneumonias; progression to pus (empyema)
occurs in more than 50% of these cases and is increasing in both incidence and associated
complications.
Classification of pleural fluid into transudates and exudates may assist the clinician in
determining causes and guiding treatment (Item C185). Transudates are plasma ultrafiltrates
and usually result from renal and liver disease or congestive heart failure. Exudates arise from
inflammatory processes (such as pneumonia, malignancies, trauma, or systemic inflammatory
diseases) or impaired lymphatic drainage, often following thoracic surgery (eg, chylothorax).
Empyema, a type of exudate, is characterized by the presence of white blood cells, a positive
Gram stain, or frank pus.
The child described in the vignette most likely has an exudate based on the lactate
dehydrogenase and protein values. A Gram stain may reveal bacteria (in this case, a grampositive cocci), although bacterial culture results may be negative, especially if the patient has
received prior antibiotic therapy. The pH value of the fluid should be low due to consumption of
glucose by the bacteria, and there should be moderate-to-high white blood cell counts,
consistent with an empyema. Lactate dehydrogenase values often are two to three times the
serum concentrations, and protein values frequently are 3.0 g/dL (30.0 g/L) or greater in pleural
effusions. Tube thoracostomy, potentially aided by video-assisted thoracic surgery, and
antibiotics are the recommended treatment.
References:
Efrati O, Barak A. Pleural effusions in the pediatric population. Pediatr Rev. 2002:23:417-426.
Available at: http://pedsinreview.aappublications.org/cgi/content/full/23/12/417
Schultz KD, Fan LL, Pinksy J, et al. The changing face of pleural empyemas in children:
epidemiology and management. Pediatrics. 2004;113:1735-1740. Available at:
http://pediatrics.aappublications.org/cgi/content/full/113/6/1735
Winnie GB. Pleurisy, pleural effusions, and empyema. In: Kliegman RM, Behrman RE, Jenson
HB, Stanton BF, eds. Nelson Textbook of Pediatrics. 18th ed. Philadelphia, Pa: Saunders
Elsevier; 2007:1832-1834

Copyright 2009 by the American Academy of Pediatrics

page 393

2009 PREP SA on CD-ROM


Question: 186

You are seeing a short 9-year-old boy. He is growing steadily in height just below the third
percentile on the growth curve. His parents ask if he will be very short when he finishes growing.
Of the following, the MOST important information needed to answer this question is

A. ethnicity of family
B. parent heights
C. sibling heights
D. upper-to-lower segment ratio
E. weight-for-age curve

Copyright 2009 by the American Academy of Pediatrics

page 394

2009 PREP SA on CD-ROM


Critique: 186

Preferred Response: B

Short stature is a statistical diagnosis representing a stature less than the third percentile for
height (3% of all children). Most children growing along lower percentiles, such as the boy
described in the vignette, are not growing slowly by the time they reach middle childhood; they
dropped to this percentile some time in the first 2 years after birth and now have a normal growth
rate. Most either have short parents or will advance through puberty slowly (constitutional delay)
and end up taller than anticipated by their childhood position on the growth curve. Accordingly,
parent heights are the most pertinent facts needed to answer the question about this child's adult
height.
Family ethnicity is only generally useful in evaluating short stature; within-family heights are
much more important for the individual child. Sibling height, unless the siblings have reached full
adult height, are not as useful as adult parental height because the sibling adult heights still are
unknown. Children who have particularly short legs or short spines may have an underlying
chondrodystrophy or skeletal dysplasia. Measurement of upper-to-lower segment ratio can be a
useful clinical examination when considering these conditions, although they are very rare.
Children who are particularly underweight for height or very overweight for height may grow
poorly. However, most overweight children grow exceedingly well because of their
overnourished status. A very malnourished child may have a decreased growth rate. The
steady growth in the child in the vignette suggests that nutritional status is not the most important
determinant of his present height.
Children who have normal growth rates are unlikely to have underlying endocrine or
metabolic disorders or serious underlying illnesses to explain their stature. In contrast, children
who have growth attenuation are much more likely to have underlying and perhaps specifically
remediable causes for short stature.
References:
Ferry RJ Jr. Short stature. eMedicine Specialties, Pediatrics: General Medicine, Endocrinology.
2007. Available at: www.emedicine.com/ped/topic2087.htm
Plotnick LP, Miller RS. Growth, growth hormone, and pituitary disorders. In: McMillan JA, Feigin
RD, DeAngelis C, Jones MD Jr. Oski's Pediatrics, Principles & Practice. Philadelphia, Pa:
Lippincott, Williams & Wilkins; 2006:2084-2092
Rogol AD. Causes of short stature. UpToDate Online 15.3. 2008. Available for subscription at:
http://www.uptodateonline.com/utd/content/topic.do?topicKey=pediendo/2279

Copyright 2009 by the American Academy of Pediatrics

page 395

2009 PREP SA on CD-ROM


Question: 187

A 4-year-old boy cannot attend a local nursery school because he is not toilet trained. His
development is otherwise normal. His parents explain that when they attempt to put him on the
toilet, he refuses and runs out of the bathroom. They ask how they can train him to use the toilet.
Of the following, the MOST appropriate approach is to

A. develop a behavioral modification program to encourage him to use the toilet


B. have the parents gently scold him when he has accidents
C. insist he sit on the toilet every 2 hours during the day
D. recommend the family find a different nursery school that allows children who are not toilet
trained

E. tell the parents to have him clean his own clothes after toilet accidents

Copyright 2009 by the American Academy of Pediatrics

page 396

2009 PREP SA on CD-ROM


Critique: 187

Preferred Response: A

In the United States, 98% of children are continent during the day by the time they are 36
months old. Toilet training usually requires about 3 to 6 months for successful completion. For
children such as the boy described in the vignette, who appears to be late in achieving toilet
training, a behavioral modification program can be established to encourage use of the toilet. It is
best to have the child take responsibility for being toilet trained, and a reward system using a
star chart to earn a desired object may help him to meet this goal. Punishing him for not using
the toilet or insisting that he clean his own clothes could lead to noncompliance and adversely
affect his self-esteem. Insisting a child sit on the potty chair or forcing him or her to sit may
increase the child's resistance. Often, a parent pushing a child to become toilet trained due to a
preschool requirement leads to a power struggle, and the child does not achieve continence.
Recommending that the parents find a different nursery school that does not require a child to be
toilet trained may alleviate some of their stress, but it does not address how to toilet train a child.
Toilet training begins with the parent encouraging the child to practice running to the potty
chair. The parent should praise or reward the child for complying with the practice session. The
child should be changed after any accidents, and the parents should avoid use of physical or
verbal punishment. The use of underwear, time-in (provide the child with positive reinforcement),
and incentives may help to increase a child's motivation.
References:
Parker S. Toilet training. In: Parker S, Zuckerman B, Augustyn M, eds. Developmental and
Behavioral Pediatrics: A Handbook for Primary Care. 2nd ed. Philadelphia, Pa: Lippincott
Williams & Wilkins; 2005:355-357
Schmitt B. Toilet training: getting it right the first time. Contemp Pediatr. 2004;21:105
Wolraich ML, Tippins S, ed. Guide to Toilet Training. Elk Grove Village, Ill: American Academy of
Pediatrics; 2003

Copyright 2009 by the American Academy of Pediatrics

page 397

2009 PREP SA on CD-ROM


Question: 188

The nurse caring for a 5-day-old infant you have hospitalized calls your office to report that the
infants blood culture is growing gram-positive rods. You admitted the infant to the hospital
because of a rectal temperature of 102.0F (38.9C) measured by his mother at home.
Of the following, the MOST likely pathogen is

A. Enterococcus sp
B. Escherichia coli
C. Listeria monocytogenes
D. Proteus mirabilis
E. Staphylococcus epidermidis

Copyright 2009 by the American Academy of Pediatrics

page 398

2009 PREP SA on CD-ROM


Critique: 188

Preferred Response: C

Listeria monocytogenes is a small, motile, gram-positive rod. Three major serotypes of Listeria
infect humans: 1a, 1b, and 4b. Infections in the newborn result from asymptomatic fecal or
vaginal carriage in pregnant women, with the organism colonizing the baby during passage
through the birth canal. Although bacterial infections due to group B Streptococcus and gramnegative organisms such as Escherichia coli or Klebsiella pneumoniae are more common,
neonatal sepsis and meningitis due to Listeria represent a major risk to the neonate, especially in
certain geographic regions of the United States (eg, the southwestern area).
Illness with Listeria can present as early-onset disease (from birth to 7 days) or late-onset
disease (>7 days after birth). The typical characteristics of early-onset infections are bacteremia
or pneumonia and preterm birth; late-onset disease usually results in meningitis. For older
infants and children, listerial infection is associated with ingestion of contaminated foods;
unpasteurized milk, soft cheese, and prepared meats (eg, hot dogs, deli meats) are the sources
of many infections. Most of these infections present with fever, malaise, headache,
gastrointestinal tract symptoms, or back pain. Bacteremia and meningitis also can occur.
Proteus mirabilis and E coli are gram-negative rods, and Enterococcus sp and Staphylococcus
epidermidis are gram-positive cocci.
References:
American Academy of Pediatrics. Listeria monocytogenes infections (listeriosis). In: Pickering
LK, Baker CJ, Long SS, McMillan JA, eds. Red Book: 2006 Report of the Committee on
Infectious Diseases. 27th ed. Elk Grove Village, Ill: American Academy of Pediatrics; 2006:426428
Posfay-Barbe KM, Wald ER. Listeriosis. Pediatr Rev. 2004;25:151-159. Available at:
http://pedsinreview.aappublications.org/cgi/content/full/25/5/151

Copyright 2009 by the American Academy of Pediatrics

page 399

2009 PREP SA on CD-ROM


Question: 189

A mother calls you to report that her 7-year-old son came home with a notice from school stating
that a child in his class was diagnosed with mumps. The mother does not know the immunization
status of the infected child but states that her son has received two measles-mumps-rubella
(MMR) vaccines and is up to date on all his other immunizations. Her son has been
asymptomatic, with no fever or other systemic complaints.
Of the following, the MOST appropriate action is to

A. administer a dose of mumps immune globulin to her son


B. confirm that her son has received two doses of MMR vaccine
C. keep her son home from school for 9 days to observe for the development of symptoms
D. treat her son with a course of ribavirin
E. vaccinate her son immediately with another dose of MMR to prevent infection from this
exposure

Copyright 2009 by the American Academy of Pediatrics

page 400

2009 PREP SA on CD-ROM


Critique: 189

Preferred Response: B

In the United States, mumps vaccine is administered as part of the measles-mumps-rubella


(MMR) vaccine routinely to children at 12 to 15 months of age, with a second dose of MMR
typically administered at 4 to 6 years of age. Protective efficacy of the vaccine is estimated to be
more than 95%. In cases of exposure, such as described in the vignette, it is important to
ensure that the exposed person has received the recommended number of doses of MMR
vaccine because mumps outbreaks have occurred in people in highly immunized populations
who previously have received only a single dose of mumps-containing vaccine. Therefore, the
most appropriate action is to confirm with the mother that her son has received two doses of
MMR vaccine.
Mumps vaccine has not been demonstrated to be effective in preventing infection after
exposure. However, the vaccine can be administered after exposure to provide protection
against subsequent exposures in persons who are not fully vaccinated. Fully immunized
persons do not need to be excluded from school after exposure. Students who are not fully
immunized are excluded from school until they are immunized, after which they can be
readmitted immediately to school. Students who refuse mumps vaccination because of medical,
religious, or other reasons should be excluded from school for at least 26 days after the onset of
parotitis in the last person who has mumps in the affected school. Persons who have mumps
are excluded from school for 9 days from the onset of their parotid swelling.
Immune globulin (IG) and mumps IG are not effective as postexposure prophylaxis
measures. In fact, mumps IG no longer is available in the United States. Treatment of the
disease is supportive; no effective antiviral agents are available.
References:
American Academy of Pediatrics. Mumps. In: Pickering LK, Baker CJ, Long SS, McMillan JA,
eds. Red Book: 2006 Report of the Committee on Infectious Diseases. 27th ed. Elk Grove
Village, Ill: American Academy of Pediatrics; 2006:464-468
Centers for Disease Control and Prevention (CDC). Notice to readers: updated
recommendations of the Advisory Committee on Immunization Practices (ACIP) for the control
and elimination of mumps. MMWR Morb Mortal Wkly Rep. 2006;55:629-630. Available at:
http://www.cdc.gov/mmwr/preview/mmwrhtml/mm55e601a1.htm

Copyright 2009 by the American Academy of Pediatrics

page 401

2009 PREP SA on CD-ROM


Question: 190

A 4-year-old boy presents with periorbital edema. He is receiving no medications, and his family
history is negative for renal disease. On physical examination, he is afebrile; his heart rate is 88
beats/min, respiratory rate is 18 breaths/min, and blood pressure is 106/62 mm Hg; and he has
periorbital (Item Q190A) and pitting pretibial edema (Item Q190B). Laboratory evaluation shows
normal electrolyte values, blood urea nitrogen of 14.0 mg/dL (5.0 mmol/L), creatinine of 0.3
mg/dL (26.5 mcmol/L), and albumin of 1.6 g/dL (16.0 g/L). Urinalysis demonstrates a specific
gravity of 1.020; pH of 6.5; 3+ protein; and negative blood, leukocyte esterase, and nitrite.
Microscopy results are normal. Additionally, complement component (C3 and C4) values are
normal, and results of serologic testing for antinuclear antibody, hepatitis B and C, and human
immunodeficiency virus are negative.
Of the following, you are MOST likely to advise the parents that

A. a renal biopsy is warranted to determine the optimal treatment


B. disease relapse can be expected in fewer than 25% of those achieving remission
C. patients who relapse have a similar prognosis as those who do not respond to steroids
D. remission is expected in more than 75% of patients who receive corticosteroid treatment
E. tacrolimus is the preferred treatment for patients who do not respond to corticosteroids

Copyright 2009 by the American Academy of Pediatrics

page 402

2009 PREP SA on CD-ROM


Critique: 190

Preferred Response: D

The boy described in the vignette has signs and symptoms of nephrotic syndrome. Causes of
nephrotic syndrome can be subdivided into primary glomerulopathies (eg, minimal change
disease, focal segmental glomerulosclerosis, mesangial proliferative glomerulonephritis) and
secondary glomerulopathies (eg, lupus nephritis, membranoproliferative glomerulonephritis due
to hepatitis C, and human immunodeficiency virus nephropathy).
The initial evaluation of new-onset nephrotic syndrome centers on establishing whether it is
due to a primary glomerulopathy and warrants an empiric trial of corticosteroids. Clinical features
that suggest secondary glomerulopathy include moderate-to-severe hypertension or gross
hematuria. Laboratory markers suggestive of a secondary glomerulopathy include azotemia,
hypocomplementemia, a positive antinuclear antibody finding, or a positive serologic test for
human immunodeficiency virus or hepatitis B or C.
The normal blood pressure and renal function as well as the absence of hematuria,
hypocomplementemia, or a positive serologic test for lupus or viral infection described for the
boy in the vignette are consistent with primary glomerulopathy. Accordingly, this child should
receive a 4- to 6-week trial of daily corticosteroids because nearly 90% of childhood nephrotic
syndrome cases are sensitive to steroids. Although 90% of pediatric patients achieve a state of
remission with corticosteroids, 50% to 70% experience at least one relapse during their disease
course.
Unless clinical or serologic findings raise the possibility of an underlying glomerulonephritis,
renal biopsy is not warranted. The steroid response, not renal biopsy findings, is the best
predictor of the prognosis for childhood nephrotic syndrome. Steroid-sensitive patients, even
those who have a relapsing course, have a better prognosis than steroid-resistant patients. The
approximately 10% of patients who are steroid-resistant have a guarded prognosis, with a risk
of developing chronic renal failure. Steroid-resistant patients require a renal biopsy to establish
histologic information.
In most circumstances, the first-line medication for steroid-resistant nephrotic syndrome is
cyclosporine. More recently, another calcineurin inhibitor, tacrolimus, has been used in children
who fail to respond to cyclosporine. At present, tacrolimus is considered a second- or third-line
treatment for steroid-resistant nephrotic syndrome.
References:
Niaudet P. Steroid-sensitive idiopathic nephrotic syndrome in children. In: Avner ED, Harmon
WE, Niaudet P, eds. Pediatric Nephrology. 5th ed. Philadelphia, Pa: Lippincott Williams & Wilkins;
2004:543-556
Niaudet P. Steroid-resistant idiopathic nephrotic syndrome in children. In: Avner ED, Harmon
WE, Niaudet P, eds. Pediatric Nephrology. 5th ed. Philadelphia, Pa: Lippincott Williams & Wilkins;
2004:557-573
Valentini RP, Smoyer WE. Nephrotic syndrome. In: Kher KK, Schnaper HW, Makker SP, eds.
Clinical Pediatric Nephrology. 2nd ed. London, England: Informa Healthcare; 2007:155-194

Copyright 2009 by the American Academy of Pediatrics

page 403

2009 PREP SA on CD-ROM


Question: 191

You are evaluating a 14-year-old girl for seasonal allergic rhinitis. Despite a regimen of multiple
allergy medications, she continues to have significant sneezing, rhinorrhea, and nasal
congestion. You decide to evaluate for possible allergic triggers and discuss the advantages and
disadvantages of allergy skin testing and blood testing.
Of the following, a TRUE statement regarding allergy skin and blood testing is that

A. infants younger than 1 year of age cannot undergo skin testing


B. patients may experience anaphylaxis during aeroallergen or food skin testing
C. patients need to fast prior to blood allergy testing
D. patients need to stop their antihistamines prior to blood allergy testing
E. the negative predictive value of aeroallergen skin testing is poor

Copyright 2009 by the American Academy of Pediatrics

page 404

2009 PREP SA on CD-ROM


Critique: 191

Preferred Response: B

Two primary diagnostic tools are used to determine the role of indoor and outdoor aeroallergens
as triggers for allergic rhinitis or allergic asthma: skin testing and blood testing. Aeroallergen skin
testing involves the application of specific allergens (eg, oak, Bermuda grass, cat, ragweed) on
the skin, typically using a prick or puncture method. Although sometimes uncomfortable for
infants and toddlers, allergy skin testing is tolerated extremely well by most children and
adolescents and can be performed at any age. The advantages of skin testing are that a broad
array of allergens can be tested, testing materials are inexpensive, and results are immediately
evident to the patient. One disadvantage is that patients must stop their antihistamine
medication(s) 1 week prior to skin testing. Also, although most patients tolerate the local pruritus
experienced at "positive" skin test sites, those who are very sensitive (eg, severe food
anaphylaxis) may experience a systemic reaction with even a simple skin test. For patients who
have a history of severe anaphylaxis to a specific allergen, allergists may choose to perform
serum immunoglobulin (Ig) E testing instead of skin testing because blood testing does not have
a risk for anaphylaxis.
In the past, serum IgE testing employed primarily the radioallergosorbent test (RAST)
method. Because of the significant variability in results between laboratories, RAST has been
replaced in most institutions with the more sensitive and reproducible CAP-system fluorescein
enzyme immunoassay. This system uses a cellulose matrix system. The advantage of serum
IgE testing is that it is not affected by medications (ie, patients do not need to stop an
antihistamine). Patients do not need to fast prior to either allergy skin or blood testing.
While ongoing studies are comparing the sensitivity and specificity of skin testing compared
with the CAP system fluorescein enzyme immunoassay, skin testing is regarded as more
sensitive and specific. Finally, although skin testing is considered "inexpensive," most general
pediatricians find the cost of an allergy consultation with skin testing to be more expensive than
a routine battery of serum IgE tests for aeroallergens or food. The availability and clinical
application of serum IgE testing continues to expand, but clinicians who do not seek allergy
consultation should be comfortable with interpretation and application of test results for a specific
clinical scenario (eg, a wheat IgE of 10 kU/L in a patient who has atopic dermatitis has little to no
clinical significance).
References:
Cartwright RC, Dolen WK. Consultation with the specialist: who needs allergy testing and how to
get it done. Pediatr Rev. 2006;27:140-146. Available at:
http://pedsinreview.aappublications.org/cgi/content/full/27/4/140
Mahr TA, Sheth K. Update on allergic rhinitis. Pediatr Rev. 2005;26:284-289. Available at:
http://pedsinreview.aappublications.org/cgi/content/full/26/8/284

Copyright 2009 by the American Academy of Pediatrics

page 405

2009 PREP SA on CD-ROM


Question: 192

A 3-year-old boy is brought to the emergency department at 8 am after his parents found him
unresponsive in bed. The last time they had seen him awake was at 2 am, when they found him
playing in the living room as they were cleaning up after a cocktail party. On physical
examination, the child has diaphoresis and moans to painful stimuli. His vital signs include a
temperature of 96.4F (35.8C), heart rate of 145 beats/min, respiratory rate of 20 breaths/min,
blood pressure of 84/34 mm Hg, and oxygen saturation of 97% in room air. His pupils are midsized and sluggishly reactive.
Of the following, the MOST important test to obtain at this time is

A. acetylcholinesterase determination
B. bedside glucose determination
C. blood alcohol determination
D. serum osmolality
E. urine toxicology screen

Copyright 2009 by the American Academy of Pediatrics

page 406

2009 PREP SA on CD-ROM


Critique: 192

Preferred Response: B

The approach to any pediatric patient who presents with acute alteration in mental status should
include assessment of airway patency, adequacy of oxygenation, ventilation, and perfusion as
well as a bedside glucose determination. Hypoglycemia may result from a variety of causes,
including inborn errors of metabolism, poisonings, sepsis, shock, liver failure, and starvation.
Signs and symptoms of hypoglycemia are related to counterregulatory epinephrine release and
cerebral glucopenia. These include diaphoresis, tachycardia, tremulousness, pallor, irritability,
somnolence, and coma. Unrecognized hypoglycemia can cause mortality as well as significant
morbidity from seizure activity and if chronic, impaired brain development. Because
hypoglycemia is treated easily with intravenous dextrose, oral glucose, or in some settings,
glucagon, rapid diagnosis is critical.
A number of poisonings are well known causes of hypoglycemia in children and include oral
hypoglycemic agents, insulin, beta-blockers, salicylates, and ethanol. Ethanol intoxication with
hypoglycemia is a likely cause of the symptoms described for the patient in the vignette because
of the parents' report of unsupervised activity in an environment where ethanol was accessible.
Other clues to ethanol ingestion in children who have hypoglycemia include an anion gap
acidosis, an osmolar gap, and an elevated lactate value. The hypoglycemia caused by ethanol is
related to gluconeogenesis inhibition and is not responsive to glucagon.
Although acetylcholinesterase and blood alcohol determinations, measurement of serum
osmolality, and a urine toxicology screen may be indicated in the evaluation of a patient who has
acute alteration of mental status, the bedside glucose determination is the most appropriate initial
test because it permits rapid identification of hypoglycemia and prompt treatment.
References:
Ernst AA, Jones K, Nick TG, Sanchez J. Ethanol ingestion and related hypoglycemia in a
pediatric and adolescent emergency department population. Acad Emerg Med. 1996;3:46-49.
Abstract available at: http://www.ncbi.nlm.nih.gov/pubmed/8749967
Sperling M. Hypoglycemia. In: Kleigman RM, Behrman RE, Jenson HB, Stanton BF, eds. Nelson
Textbook of Pediatrics. 18th ed. Philadelphia, Pa: Saunders Elsevier; 2007:655-670
Sunehag A, Haymond MW. Etiology of hypoglycemia in infants and children. UpToDate Online
15.3. 2008. Available for supscription at:
http://www.utdol.com/utd/content/topic.do?topicKey=pediendo/11162&selectedTitle=4~29&sourc
e=search_result

Copyright 2009 by the American Academy of Pediatrics

page 407

2009 PREP SA on CD-ROM


Question: 193

A 3-year-old child presents to your office with chronic recurrent diarrhea of 3 months duration.
He attends child care during the week. He is one of four children in the family, the oldest of
whom is 8 years old. Stool microscopic analysis identifies Giardia lamblia. You treat the boy with
metronidazole for 10 days. On a follow-up visit 30 days after initiating treatment, the mother
states that the symptoms initially improved, but have recurred.
Of the following, the BEST explanation for the persistent symptoms is

A. Clostridium difficile infection


B. metronidazole resistance
C. persistent giardiasis
D. superimposed milk protein allergy
E. undiagnosed celiac disease

Copyright 2009 by the American Academy of Pediatrics

page 408

2009 PREP SA on CD-ROM


Critique: 193

Preferred Response: C

Based on the documented history of giardiasis and ongoing diarrhea for the patient described in
the vignette, the most likely cause of his symptoms is persistent giardiasis, most likely from
reinfection from one of his siblings or a family pet. Many Giardia infections are asymptomatic,
and siblings, other children in child care, or family pets frequently can be reservoirs for the
organism. Therefore, a patient treated with a course of metronidazole is susceptible to
reinfection if he or she is exposed to asymptomatic carriers. Metronidazole resistance is
extremely rare in Giardia parasites, and based on the history provided, there is no reason to
suspect Clostridium difficile infection, celiac disease, or food allergy at this time.
Giardia is a flagellated enteric-coated protozoan that is the most common cause of intestinal
parasitic infections in the United States. The parasite exists in two forms, cysts and
trophozoites. Cysts (Item C193A) are extremely hardy and can survive in cold water for weeks
to months. The cysts also are resistant to chlorination. Once animals (eg, dogs, cats, beavers)
or humans ingest contaminated water, the cysts mature into trophozoites (Item C193B) in the
intestine, where they cause small intestinal inflammation, villous atrophy, and malabsorption.
Asymptomatic carriage is more common than symptomatic disease. However, in patients who
do develop symptoms, the most common features are diarrhea, nausea, distention, cramping,
and anorexia. Because Giardia is a small bowel pathogen, it does not cause rectal bleeding.
The most important aspect to diagnosing Giardia infection is having a strong clinical
suspicion. A number of good diagnostic tests of the stool are available, including microscopic
examination for ova and parasites (stool O and P), enzyme-linked immunofluorescence assays,
and direct fluorescence antibody tests. The microscopic examination ("O and P" test) has
poorer sensitivity (50%) from a single sample and ideally requires microscopic examination of
three separate samples. In contrast, the Giardia antigen-based tests have a sensitivity of
approximately 90% from a single stool sample. However, the O and P examination can identify
other potential pathogens (Blastocystis, ameba) that are not identified with the antigen tests. In
some cases, endoscopy with biopsy may be useful to exclude Giardia or other pathogens and
to characterize the extent of villous atrophy.
Suspected or established giardiasis can be treated with either metronidazole (5 mg/kg per
dose administered three times a day for 7 days) or nitazoxanide for 3 days (100 mg bid for
children younger than 3 years, 200 mg bid for children ages 4 through 11 years, 500 mg bid for
patients older than 12 years of age). If asymptomatic carriers are suspected in the family, it may
be prudent to evaluate and treat the entire family.
References:
American Academy of Pediatrics. Giardia intestinalis infections (giardiasis). In: Pickering LK,
Baker CJ, Long SS, McMillan JA, eds. Red Book: 2006 Report of the Committee on Infectious
Diseases. 27th ed. Elk Grove Village, Ill: American Academy of Pediatrics; 2006:296-301
Huang DB, White AC. An updated review on Cryptosporidium and Giardia. Gastroenterol Clin
North Am. 2006;35:291-314. Abstract available at:
http://www.ncbi.nlm.nih.gov/pubmed/16880067

Copyright 2009 by the American Academy of Pediatrics

page 409

2009 PREP SA on CD-ROM


Question: 194

You admit a 39 weeks gestation male who has respiratory distress to the intensive care
nursery. His mother had a negative group B Streptococcus screening culture and did not
receive antibiotics in labor. She did not have chorioamnionitis or prolonged rupture of the fetal
membranes. However, the amniotic fluid was meconium-stained at the time of delivery, and the
infant required tracheal intubation, with resultant meconium suctioned from below the vocal
cords. Apgar scores were 3 and 7 at 1 and 5 minutes, respectively. On physical examination, he
has marked work of breathing with tachypnea and retractions and episodic cyanosis when
agitated. Breath sounds are coarse and equal. There is no heart murmur. While receiving hood
oxygen at an FiO2 of 0.50, his oxygen saturation by pulse oximetry is 85%. You obtain a chest
radiograph.
Of the following, the radiographic findings MOST expected for this infant are

A. air bronchograms, diffusely hazy lung fields, and low lung volume
B. cardiomegaly, hazy lung fields, and pulmonary vascular engorgement
C. fluid density in the horizontal fissure, hazy lung fields with central vascular prominence, and
normal lung volume

D. gas-filled loops of bowel in the left hemithorax and opacification of the right lung field
E. patchy areas of diffuse atelectasis, focal areas of air-trapping, and increased lung volumes

Copyright 2009 by the American Academy of Pediatrics

page 410

2009 PREP SA on CD-ROM


Critique: 194

Preferred Response: E

The infant described in the vignette, who was born at term following exposure to meconiumstained amniotic fluid (MSAF), has respiratory distress and hypoxemia. Radiographic evidence
of meconium aspiration syndrome (MAS), atelectasis and hyperinflation, is seen in Item C194A.
Meconium aspiration may follow in utero fetal gasping and aspiration of MSAF in the fetus
that has intrauterine growth restriction or asphyxia or after delivery of the newborn who has
meconium in the oropharynx, nasopharynx, or trachea. If respiratory distress follows, the
condition often is described as MAS.
The prevalence of MAS among infants born following MSAF is less than 10%, but tends to
rise beyond 40 weeks' gestation (Item C194B), especially in cases of nonreassuring fetal heart
rate monitoring or perinatal asphyxia. Affected infants present with clinical manifestations of
impaired pulmonary compliance and hypoxemia. The chest radiograph is essential in confirming
the diagnosis. In the classic case of MAS, a diffuse pneumonitis ensues, the lung often is
hyperinflated, and there are patchy infiltrates and opacification of the pulmonary parenchyma.
Complication with air-leak syndromes such as pneumothorax or pneumomediastinum is not
uncommon.
In some cases, chest radiography demonstrates pulmonary hypoperfusion without any
apparent infiltrates or air leaks, and these cases generally represent hypoxia-mediated
pulmonary vascular reactivity and pulmonary hypertension that may have been initiated in utero.
Air bronchograms in diffusely hazy low volume lungs are most consistent with surfactant
deficiency-related respiratory distress syndrome (Item C194C) in preterm infants. The same
findings in a term infant might be seen in congenital pneumonia. Cardiomegaly with hazy lung
fields and pulmonary vascular engorgement is seen in left-sided obstructive cardiac disease
states or infants who have pulmonary overcirculation (eg, truncus arteriosus, aortic stenosis,
anomalous pulmonary venous return, transposition of the great arteries) (Item C194D). Fluid
densities in the horizontal fissure with hazy lung fields, normal lung volumes, and central
vascular prominence are seen in retained fetal lung fluid, otherwise known as transient
tachypnea of the newborn (Item C194E). When gas-filled loops of bowel are seen in the chest,
diaphragmatic hernia is the diagnosis (Item C194F).
References:
Aly H. Respiratory disorders in the newborn: identification and diagnosis. Pediatr Rev.
2004;25:201-208. Available at: http://pedsinreview.aappublications.org/cgi/content/full/25/6/201
Dargaville PA, Copnell B for the Australian and New Zealand Neonatal Network. The
epidemiology of meconium aspiration syndrome: incidence, risk factors, therapies, and outcome.
Pediatrics. 2006;117:1712-1721. Available at:
http://pediatrics.aappublications.org/cgi/content/full/117/5/1712
Miller MJ, Fanaroff AA, Martin RJ. Respiratory disorders in preterm and term infants. In: Martin
RJ, Fanaroff AA, Walsh MC, eds. Fanaroff and Martin's Neonatal-Perinatal Medicine. 8th ed.
Philadelphia, Pa: Mosby Elsevier; 2006:1122-1145
Ross MG. Meconium aspiration syndrome-more than intrapartum meconium. N Engl J Med.
2005;353:946-948. Extract available at: http://content.nejm.org/cgi/content/extract/353/9/946

Copyright 2009 by the American Academy of Pediatrics

page 411

2009 PREP SA on CD-ROM


Question: 195

A mother brings her 5-year-old girl to your office because she noticed a "lump" in her daughters
neck over the past several weeks. The girl appears well and has normal vital signs and no
fever. A 1x1-cm slightly soft mass (Item Q195) is apparent in the middle of her neck, and when
she swallows, the mass moves vertically. There is no drainage or overlying erythema.
Of the following, the MOST likely diagnosis is

A. branchial cleft cyst


B. cystic hygroma
C. reactive lymphadenopathy
D. thyroglossal duct cyst
E. thyroid nodule

Copyright 2009 by the American Academy of Pediatrics

page 412

2009 PREP SA on CD-ROM


Critique: 195

Preferred Response: D

The differential diagnosis of neck masses in children is extensive and includes inflammatory
masses, neoplasms, and congenital lesions. The most common masses are inflammatory and
include reactive lymphadenopathy and lymphadenitis. Reactive lymphadenopathy occurs when
an area drained by the lymph node becomes infected, as in anterior cervical adenopathy with
streptococcal or viral pharyngitis. The affected lymph nodes are enlarged, mobile, and usually
tender, but overt signs of infection, such as erythema and warmth, are absent. Lymphadenitis
occurs when the node itself becomes infected, and erythema, tenderness, warmth, and
occasionally fluctuance are present.
Congenital lesions, although present from birth, may not become evident until later in
childhood, often after a simple upper respiratory tract infection. These result from failure of
branchial clefts to close properly during embryonic development. The most common is the
branchial cleft cyst (Item C195A), which may be unilateral or bilateral and is nonmobile. A
thyroglossal duct cyst is a soft midline mass (Item C195B) that moves vertically when the child
swallows or protrudes the tongue, as described for the girl in the vignette. Drainage onto the skin
may be seen with these lesions. Both types of lesions may become infected and appear
indistinguishable from lymphadenitis (especially unilateral branchial cleft cysts), but their
persistence after appropriate antibiotic treatment suggests the diagnosis.
Neoplasms, such as leukemia and lymphoma, may present with enlarged lymph nodes in
the cervical area. Characteristics include nontender, firm nodes that are matted to the adjacent
tissues. A cystic hygroma (Item C195C), or lymphangioma, is a soft mass of dilated lymph
vessels. These benign tumors of the lymphatic system can grow to a large size and infiltrate
local structures and blood vessels. Thyroid nodules are infrequent in children. They may be firm
or soft masses and are usually lateral rather than midline. Movement on swallowing is not
typical.
References:
Camitta BM. The lymphatic system. In: Kliegman RM, Behrman RE, Jenson HB, Stanton BF,
eds. Nelson Textbook of Pediatrics. 18th ed. Philadelphia, Pa: Elsevier Saunders; 2007:20922096
Tracy TF Jr, Muratore CS. Management of common head and neck masses. Semin Pediatr
Surg. 2007;16:3-13. Abstract available at: http://www.ncbi.nlm.nih.gov/pubmed/17210478
Waldhausen JH. Branchial cleft and arch anomalies in children. Semin Pediatr Surg. 2006;15:6469. Abstract available at: http://www.ncbi.nlm.nih.gov/pubmed/16616308

Copyright 2009 by the American Academy of Pediatrics

page 413

2009 PREP SA on CD-ROM


Question: 196

You have been treating a 2-year-old girl for 1 month with ferrous sulfate after her hemoglobin
was 10.0 g/dL (100.0 g/L) and hematocrit was 29% (0.29). Today, her reticulocyte count is 4.2%
(0.042), hemoglobin is 11.5 g/dL (115.0 g/L), and hematocrit is 33% (0.33). The nurse
practitioner student with whom you are working asks if she can stop the iron supplement.
Of the following, the BEST reason for continuing iron therapy in this child is to

A. increase her appetite


B. prevent infection
C. prevent lead poisoning
D. reinforce vitamin supplementation
E. replenish iron stores

Copyright 2009 by the American Academy of Pediatrics

page 414

2009 PREP SA on CD-ROM


Critique: 196

Preferred Response: E

Treatment of iron deficiency anemia involves iron replacement, usually with ferrous sulfate, for at
least 2 months after the anemia has been corrected to replenish iron stores.
Most children who have mild iron deficiency anemia are asymptomatic and are diagnosed
following routine screening as part of health supervision visits, screening based on dietary risk
factors, or a complete blood count obtained for evaluation of illness, particularly fever without a
focus.
Contrary to popular belief, there is no evidence that treatment of iron deficiency stimulates
appetite. Treatment of iron deficiency also does not prevent lead poisoning, although lead
poisoning and iron deficiency may coexist because iron deficiency may lead to pica.
Neither iron supplementation nor therapy has been shown to prevent clinical infection.
However, studies in children of low socioeconomic status have shown that iron supplementation
may improve T-cell counts, delayed hypersensitivity skin reactions, and interleukin-2 production.
Vitamin and mineral supplementation is not recommended for children who are eating normal
diets and have no evidence for deficiency; such supplementation may increase the risk of
ingesting toxic amounts of iron.
References:
Glader B. Iron deficiency anemia. In: Kliegman RM, Behrman RE, Jenson HB, Stanton BF, eds.
Nelson Textbook of Pediatrics. 18th ed. Philadelphia, Pa: Elsevier Saunders; 2007:2014-2016
Kleinman RE. Nutrition and immunity. In: Pediatric Nutrition Handbook. 5th ed. Elk Grove Village,
Ill: American Academy of Pediatrics; 2004:609-628
Richardson M. Microcytic anemia. Pediatr Rev. 2007;28:5-14. Available at:
http://pedsinreview.aappublications.org/cgi/content/full/28/1/5

Copyright 2009 by the American Academy of Pediatrics

page 415

2009 PREP SA on CD-ROM


Question: 197

You are performing screening sports participation examinations at the local high school. One of
the students, a 16-year-old boy, reports that his father has hypertrophic cardiomyopathy but
that none of his three older brothers has it. He also reports that he was seen by a cardiologist at
age 10 years and was "fine." As you take his history, you find that he has never had shortness
of breath, chest pain, exercise intolerance, dizziness, or fainting. He has always participated in
sports and has excelled.
Of the following, the BEST plan of management for this boy is

A. chest radiography
B. electrocardiography
C. genetic testing for hypertrophic cardiomyopathy
D. referral to a cardiologist
E. unrestricted sports participation unless symptoms develop

Copyright 2009 by the American Academy of Pediatrics

page 416

2009 PREP SA on CD-ROM


Critique: 197

Preferred Response: D

Hypertrophic cardiomyopathy (HCM) typically is inherited as an autosomal dominant trait due to


a gene abnormality on chromosome 15. However, multiple genes have been found to be
responsible for this disorder, and this fact contributes to the variability of severity. Interestingly,
HCM may not be apparent on echocardiography until adolescence and even young adulthood in
some patients, necessitating regular, repeat evaluation for asymptomatic children who have an
affected first-degree relative. Alternatively, genetic testing may be an option for some, but
genetic variability and the expense of testing may make this impractical in some cases.
Accurate diagnosis of HCM in children and adolescents is essential because the condition can
be the source of morbidity and mortality. Thoughtful counseling is an important component of
care for the affected child; some sports and activities, such as competitive soccer, basketball,
and football, are believed to be particularly high risk for the development of arrhythmia and even
sudden death. In fact, HCM is the most common reason for a young person to experience
sudden death during sports activities, occurring with nearly twice the frequency of coronary
artery anomalies.
The boy in the vignette has a family history of HCM (present in his father); each of his
siblings has as great as a 50% chance of having inherited the HCM gene from the father.
Current recommendations suggest evaluating such children regularly (perhaps as often as
yearly) with detailed history, physical examination, and diagnostic testing that includes
electrocardiography and echocardiography (Item C197). Therefore, the boy in the vignette and
his siblings should be referred to a cardiologist for further evaluation.
Chest radiography, a relatively nonspecific imaging study for cardiac structure, may yield
normal results in patients who have HCM and would not be used either to confirm or rule out the
diagnosis. Similarly, electrocardiography tracings can appear normal in a small percentage of
patients who have HCM, making this study inappropriate for confirmation or ruling out of the
diagnosis. Genetic testing for the HCM gene can be very effective if the specific gene mutation
is well-identified in the index case of the family, but undertaking such testing for the boy in the
vignette without testing his father first is not productive. Recognition of HCM and referral to a
cardiologist for thorough preparticipation screening are critical.
References:
Berger S, Utech L, Hazinski MF. Sudden death in children and adolescents. Pediatr Clin North
Am. 2004;51:1653-1677. Abstract available at: http://www.ncbi.nlm.nih.gov/pubmed/15561179
Corrado D, Basso C, Schiavon M, Thiene G. Screening for hypertrophic cardiomyopathy in
young athletes. N Engl J Med. 1998;339:364-369. Abstract available at:
http://www.ncbi.nlm.nih.gov/pubmed/9691102
Ho CY, Seidman CE. A contemporary approach to hypertrophic cardiomyopathy. Circulation.
2006;113:e858-e862. Available at: http://circ.ahajournals.org/cgi/content/full/113/24/e858
Maron BJ, Chaitman BR, Ackerman MJ, et al. Recommendations for physical activity and
recreational sports participation for young patients with genetic cardiovascular diseases.
Circulation. 2004;109:2807-2816. Available at:
http://circ.ahajournals.org/cgi/content/full/109/22/2807
Maron BJ, Thompson PD, Ackerman MJ, et al. AHA scientific statements. Recommendations
and considerations related to preparticipation screening for cardiovascular abnormalities in
competitive athletes: 2007 update. A scientific statement from the American Heart Association
Council on Nutrition, Physical Activity, and Metabolism: endorsed by the American College of
Cardiology Foundation. Circulation. 2007;115:1643-1655. Available at:
http://circ.ahajournals.org/cgi/content/full/115/12/1643

Copyright 2009 by the American Academy of Pediatrics

page 417

2009 PREP SA on CD-ROM


Question: 198

A 4-year-old boy recently underwent hematopoietic stem cell transplantation for acute
myelogenous leukemia. Fourteen days after his transplant, he experiences a seizure and
confusion. He is receiving cyclosporine, prednisone, ganciclovir, fluconazole, cefotaxime,
tobramycin, and omeprazole. Magnetic resonance imaging shows signal changes in bilateral
occipital lobes.
Of the following, the MOST likely cause of the seizures is

A. cyclosporine
B. fluconazole
C. ganciclovir
D. prednisone
E. tobramycin

Copyright 2009 by the American Academy of Pediatrics

page 418

2009 PREP SA on CD-ROM


Critique: 198

Preferred Response: A

Seizures in children are not commonly iatrogenic, but medications used in certain disciplines,
most commonly oncology and psychiatry, may precipitate them. Chemotherapy for childhood
cancers or organ transplantation often has adverse effects on the central nervous system,
causing neuronal cell death, changes in white matter, or vascular lesions. Patients receiving
cyclosporine or intrathecal methotrexate may develop diffuse or predominantly occipital white
matter changes.
The child described in the vignette has characteristic changes associated with cyclosporine.
In addition to seizures, occipital blindness or strokelike weakness may occur. The changes
seen on magnetic resonance imaging can be reversible. Fluconazole, ganciclovir, prednisone,
and tobramycin are unlikely causes of seizures. Other medications that may cause or
exacerbate seizures in children include isoniazid, theophylline, cocaine, psychostimulants,
bupropion, insulin, and oral hypoglycemic agents.
References:
Abbott MB, Levin RH, Wu S. Medication potpourri. Pediatr Rev. 2006;27:283-288. Available at:
http://pedsinreview.aappublications.org/cgi/content/full/27/8/283
Norman JK, Parke JT, Wilson DA, McNall-Knapp RY. Reversible posterior leukoencephalopathy
syndrome in children undergoing induction therapy for acute lymphoblastic leukemia. Pediatr
Blood Cancer. 2007;49:198-203. Abstract available at:
http://www.ncbi.nlm.nih.gov/pubmed/16123992
Pound CM, Keene DL, Udjus K, Humphreys P, Johnston DL. Acute encephalopathy and
cerebral vasospasm after multiagent chemotherapy including PEG-asparaginase and intrathecal
cytarabine for the treatment of acute lymphoblastic leukemia. J Pediatr Hematol Oncol.
2007;29:183-186. Abstract available at: http://www.ncbi.nlm.nih.gov/pubmed/17356399

Copyright 2009 by the American Academy of Pediatrics

page 419

2009 PREP SA on CD-ROM


Question: 199

During the health supervision visit of a 2-week-old infant, you note that his weight remains below
his birthweight. The baby was delivered at term by a midwife in the parents home. There were
no complications, and the parents have declined all perinatal testing. His mother says that he
breastfeeds well, and her milk supply is good compared with that for her previous two children.
Recently, though, the infant has been vomiting after feedings. On physical examination, he has
total body jaundice, and his liver is enlarged to palpation. He is alert.
Of the following, the test that is MOST likely to aid in diagnosis is

A. abdominal ultrasonography
B. serum transaminases measurement
C. total and direct bilirubin measurement
D. urine organic acids measurement
E. urine reducing substances measurement

Copyright 2009 by the American Academy of Pediatrics

page 420

2009 PREP SA on CD-ROM


Critique: 199

Preferred Response: E

The infant described in the vignette has a history and physical findings suggestive of
galactosemia, an autosomal recessive inborn error of metabolism that results in the inability to
oxidize galactose. Galactose dimerizes with glucose to form lactose, which is abundant in
human and cow milk. Individuals who have classic galactosemia are deficient in galactose-1phosphate uridyltransferase (GALT) activity, resulting in an elevated concentration of galactose,
a reducing substance, in the urine. Although the presence of reducing substances in the urine is
not specific for galactosemia, this finding, especially in the presence of poor weight gain,
jaundice, and hepatomegaly, is strongly suggestive of galactosemia. Galactosemia can be
detected in almost 100% of affected infants by newborn screening programs that test for it.
The infant who has classic galactosemia typically presents within days of the initiation of milk
feedings with vomiting and jaundice. If untreated, the child develops hepatomegaly and failure to
thrive. Escherichia coli sepsis may occur, and approximately 10% of infants have cataracts at
the time of diagnosis. Treatment is aimed at removing lactose from the diet, usually by feeding
the baby a soy-based formula.
The natural history of galactosemia is more complicated than once believed. Studies are
showing that, despite early diagnosis and strict dietary management, affected older children and
adults can have learning disabilities, neurologic disorders, verbal dyspraxia, and ovarian failure.
For infants presenting with signs of galactosemia, abdominal ultrasonography may show
hepatomegaly. Serum transaminases are elevated, as are total and direct bilirubin values.
However, these abnormalities are not sufficiently informative to point to a diagnosis. Urine
organic acids assessment may document elevated lactate, which is a nonspecific finding.
References:
Elsas LJ. Galactosemia. GeneReviews. 2007. Available at:
http://www.geneclinics.org/servlet/access?db=geneclinics&site=gt&id=8888891&key=xAcWBcrj
mZrVo&gry=&fcn=y&fw=4Vlc&filename=/profiles/galactosemia/index.html
Nyhan WL, Barshop BA, Ozand PT. Organic acidemias. In: Atlas of Metabolic Diseases. 2nd ed.
London, England: Hodder Arnold; 2005:1-108

Copyright 2009 by the American Academy of Pediatrics

page 421

2009 PREP SA on CD-ROM


Question: 200

A 16-year-old young man presents to the emergency department with a 12-hour history of pain
in the scrotal area. He states that the pain started gradually and describes it as on the left side
and moderate in intensity. He is sexually active and uses condoms. He has some burning pain
with urination, but no penile discharge. He has felt warm but has not taken his temperature. He
has had no vomiting or diarrhea. He has had no previous similar symptoms. On physical
examination, the young man is afebrile and has normal findings on abdominal evaluation. He has
moderate swelling of the left scrotum without erythema and marked tenderness that involves
more of the posterolateral area. The testicular position is lower on the left than on the right. The
left spermatic cord is very tender. Urinalysis shows more than 10 white blood cells per highpower field on a first-void specimen.
Of the following, the MOST likely diagnosis is

A. epididymitis
B. testicular torsion
C. testicular tumor
D. urinary tract infection
E. varicocele

Copyright 2009 by the American Academy of Pediatrics

page 422

2009 PREP SA on CD-ROM


Critique: 200

Preferred Response: A

The differential diagnosis for a painful scrotal mass or swelling in an adolescent includes
testicular torsion, torsion of the spermatic cord or appendix testis, epididymitis (Item C200),
orchitis, trauma with hematoma, and incarcerated hernia. Other causes could be HenochSchnlein purpura or other trauma such as an infected piercing or insect sting. The features
exhibited by the young man in the vignette that support the diagnosis of epididymitis include:
sexual activity, gradual pain onset, localization of pain to the spermatic cord and posterolateral
area of the scrotum, dysuria, and pyuria.
With testicular torsion, a true surgical emergency, pain often is more abrupt in onset, and
nausea and vomiting are frequent. The affected testis frequently is located more superiorly than
the contralateral testis. Testicular tumors and varicoceles generally are not painful and are not
accompanied by spermatic cord tenderness. A urinary tract infection is unlikely in an adolescent
who has no congenital urinary tract abnormalities and would not produce a painful, swollen
testis.
References:
Adelman WP, Joffe A. Scrotal disorders. In: Neinstein, LS, ed. Adolescent Health Care: A
Practical Guide. 5th ed. Philadelphia, Pa: Lippincott Williams & Wilkins; 2008:401-410
Centers for Disease Control and Prevention. Update to CDC's sexually transmitted diseases
treatment guidelines, 2006: fluoroquinolones no longer recommended for treatment of
gonococcal infections. MMWR Morbid Mortal Wkly Rep. 2007;56:332-336. Available at:
http://www.cdc.gov/mmwr/preview/mmwrhtml/mm5614a3.htm
Workowski AK, Berman SM, Centers for Disease Control and Prevention. Sexually transmitted
diseases treatment guidelines, 2006. MMWR Recomm Rep. 2006;55(RR11):1-94. Available at:
http://www.cdc.gov/mmwr/preview/mmwrhtml/rr5511a1.htm

Copyright 2009 by the American Academy of Pediatrics

page 423

2009 PREP SA on CD-ROM


Question: 201

A 1-year-old boy who is intubated for severe asthma is demonstrating significant acute
respiratory and cardiac deterioration, as evidenced by tachycardia, tachypnea, decreased blood
pressure, and oxygen saturation of 75%. During your examination, you note a marked shift of
the trachea to the left and markedly decreased aeration on the right side.
Of the following, the MOST likely cause of this boys sudden respiratory deterioration is

A. endotracheal tube obstruction


B. exacerbation of the underlying asthma
C. intubation of the right mainstem bronchus
D. tension pneumothorax
E. ventilator-associated pneumonia

Copyright 2009 by the American Academy of Pediatrics

page 424

2009 PREP SA on CD-ROM


Critique: 201

Preferred Response: D

The tachypnea, tachycardia, decreased breath sounds on the involved side, and decreased
oxygenation described for the boy in the vignette are strongly suggestive of pneumothorax, the
accumulation of air within the extrapleural space. Pneumothoraces typically are classified as
simple, communicating, or tension. Simple pneumothoraces are the most common and have no
communication with the atmosphere. They usually are a result of a spontaneous pneumothorax
with or without underlying lung pathology, blunt chest trauma, or mechanical ventilation. A
communicating pneumothorax normally is caused by open chest trauma, although rapid
accumulation of pleural air is possible in the spontaneously breathing patient due to entrainment
of air with inspiration. Tension pneumothoraces represent a medical emergency and develop
when continued accumulation of air produces a mediastinal shift to the contralateral side and
subsequent compression and decreased function of the lung and vascular structures. The
tracheal shift and hypotension reported for the boy in the vignette indicate that he has a tension
pneumothorax.
Rapid diagnosis and treatment of symptomatic pneumothoraces is critical. Chest
radiography (Item C201) is the gold standard for diagnosis, but clinical conditions may
necessitate rapid treatment with thoracentesis or thoracic tube placement prior to imaging.
Development of a pneumothorax along with other potential ventilator complications always
should be considered promptly when a mechanically ventilated patient deteriorates.
Endotracheal tube obstruction normally presents with bilateral decreased aeration, difficulty
in passing a suction catheter, and lack of tracheal shift. A right bronchus intubation is
characterized by decreased aeration on the left side and oxygen desaturation, but
cardiovascular compromise is less common than with a tension pneumothorax. Progression of
underlying disease processes such as asthma or development of ventilator-associated
pneumonia is more gradual than described in the vignette.
References:
Chase MA, Wheeler DS. Disorders of the pediatric chest. In: Wheeler DS, Wong HR, Shanley T,
eds. Pediatric Critical Care Medicine: Basic Science and Clinical Evidence. New York, NY:
Springer-Verlag; 2007:361-375
Winnie GB. Pnuemothorax. In: Kliegman RM, Behrman RE, Jenson HB, Stanton BF, eds. Nelson
Textbook of Pediatrics. 18th ed. Philadelphia, Pa: Saunders Elsevier; 2007:1835-1836

Copyright 2009 by the American Academy of Pediatrics

page 425

2009 PREP SA on CD-ROM


Question: 202

During the health supervision visit of a 9}-year-old girl, you note that her height is just above the
97th percentile for age and her weight is at the 85th percentile. Her mother is 5 feet 5 inches tall
and father is 5 feet 10 inches. Her parents ask if she will be very tall when she has finished
growing.
Of the following, the MOST important element of the physical examination to help answer this
question is

A. arm span measurement


B. body mass index
C. eye examination
D. Sexual Maturity Rating
E. upper-to-lower segment ratio

Copyright 2009 by the American Academy of Pediatrics

page 426

2009 PREP SA on CD-ROM


Critique: 202

Preferred Response: D

Most children of tall stature either have tall parents or have matured early and are tall in late
childhood, although adult height after early puberty will be well within the mid-parental range.
Therefore, if the girl described in the vignette is in puberty, an explanation for her tall stature at
age 9 years will be apparent, and a bone age radiograph can be used to predict her ultimate
adult height. Accordingly, it is most important to determine her Sexual Maturity Rating. Lack of
puberty suggests that she will, indeed, be tall.
Further studies might be indicated to look for other reasons for her tall stature, particularly if
she has had growth acceleration. Rarely, children of tall stature have Marfan syndrome, which is
characterized by long arm span and a decreased body mass index. Increased body mass index
and obesity may be associated with tall stature and early puberty. If there is worry about growth
hormone excess and a pituitary tumor, eye examination and careful assessment of visual fields
should be undertaken, but growth hormone-producing pituitary tumors often are too small to
produce visual symptoms. They are detected best by physical examination showing signs of
growth hormone excess: widened spaces between the teeth, large hands and feet, and
increased soft-tissue thickening. Laboratory measurement of insulin-like growth factor I and
magnetic resonance imaging of the pituitary may be helpful. Measurement of the upper-to-lower
segment ratio may be useful in the clinical diagnosis of Marfan syndrome because long legs
compared with the trunk result in a lower ratio than expected for age. Other congenital causes of
tall stature, such as Sotos syndrome, are associated with specific physical and behavioral
findings, including large head circumference and decreased intellectual functioning. Diagnostic
genetic testing is available for this disorder.
References:
Boom JA. Normal pediatric growth. UpToDate Online 15.3. 2008.Available for subscription at:
http://www.uptodateonline.com/utd/content/topic.do?topicKey=gen_pedi/13648
Cohen P, Shim M. Hyperpituitarism, tall stature, and overgrowth syndromes. In: Kliegman RM,
Behrman RE, Jenson HB, Stanton BF, eds. Nelson Textbook of Pediatrics. 18th ed. Philadelphia,
Pa: Saunders Elsevier; 2007:2303-2307
Richmond EJ, Rogol AD. The child with abnormally rapid growth. UpToDate Online 15.3. 2008.
Available for subscription at:
http://www.uptodateonline.com/utd/content/topic.do?topicKey=pediendo/7226

Copyright 2009 by the American Academy of Pediatrics

page 427

2009 PREP SA on CD-ROM


Question: 203

A mother brings in her 10-year-old daughter and 8-year-old son because they are fighting
constantly. The son says he hates having a sister and complains that his parents favor her and
give her everything she wants. The daughter says that her brother is spoiled and always
touches her stuff. The mother is frustrated by their constant fighting and asks for assistance in
handling the children.
Of the following, the BEST initial guidance for the mother is to

A. explain that this is typical of siblings and she should ignore the behavior
B. give her a list of books on parenting
C. refer her to a behavioral therapist to improve her parenting skills
D. suggest she use behavioral modification techniques to diminish fighting
E. tell her to return in 6 months if the siblings are still fighting

Copyright 2009 by the American Academy of Pediatrics

page 428

2009 PREP SA on CD-ROM


Critique: 203

Preferred Response: D

Although sibling rivalry is common, the actions described for the children in the vignette indicate
the need for the mother to employ behavioral techniques to decrease their fighting in the home.
Children initially should be allowed to resolve their differences, but parents need to intervene if
one sibling is being abused either physically or verbally. When the fighting is heading toward a
dangerous situation, the parents need to describe the actions of the siblings, establish limits, and
separate the siblings. The purpose of the behavioral intervention is to open lines of
communication so the siblings can begin to work out their differences. If the behavior continues
to be challenging and not responsive to such initial parental interventions, referral to a therapist
may be considered. Telling the mother either to ignore the sibling rivalry or wait 6 months will not
aid in changing the children's behaviors. A book on sibling rivalry may supplement an initial
therapeutic plan, but the family should be provided direct guidance on implementing a behavior
intervention strategy.
A common time for the emergence of sibling rivalry is after a recent birth. It is common for a
child to be upset and have some regression in behavioral skills following the birth of a new
sibling. Parents need to intervene more with younger children to prevent any injuries.
References:
Faber A, Mazlish E. When the kids fight: how to step in so we can step out. In: Siblings Without
Rivalry: How to Help Your Children Live Together So You Can Live Too. New York, NY: Quill;
2002:146-177
Needlman R. Sibling rivalry. In: Parker S, Zuckerman B, Augustyn M, eds. Developmental and
Behavioral Pediatrics: A Handbook for Primary Care. 2nd ed. Philadelphia, Pa: Lippincott
Williams & Wilkins; 2005:412-415

Copyright 2009 by the American Academy of Pediatrics

page 429

2009 PREP SA on CD-ROM


Question: 204

You are evaluating an 8-month-old boy who is having multiple "coughing spells." During these
spells, the boy sometimes turns blue and even vomits. You inform the mother that you are going
to prescribe an antimicrobial agent. She wants to know why you are giving her infant an
antimicrobial agent when he needs something for the cough.
Of the following, the BEST reason to prescribe an antimicrobial agent for this boy is that
treatment will decrease the

A. chance of death
B. cough
C. hypoxic episodes
D. infectivity
E. posttussive vomiting

Copyright 2009 by the American Academy of Pediatrics

page 430

2009 PREP SA on CD-ROM


Critique: 204

Preferred Response: D

The clinical findings for the child described in the vignette indicate the diagnosis of pertussis
(whooping cough). The clinical course of pertussis can be ameliorated with antimicrobial agents
only in patients whose disease is limited to mild upper respiratory tract symptoms (catarrhal
stage). Once patients develop the paroxysmal cough (paroxysmal stage), the use of
antimicrobial agents does not shorten the disease course or affect the cough, posttussive
vomiting, hypoxic episodes, or chance of death. However, antimicrobial therapy is used for
patients in the paroxysmal stage to decrease the spread of Bordetella pertussis to others.
Erythromycin products have been the treatment of choice for pertussis until recently. Data
now demonstrate that azithromycin and clarithromycin have microbiologic effectiveness similar
to erythromycin in individuals 6 months of age or older. The advantages of the newer macrolides
over erythromycin for the treatment of pertussis are: 1) higher tissue concentrations, 2) longer
half-life, 3) fewer daily doses required (one or two per day), and 4) short treatment courses (5 to
7 days). Azithromycin and clarithromycin also are approved as first-line agents for prophylaxis
of exposed individuals. In most situations of treatment or prophylaxis, patients tolerate and are
more compliant with azithromycin or clarithromycin than erythromycin products. Erythromycin
should not be used in infants younger than 1 month of age due to the agent's association with
infantile pyloric stenosis. There are no safety data for the use of clarithromycin in neonates,
making azithromycin the macrolide of choice for this age group. Trimethoprim-sulfamethoxazole
is an alternative agent that can be used for children older than 2 months of age who cannot take
macrolides for pertussis treatment or prophylaxis.
References:
American Academy of Pediatrics. Pertussis (whooping cough). In: Pickering LK, Baker CJ, Long
SS, McMillan JA, eds. Red Book: 2006 Report of the Committee on Infectious Diseases. 27th ed.
Elk Grove Village, Ill: American Academy of Pediatrics; 2006:498-520
Tiwari T, Murphy RV, Moran J. Recommended antimicrobial agents for the treatment and
postexposure prophylaxis of pertussis. 2005 CDC guidelines. MMWR Recomm Rep.
2005;54(RR14):1-16. Available at: http://www.cdc.gov/mmwr/preview/mmwrhtml/rr5414a1.htm
Waseem M, Kin LL. Index of suspicion: case 6. Pediatr Rev. 2005;26:23-33. Available at:
http://pedsinreview.aappublications.org/cgi/content/full/26/1/23

Copyright 2009 by the American Academy of Pediatrics

page 431

2009 PREP SA on CD-ROM


Question: 205

You are evaluating a 17-month-old previously healthy girl who presents with an 8-month history
of recurrent cellulitis and abscesses on her lower right abdomen. Eight months ago, she
developed a "pimple" on her abdomen that rapidly enlarged to the size of a golf ball and became
very red, hard, and tender over 2 days. The lesion spontaneously drained a purulent material
and resolved. Over the last 8 months, the girl has had nine similar episodes. She was seen on
several occasions in an urgent care center and each time was placed on a course of
cephalexin, which resulted in no improvement until the lesion drained spontaneously. The patient
has no fever with the episodes. Physical examination shows a 2x3-cm erythematous, indurated,
very tender, fluctuant lesion on the patients right flank.
Of the following, the MOST likely organism causing this patients recurrent infections is

A. methicillin-resistant Staphylococcus aureus


B. methicillin-sensitive Staphylococcus aureus
C. Staphylococcus epidermidis
D. Streptococcus pneumoniae
E. Streptococcus pyogenes

Copyright 2009 by the American Academy of Pediatrics

page 432

2009 PREP SA on CD-ROM


Critique: 205

Preferred Response: A

The first-generation cephalosporins (eg, cephalexin) have a spectrum of activity focused


primarily on gram-positive bacteria. They have been used widely as alternatives to penicillin for
the treatment of staphylococcal and nonenterococcal streptococcal infections. These agents
are effective in the treatment of infections caused by susceptible strains of groups A, B, C, and
G streptococci; S pneumoniae; viridans streptococci; methicillin-susceptible Staphylococcus
aureus (MSSA); and some strains of S epidermidis. Historically, first-generation cephalosporins
have been the mainstay of therapy for skin and soft-tissue infections. The gram-negative
spectrum of the first-generation cephalosporins is much more limited. These drugs are effective
against susceptible strains of Neisseria gonorrhoeae, Escherichia coli, Klebsiella sp, and
Proteus mirabilis. However, they have poor activity against penicillin-resistant pneumococci,
Haemophilus influenzae, and Moraxella catarrhalis and are not recommended in the treatment of
sinusitis, otitis media, and lower respiratory tract infections. All isolates of methicillin-resistant S
aureus (MRSA) are resistant to the cephalosporin class of antibiotics. Antibiotics used to treat
MRSA include clindamycin, trimethoprim-sulfamethoxazole, and linezolid.
The patient described in the vignette has had episodes of recurrent cellulitis and abscesses
that improve only after spontaneous drainage, despite receiving a first-generation cephalosporin
for treatment. The most common organisms associated with skin and soft-tissue infections are
S aureus and S pyogenes (group A streptococci). The organism most likely to be the cause of
this patient's infection is MRSA, which is associated with recurrent episodes of cellulitis and
abscesses. MRSA is resistant to first-generation cephalosporins such as cephalexin. MSSA and
S pyogenes both are susceptible to the first-generation cephalosporins, and infection caused by
these agents should improve with treatment. S pneumoniae is a cause of preseptal cellulitis and
rarely lymphadenitis, but it is not associated with cellulitis and abscesses. S epidermidis usually
does not cause cellulitis or soft-tissue infections.
References:
Andes DR, Craig WA. Cephalosporins. In: Mandell GL, Bennett JE, Dolin R, eds. Mandell,
Douglas, and Bennett's Principles and Practice of Infectious Diseases. 6th ed. Philadelphia, Pa:
Elsevier Churchill Livingstone; 2005:294-310
Jantausch BA. Peripheral brain: cephalosporins. Pediatr Rev. 2003;24:128-136. Available at:
http://pedsinreview.aappublications.org/cgi/content/full/24/4/128

Copyright 2009 by the American Academy of Pediatrics

page 433

2009 PREP SA on CD-ROM


Question: 206

A mother brings in her 4-year-old daughter because of decreased energy following a 3-day
history of diarrhea without vomiting. On physical examination, the girls temperature is 100.2F
(37.9C), heart rate is 130 beats/min, respiratory rate is 18 breaths/min, and blood pressure is
122/84 mm Hg. She has pale conjunctivae, a hyperdynamic precordium, and mild pretibial
edema. Laboratory evaluation reveals:
Sodium, 133.0 mEq/L (133.0 mmol/L)
Potassium, 5.2 mEq/L (5.2 mmol/L)
Chloride, 100.0 mEq/L (100.0 mmol/L)
Bicarbonate, 16.0 mEq/L (16.0 mmol/L)
Albumin, 2.5 g/dL (25.0 g/L)
Blood urea nitrogen, 40.0 mg/dL (14.3 mmol/L)
Creatinine, 1.4 mg/dL (123.8 mcmol/L)
Hemoglobin, 6.1 g/dL (610.0 g/L)
White blood cell count, 21.5x10 3/mcL (21.5x109/L)
Platelet count, 90.0x10 3/mcL (90.0x109/L)
Of the following, the MOST likely additional laboratory abnormality is

A. a low reticulocyte count


B. elevated lactate dehydrogenase concentration
C. elevated parathyroid hormone concentration
D. positive Coombs test
E. prolonged prothrombin time

Copyright 2009 by the American Academy of Pediatrics

page 434

2009 PREP SA on CD-ROM


Critique: 206

Preferred Response: B

The diarrheal prodrome, tachycardia, hyperdynamic precordium, anemia, thrombocytopenia,


and elevated creatinine value described for the girl in the vignette suggests the diagnosis of
hemolytic-uremic syndrome (HUS). HUS is a microangiopathic process often linked to Shiga
toxin-producing enterohemorrhagic Escherichia coli, which results in the triad of
microangiopathic hemolytic anemia, thrombocytopenia, and acute renal failure (ARF). HUS is
categorized as a thrombotic microangiopathy that has a predilection for arterioles and capillaries,
especially those involving the renal circulation.
When HUS is preceded by diarrhea (90% of cases), it is designated D(+). The diarrheal
prodrome may be bloody or nonbloody and is associated most commonly with
enterohemorrhagic E coli, with serotype O157:H7 being the most common in the United States.
This Shiga toxin-producing E coli often is acquired by ingesting undercooked hamburger,
unpasteurized apple cider, or other contaminated foods. Patients who have this organism in their
gastrointestinal tracts can develop diarrhea (bloody or nonbloody) alone or HUS. Interestingly,
only 5% to 8% of children who have hemorrhagic colitis from a Shiga toxin-producing
enterohemorrhagic E coli actually develop HUS, which suggests that virulence and host factors
likely play a role in who develops HUS. Those who exhibit HUS develop endothelial cell injury
and small vessel thromboses involving the glomerular capillary, arteriolar, and interlobular
arteries. The resultant ischemic injury affects the glomeruli and tubules, leading to ARF. The
endothelial injury results in von Willebrand factor release, platelet adhesion, and microthrombi
formation. This thrombotic microangiopathic process leads to a hemolytic anemia and
thromobocytopenia.
Children who have D(+) HUS may have symptoms mimicking those of ulcerative colitis or
an acute abdomen. To detect HUS in its early stages, it is important to monitor hemoglobin,
platelet count, and serum creatinine as well as urinalysis in a child who has severe colitis.
Because oliguria due to dehydration is common in children who have severe diarrhea, the
diagnosis of ARF due to HUS can be delayed. When HUS is suspected, a peripheral blood
smear should be obtained. A smear that shows evidence of fragmented red blood cells
(schistocytes) (Item C206), low platelet number, and reticulocytosis is diagnostic. Of note, the
reticulocyte count may not be markedly elevated in severe renal dysfunction due to impaired
erythropoietin production. Leukocytosis is common, with higher white blood cell counts
associated with a worse renal prognosis. In addition, due to the severity of the hemolysis, other
laboratory abnormalities include elevated lactate dehydrogenase and low haptoglobin values.
The Coombs test is negative because this is not an antibody-mediated process. The
prothrombin and partial thromboplastin times are normal because HUS is not a consumptive
coagulopathy. The parathyroid hormone concentration would not be expected to be elevated in
the setting of acute renal failure; this is a manifestation of renal osteodystrophy seen in severe
forms of chronic kidney disease.
Treatment of HUS is largely supportive, including intravenous volume expansion in those
who do not have oliguria and dialysis in those who have oligoanuria. Of note, empiric antibiotics
should be avoided in children who have bloody diarrhea because if the diarrhea is due to
enterohemorrhagic E coli, antibiotic therapy is associated with an increased risk of developing
HUS.
References:
Ake JA, Jelacic S, Ciol MA, et al. Relative nephroprotection during Escherichia coli O157:H7
infections: association with intravenous volume expansion. Pediatrics. 2005;115:e673-e680.
Available at: http://pediatrics.aappublications.org/cgi/content/full/115/6/e673
Oakes RS, Siegler RL, McReynolds MA, Pysher T, Pavia AT. Predictors of fatality in
postdiarrheal hemolytic uremic syndrome. Pediatrics. 2006;117:1656-1662. Available at:
http://pediatrics.aappublications.org/cgi/content/full/117/5/1656
Mahan JD. Hemolytic uremic syndrome. In: Kher KK, Schnaper HW, Makker SP, eds. Clinical

Copyright 2009 by the American Academy of Pediatrics

page 435

2009 PREP SA on CD-ROM

Pediatric Nephrology. 2nd ed. London, England: Informa Healthcare; 2007:235-244


Wong CS, Jelacic S, Habeeb RL, Watkins SL, Tarr PI. The risk of the hemolytic-uremic
syndrome after antibiotic treatment of Escherichia coli O157:H7 infections. N Engl J Med.
2000;342:1930-1936. Available at: http://content.nejm.org/cgi/content/full/342/26/1930

Copyright 2009 by the American Academy of Pediatrics

page 436

2009 PREP SA on CD-ROM


Question: 207

A 16-year-old girl who has moderate persistent asthma presents to the emergency department
with coughing, wheezing, and increasing dyspnea. She states that she was feeling fine until she
was exposed to cologne that one of her classmates was wearing. An ambulance was called
after her symptoms did not improve following administration of two puffs of her beta2 agonist
inhaler. On physical examination, the teenager has a respiratory rate of 30 breaths/min, heart
rate of 90 beats/min, and pulse oximetry of 98% on room air. She has difficulty completing a
sentence and points to her neck, saying it is "hard to get air in." Her lungs are clear to
auscultation, and rhinolaryngoscopy demonstrates adduction of one of the vocal cords during
inspiration. Pulmonary function testing shows a blunted inspiratory loop (Item Q207).
Of the following, the MOST likely cause for this patients symptoms is

A. allergic rhinitis
B. asthma exacerbation
C. habit cough
D. sinusitis
E. vocal cord dysfunction

Copyright 2009 by the American Academy of Pediatrics

page 437

2009 PREP SA on CD-ROM


Critique: 207

Preferred Response: E

The teenager described in the vignette has signs and symptoms consistent with vocal cord
dysfunction (VCD), a condition that can mimic or coexist with asthma. In contrast to an asthma
exacerbation, the key features of VCD exhibited by this girl include a normal room air pulse
oximetry reading, failure to improve with her beta2 agonist inhaler, clear lungs, and difficulty with
inspiration instead of expiration. A blunted inspiratory loop on spirometry also is supportive of
VCD, although affected patients usually have normal spirometry readings when not experiencing
symptoms. Triggers for VCD can include viral upper respiratory tract infections, chemicals,
fumes, pollution, emotional changes, laughing, exercise, gastroesophageal reflux (GER), and
cold air.
GER can cause cough and be a trigger for asthma. It may worsen during exercise, eating,
or when supine. Although GER is a cause of chronic cough, the patient in the vignette does not
admit to GER symptoms, making this diagnosis unlikely.
Postnasal drip syndrome, now termed upper airway cough syndrome, can result in
coughing due to allergic rhinitis, nonallergic rhinitis, or sinusitis. The lack of nasal congestion,
rhinorrhea, or postnasal drip for this girl makes this an unlikely cause of her acute symptoms.
Psychogenic cough, also called habit cough syndrome, is a well-described chronic cough
that may begin after a viral upper respiratory tract infection. The cough usually is nonproductive
and does not occur during sleep. Teenagers who have asthma may use coughing as a method
to avoid school (factitious or malingering), but the girl in the vignette is not having symptoms
during a specific class or time of day.
References:
Liu AH, Covar RA, Spahn JD, Leung DYM. Childhood asthma. In: Kleigman RM, Behrman RE,
Jenson HB, Stanton BF, eds. Nelson Textbook of Pediatrics. 18th ed. Philadelphia, Pa:
Saunders Elsevier; 2007:953-969
Weinberger M, Abu-Hasan M. Pseudo-asthma: when cough, wheezing, and dyspnea are not
asthma. Pediatrics. 2007;120:855-864. Available at:
http://pediatrics.aappublications.org/cgi/content/full/120/4/855

Copyright 2009 by the American Academy of Pediatrics

page 438

2009 PREP SA on CD-ROM


Question: 208

A 7-year-old boy is brought to the emergency department because of altered mental status. His
parents report that he was well when he came home from school today, but when he came in
the house for dinner after playing outside with his friends, he complained of abdominal pain and
had an episode of nonbilious and nonbloody emesis. Over the next 30 minutes, he became
increasingly lethargic until his parents could not arouse him. They called emergency medical
services, and he was transported to the emergency department by ambulance. On physical
examination, he is unresponsive and drooling, his temperature is 98.8F (37.1C), heart rate is
50 beats/min, respiratory rate is 36 breaths/min, blood pressure is 100/60 mm Hg, and oxygen
saturation is 82% on room air. His pupils are mid-size and sluggishly reactive, and his breath
sounds are coarse bilaterally, with increased work of breathing. You suspect a toxin exposure.
Of the following, the MOST appropriate treatment of this patient is

A. atropine
B. N-acetylcysteine
C. naloxone
D. octreotide
E. physostigmine

Copyright 2009 by the American Academy of Pediatrics

page 439

2009 PREP SA on CD-ROM


Critique: 208

Preferred Response: A

The patient described in the vignette is exhibiting the classic symptoms of cholinergic poisoning.
These symptoms can be remembered using the mnemonic "SLUDGE" (salivation, lacrimation,
urination, diarrhea, gastric emesis) and are due to irreversible inhibition of acetylcholinesterase
and excess acetylcholine at the neuromuscular junction. The resulting overstimulation of
cholinergic receptors causes the muscarinic symptoms noted previously. Nicotinic symptoms,
which include muscle twitching, weakness, and paralysis, also may result. Organophosphate
exposure can occur though ingestion as well as dermal absorption and inhalation. These
compounds are found commonly in the home environment as components of lawn and garden
care products, scabicides, and interior insecticides.
The treatment of cholinergic poisoning involves patient stabilization, decontamination, and
administration of antidotes. Significantly symptomatic patients may require intubation for airway
protection and pulmonary toilet as well as assisted ventilation with 100% oxygen for respiratory
muscle weakness and hypoxia. Systemic decontamination with activated charcoal as well as
dermal decontamination should be performed. Dermal decontamination may be facilitated by
cleansing the skin with a dilute bleach solution. Contaminated clothes should be discarded
because laundering may not remove the toxin.
The mainstay of stabilization and treatment for cholinergic poisonings is the administration of
atropine. Atropine competes with acetylcholine at the cholinergic receptors and decreases the
muscarinic cholinergic effects. The doses of atropine used in this setting are higher than those
used for symptomatic bradycardia from other causes. An initial atropine dose of 0.05 mg/kg
should be administered and doubled every 3 to 5 minutes until the pulmonary muscarinic
symptoms (bronchorrhea, bronchospasm) are controlled. Nicotinic neuromuscular symptoms
are treated by adding pralidoxime, a cholinesterase reactivating agent, to atropine therapy.
N-acetylcysteine is used to treat acetaminophen poisoning, which causes few, if any, acute
symptoms. Naloxone is an opiate antagonist used to treat symptomatic opiate overdoses, which
are characterized by miosis, bradycardia, and respiratory depression without bronchorrhea.
Octreotide is a somatostatin analog that inhibits insulin release and is indicated in the treatment
of sulfonylurea overdoses, which cause profound hypoglycemia unresponsive to dextrose
administration. Physostigmine is a cholinergic agent that may be used in significantly
symptomatic anticholinergic poisonings. Because the patient's symptoms are not consistent with
overdoses of any of these drugs, treatment with these agents is not indicated.
References:
Bird S. Organophosphate and carbamate toxicity. UpToDate Online 15.3. 2008. Available for
subscription at:
http://www.utdol.com/utd/content/topic.do?topicKey=ad_tox/9425&selectedTitle=1~150&source=
search_result
Karr CJ, Solomon GM, Brock-Utne AC. Health effects of common home, lawn, and garden
pesticides. Pediatr Clin North Am. 2007;54:63-80. Abstract available at:
http://www.ncbi.nlm.nih.gov/pubmed/17306684
Peter JV, Moran JL, Graham PL. Advances in the management of organophosphate poisoning.
Expert Opin Pharmacother. 2007;8:1451-1464. Abstract available at:
http://www.ncbi.nlm.nih.gov/pubmed/17661728

Copyright 2009 by the American Academy of Pediatrics

page 440

2009 PREP SA on CD-ROM


Question: 209

You are evaluating a 2-day-old term infant because of abdominal distention. He fed normally the
first day after birth, but has had progressively increasing vomiting, which now is bilious. Physical
examination demonstrates upslanted palpebral fissures, a prominent tongue, and mild hypotonia.
Upon passage of a nasogastric tube, you aspirate 80 mL of green-yellow material from his
stomach. Abdominal radiographs, including a left lateral decubitus film, reveal dilated loops of
bowel and air-fluid levels but no evidence of pneumatosis (Item Q209).
Of the following, the condition that BEST explains this babys clinical findings is

A. duodenal atresia
B. Hirschsprung disease
C. meconium ileus
D. necrotizing enterocolitis
E. neonatal intussusception

Copyright 2009 by the American Academy of Pediatrics

page 441

2009 PREP SA on CD-ROM


Critique: 209

Preferred Response: B

The infant described in the vignette has clinical features of Down syndrome (Item C209A).
Infants who have Down syndrome are at risk for a number of gastrointestinal malformations,
most notably duodenal atresia and Hirschsprung disease. The air in the distal small bowel
apparent in the radiograph obtained for the infant in the vignette excludes duodenal atresia and
annular pancreas. The lack of pneumatosis on the radiograph for this term infant makes
necrotizing enterocolitis unlikely. Neonatal intussusception is extremely rare, and meconium
ileus is associated with cystic fibrosis, not Down syndrome.
Hirschsprung disease is characterized by congenital absence of a portion of the enteric
nervous system (aganglionosis). The aganglionic segment of bowel typically begins at the anal
verge and extends proximally. Disease limited to the rectosigmoid colon ("short segment"
Hirschsprung) accounts for 80% to 90% of cases. The remainder of cases can involve a larger
portion of the distal colon ("long segment Hirschsprung"), the entire colon ("total colonic
Hirschsprung"), or the colon and small bowel. The aganglionic segment of bowel is unable to
contract, leading to either severe constipation or a functional bowel obstruction.
It can be difficult to distinguish between functional constipation and Hirschsprung disease in
childhood. Some distinguishing features are shown in Item C209B. In general, patients who have
Hirschsprung disease have a history of delayed passage of meconium; 95% of affected infants
fail to pass meconium in the first day after birth. Stools of infants and toddlers who have
Hirschsprung disease often are very thin. Because of the hypertensive anal sphincter and
aperistaltic distal colon, affected children almost never have encopresis. On rectal examination,
an infant or child who has short segment Hirschsprung may have an "explosion of stool" after
the examiner's finger is removed.
If Hirschsprung disease is suspected strongly, the patient should undergo further diagnostic
testing. Anorectal manometry demonstrates a hypertensive anal sphincter that fails to relax.
Barium enema may demonstrate a narrowed rectosigmoid (aganglionic segment) (Item C209C)
and a dilated proximal colon, with a "transition zone" (area of caliber change between the normal
and aganglionic segment). The definitive test for Hirschsprung disease is rectal biopsy, which
demonstrates absent ganglion cells in the submucosa and muscularis propria of the rectum. If
Hirschsprung disease is identified, surgical resection of the aganglionic colon and anastomosis
of the normal colon to the anorectal canal is the treatment of choice.
References:
de Lorijn F, Kremer LC, Reitsma JB, Benninga MA. Diagnostic tests in Hirschsprung disease: a
systematic review. J Pediatr Gastroenterol Nutr. 2006;42:496-505. Abstract available at:
http://www.ncbi.nlm.nih.gov/pubmed/16707970
Imseis E, Gariepy CE. Hirschsprung disease. In: Walker WA, Goulet O, Kleinman RE, Sherman
PM, Shneider BL, Sanderson IR, eds. Pediatric Gastrointestinal Disease. 4th ed. Hamilton,
Ontario, Canada: BC Decker; 2004:1031-1043

Copyright 2009 by the American Academy of Pediatrics

page 442

2009 PREP SA on CD-ROM


Question: 210

You are called to the neonatal intensive care unit to examine a newborn who has abdominal
distention and respiratory distress. She was born at 38 weeks gestation and weighs 4 kg. Apgar
scores were 3 and 6 at 1 and 5 minutes, respectively. She required tracheal intubation and
assisted ventilation. On physical examination, she has a large, distended, and tense abdomen
without bowel sounds. The abdominal wall is not erythematous, and there is no clearly palpable
mass. She does not display other evidence of body wall or scalp edema. The breath sounds are
coarse and equal bilaterally. There is no heart murmur. Radiograph of the chest appears normal,
but abdominal radiography shows background granular density, paucity of intraluminal bowel
gas, and a calcified mass in the left lower quadrant (Item Q210).
Of the following, the BEST explanation for this infants abdominal findings is

A. congenital lymphangioma
B. erythroblastosis fetalis
C. meconium peritonitis
D. ovarian cyst
E. urinary ascites

Copyright 2009 by the American Academy of Pediatrics

page 443

2009 PREP SA on CD-ROM


Critique: 210

Preferred Response: C

The respiratory distress described for the infant in the vignette may be related to diaphragmatic
impingement from abdominal distention. Evaluation of the cause of abdominal distention includes
a plain radiograph that, in this case, has findings consistent with ascites and a calcified mass
(Item C210A). Subsequent abdominal ultrasonography may prove to be valuable diagnostically.
The mass is a meconium pseudocyst, indicative of a bowel rupture with contained (walled-off)
meconium that has become calcified. This condition, seen in meconium peritonitis, usually is
associated with meconium ileus and is a result of cystic fibrosis (CF).
An autosomal recessive disease, CF is believed to affect 1 in 2,000 to 4,000 live births of
white children. It occurs much less frequently in African Americans (estimated at 1 in 17,000 live
births) and is rare in infants of Asian descent (estimated at 1 in 90,000 live births). The disease
is related to altered epithelial cell ion transport and is associated with exocrine pancreatic
insufficiency and pulmonary decompensation in later life. More than 1,000 mutations (mapped to
the 7th chromosome) have been identified, but the most common defect is in a chloride channel
transmembrane transport protein regulated by the delta-F508 mutation, which accounts for more
than 70% of all CF cases. Problems that may present in the neonatal period include:
1. Meconium ileus, a thick meconium obstruction of the distal ileum characteristically
presenting clinically as a small bowel obstruction. This condition occurs in approximately 17% of
infants who have CF. A history may indicate polyhydramnios, a prenatal ultrasonographically
observed bowel dilation, or delayed postnatal passage of stool. Physical examination may reveal
abdominal distention and bile-stained emesis. A plain abdominal radiograph may show stacked
loops of variably dilated bowel, soap-suds bubbly-like appearance of meconium stool (most
often in the right lower quadrant), and a ground-glass character of the distal bowel with a relative
paucity of distal bowel gas. A contrast enema may reveal a microcolon and failure to pass
contrast beyond the ileocecal valve. Surgical exploration and removal of inspissated meconium
is generally necessary, as may be a temporizing ileostomy and later reanastamosis.
2. Meconium peritonitis is associated with bowel obstruction leading to perforation and
spillage of meconium into the peritoneal cavity, occasionally involving a walled-off calcified
meconium "pseudocyst" apparent on plain abdominal radiograph or abdominal ultrasonography.
The perforation may have occurred in utero and may be associated with fetal ascites or
hydrops, pulmonary hypoplasia and respiratory failure, and impaired bowel or liver function. The
newborn may have ascites or anasarca on physical examination, with a tense, distended
abdomen. Plain abdominal radiography may reveal a diffuse ground-glass density and little
intraluminal bowel gas; there also may be diffuse or focal calcification (Item C210B). Medical
management is directed at stabilizing pulmonary function, fluid and electrolyte balance, and
hepatic function before addressing the surgical condition.
3. Prolonged jaundice with a predominance of conjugated hyperbilirubinemia also is seen in
infants who have CF and may result from inspissated bile or prolonged use of parenteral
nutrition.
Congenital lymphangiomas may present with ascites and hydrothoraces requiring drainage;
analysis of the fluid confirms the diagnosis. Erythroblastosis fetalis is an isoimmune hemolytic
condition that involves fetal hydrops (neonatal anasarca) and profound anemia, but it is not
related to bowel obstruction and does not result in meconium peritonitis. Ovarian cysts may be
massive, occasionally resulting in bowel compression and extrinsic obstruction that can require
surgical excision. They are discernible by abdominal and pelvic ultrasonography as being
distinct from the bowel lumen. Urinary ascites develops following urinary tract obstruction, with
perforation and leakage of urine into the peritoneal space. It occurs most commonly in boys who
have posterior urethral valves. Analysis of the ascites fluid reveals an elevated creatinine
concentration.
References:
Albanese CT, Sylvester KG. Pediatric surgery. In: Doherty GM, Way LW. Current Surgical
Diagnosis and Treatment. 12th ed. New York, NY: The McGraw-Hill Companies, Inc; 2006:chap
45

Copyright 2009 by the American Academy of Pediatrics

page 444

2009 PREP SA on CD-ROM

Chaudry G, Navarro OM, Levine DS, Oudjhane K. Abdominal manifestations of cystic fibrosis in
children. Pediatr Radiol. 2006;36:233-240. Abstract available at:
http://www.ncbi.nlm.nih.gov./pubmed/16391928
Davis PB. Cystic fibrosis. Pediatr Rev. 2001;22:257-264. Available at:
http://pedsinreview.aappublications.org/cgi/content/full/22/8/257
Kaye CI and the Committee on Genetics. Newborn screening fact sheets. Pediatrics.
2006;118:e934-e963. Available at:
http://pediatrics.aappublications.org/cgi/content/full/118/3/e934

Copyright 2009 by the American Academy of Pediatrics

page 445

2009 PREP SA on CD-ROM


Question: 211

A 13-year-old girl comes to your office with a 1-day history of right eye pain and tearing. She
denies trauma, but says she rubbed her eyes a lot the day before because it was windy outside.
Her right bulbar and palpebral conjunctivae are very injected, and copious clear discharge is
present. There is no hyphema, and the pupils are normal. She complains of pain with the eye
examination. After applying fluorescein to the eye, you see a single linear abrasion on the
cornea. When you evert the eyelid, you find no foreign body.
Of the following, the MOST appropriate management for this condition is

A. oral analgesic
B. oral antistaphylococcal antibiotic
C. tight patching of the eye
D. topical anesthetic drops
E. topical steroid drops

Copyright 2009 by the American Academy of Pediatrics

page 446

2009 PREP SA on CD-ROM


Critique: 211

Preferred Response: A

Corneal abrasions occur commonly in children as a result of vigorous rubbing of the eye or a
foreign body. Symptoms are tearing, photophobia, and pain. The physical examination may
reveal injection of the conjunctivae, copious clear discharge, or a dull corneal light reflex. It is
important to perform fluorescein dye staining in children suspected of having a corneal abrasion.
This dye stains the damaged cornea but is not taken up by intact corneal epithelium. The
staining is seen best with a Wood lamp (Item C211), but large abrasions may be visible with
regular white light or an ophthalmoscope. The presence of a linear abrasion, as described for
the girl in the vignette, suggests the possibility of a retained foreign body underneath the eyelid,
and care should be taken to evert the eyelid to evaluate for this.
Topical anesthetic drops, such as tetracaine, may be used to facilitate the eye examination
and may provide significant relief to the patient, but providing these for home use is not
recommended because they may slow healing and mask persistence of symptoms. Oral
analgesics usually are all that is needed to control pain while the abrasion heals. Topical
nonsteroidal anti-inflammatory drops also may provide relief, but topical steroids are not
indicated in the management of uncomplicated corneal abrasions (due to concerns about
adverse effects on wound healing and exacerbation of coexisting viral infection). Antibiotic
ointment may provide lubrication until the abrasion has healed, but the risk of secondary
infection is low, so routine use of antibiotics is not needed. Patching does not speed the healing
process and has been shown to interfere with activities of daily living; it no longer is
recommended for routine care of simple corneal abrasions.
Most corneal abrasions heal within 2 to 3 days, so if symptoms continue beyond this period,
evaluation by an ophthalmologist is warranted to rule out secondary infection or retained foreign
body.
References:
Calder LA, Balasubramanian S, Fergusson D. Topical nonsteroidal anti-inflammatory drugs for
corneal abrasions: meta-analysis of randomized trials. Acad Emerg Med. 2005;12: 467-473.
Abstract available at: http://www.ncbi.nlm.nih.gov/pubmed/15860701
Michael JG, Hug D, Dowd MD. Management of corneal abrasion in children: a randomized
clinical trial. Ann Emerg Med. 2002;40:67-72. Abstract available at:
http://www.ncbi.nlm.nih.gov/pubmed/12085075
Stout AU. Technical tip: corneal abrasions. Pediatr Rev. 2006;27:433-434. Available at:
http://pedsinreview.aappublications.org/cgi/content/full/27/11/433
Turner A, Rabiu M. Patching for corneal abrasion. Cochrane Database Syst Rev.
2006;2:CD004764. Available at:
http://www.mrw.interscience.wiley.com/cochrane/clsysrev/articles/CD004764/frame.html

Copyright 2009 by the American Academy of Pediatrics

page 447

2009 PREP SA on CD-ROM


Question: 212

A previously healthy 15-year-old girl returns from summer camp in the mountains complaining of
dysuria, frequency, and urgency. You diagnose cystitis and prescribe trimethoprimsulfamethoxazole. Her mother phones 3 days later to report that the girl is very tired and
appears pale. You advise her mother to bring her to your office. On examination, she appears
pale and your order laboratory tests. The girls hemoglobin is 8.5 g/dL (85.0 g/L), a decrease
from the value of 11.5 g/dL (115.0 g/L) that was measured during her pre-camp physical
examination. Her reticulocyte count is 5.0% (0.050), and the red cell indices are normal except
for mild microcytosis with a mean corpuscular volume of 76 fL. You review a smear (Item
Q212).
Of the following, the MOST likely cause of this girls rapid onset of anemia is

A. glucose-6-phosphate dehydrogenase deficiency


B. hemoglobin SC disease
C. hereditary elliptocytosis
D. inadequate dietary iron
E. pyelonephritis

Copyright 2009 by the American Academy of Pediatrics

page 448

2009 PREP SA on CD-ROM


Critique: 212

Preferred Response: A

The girl described in the vignette has a peripheral blood smear and clinical signs and symptoms
consistent with hemolytic anemia following the administration of trimethoprim-sulfamethoxazole.
In people who have glucose-6-phosphatase dehydrogenase (G6PD) deficiency, hemolysis can
occur after the administration of sulfonamides and other antibiotics.
G6PD deficiency is the most common disease-producing enzymopathy in humans. Inherited
as an X-linked disorder, primarily as single-base mutations in the G6PD locus at Xq28, G6PD
deficiency affects 400 million people worldwide. The highest prevalence rates (with gene
frequencies from 5% to 25%) are found in tropical Africa, the Middle East, tropical and
subtropical Asia, some areas of the Mediterranean, and Papua New Guinea. The defect occurs
in approximately 13% of African American males. Homozygous women are found in populations
in which the frequency of G6PD deficiency is high. Heterozygous (carrier) women may develop
hemolytic attacks. The gene confers some protection against malaria, which probably accounts
for its high gene frequency in modern populations.
The G6PD enzyme catalyzes the oxidation of glucose-6-phosphate to 6-phosphogluconate
while reducing the oxidized form of nicotinamide adenine dinucleotide phosphate (NADP+) to
nicotinamide adenine dinucleotide phosphate (NADPH). NADPH, a required cofactor in many
biosynthetic reactions, maintains glutathione in its reduced form.
Reduced glutathione is a scavenger for dangerous oxidative metabolites in the cell. Red
blood cells depend on G6PD activity as the only protection against oxidative stresses. People
deficient in G6PD, therefore, are at risk for hemolysis and its sequelae when exposed to
oxidative stress. The degree of hemolysis varies with dose of the inciting agent.
The enzymatic defect in Americans of African descent is statistically less severe than that of
people of Mediterranean descent. The enzyme also is found in lower quantities in older red blood
cells due to senescence of the enzyme.
Affected individuals typically do not display symptoms or signs in the absence of oxidative
stress. However, those who have very low enzyme concentrations may present with neonatal
jaundice and acute hemolytic anemia. Neonatal jaundice usually appears by age 1 to 4 days, at
the same time as or slightly earlier than so-called physiologic jaundice; kernicterus is rare. Acute
hemolytic anemia results from stress factors such as oxidative drugs or chemicals, infection, or
ingestion of fava beans (common to a Mediterranean diet).
Gallstones may be a prominent feature in affected individuals. As with hemolysis of different
causes, jaundice and splenomegaly may be present during a crisis. Immediately after an
episode, younger red blood cells are released that contain a higher concentration of enzyme.
Accordingly, testing should be delayed in the event of undiagnosed hemolysis in children for a
few weeks after the hemolysis to allow G6PD to decrease to normal concentrations.
The clinician should have a high suspicion for G6PD deficiency in immigrants of the ethnic
groups noted previously and consider testing potentially affected males who exhibit hemolysis
as part of a significant illness (such as diabetic acidosis, hepatitis) or trauma, especially prior to
administering medications that may precipitate hemolysis. Precipitants include antibacterials
(especially sulfonamides), antimalarials (chloroquine, primaquine), and other medications
(aspirin, vitamin K analogs). Chemicals that may induce hemolysis include naphthalene, which is
found in mothballs.
Neonatal screening and health education have been effective in some countries at detecting
individuals at risk and preventing hemolysis. Widespread screening of asymptomatic individuals
is rare in the United States because most affected individuals have only a mild form of the
disease.
Hemoglobin SC disease and hereditary elliptocytosis are unlikely diagnoses in a previously
healthy 15-year-old girl because these conditions are associated with clinical disease in early
childhood. In addition, hemoglobin SC disease often is diagnosed in the neonatal period because
of the availability of neonatal screening. Hereditary elliptocytosis (Item C212) also is detected in
infancy and early childhood based on hemolysis without stressors. Iron deficiency does not
cause hemolysis or jaundice. Pyelonephritis may result from ascending infection from cystitis
but normally is associated with fever and other constitutional symptoms in addition to fatigue.
Hemolysis can result from infection in persons unaffected by G6PD deficiency but usually not to
Copyright 2009 by the American Academy of Pediatrics

page 449

2009 PREP SA on CD-ROM

the degree noted in the child described in the vignette.


References:
Carter SM, Gross SJ. Glucose-6-phosphate dehydrogenase deficiency. eMedicine Specialties,
Medicine, Ob/Gyn, Psychiatry, and Surgery, Hematology. 2005. Available at:
http://www.emedicine.com/med/topic900.htm
Frank JE. Diagnosis and management of G6PD deficiency. Am Fam Physician. 2005;72:12771282. Available at: http://www.aafp.org/afp/20051001/1277.html
Segal GB. Enzymatic defects. In: Kliegman RM, Behrman RE, Jenson HB, Stanton BF, eds.
Nelson Textbook of Pediatrics. Philadelphia, Pa: Saunders Elsevier; 2007:2039-2041
Segel GB, Hirsh MG, Feig SA. Managing anemia in a pediatric office practice: part 2. Pediatr
Rev. 2002;23:111-122. Available at:
http://pedsinreview.aappublications.org/cgi/content/full/23/4/111

Copyright 2009 by the American Academy of Pediatrics

page 450

2009 PREP SA on CD-ROM


Question: 213

A 13-year-old boy who has a bicuspid aortic valve and aortic stenosis with a 20-mm Hg (mild)
gradient by echocardiography (Item Q213) is interested in participating in sports. He asks for
your advice.
Of the following, the BEST response is that

A. aerobic activities should be well tolerated


B. football is contraindicated
C. he should not participate in weight training
D. he should refrain from highly competitive sports
E. wrestling is contraindicated

Copyright 2009 by the American Academy of Pediatrics

page 451

2009 PREP SA on CD-ROM


Critique: 213

Preferred Response: A

Congenital heart disease is the most commonly encountered malformation, with an incidence of
approximately 8 in 1,000 live births or slightly less than 1%. Today, congenital heart disease is
diagnosed early, often in utero, and medical, catheter-based, and surgical management
strategies have progressed to a point that most affected children lead active and healthy lives.
Exercise and sports participation are healthy and normal components of many children's lives,
including those affected by congenital heart disease. Various professional organizations have
recommended levels of sports participation based on the underlying cardiac malformation and its
degree of severity.
The most common congenital heart abnormality is a bicommissural (bicuspid) aortic valve
(Item C213), as described for the boy in the vignette. Although many of the children who have
bicommissural valves do not have associated aortic stenosis during childhood, it is the most
common cause of aortic stenosis in children. The pressure gradient across the aortic valve
during systole determines the severity of the stenosis. Pressure gradients that are less than 25
mm Hg generally are considered mild, those between 25 and 50 mm Hg are considered
moderate, and those in excess of 50 mm Hg are considered severe. Patients who have aortic
stenosis should be monitored regularly, with electrocardiography and echocardiography
supplementing a thorough history and physical examination because aortic stenosis can
progress with patient growth and age. Patients who have mild aortic stenosis, such as the boy
described in the vignette, generally have no restrictions on physical activity and exercise.
Children who have moderate aortic stenosis may participate in self-limited aerobic activities and
should avoid isometric activities such as weightlifting and wrestling, which increase the systemic
afterload and workload of the left ventricle.
The boy in the vignette should be able to play football or any other competitive sport if the
remainder of his physical examination and cardiac evaluation is benign. He also may participate
in weight training, particularly the lighter weights that build endurance and muscle speed rather
than the heavier weights that build large muscle bulk. Wrestling is not contraindicated for children
who have asymptomatic mild aortic stenosis, although it may be in patients who have moderate
aortic stenosis.
References:
Dickhuth H-H, Kececioglu D, Schumacher YO. FIMS Position Statement: Congenital Heart
Disease and Sports. International Federation of Sports Medicine; January 2006. Available at:
http://www.fims.org/default.asp?PageID=120975716
Maron BJ, Chaitman BR, Ackerman MJ, et al. Recommendations for physical activity and
recreational sports participation for young patients with genetic cardiovascular diseases.
Circulation. 2004;109:2807-2816. Available at:
http://circ.ahajournals.org/cgi/content/full/109/22/2807
Singh A, Silberbach M. Consultation with the specialist: cardiovascular preparticipation sports
screening. Pediatr Rev. 2006;27:418-424. Available at:
http://pedsinreview.aappublications.org/cgi/content/full/27/11/418
Stefani L, Galanti G, Tonicelli L, et al. Bicuspid aortic valve in competitive athletes. Br J Sports
Med. 2008;42:31-35. Abstract available at: http://www.ncbi.nlm.nih.gov/pubmed/17548371

Copyright 2009 by the American Academy of Pediatrics

page 452

2009 PREP SA on CD-ROM


Question: 214

The mother of a 7-year-old girl who has epilepsy phones your office because her child has
developed a rash. The mother is worried that the rash may be due to her new antiseizure
medication.
Of the following, the MOST appropriate next step is to

A. defer evaluation until the next health supervision visit


B. examine the child in your office promptly
C. refer the child to a dermatologist
D. refer the child to her neurologist
E. send the child to the laboratory to obtain a complete blood count

Copyright 2009 by the American Academy of Pediatrics

page 453

2009 PREP SA on CD-ROM


Critique: 214

Preferred Response: B

Antiseizure medications can cause a wide variety of adverse effects, including central nervous
system effects such as sedation, dizziness, personality changes, and occasionally, worsening
of seizures. The neurologist, psychiatrist, or other physician who prescribes such medications
should be familiar with adverse effect profiles of these medications. Adverse effects of
antiseizure medications may occur outside the central nervous system as well. Bone marrow
suppression and liver toxicity are among the more serious possible systemic effects.
Rashes are common in children and can occur as an adverse effect of most antiseizure
medications. Such drug rashes tend to occur during the first month of use, as described for the
girl in the vignette. Rapid assessment of the child is important because in some cases, the rash
may be a harbinger of a more global, serious allergic reaction or of Stevens-Johnson syndrome
(erythema multiforme major) (Item C214). A general pediatrician often is better trained than a
neurologist in the identification of the many rashes that can occur in childhood. Therefore,
prompt assessment should be undertaken by the primary care physician in the office. A
dermatologist referral may be needed, and laboratory testing may be useful, but these decisions
should be made after the clinical assessment by the child's primary care physician.
References:
French JA, Kanner AM, Bautista J, et al. Efficacy and tolerability of the new antiepileptic drugs I:
treatment of new onset epilepsy. Report of the Therapeutics and Technology Assessment
Subcommittee and Quality Standards Subcommittee of the American Academy of Neurology and
the American Epilepsy Society. Neurology. 2004;62:1252-1260. Available at:
http://www.neurology.org/cgi/content/full/62/8/1252
Johnston MV. Seizures in childhood. In: Kliegman RM, Behrman RE, Jenson HB, Stanton BF,
eds. Nelson Textbook of Pediatrics. 18th ed. Philadelphia, Pa: Saunders Elsevier; 2007:24572475

Copyright 2009 by the American Academy of Pediatrics

page 454

2009 PREP SA on CD-ROM


Question: 215

A newborn male experiences prolonged oozing following circumcision. Hematologic evaluation


reveals that he has less than 1% of factor VIII clotting activity and a prolonged partial
thromboplastin time, consistent with severe hemophilia A. His family history is negative for any
individuals affected by clotting disorders.
Of the following, the MOST accurate statement for counseling this childs parents is that

A. another family member likely is affected, but the condition is so mild that the person has not
been diagnosed

B. in families such as this, 50% of affected boys have a spontaneous gene mutation
C. molecular genetic testing can detect mutations in 50% of affected individuals
D. severe hemophilia is the rarest of all types
E. there is an 80% chance that the mother is a hemophilia carrier

Copyright 2009 by the American Academy of Pediatrics

page 455

2009 PREP SA on CD-ROM


Critique: 215

Preferred Response: E

One third to one half of all males who have hemophilia A have no family history of the disorder,
as described for the infant in the vignette. In these cases, it is important to recognize that
multiple genetic scenarios can result in such an outcome.
In about 20% of the cases involving no previous family history for hemophilia, the mother is
not a carrier, in which case her son has a de novo disease-causing mutation. In some of these
situations, the son is a somatic mosaic for the mutation, ie, the mutation is not present in all of
the cells of his body.
In 80% of such cases, the mother is a carrier, and she might have a de novo gene mutation.
This mutation could have occurred in the egg or sperm cell from which she was conceived (in
which case it is present in all of her cells). Alternatively, it may be due to a somatic mutation that
occurred in early embryogenesis or to germline mosaicism, in which case it is present in only
some of her germ cells. The mother could have inherited the disease-causing mutation from her
mother or unaffected father (if he is a somatic or germline mosaic). Finally, the mother could
have inherited the mutation from a previous generation, which may have been passed on only
through daughters, for example, so that no one was affected with hemophilia A.
Cases in which the mutation for hemophilia A is present in a mosaic state are unusual, and
the gene mutation runs true in families. Therefore, every boy in a family who has the same
mutation will have the same severity of disease.
Severe hemophilia (<1% normal factor VIII activity) is the most common of all types of
hemophilia A. Molecular genetic testing can detect mutations in 98% of those who have severe
disease.
References:
Hamosh A. Clinical case studies illustrating genetic principles. In: Nussbaum RL, McInnes RR,
Willard HR, eds. Thompson & Thompson Genetics in Medicine. 7th ed. Philadelphia, Pa: Elsevier
Saunders; 2007:268-269
Johnson MJ, Thompson AR. Hemophilia A. GeneReviews. 2005. Available at:
http://www.geneclinics.org/servlet/access?db=geneclinics&site=gt&id=8888891&key=xAcWBcrj
mZrVo&gry=&fcn=y&fw=8cy5&filename=/profiles/hemo-a/index.html

Copyright 2009 by the American Academy of Pediatrics

page 456

2009 PREP SA on CD-ROM


Question: 216

An 18-year-old young woman reports that she has "bumps" in her vaginal area. She recently
became sexually active with a single partner. She says that the lesions are not tender, and she
has no vaginal discharge or itching. Genital examination reveals several clusters of fleshcolored, pedunculated lesions, primarily in the posterior fourchette, compatible with genital warts
(Item Q216). You counsel her about treatment options.
Of the following, the MOST accurate statement regarding management and treatment of genital
warts is that

A. human papillomavirus infectivity is eradicated by treatment of external warts


B. no definitive evidence supports the superiority of any of the available genital wart treatments
C. single treatment with clinician- or patient-applied methods eradicates all lesions in most
patients

D. waiting for spontaneous resolution of warts is not acceptable


E. with her lesions, the patient is not currently a candidate for the human papillomavirus vaccine

Copyright 2009 by the American Academy of Pediatrics

page 457

2009 PREP SA on CD-ROM


Critique: 216

Preferred Response: B

Human papillomavirus (HPV) is the cause of genital warts, which usually appear as flat, papular,
or pedunculated lesions of the genital mucosa (Item C216). Research data indicate that
currently available therapies for the treatment of genital warts might reduce, but probably do not
eradicate, HPV infectivity. When no treatment is given, genital warts may resolve spontaneously,
remain unchanged, or increase in number or size. An acceptable alternative for some patients is
to forego treatment and wait for spontaneous resolution. No definitive evidence suggests that
any of the available treatments, including topical patient- or clinician-applied medications
(podofilox, imiquimod, podophyllin resin, or bichloro- or trichloroacetic acid), cryotherapy,
surgical removal, intralesional interferon, or laser therapy, is superior to any other. Most patients
require a course of therapy rather than a single treatment to eradicate warts. Vaccination
against HPV still is recommended for a patient who has external warts because the vaccine can
protect against other strains of HPV to which the patient may not have been exposed.
References:
Markowitz LE, Dunne EF, Saraiya M, Lawson HW, Chesson H, Unger ER. Quadrivalent human
papillomavirus vaccine: recommendations of the Advisory Committee on Immunization Practices
(ACIP). MMWR Recomm Rep. 2007;56(RR02):1-24. Available at:
http://www.cdc.gov/mmwr/preview/mmwrhtml/rr5602a1.htm
Workowski KA, Berman SM, Centers for Disease Control and Prevention. sexually transmitted
diseases treatment guidelines, 2006. MMWR Recomm Rep. 2006;55(RR11):1-94. Available at:
http://www.cdc.gov/mmwr/preview/mmwrhtml/rr5511a1.htm

Copyright 2009 by the American Academy of Pediatrics

page 458

2009 PREP SA on CD-ROM


Question: 217

A 2-year-old girl is brought to the emergency department after being found unconscious at her
grandparents home. Her mother reports that she was in her usual good health when she was
dropped off at her grandparents 2 hours ago and that there is no history of trauma. Of note, the
grandmother found a spilled, opened bottle of her "blood pressure" medicine in the bathroom. On
physical examination, the girl is somnolent but arouses with stimulation. There is no sign of
trauma on physical examination. Her temperature is 98.0F (37.0C), heart rate is 60 beats/min,
respiratory rate is 25 breaths/min, and oxygen saturation is 93% on room air. Her pupils are 2
mm and reactive bilaterally. Her mouth and mucous membranes are dry, and she has no
rashes. You order serum electrolyte measurement and a urine toxicology screen.
Of the following, the MOST appropriate additional tests to obtain are

A. chest radiography and electrocardiography


B. computed tomography (CT) scan of the abdomen and electrocardiography
C. CT scan of the head and electrocardiography
D. CT scan of the head and lumbar puncture
E. electrocardiography and skull radiographs

Copyright 2009 by the American Academy of Pediatrics

page 459

2009 PREP SA on CD-ROM


Critique: 217

Preferred Response: C

A child who has an altered level of consciousness can present a diagnostic dilemma. Patient
age, a clear history of the present illness, and a comprehensive past medical history can help
guide the diagnostic evaluation and narrow the differential diagnosis. A useful guide for
remembering potential causes of an altered level of consciousness is the mnemonic AEIOU
TIPS (tips on the vowels) (Item C217).
Toxic ingestions are a common cause of an acute altered level of consciousness in a
toddler. Serum and urine toxicology screens should be performed if the ingestion is not
witnessed because multiple drugs may have been ingested. The previously healthy child
described in the vignette has signs and symptoms consistent with a clonidine overdose, as
evidenced by her being awakened with stimulation and having dry mucous membranes and
pupillary constriction. The onset of action of this drug is less than 1 hour, and toxic effects
include neurologic and respiratory depression, hypotonia, bradycardia, hypotension, and cardiac
arrhythmias. Treatment is supportive, with both intensive care unit monitoring and baseline
electrocardiography indicated. In this case, CT scan of the head also is indicated because the
child might have had an unwitnessed fall.
Primary pulmonary processes rarely present as an altered level of consciousness unless
there is an associated oxygen deficiency. Therefore, chest radiography is not necessary in this
case. Abdominal CT scan is indicated if abdominal trauma or an intra-abdominal infectious
process is suspected. Lumbar puncture should be undertaken if meningitis is suspected, but the
child in the vignette is afebrile and was reported in good health just 2 hours ago. Skull
radiography has limited utility in the evaluation of an altered level of consciousness.
References:
Avner JR. Altered states of consciousness. Pediatr Rev. 2006:27:331-338. Available at:
http://pedsinreview.aappublications.org/cgi/content/full/27/9/331
Frankel LR. Neurological emergencies and stabilization. In: Kliegman RM, Behrman RE, Jenson
HB, Stanton BF, eds. Nelson Textbook of Pediatrics. 18th ed. Philadelphia, Pa: Saunders
Elsevier: 2007:405-412

Copyright 2009 by the American Academy of Pediatrics

page 460

2009 PREP SA on CD-ROM


Question: 218

The parents of a 6-year-old boy are concerned because he has been developing pubic hair over
the past 6 months. On physical examination, you note a recent growth spurt, Sexual Maturity
Rating 3 pubic hair, a penis that is 8 cm in length and androgenized, and testes that are 5 mL in
volume. Other findings are normal. His bone age is 7 years. You order measurements of serum
testosterone, 17-hydroxyprogesterone, dehydroepiandrosterone, luteinizing hormone, and
follicle-stimulating hormone.
Of the following, the MOST important additional test is measurement of serum

A. adrenocorticotropic hormone
B. estradiol
C. free testosterone
D. human chorionic gonadotropin
E. prolactin

Copyright 2009 by the American Academy of Pediatrics

page 461

2009 PREP SA on CD-ROM


Critique: 218

Preferred Response: D

The child described in the vignette has sexual precocity, with testes that are increased in
volume and definite evidence of increased phallus size and pubic hair. Therefore, it is likely that
the increased androgen is being produced by the child's testes. Such production could be
related to autonomous testicular functioning, as in "testitoxicosis," or gonadotropin-independent
sexual precocity, but it is more likely related to testicular stimulation by gonadotropins. With true
central sexual precocity, luteinizing hormone (LH), follicle-stimulating hormone (FSH), and
circulating concentrations of testosterone are elevated. However, testosterone also could be
produced if the testes are stimulated by human chorionic gonadotropin (HCG), which can mimic
LH and stimulate growth of the Leydig cells that produce testosterone. HCG may be produced
by germ cell or other tumors located in the central nervous system, the mediastinum, the liver
(hepatoblastomas), and other locations. This hormone should be measured specifically in the
blood as beta-HCG. Elevation of HCG concentrations causes pubertal change in boys but not in
girls because girls require LH and FSH to stimulate ovarian estrogen production. Boys who have
sexual precocity as a result of HCG secretion have a smaller testicular volume than expected
for pubertal stage because FSH-stimulated Sertoli cell numbers do not increase.
Measurement of adrenocorticotropic hormone is not useful because concentrations of this
hormone fluctuate with stress and are elevated persistently only in the presence of Cushing
disease or adrenal insufficiency, not precocious puberty. Serum estradiol values are likely to be
slightly elevated because testosterone is converted to estrogen peripherally in fat and in the
liver, so if testosterone or other androgen concentrations are elevated, so are estrogens.
Estrogen values in this child are likely to be low because no breast enlargement is reported.
Measurement of free testosterone is unlikely to be useful in a child whose physical examination
reveals so much androgen effect. Free testosterone measurement may be useful to examine
androgen effect when normal testosterone concentrations are associated with mild clinical
hyperandrogenism in women. Serum prolactin does not stimulate production of androgen in
boys. Sometimes, prolactinomas are associated with mild hyperandrogenism and irregular
menses or amenorrhea in women.
References:
Ferry RJ Jr. Precocious pseudopuberty. eMedicine Specialties, Pediatrics: General Medicine,
Endocrinology. 2007. Available at: http://www.emedicine.com/ped/topic1881.htm
Kaplowitz P. Precocious puberty. eMedicine Specialties, Pediatrics: General Medicine,
Endocrinology. 2007. Available at: http://www.emedicine.com/ped/topic1882.htm
Muir A. Precocious puberty. Pediatr Rev. 2006;27:373-381. Available at:
http://pedsinreview.aappublications.org/cgi/content/full/27/10/373
Rivarola MA, Belgorsky A, Mendilaharzu H, Vidal G. Precocious puberty in children with tumours
of the suprasellar and pineal areas: organic central precocious puberty. Acta Paediatr.
2001;90:751-756. Abstract available at: http://www.blackwellsynergy.com/doi/abs/10.1111/j.1651-2227.2001.tb02800.x?journalCode=apa
Saenger P. Overview of precocious puberty. UpToDate Online 15.3. 2008. Available for
subscription at: http://www.uptodateonline.com/utd/content/topic.do?topicKey=pediendo/14867

Copyright 2009 by the American Academy of Pediatrics

page 462

2009 PREP SA on CD-ROM


Question: 219

During the health supervision visit for an infant, her mother mentions that the child has been
tolerating solid foods with no problem. When placed on her back to be examined, she brings her
feet to her mouth. Her mother holds a small mirror to the childs face to distract her during your
examination, and the baby reaches for the mirror and pats her image.
Of the following, these developmental milestones are MOST typical for an infant whose age is

A. 2 months
B. 4 months
C. 6 months
D. 9 months
E. 12 months

Copyright 2009 by the American Academy of Pediatrics

page 463

2009 PREP SA on CD-ROM


Critique: 219

Preferred Response: C

Tolerating solid foods well, placing her feet to her mouth while supine, and reaching for a mirror
and patting the image are socioemotional milestones most typical of a 6-month-old child. At 2
months of age, infants bring their hands to their mouths, swipe at dangling objects with their
hands, and grasp and shake hand toys. A 4-month-old infant has a spontaneous social smile
and, when shown a mirror, smiles and vocalizes. An infant of this age also brings items to his or
her mouth, can maintain an upright position if placed upright with the assistance of pillows, and
knows the difference between strangers and family. A 9-month-old infant can imitate nursery
games in response to a mother's demonstrations and may give a toy in response to a request. A
12-month-old infant points for requests, may throw a toy in play or refusal, helps with dressing,
may hug a stuffed animal, and may offer a toy to an image in a mirror.
References:
Knobloch H, Stevens FM, Malone AF. The revised developmental stages. In: Manual of
Developmental Diagnosis: The Administration and Interpretation of the Revised Gesell and
Amatruda Developmental and Neurologic Examination. Albany, NY: Developmental Evaluation
Materials, Inc; 1987:17-120
Whitaker T, Palmer F. The developmental history. In: Accardo PJ. Capute & Accardo's
Neurodevelopmental Disabilities in Infancy and Childhood. Volume I: Neurodevelopmental
Diagnosis and Treatment. 3rd ed. Baltimore, Md: Paul H. Brookes Publishing Co; 2008:297-310

Copyright 2009 by the American Academy of Pediatrics

page 464

2009 PREP SA on CD-ROM


Question: 220

You admitted a patient to the hospital yesterday who had acute onset of fever (temperature of
103.0oF [39.4oC]), a petechial rash, meningismus, and shock. She required blood pressure
support and mechanical ventilation during the night. As per the protocol for your hospital, you
placed this child into respiratory isolation upon admission. Today you are told that her blood
culture is growing Neisseria meningitidis. The nurse taking care of her asks you how long the
child needs to remain in respiratory isolation.
Of the following, the BEST answer is until the child

A. completes 1 day of antimicrobial therapy


B. defervesces
C. is clinically stable
D. is extubated
E. is proven not to have meningitis

Copyright 2009 by the American Academy of Pediatrics

page 465

2009 PREP SA on CD-ROM


Critique: 220

Preferred Response: A

Standard isolation precautions should be observed for patients who have meningococcal
disease, and they should remain in respiratory isolation until 24 hours after the initiation of
effective antimicrobial therapy. The presence of continued fever, prolonged intubation, clinical
instability, or meningitis is irrelevant to the issue of isolation. Patients treated for invasive disease
with any antimicrobial agent other than cefotaxime or ceftriaxone also must receive 2 days of
rifampin, one dose of ceftriaxone, or a single dose of ciprofloxacin prior to hospital discharge to
eradicate the nasopharyngeal carriage of Neisseria meningitidis. Family members and
individuals who have had close contact with the patient also should be given chemoprophylaxis.
Exposed individuals who have received either the polysaccharide or conjugate meningococcal
vaccine still should receive chemoprophylaxis because the vaccine may not contain the
serotype that is specific for the patient's illness (eg, serotype B).
Invasive disease caused by N meningitidis usually presents in one of three forms.
Meningococcemia is characterized by symptoms suggestive of an upper respiratory tract
infection and complaints of headache, lethargy, myalgias, or joint pain. Within hours, a petechial
or purpuric rash (Item C220) may develop, as might circulatory collapse. Meningococcal
meningitis often begins with similar symptoms to meningococcemia, but affected children
become irritable and have signs of meningeal irritation on physical examination. Occult
bacteremia with N meningitidis also has been described. Affected children present with a febrile
illness, and a blood culture grows N meningitidis, but the children never progress to the clinical
picture of fulminant meningococcemia.
References:
American Academy of Pediatrics. Meningococcal infections. In: Pickering LK, Baker CJ, Long
SS, McMillan JA, eds. Red Book: 2006 Report of the Committee on Infectious Diseases. 27th ed.
Elk Grove Village, Ill: American Academy of Pediatrics; 2006:452-460
Bilukha OO, Rosenstein N. Prevention and control of meningococcal disease: recommendations
of the Advisory Committee on Immunization Practices (ACIP). MMWR Recomm Rep.
2005;54(RR07):1-21. Available at: http://www.cdc.gov/mmwr/preview/mmwrhtml/rr5407a1.htm
Hugo B. Index of suspicion: case 4. Pediatr Rev. 2005;26:295-301. Available at:
http://pedsinreview.aappublications.org/cgi/content/full/26/8/295
Notice to readers: revised recommendations of the Advisory Committee on Immunization
Practices to vaccinate all persons aged 11-18 years with meningococcal conjugate vaccine.
MMWR Morbid Mortal Wkly Rep. 2007;56:794-795. Available at:
http://www.cdc.gov/mmwr/preview/mmwrhtml/mm5631a3.htm

Copyright 2009 by the American Academy of Pediatrics

page 466

2009 PREP SA on CD-ROM


Question: 221

You are speaking to the mother of a previously healthy boy who has just broken out with
chickenpox lesions. His mother states that one of her sons classmates also has the disease.
No one else in the household is ill. He did not receive varicella vaccine, but all of his other
immunizations are up to date. His mother asks whether her son is at risk for developing a
severe case of the disease.
Of the following, the factor that is MOST likely to increase his risk for moderate-to-severe
varicella disease is

A. age younger than 12 years


B. being a secondary case in the household
C. concurrent ear infection
D. receipt of a recent course of antibiotics
E. receipt of short-term salicylate therapy

Copyright 2009 by the American Academy of Pediatrics

page 467

2009 PREP SA on CD-ROM


Critique: 221

Preferred Response: B

Chickenpox or varicella infection is caused by the varicella-zoster virus. Infection tends to be


more severe in adolescents and adults and in those who have immunocompromising conditions.
Complications may include bacterial superinfection of the skin lesions (Item C221A), pneumonia
(Item C221B), central nervous system involvement (cerebellar ataxia, encephalitis),
thrombocytopenia, glomerulonephritis, arthritis, and hepatitis. The decision of whether a patient
should receive antiviral therapy, the route by which it is administered, and the duration of therapy
is determined by specific host factors, the extent of infection, the timing of exposure, and initial
response to therapy.
Oral acyclovir is not recommended for routine use in healthy children who have varicella.
However, oral acyclovir should be considered in immunocompetent hosts who are at increased
risk for moderate-to-severe varicella. Persons who fall into this category include: those who are
older than 12 years of age; those who have chronic cutaneous or pulmonary disorders; persons
receiving long-term salicylate therapy; people receiving short, intermittent, or aerosolized
courses of corticosteroids; and secondary household cases (disease usually is more severe
than in the primary case). The factor that would increase the risk for developing moderate-tosevere varicella disease for the patient described in the vignette is being a secondary case in
the household. His age, ear infection, or receipt of antibiotics or short-term salicylate therapy is
not associated with increased risk for moderate-to-severe disease.
Intravenous antiviral therapy is recommended for immunocompromised patients who have
varicella, including patients being treated with chronic corticosteroids. To maximize efficacy,
therapy should be initiated early in the course of the illness, preferably within 24 hours of rash
onset. Oral acyclovir generally should not be used to treat immunocompromised children who
have varicella because of poor oral bioavailability. However, high-dose oral acyclovir therapy
may be used in selected immunocompromised patients who are believed to be at lower risk of
developing severe varicella. Such patients include those infected with human immunodeficiency
virus who have relatively normal concentrations of CD4+ lymphocytes and children who have
leukemia in whom careful follow-up is guaranteed. Varicella-zoster immune globulin, if available,
or intravenous immunoglobulin administered shortly after exposure can prevent or modify the
course of varicella disease, but immune globulin preparations are not effective once disease is
established.
References:
American Academy of Pediatrics. Varicella-zoster infections. In: Pickering LK, Baker CJ, Long
SS, McMillan JA, eds. Red Book: 2006 Report of the Committee on Infectious Diseases. 27th ed.
Elk Grove Village, Ill: American Academy of Pediatrics; 2006:711-725
Arvin AM. Antiviral therapy for varicella and herpes zoster. Semin Pediatr Infect Dis. 2002;13:1221. Abstract available at: http://www.ncbi.nlm.nih.gov/pubmed/12118839
Balfour HH Jr, Rotbart HA, Feldman S, et al. Acyclovir treatment of varicella in otherwise healthy
adolescents. The Collaborative Acyclovir Varicella Study Group. J Pediatr. 1992;120:627-633.
Abstract available at: http://www.ncbi.nlm.nih.gov/pubmed/1313098
Whitley RJ. Approaches to the treatment of varicella-zoster virus infections. Contrib Microbiol.
1999;3:158-172
Whitley RJ. Therapeutic approaches to varicella-zoster virus infections. J Infect Dis.
1992;166(suppl 1):S51-S57. Abstract available at: http://www.ncbi.nlm.nih.gov/pubmed/1378081

Copyright 2009 by the American Academy of Pediatrics

page 468

2009 PREP SA on CD-ROM


Question: 222

A father brings in his 8-year-old son because the boy has been "feeling tired" for the past few
weeks. The remainder of the history is unremarkable. His weight is at the 5th percentile, height
is less than the 5th percentile, temperature is 98.6F (37C), heart rate is 88 beats/min,
respiratory rate is 16 breaths/min, and blood pressure is 124/84 mm Hg. Urinalysis findings
include a specific gravity of 1.005, pH of 6.5, no blood, and 2+ protein. Other laboratory results
are:
Sodium, 134.0 mEq/L (134.0 mmol/L)
Potassium, 5.4 mEq/L (5.4 mmol/L)
Chloride, 96.0 mEq/L (96.0 mmol/L)
Bicarbonate, 14.0 mEq/L (14.0 mmol/L)
Blood urea nitrogen, 96.0 mg/dL (34.3 mmol/L)
Creatinine, 8.4 mg/dL (742.6 mcmol/L)
Of the following, the MOST likely additional finding expected for this child is

A. enlarged kidneys on ultrasonography


B. hypomagnesemia
C. increased parathyroid hormone concentration
D. low insulin-like growth factor I concentration
E. reticulocytosis

Copyright 2009 by the American Academy of Pediatrics

page 469

2009 PREP SA on CD-ROM


Critique: 222

Preferred Response: C

The boy described in the vignette has fatigue, growth failure, and elevated blood urea nitrogen
(BUN) and creatinine values, which are consistent with a diagnosis of chronic renal failure
(CRF). CRF in children is characterized by the marked loss of excretory function. The normal
glomerular filtration rate (GFR) is 100 to 120 mL/min per 1.73 m2, and children who have severe
renal impairment should be considered for renal replacement therapy (dialysis or
transplantation) when the GFR approaches 20 mL/min per 1.73 m2. Prior to reaching this
degree of renal dysfunction, other abnormalities occur due to impaired renal excretory function,
reduced functional nephron mass, and impaired endocrine function.
Because of the CRF, affected children often have poor appetites and impaired growth,
leading to failure to thrive and short stature in those who have long-standing disease. More
specifically, affected children actually have elevated growth hormone and insulin-like growth
factor (IGF) I concentrations, but because of high concentrations of IGF-binding proteins, the
bioavailability is reduced, resulting in abnormal growth. Decreased urine output with impaired
excretion of sodium and water can lead to hypertension. Other minerals, including phosphorus,
potassium, and magnesium, tend to accumulate in renal failure. With decreased nephron mass,
25-hydroxyvitamin D3 exiting the liver cannot be activated in the kidney to 1,25-dihydoxyvitamin
D3 due to low concentrations of 1-alpha hydroxylase activity. Low bioactive forms of vitamin D3
accompanied by phosphorus retention and secondary hyperparathyroidism lead to renal
osteodystrophy (renal rickets) (Item C222). The low erythropoietin values that are seen in
advanced renal disease (usually when the GFR is less than 35 mL/min per 1.73 m2) lead to
anemia associated with a low reticulocyte count.
Although children who have acute renal failure often have inflamed kidneys that are enlarged
on ultrasonography, children who have CRF usually have small, shrunken kidneys on
ultrasonography. Thus, kidney size sometimes can help distinguish between acute and chronic
renal failure.
References:
Fine RN, Whyte DA, Boydstun II. Conservative management of chronic renal insufficiency. In:
Avner ED, Harmon WE, Niaudet P, eds. Pediatric Nephrology. 5th ed. Philadelphia, Pa: Lippincott
Williams & Wilkins; 2004:1291-1311
Wong CS, Mak RH. Chronic kidney disease. In: Kher KK, Schnaper HW, Makker SP, eds.
Clinical Pediatric Nephrology. 2nd ed. London, England: Informa Healthcare; 2007:339-352

Copyright 2009 by the American Academy of Pediatrics

page 470

2009 PREP SA on CD-ROM


Question: 223

A 4-year-old girl presents with a 2-week history of bilateral discolored rhinorrhea, nasal
congestion, and decreased oral intake. Her mother states that at the onset of this illness, she
developed clear rhinorrhea 2 days after attending child care. Despite using over-the-counter
antihistamines and decongestants, the childs symptoms have persisted. A quick review of her
chart shows that her immunizations are up to date, including her pneumococcal conjugate
vaccine series and her annual influenza vaccination. On physical examination, the child has
appropriate vital signs for her age, infraorbital edema bilaterally, and yellowish mucus in her
nares. You suspect acute bacterial rhinosinusitis (ABRS) and discuss evaluation and treatment
options with the mother.
Of the following, a TRUE statement regarding this childs condition is that

A. a sinus radiograph should be performed prior to initiating antibiotic therapy for ABRS
B. ABRS can be distinguished easily from a viral upper respiratory tract infection
C. allergic rhinitis is the most common risk factor for developing ABRS
D. the gold standard test for organism identification in ABRS is a nasal swab culture
E. the most likely bacterial pathogen is Haemophilus influenzae

Copyright 2009 by the American Academy of Pediatrics

page 471

2009 PREP SA on CD-ROM


Critique: 223

Preferred Response: E

Acute bacterial rhinosinusitis (ABRS) is defined as sinusitis symptoms lasting for fewer than 4
weeks. Common symptoms include purulent nasal discharge and sinus pressure. Because the
symptoms of ABRS are difficult to distinguish from a viral upper respiratory tract infection (URI),
symptoms of a viral URI that persist past 7 to 10 days, as described for the girl in the vignette,
generally are regarded as indicative of bacterial sinusitis. Viral URI is the most common
preceding condition for ABRS, but allergic rhinitis is a precipitant in approximately 20% of cases.
In 2001, the American Academy of Pediatrics published management guidelines for the
evaluation and treatment of ABRS. Because the initial symptoms of ABRS usually are
indistinguishable from a viral URI, physical examination, laboratory tests, and radiographic
studies such as a sinus radiograph or sinus computed tomography scan are not helpful in
distinguishing a prolonged viral URI from bacterial sinusitis. A Waters view sinus radiograph may
help confirm unilateral sinusitis by showing maxillary opacification, but it does not need to be
performed prior to initiating antibiotics.
An antral puncture is considered the gold standard for isolation of the causative organism,
but it is invasive and rarely performed today. A nasal swab culture of the nares may isolate the
causative organism, but it also will include many other normal flora. With the initiation of the
pneumococcal conjugate vaccine, nontypeable Haemophilus influenzae has replaced
Streptococcus pneumoniae as the predominant bacterial pathogen in ABRS.
When determining appropriate antibiotic choices for the treatment of ABRS, the clinician
should consider the most likely pathogen, recent antibiotic use, and local antibiotic resistance
rates. Depending on the organism, 15% to 80% of ABRS cases resolve spontaneously without
treatment. Most clinicians, however, are reluctant to use a "watch-and-wait" approach after 10 to
14 days of unremitting symptoms. Initial recommended oral therapies include high-dose
amoxicillin (90 mg/kg per day), amoxicillin/clavulanate, cefdinir, cefpodoxime proxetil, or
cefuroxime axetil. Patients who have had recent antibiotic therapy or who attend child care may
require amoxicillin/clavulanate or intramuscular ceftriaxone as initial therapy.
Many adjunctive therapies for ABRS are available, including nasal corticosteroids, oral
decongestants, cough expectorants, pine oil, nasal saline rinses, and nasal decongestants.
However, none has shown clinical benefit.
References:
American Academy of Pediatrics Subcommittee on Management of Sinusitis and Committee on
Quality Improvement. Clinical practice guideline: management of sinusitis. Pediatrics.
2001;108:798-808. Available at: http://pediatrics.aappublications.org/cgi/content/full/108/3/798
Brook I, Foote PA, Hausfeld JN. Frequency of recovery of pathogens causing acute maxillary
sinusitis in adults before and after introduction of vaccination of children with the 7-valent
pneumococcal vaccine. J Med Microbiol. 2006;55:943-946. Available at:
http://jmm.sgmjournals.org/cgi/content/full/55/7/943
Taylor A, Adam HM. In brief: sinusitis. Pediatr Rev. 2006;27:395-397. Available at:
http://pedsinreview.aappublications.org/cgi/content/full/27/10/395

Copyright 2009 by the American Academy of Pediatrics

page 472

2009 PREP SA on CD-ROM


Question: 224

A 5-year-old boy is brought to the emergency department following 2 days of headache, nausea,
and vomiting. His mother reports that he has had no fever or diarrhea and that everyone at
home, "including the dog," has the same symptoms. Physical examination demonstrates a heart
rate of 120 beats/min, respiratory rate of 24 breaths/min, blood pressure of 100/60 mm Hg, and
oxygen saturation of 100% on room air. The boy is mildly irritable, and his mucous membranes
appear bright red. His lungs are clear, and abdominal examination findings are unremarkable. As
you are completing your evaluation, the mother tells you that the furnace in their house has been
malfunctioning.
Of the following, the MOST appropriate next step is to

A. administer 2 L of oxygen by nasal cannula


B. arrange for emergent hyperbaric oxygen therapy
C. monitor the patients oxygen saturation continuously with pulse oximetry
D. obtain a venous carboxyhemoglobin measurement
E. obtain arterial blood gases

Copyright 2009 by the American Academy of Pediatrics

page 473

2009 PREP SA on CD-ROM


Critique: 224

Preferred Response: D

Carbon monoxide (CO) is an odorless, colorless gas that, when inhaled in a closed space, leads
to nonspecific clinical symptoms that can be mistaken easily for those seen in a variety of
unrelated medical conditions. Because unrecognized CO poisoning can lead to significant
morbidity, including delayed cognitive and focal neurologic deficits, as well as mortality,
physicians should have a high index of suspicion for this diagnosis when examining patients who
have vague, "flulike" symptoms. This is especially important if there is no accompanying fever or
others in the same environment have similar symptoms, such as described in the vignette.
Although a "cherry red" appearance of the skin and mucous membranes traditionally was
believed to be a clue to the diagnosis, this now is recognized as a highly insensitive sign.
CO poisoning results from CO binding to hemoglobin 250 times more readily than oxygen.
The resultant carboxyhemoglobin (COHgb) is unable to transport oxygen to tissues and
interferes with oxygen dissociation from the remaining oxyhemoglobin. Tissue hypoxia results,
with effects manifested in every organ system. Central nervous system (headache, confusion,
lethargy, dizziness, coma), pulmonary (dyspnea on exertion, tachypnea), cardiovascular
(tachycardia, palpitations), and gastrointestinal (nausea and vomiting) symptoms predominate.
Evaluation and treatment of CO poisoning include removal of the patient from the
contaminated environment, patient stabilization, high-flow oxygen administration, and
measurement of a COHgb concentration. A COHgb value of greater than 3% to 5% in a
nonsmoker is indicative of exposure. Although symptomatic patients typically have values
greater than 10%, such values do not correlate well with type or magnitude of symptoms. Pulse
oximetry cannot distinguish between oxyhemoglobin and COHgb, so oxygen saturation
measurements typically are normal. Arterial blood gases should be measured in those who have
severe symptoms to quantify the degree of acidosis, an indicator of anaerobic metabolism in the
face of poor tissue oxygenation.
The goal of oxygen therapy in the treatment of CO poisoning is to convert COHgb rapidly to
oxyhemoglobin. The administration of 100% oxygen using a nonrebreather mask can decrease
the half-life of COHgb from 300 minutes to 90 minutes and should be continued until the COHgb
concentration is less than 5%. Although hyperbaric oxygen can decrease the COHgb half-life
further to 30 minutes, its benefit in improving patient outcome has not been demonstrated
clearly. At present, expert consensus guidelines recommend its use only for severe
intoxications.
References:
Clardy PF, Manakar S. Carbon monoxide poisoning. UpToDate Online 15.3. 2008. Available for
subscription at:
http://www.utdol.com/utd/content/topic.do?topicKey=ad_tox/2932&selectedTitle=1~26&source=s
earch_result
Juurlink DN, Buckley NA, Stanbrook MB, Isbister GK, Bennett M, McGuigan MA. Hyperbaric
oxygen for carbon monoxide poisoning. Cochrane Database Syst Rev. 2005;1:CD002041.
Available at:
http://www.mrw.interscience.wiley.com/cochrane/clsysrev/articles/CD002041/frame.html
Kind T, Etzel RA. In brief: carbon monoxide. Pediatr Rev. 2005;26:150-151. Available at:
http://pedsinreview.aappublications.org/cgi/content/full/26/4/150

Copyright 2009 by the American Academy of Pediatrics

page 474

2009 PREP SA on CD-ROM


Question: 225

A 15-year-old girl presents with a history of abdominal pain. She is a competitive runner and has
required frequent ibuprofen for treatment of knee pain. On physical examination, you note
epigastric tenderness. Fecal occult blood test results are positive.
Of the following, the test that is MOST likely to provide a definitive diagnosis is

A. abdominal computed tomography scan


B. Helicobacter pylori serology
C. serum amylase measurement
D. upper endoscopy with biopsies
E. upper gastrointestinal radiographic series

Copyright 2009 by the American Academy of Pediatrics

page 475

2009 PREP SA on CD-ROM


Critique: 225

Preferred Response: D

The epigastric pain, use of nonsteroidal anti-inflammatory agents, and guaiac-positive stool
described for the patient in the vignette suggest the possibility of peptic ulcer disease. Peptic
ulcers are erosions of the gastric or duodenal mucosa that disrupt the epithelium and predispose
to abdominal pain or gastrointestinal bleeding. The most common predisposing factors placing
patients at risk for the development of gastric ulcers are Helicobacter pylori infection,
medications (particularly chemotherapeutic agents or nonsteroidal anti-inflammatory drugs),
Crohn disease, vasculitis, or severe illness requiring intensive care unit hospitalization. Ulcers
also may be idiopathic and without an identifiable inflammatory or infectious cause.
If peptic ulcer is suspected and definitive diagnosis is required, the test of choice is upper
gastrointestinal endoscopy or esophagogastroduodenoscopy (EGD). EGD is superior to other
imaging modalities such as ultrasonography or abdominal computed tomography scan for
identifying mucosal lesions and allows for biopsy of the stomach to evaluate for Crohn disease
or H pylori. If a patient has a bleeding ulcer (Item C225) that has a visible vessel, therapeutic
endoscopy (in which the bleeding area is injected with epinephrine or cauterized) can be
performed at the same time as the diagnostic procedure. Because endoscopy is an invasive
test that requires sedation or anesthesia, the benefits of the procedure need to be weighed
against the risks. In particular, the endoscopy should be undertaken with caution in a patient
who has a bleeding disorder or a critically ill patient who has gastrointestinal bleeding. In such
patients, empiric therapy with proton pump antagonists and hemodynamic stabilization should be
attempted before endoscopy is performed.
H pylori serology and serum amylase measurement cannot diagnose peptic ulcer disease.
Upper gastrointestinal radiographic series and abdominal computed tomography scan have poor
sensitivity for ulcers.
References:
De Giacomo C. Helicobacter pylori gastritis and peptic ulcer disease. In: Guandalini S, ed.
Textbook of Pediatric Gastroenterology and Nutrition. London, England: Taylor & Francis;
2004:73-94
Fox VL. Pediatric endoscopy. Gastrointest Endosc Clin North Am. 2000;10: 175-194. Abstract
available at: http://www.ncbi.nlm.nih.gov/pubmed/10618461

Copyright 2009 by the American Academy of Pediatrics

page 476

2009 PREP SA on CD-ROM


Question: 226

You are seeing a 30-year-old multigravid woman for prenatal counseling. She has had immune
thrombocytopenic purpura for the past 5 years, and her spleen was removed 2 years ago. She
asks you about the effects that her disease might have on her unborn child.
Of the following, you are MOST likely to tell her that

A. if her newborn has thrombocytopenia, he or she will be treated with intravenous


immunoglobulin

B. maternal platelet counts predict fetal risks of intracranial hemorrhage


C. maternal platelet transfusion during pregnancy will minimize the risk for neonatal
thrombocytopenia

D. operative delivery of the newborn will reduce the risk of intracranial hemorrhage
E. the newborn will require a platelet transfusion soon after birth

Copyright 2009 by the American Academy of Pediatrics

page 477

2009 PREP SA on CD-ROM


Critique: 226

Preferred Response: A

The fact that the mother described in the vignette has immune thrombocytopenia is noteworthy.
Because this condition is associated with immunoglobulin G (IgG) directed against maternal
platelet antigens, transplacental acquisition of these antibodies by the fetus may occur after 30
weeks' gestation. As such, maternal-fetal surveillance is recommended, but no direct correlation
exists between maternal platelet counts and fetal platelet counts or the risk of intracranial
hemorrhage. Maternal platelet transfusion during pregnancy may help avoid concerns for the
mother, but not the fetus or newborn. Although the risk for intracranial hemorrhage is only about
5% overall, it is greatest when fetal platelet counts are less than 20.0x103/mcL (20.0x109/L)
because spontaneous intracranial hemorrhage may occur. An operative delivery of the newborn
does not reduce the risk of intracranial hemorrhage significantly.
Treatment of the newborn generally is supportive, with attention to the risk of
thrombocytopenia significant enough to warrant platelet transfusion. Ninety percent of affected
newborns require no treatment. However, platelet transfusion may be in order for any infant of a
mother who has immune thrombocytopenic purpura when the neonate exhibits bleeding or has a
platelet count of less than 20.0x103/mcL (20.0x109/L). The best treatment for affected infants is
intravenous immunoglobulin (IVIg), which blocks circulating maternal IgG directed against
platelet antigens and allows the newborn's platelet counts to rise. Platelet transfusion for the
newborn is less effective than IVIg and may risk exposure to platelet-associated antigens that
could induce a postnatal immune thrombocytopenia.
References:
Buyon JP, Nugent D, Mellins E, Sandborg C. Maternal immunologic diseases and neonatal
disorders. NeoReviews. 2002;3:e3-e10. Available for subscription at:
http://neoreviews.aappublications.org/cgi/content/full/3/1/e3
Murphy MF, Bussel JB. Advances in the management of alloimmune thrombocytopenia. Br J
Haematol. 2007;136:366-378. Abstract available at:
http://www.ncbi.nlm.nih.gov/pubmed/17233844
Wong W, Glader B. Approach to the newborn who has thrombocytopenia. NeoReviews.
2004;5:e444-e450. Available for subscription at:
http://neoreviews.aappublications.org/cgi/content/full/5/10/e444

Copyright 2009 by the American Academy of Pediatrics

page 478

2009 PREP SA on CD-ROM


Question: 227

You are addressing a group of expectant mothers who are due to deliver their infants in the next
few weeks. You discuss the benefits of breastfeeding and explain that it is the best nutrition for
most babies. One woman asks you if it is acceptable to breastfeed if she has had hepatitis in the
past. You explain that there are only a few infections that would prevent a mother from being
able to breastfeed her baby.
Of the following, breastfeeding is MOST likely to be contraindicated if a mother

A. has active untreated pulmonary tuberculosis


B. has genital herpes without breast lesions
C. is a cytomegalovirus carrier
D. is hepatitis B surface antigen-positive
E. is hepatitis C antibody-positive

Copyright 2009 by the American Academy of Pediatrics

page 479

2009 PREP SA on CD-ROM


Critique: 227

Preferred Response: A

Human milk is the optimal nutrition for infants. Benefits include transference of protective
maternal antibodies, improved bonding between mother and child, and probable improvement in
cognitive and developmental function of the infant. Most mothers can breastfeed successfully,
although there are contraindicated conditions for both infants and mothers. Infants who have
galactosemia should not receive human milk, and infants who have other forms of metabolic
disease, such as urea cycle defects or phenylketonuria, may receive only a limited amount.
Mothers infected with human immunodeficiency virus (in the United States) or human T-cell
lymphotrophic virus-1 or -2 and those who have active untreated tuberculosis or active herpes
lesions on the breast should not breastfeed their infants. However, mothers who are hepatitis B
surface antigen-positive or hepatitis C antibody-positive, are cytomegalovirus carriers, or have
genital herpes without breast lesions can breastfeed safely. Maternal medications that preclude
breastfeeding include antineoplastic agents, immunosuppressants, lithium, and
radiopharmaceutical agents.
Several disorders of the breast may make breastfeeding difficult, but they are not
contraindications to breastfeeding. Previous breast surgery may cause ineffective lactation, but
this varies among mothers. Women who have inverted or flat nipples may experience difficulties
with latch-on, but this can be improved with early feedings, use of nipple shields, and lactation
consultation. Use of a breast pump also may help. Women who have breast cancer may be able
to breastfeed if they are not taking antineoplastic medications. Mastitis, inflammation of the
breast usually caused by obstruction of ducts, may make breastfeeding painful, but more
frequent nursing is the best recommendation to help resolve this condition. Sore or cracked
nipples may develop, especially if the infant has oral-motor dysfunction, and adjusting the infant's
latch-on may improve these symptoms. A mother who has Candida infection of the breast may
continue to breastfeed, but both she and her infant should be treated for the infection to avoid a
cycle of reinfection.
References:
American Academy of Pediatrics Section on Breastfeeding. Breastfeeding and the use of human
milk. Pediatrics. 2005;115:496-506. Available at:
http://pediatrics.aappublications.org/cgi/content/full/115/2/496
Chandran L, Gelfer P. Breastfeeding: the essential principles. Pediatr Rev. 2006;27:409-417.
Available at: http://pedsinreview.aappublications.org/cgi/content/full/27/11/409
Powers NG, Slusser W. Breastfeeding update 2: clinical lactation management. Pediatr Rev.
1997;18:147-161. Available at: http://pedsinreview.aappublications.org/cgi/content/full/18/5/147

Copyright 2009 by the American Academy of Pediatrics

page 480

2009 PREP SA on CD-ROM


Question: 228

A 6-month-old girl, who was born in Nigeria, presents for an urgent visit as soon as the family
arrives in the United States because of fever and irritability. Physical examination reveals a
fussy infant who has anorexia, a temperature of 100F (37.8C), and swelling of all of the fingers
of the right hand (Item Q228). The remainder of the examination findings are negative.
Of the following, the MOST likely cause of this pattern of swelling in this child is

A. cellulitis
B. juvenile idiopathic arthritis
C. malaria
D. sickle cell disease
E. trauma

Copyright 2009 by the American Academy of Pediatrics

page 481

2009 PREP SA on CD-ROM


Critique: 228

Preferred Response: D

Sickle cell disease (SCD) usually is diagnosed in the United States in early infancy because of
mandated newborn screening. If screening is not performed, the diagnosis may be delayed until
the development of symptoms from a vaso-occlusive crisis. In infants and toddlers, the first
vaso-occlusive crisis may be heralded by painful and swollen hands or feet, also known as
dactylitis or "hand and foot syndrome." The examination findings for the child described in the
vignette suggest the diagnosis of dactylitis that is most likely due to SCD.
Dactylitis due to SCD usually presents with swelling and tenderness of the hands (Item
C228) or feet that is self-limited and resolves in 5 to 31 days. Fever and leukocytosis also may
be noted. Radiographs of the hands and feet taken 7 to 14 days after the swelling begins may
reveal periosteal new bone formation or intramedullary densities. Such lesions resolve in 2 to 3
months. Diagnostic confusion with cellulitis or osteomyelitis (which rarely may occur in
conjunction with dactylitis) is frequent, but dactylitis often involves most, if not all, of the digits
equally and is not accompanied by overlying cellulitis of the skin.
Trauma generally does not involve all digits equally without contusion or laceration of the
overlying skin. Arthritis, including juvenile idiopathic arthritis, usually affects only a few joints and
is uncommon in this age group. Malaria causes fever and anemia but does not commonly cause
dactylitis or other extremity findings.
Dactylitis may be seen in other conditions, including psoriasis, insect bites with angioedema,
and other conditions that are more common in older children and adults.
References:
American Academy of Pediatrics Section on Hematology/Oncology and Committee on Genetics.
Health supervision for children with sickle cell disease. Pediatrics. 2002;109:526-535. Available
at: http://pediatrics.aappublications.org/cgi/content/full/109/3/526
Gill FM, Sleeper LA, Weiner SJ, et al for the Cooperative Study of Sickle Cell Disease. Clinical
events in the first decade in a cohort of infants with sickle cell disease. Blood. 1995;86:776-783.
Available at: http://bloodjournal.hematologylibrary.org/cgi/reprint/86/2/776
Meremikwu MM. Sickle cell disease (updated). BMJ Clinical Evidence. 2007. Available for
subscription at: http://clinicalevidence.bmj.com/ceweb/conditions/bly/2402/2402.jsp
Miller ST, Sleeper LA, Pegelow CH, et al. Prediction of adverse outcomes in children with sickle
cell disease. N Engl J Med. 2000:342:2:83-89. Abstract available at:
http://www.ncbi.nlm.nih.gov/pubmed/10631276
Sickle-cell dactylitis. In: Wheeless' Textbook of Orthopaedics. Available at:
http://www.wheelessonline.com/ortho/sickle_cell_dactylitis

Copyright 2009 by the American Academy of Pediatrics

page 482

2009 PREP SA on CD-ROM


Question: 229

You work as a voluntary attending pediatrician in the resident continuity clinic at your local
hospital. You are precepting a resident, who tells you that she has just evaluated a 16-year-old
varsity volleyball player. The girls height is 71 inches, weight is 125 lb, and blood pressure is
115/74 mm Hg. The resident is concerned about scoliosis and a 3/6 holosystolic murmur heard
at the cardiac apex with radiation to the left axilla (Item Q229).
Of the following, the MOST likely diagnosis for this patient is

A. EhlersDanlos syndrome
B. infective endocarditis
C. Marfan syndrome
D. rheumatic heart disease
E. Williams syndrome

Copyright 2009 by the American Academy of Pediatrics

page 483

2009 PREP SA on CD-ROM


Critique: 229

Preferred Response: C

The patient in the vignette has several physical examination findings that suggest the diagnosis
of Marfan syndrome, including scoliosis, tall stature, and a holosystolic murmur that could be
due to mitral valve disease. The murmur is heard at the cardiac apex and is of the quality and
location typical for mitral regurgitation (Item C229A), a common result of the prolapsing and
redundant valve. The patient should be referred to a specialist, usually a geneticist, who will use
the Ghent criteria to determine if the child has Marfan syndrome.
Marfan syndrome is an autosomal dominant connective tissue disorder that is caused by
mutations in the fibrillin gene on chromosome 15. The abnormal fibrillin affects the organization of
extracellular myofibrils that support the formation of elastin networks in connective tissue. The
abnormal fibrillin is associated with variable cardiovascular, skeletal, and ocular features. Not all
people who have Marfan syndrome have cardiovascular involvement, but the blood vessels,
cardiac valves, and myocardium all contain substantial amounts of fibrillin. Cardiovascular
involvement in children most commonly results in dilation of the aortic root at the sinuses of
Valsalva and abnormality of the atrial ventricular valves. The mitral valve frequently is affected
and demonstrates dysplasia, prolapse, and dysfunction, as may the tricuspid valve.
The skeletal findings of Marfan syndrome may include scoliosis, a tall and thin habitus,
pectus deformities (Item C229B), arachnodactyly (Item C229C), and hypermobility of the joints.
The ocular findings may include ectopia lentis (lens dislocation) (Item C229D). Often, the arm
span-to-height ratio exceeds 1.05, and the upper-to-lower segment ratio is less than 0.88.
Patients in whom Marfan syndrome is diagnosed or highly suspected should be counseled
to avoid participation in competitive contact sports. Avoidance of contact sports is important
because there is an increased risk of cardiac, skeletal, and ophthalmologic problems resulting
from injury. A specific potential problem with the cardiovascular system is tearing of the
weakened, abnormal aortic wall, which can lead to a catastrophic outcome. It is important to
remember that sports practice often can be as competitive and intense as the games; both
should be avoided. Consultation with an experienced pediatric cardiologist is important, and
echocardiographic imaging of the heart is essential to evaluate for the possibility of
cardiovascular involvement.
The patient described in the vignette does not have the findings of either Ehlers-Danlos or
Williams syndromes. She has no findings to suggest infective carditis or rheumatic fever/heart
disease.
References:
Maron BJ, Chaitman BR, Ackerman MJ, et al. Recommendations for physical activity and
recreational sports participation for young patients with genetic cardiovascular diseases.
Circulation. 2004;109:2807-2816. Available at:
http://circ.ahajournals.org/cgi/content/full/109/22/2807
Maron BJ, Thompson PD, Ackerman MJ, et al. AHA scientific statements. Recommendations
and considerations related to preparticipation screening for cardiovascular abnormalities in
competitive athletes: 2007 update. A scientific statement from the American Heart Association
Council on Nutrition, Physical Activity, and Metabolism: endorsed by the American College of
Cardiology Foundation. Circulation. 2007;115:1643-1655. Available at:
http://circ.ahajournals.org/cgi/content/full/115/12/1643
Moodie DS. AAP: health supervision for children with Marfan syndrome. Clin Pediatr (Phila).
1997;36:489
Peirpont MEM. Connective tissue diseases. In: Moller JH, Hoffman JIE, eds. Pediatric
Cardiovascular Medicine. Philadelphia, Pa: Churchill Livingstone; 2000:901-912
von Kodolitsch Y, Robinson PN. Marfan syndrome: an update of genetics, medical and surgical
management. Heart. 2007;93:755-760. Extract available at:

Copyright 2009 by the American Academy of Pediatrics

page 484

2009 PREP SA on CD-ROM

http://heart.bmj.com/cgi/content/extract/93/6/755

Copyright 2009 by the American Academy of Pediatrics

page 485

2009 PREP SA on CD-ROM


Question: 230

The parents of a 6-month-old previously well infant bring her to your office. She had been
developing normally, but she stopped interacting with her parents over the last 24 hours. For
several days prior to this development, she had had unusual spells during which her head and
chin dropped to her chest. Now she is having clusters of these spells involving head drop and
body flexion. On physical examination, there is no bruising. The infant is afebrile and alert, her
tone is low, and she does not make persistent eye contact or track visually. You refer her to the
emergency department, where results of a complete blood count, electrolyte panel, urinalysis,
and a noncontrast head computed tomography scan are normal.
Of the following, the test that is MOST likely to reveal the correct diagnosis is

A. electroencephalography
B. electroretinography
C. lumbar puncture
D. muscle biopsy
E. serum lactate measurement

Copyright 2009 by the American Academy of Pediatrics

page 486

2009 PREP SA on CD-ROM


Critique: 230

Preferred Response: A

The infant described in the vignette has an encephalopathy, a "confusional" state. It is critical to
consider focal, ischemic, ictal, infectious, and toxic/metabolic causes for any patient presenting
with mental status changes. The noncontrast head computed tomography scan and laboratory
tests were appropriate initial decisions in the emergency department to rule out emergent,
treatable problems. For this infant, the key to the diagnosis is recognition that the cluster of body
spasms combined with the loss of developmental milestones is characteristic of infantile
spasms, which should be assessed by electroencephalography (EEG). EEG is used to identify
the neurophysiologic pattern that confirms the diagnosis of infantile spasms. The characteristic
diffuse, slow, disorganized, and high-amplitude pattern is known as "hypsarrhythmia."
Infantile spasms describes a syndrome of epilepsy and encephalopathy in infancy, with a
peak onset at 6 months. The spasms (Item C230) can range from subtle, quick movements
involving just the head to dramatic "clasp-knife" full body spasms. Infantile spasms can have a
number of causes or can be idiopathic. The prognosis is often poor, particularly if the child's
neurologic development was abnormal prior to the onset of spasms. Potential causes of the
infantile spasms may be structural, genetic, or metabolic.
Electroretinography (ERG) is used to assess the neurophysiology of the retina. For
example, ERG responses are abnormal in Leber congenital amaurosis and various rod/cone
dystrophies. This child's new visual impairment associated with hypotonia and spasms is not
likely related to retinal pathology, but more likely is due to a central nervous system problem
causing abnormal visual processing.
A lumbar puncture should be considered in an infant who has fever and an acute
encephalopathy to rule out infectious causes, but establishing the diagnosis of infantile spasms
is more important in this case. A lumbar puncture, if it is obtained, might be more valuable as part
of a metabolic evaluation, as would measurement of serum lactate and a muscle biopsy, if
needed to assess for the possibility of a mitochondrial disorder.
References:
Kossoff EH. Infantile spasms. In: Singer HS, Kossoff EH, Hartman AL, Crawford TO, eds.
Treatment of Pediatric Neurologic Disorders. Boca Raton, Fla: Taylor & Francis; 2005:111-116
Mackay MT, Weiss SK, Adams-Webber T, et al. Practice parameter: medical treatment of
infantile spasms. Report of the American Academy of Neurology and the Child Neurology
Society. Neurology. 2004;62:1668-1681. Available at:
http://www.neurology.org/cgi/content/full/62/10/1668

Copyright 2009 by the American Academy of Pediatrics

page 487

2009 PREP SA on CD-ROM


Question: 231

While examining an infant in the newborn nursery, you note that the pupil of one eye seems
abnormally large, and little of the iris is visible. The baby appears otherwise normal. A
subsequent ophthalmologic evaluation confirms the diagnosis of partial aniridia.
Of the following, the MOST accurate statement regarding the diagnosis is that

A. aniridia is associated with hepatoblastoma


B. few individuals who have aniridia have an affected parent
C. isolated aniridia has an autosomal recessive inheritance pattern
D. molecular testing is available to determine risk for Wilms tumor
E. routine abdominal ultrasonography should be performed every 3 months until age 5 years in
affected individuals

Copyright 2009 by the American Academy of Pediatrics

page 488

2009 PREP SA on CD-ROM


Critique: 231

Preferred Response: D

Aniridia is defined as complete or partial iris hypoplasia (Item C231), often associated with foveal
hypoplasia and reduced visual acuity. Sometimes it can be detected in the newborn period with
the use of direct ophthalmoscopy; the red reflex may appear unusual in shape or is too large
due to the lack of intervening iris tissue. Infants who have such a finding should be referred to
ophthalmology for confirmation and further delineation of the defect. The ophthalmologist plays a
major role in the management of affected children. Isolated aniridia is an autosomal dominant
trait, and most affected individuals have an affected parent.
It is important to note that aniridia is associated with Wilms tumor in some individuals. It is
now possible to evaluate a blood specimen from an individual who has aniridia to determine if he
or she has a PAX6 gene mutation, which is detectable in 90% of isolated cases. If a PAX6
mutation is detected, Wilms tumor is not a concern. However, if no mutation of PAX6 is detected,
the aniridia could be part of a condition, such as aniridia-Wilms tumor or the Wilms tumor-aniridiagenital anomalies-retardation (WAGR) syndrome. These conditions are caused by genes
adjacent to PAX6, which, when deleted, result in aniridia plus other anomalies. In such cases, it
is important to follow a regular surveillance protocol for Wilms tumor that includes regular
urinalysis and routine renal ultrasonography.
Wilms tumor also occurs with increased frequency in conditions such as BeckwithWiedemann syndrome (BWS) and hemihypertrophy. In these conditions, published surveillance
protocols for the development of associated neoplasms should be followed.
Aniridia is not associated with an increased risk for hepatoblastoma, although BWS and
hemihypertrophy are.
References:
Clericuzio C. Recognition and management of childhood cancer syndromes: a systemic
approach. Am J Med Genet. 1999;25:89:81-90. Abstract available at:
http://www.ncbi.nlm.nih.gov/pubmed/10559762
Hingorani M, Moore A. Aniridia. GeneReviews. 2005. Available at:
http://www.geneclinics.org/servlet/access?db=geneclinics&site=gt&id=8888891&key=xAcWBcrj
mZrVo&gry=&fcn=y&fw=581E&filename=/profiles/aniridia/index.html
Lapunzina P. Risk of tumorigenesis in overgrowth syndromes: a comprehensive review. Am J
Med Genet C Semin Med Genet. 2005;137:53-71. Abstract available at:
http://www.ncbi.nlm.nih.gov/pubmed/16010678

Copyright 2009 by the American Academy of Pediatrics

page 489

2009 PREP SA on CD-ROM


Question: 232

A 15-year-old girl presents with vaginal pain and burning accompanied by feelings of warmth and
generalized muscle aches for the past 24 hours. She has had no previous similar symptoms.
She is sexually active and does not use barrier methods for contraception. On physical
examination, you find multiple shallow ulcers of the labia minora that are surrounded by
erythema and are exquisitely tender to touch (Item Q232). There is no vaginal discharge.
Of the following, the most appropriate treatment is

A. acyclovir 5 to 10 mg/kg intravenously every 8 hours for 2 to 7 days


B. acyclovir 400 mg orally once a day for 1 year
C. acyclovir 400 mg orally 3 times a day for 7 to 10 days
D. acyclovir 800 mg orally twice a day for 2 days
E. no antiviral chemotherapy

Copyright 2009 by the American Academy of Pediatrics

page 490

2009 PREP SA on CD-ROM


Critique: 232

Preferred Response: C

Most genital herpes simplex virus (HSV) infections are caused by HSV type 2 (HSV-2), although
the prevalence of HSV type 1 (HSV-1) appears to be increasing. Primary infection with either of
the two types may go undetected, but the labial lesions (Item C232), systemic symptoms of
generalized muscle aches, and no previous symptoms described for the girl in the vignette
suggest that this is a primary HSV infection. The clinical diagnosis should be confirmed by typespecific laboratory testing. Recurrences and subclinical shedding are much less frequent for
genital HSV-1 infection than HSV-2 infection, and the prognosis and counseling can be
influenced by the type detected.
Antiviral therapy for a first clinical episode of genital herpes is recommended, even if
manifestations are mild, because some patients develop severe or prolonged symptoms at a
later time. A 7- to 10-day course of oral acyclovir (400 mg three times a day)is recommended.
Famciclovir and valacyclovir are other treatment options.
References:
American Academy of Pediatrics. Herpes simplex. In: Pickering LK, Baker CJ, Long SS,
McMillan JA, eds. Red Book: 2006 Report of the Committee on Infectious Diseases. 27th ed. Elk
Grove Village, Ill: American Academy of Pediatrics; 2006:361-371
Workowski KA, Berman SM, Centers for Disease Control and Prevention. Sexually transmitted
diseases treatment guidelines, 2006. MMWR Recomm Rep. 2006;55(RR11):1-94. Available at:
http://www.cdc.gov/mmwr/preview/mmwrhtml/rr5511a1.htm

Copyright 2009 by the American Academy of Pediatrics

page 491

2009 PREP SA on CD-ROM


Question: 233

A 16-year-old boy presents to the emergency department with an acute change in his mental
status. According to his parents, he was previously healthy and has suffered no recent trauma.
On physical examination, he is somnolent, has pinpoint pupils and mild hypotension, and
demonstrates shallow breathing.
Of the following, the test that is MOST likely to help determine the cause of his altered level of
consciousness is

A. brainstem auditory evoked response


B. chest radiography
C. electroencephalography
D. serum amino acid measurement
E. urine toxicology screen

Copyright 2009 by the American Academy of Pediatrics

page 492

2009 PREP SA on CD-ROM


Critique: 233

Preferred Response: E

Evaluation of the patient who has an altered level of consciousness consists of a thorough
medical history, a detailed physical examination, and appropriate diagnostic studies. A sudden
onset over minutes in a previously healthy individual suggests trauma or a cerebrovascular
accident; effects of toxic ingestions may present over several hours; and a more gradual onset
suggests infection, metabolic disease, or a space-occupying lesion. Vital signs, level of
consciousness, pupil size and reaction, motor responses, and skin findings can be crucial in
narrowing potential causes and determining appropriate diagnostic studies. Diagnostic studies
might include blood and urine testing (for potential metabolic, infectious, and toxic causes),
lumbar puncture, computed tomography scan and magnetic resonance imaging (with or without
angiography), brainstem evoked potentials, electroencephalography, and electrocardiography.
The teenager described in the vignette has had an acute mental status change, and his
somnolence, hypotension, shallow and slow respirations, and pinpoint pupils are consistent with
an opiate ingestion. Therefore, toxicology screens for confirmation as well as to evaluate for
coingestions are indicated. Evoked potentials may be useful in testing the integrity of the
brainstem and in providing prognostic information for comatose patients, but results are
nonspecific and rarely helpful for initial diagnosis. Chest radiography generally is of little value in
the assessment of a comatose patient. Although seizures may produce an altered level of
consciousness, no suggestion of seizure activity is present in this patient that would necessitate
electroencephalography. The patient's age makes an inborn error of metabolism unlikely, and
serum amino acid measurement can be deferred.
References:
Avner JR. Altered states of consciousness. Pediatr Rev. 2006:27:331-338. Available at:
http://pedsinreview.aappublications.org/cgi/content/full/27/9/331
Frankel LR. Neurological emergencies and stabilization. In: Kliegman RM, Behrman RE, Jenson
HB, Stanton BF, eds. Nelson Textbook of Pediatrics. 18th ed. Philadelphia, Pa: Saunders
Elsevier; 2007:405-412

Copyright 2009 by the American Academy of Pediatrics

page 493

2009 PREP SA on CD-ROM


Question: 234

The parents of a 3-year-old boy in whom you recently diagnosed type 1 diabetes mellitus are
anxious about providing the best diabetes control for their son, but wish to avoid frequent
fingersticks to measure blood glucose. They have read that a hemoglobin A1c gives a measure
of blood glucose control and correlates with long-term complications of diabetes. They request
that this blood test be obtained at weekly intervals to give them assurance of good control.
Of the following, the MOST important information to provide them about hemoglobin A1c
measurement is that it

A. can replace self blood glucose monitoring


B. is not useful in children younger than 5 years of age
C. should be obtained every month
D. should be obtained every 2 to 3 months
E. should be obtained every 6 months

Copyright 2009 by the American Academy of Pediatrics

page 494

2009 PREP SA on CD-ROM


Critique: 234

Preferred Response: D

The hemoglobin A1c (HbA1c) is a measure of nonenzymatic glycosylation of hemoglobin A and


correlates with concentrations of circulating blood glucose over the life of the red blood cell. It is
not a useful measure of daily changes in blood glucose or of amplitude of excursion of blood
glucose over the course of a day. Because the red cell has a life of approximately 120 days,
measurement of HbA1c more frequently than every 2 months is not likely to be helpful in clinical
management. In general, this measurement is made every 2 to 3 months. Results permit broad
adjustments in insulin therapy and monitoring of large trends in management. If the HbA1c is
measured less frequently, glycemic control may deteriorate without recognition. As children
have been reported to say, it is "the test that does not lie." It is useful in a child of any age,
although the presence of fetal hemoglobin in very young children may make interpretation of the
results more difficult. In addition, measurement of HbA1c in children who have
hemoglobinopathies may not be entirely accurate. Instead, measurement of total glycosylated
hemoglobin can be substituted. For children who have rapid red cell destruction, the published
relationship between serum glucose and HbA1c may be inaccurate.
Self blood glucose monitoring gives a relatively dynamic picture of blood glucose changes
over the course of a day and allows decisions to be made about adjustment of basal insulin
requirements as well as adjustments for meals, exercise, and other life activities. A combination
of self blood glucose monitoring and periodic measurement of HbA1c permits reasoned
decisions and improved glycemic control in this most difficult disorder.
References:
McCulloch DK. Estimation of blood glucose control in diabetes mellitus. UpToDate Online 15.3.
2008. Available for subscription at:
http://www.uptodateonline.com/utd/content/topic.do?topicKey=diabetes/7913
McCulloch DK. Glycemic control and vascular complications in type 1 diabetes. UpToDate
Online 15.3. 2008. Available for subscription at:
http://www.uptodateonline.com/utd/content/topic.do?topicKey=diabetes/10573
Silverstein J, Klingensmith G, Copeland K, et al, Care of children and adolescents with type 1
diabetes: a statement of the American Diabetes Association. Diabetes Care. 2005;28:186-212.
Available at: http://care.diabetesjournals.org/cgi/content/full/28/1/186

Copyright 2009 by the American Academy of Pediatrics

page 495

2009 PREP SA on CD-ROM


Question: 235

A mother brings in her child for a health supervision visit. He is able to pull to stand, take a few
independent steps, and use two fingers to grasp pieces of cereal.
Of the following, these developmental milestones are MOST typical for a child whose age is

A. 6 months
B. 9 months
C. 12 months
D. 15 months
E. 18 months

Copyright 2009 by the American Academy of Pediatrics

page 496

2009 PREP SA on CD-ROM


Critique: 235

Preferred Response: C

The motor milestones described in the vignette are appropriate for a 12-month-old child. Such
children can pull to a stand and cruise, take a few independent steps (Item C235A), and use a
neat pincer grasp to handle a raisin or pellet. Motor milestones for a 6-month-old child include
sitting with support, rolling back to front (Item C235B), and transferring an object between hands.
A 9-month-old child can sit indefinitely, may creep on hands and knees, and can use a radial
digital grasp to pick up a cube. A 15-month-old child can run stiff-legged. An 18-month-old child
can walk up and down stairs with one hand held, climb on a chair to reach items, scribble
spontaneously, turn two or three pages at once in a picture book, drink from a regular cup, and
get a spoon to his or her mouth.
Parents typically first report delays in motor milestones between the ages of 6 and 12
months. A child's motor developmental milestones are determined from both parental history and
direct observation during the neurodevelopmental evaluation. A motor quotient may be
determined by dividing the developmental age level by the chronologic age and multiplying it by
100. Gross motor quotients below 50 usually are indicative of cerebral palsy. Motor quotients in
the 50 to 75 range may be more indicative of low muscle tone and developmental coordination
disorder. Children who have milder motor delays may have comorbid neurodevelopmental
disorders such as learning disorders or attention-deficit/hyperactivity disorder.
References:
Accardo PJ, Accardo J, Capute A. A neurodevelomental perspective on the continuum of
developmental disabilities. In: Accardo PJ. Capute & Accardo's Neurodevelopmental Disabilities
in Infancy and Childhood. Volume I: Neurodevelopmental Diagnosis and Treatment. 3rd ed.
Baltimore, Md: Paul H. Brookes Publishing Co; 20083-10
Blasco PA. Motor delays. In: Parker S, Zuckerman B, Augustyn M, eds. Developmental and
Behavioral Pediatrics: A Handbook for Primary Care. 2nd ed. Philadelphia, Pa: Lippincott
Williams & Wilkins; 2005:242-247
Knobloch H, Stevens FM, Malone AF. The revised developmental stages. In: Manual of
Developmental Diagnosis: The Administration and Interpretation of the Revised Gesell and
Amatruda Developmental and Neurologic Examination. Albany, NY: Developmental Evaluation
Materials, Inc; 1987:17-120

Copyright 2009 by the American Academy of Pediatrics

page 497

2009 PREP SA on CD-ROM


Question: 236

A 5-year-old girl presents approximately 96 hours after being bitten by a dog on her leg. Her
mother states that she developed fever and swelling of the leg around the bite site over the past
12 hours. Physical examination reveals a nontoxic-appearing girl who has a temperature of
101.8F (38.8C) and an open wound with visible purulence and surrounding erythema.
Of the following, the MOST likely pathogen responsible for these symptoms is

A. Eikenella corrodens
B. Kingella kingae
C. Pasteurella multocida
D. Staphylococcus aureus
E. Streptococcus pyogenes

Copyright 2009 by the American Academy of Pediatrics

page 498

2009 PREP SA on CD-ROM


Critique: 236

Preferred Response: D

Approximately 10% to 15% of patients who sustain a bite wound from a dog develop infection in
the wound. The patient described in the vignette developed an infection 4 days after sustaining
her injury and has visible purulence at the bite site. Late infections that produce purulence are
usually due to Staphylococcus aureus. Infections with Pasteurella multocida can occur following
an animal bite, but they usually develop rapidly (within 24 hours) and exhibit erythema,
tenderness, and edema. Eikenella corrodens is associated more commonly with human than
animal bites. Although dogs have been found to be colonized with group A Streptococcus (GAS),
wound infections with this organism after a bite are uncommon. In addition, GAS is less likely to
produce overt purulence than S aureus. Kingella kingae are found in the human oropharynx and
are an important pathogen in bone and joint disease among children younger than 5 years of
age.
References:
American Academy of Pediatrics. Bite wounds. In: Pickering LK, Baker CJ, Long SS, McMillan
JA, eds. Red Book: 2006 Report of the Committee on Infectious Diseases. 27th ed. Elk Grove
Village, Ill: American Academy of Pediatrics; 2006:191-195
Sagerman PJ. Wounds. Pediatr Rev. 2005;26:43-49. Available at:
http://pedsinreview.aappublications.org/cgi/content/full/26/2/43

Copyright 2009 by the American Academy of Pediatrics

page 499

2009 PREP SA on CD-ROM


Question: 237

A 10-year-old boy was bitten by a dog 2 days ago while visiting relatives in rural Mexico. He was
playing outside with his cousin when a stray dog suddenly ran up and bit him on the arm. After
the incident, the dog ran off and could not be found. His mother washed the wound with soap
and water, but no other medical attention was sought at that time. Physical examination today
reveals a moderately deep bite wound on the boys right forearm that is erythematous, mildly
indurated, and tender, with seropurulent drainage. You prescribe appropriate antibiotic therapy.
Of the following, the MOST appropriate postexposure prophylaxis regimen for this patient is

A. rabies immune globulin alone


B. rabies immune globulin and rabies vaccine
C. rabies immune globulin and tetanus vaccine
D. rabies vaccine and tetanus vaccine
E. rabies vaccine alone

Copyright 2009 by the American Academy of Pediatrics

page 500

2009 PREP SA on CD-ROM


Critique: 237

Preferred Response: B

Infection with the rabies virus produces an acute illness that has rapidly progressive central
nervous system findings, including anxiety, dysphagia, seizures, and encephalitis that, in most
cases, progresses to death. Postexposure prophylaxis for rabies is recommended for all
persons: 1) bitten by wild mammalian carnivores, bats, or domestic animals that may be infected
and 2) who report an open wound, scratch, or mucous membrane that has been contaminated
with saliva or other potentially infectious material from a rabid animal or human. Prophylaxis
should be initiated as soon as possible after bites by known or suspected rabid animals.
The goal of postexposure prophylaxis is to prevent virus from entering neural tissue. Prompt
local treatment of the wound is critical. All wounds should be flushed thoroughly and cleaned with
soap and water. If possible, wounds should not be sutured. The need for tetanus prophylaxis
and antibiotic therapy also should be considered. After wound care is completed, concurrent use
of passive (human rabies immune globulin) (HRIG) and active (rabies vaccine)
immunoprophylaxis is required for optimal therapy. Prophylaxis should begin as soon as
possible after exposure, ideally within 24 hours, but prophylaxis still should be initiated if
indicated, regardless of the interval between exposure and initiation of therapy. Therefore, the
boy described in the vignette should receive both HRIG and rabies vaccine. HRIG should be
used concomitantly with the first dose of vaccine for postexposure prophylaxis. If vaccine is not
available immediately, HRIG should be given alone and immunization started as soon as
possible. Similarly, if HRIG is not available immediately, vaccine should be administered and
HRIG given if it can be obtained within 7 days after initiating immunization. If administration of
both vaccine and HRIG is delayed, both should be used regardless of the interval between
exposure and treatment.
The recommended dose of HRIG is 20 IU/kg. As much of the dose as possible should be
used to infiltrate the wound, if present. The remainder is administered intramuscularly. A 1.0-mL
dose of rabies vaccine is given intramuscularly in the deltoid region or the anterolateral aspect of
the thigh on the first day of postexposure prophylaxis, and repeated doses are provided on days
3, 7, 14, and 28 after the first dose for a total of five doses.
HRIG or rabies vaccine alone or in combination with tetanus vaccine is inadequate
postexposure prophylaxis.
References:
American Academy of Pediatrics. Rabies. In: Pickering LK, Baker CJ, Long SS, McMillan JA,
eds. Red Book: 2006 Report of the Committee on Infectious Diseases. 27th ed. Elk Grove
Village, Ill: American Academy of Pediatrics; 2006:552-559
Centers for Disease Control and Prevention. Human rabies prevention - United States 1999:
recommendations of the Advisory Committee on Immunization Practices (ACIP). MMWR
Recomm Rep. 1999;48(RR-1):1-21. Available at:
http://www.cdc.gov/mmwr/preview/mmwrhtml/00056176.htm
Rupprecht CE, Gibbons RV. Prophylaxis against rabies. N Engl J Med. 2004;351:2626-2635.
Extract available at: http://content.nejm.org/cgi/content/extract/351/25/2626

Copyright 2009 by the American Academy of Pediatrics

page 501

2009 PREP SA on CD-ROM


Question: 238

An 8-year-old boy presents with gross hematuria associated with intermittent right-sided flank
pain. There is no history of dysuria, urgency, frequency, or trauma. Physical examination
reveals a temperature of 98.6F (37C), heart rate of 76 beats/min, respiratory rate of 20
breaths/min, blood pressure of 106/66 mm Hg, and no abdominal or costovertebral angle
tenderness. Urinalysis shows a specific gravity of 1.025, pH of 6, 3+ blood, and trace protein.
Microscopy documents 20 to 50 red blood cells/high-power field. Renal ultrasonography reveals
a normal bladder with mild hydronephrosis on the right and an echogenic focus (Item Q238) with
shadowing in the right kidney.
Of the following, the MOST likely additional expected laboratory feature contributing to this
patients condition is

A. a positive urine culture


B. elevated urinary calcium excretion
C. elevated urinary citrate excretion
D. hypercalcemia
E. metabolic alkalosis

Copyright 2009 by the American Academy of Pediatrics

page 502

2009 PREP SA on CD-ROM


Critique: 238

Preferred Response: B

The renal colic (intermittent flank pain) and gross hematuria combined with red blood cells in the
urine and hydronephrosis with a shadowing echogenic focus in the kidney described for the
patient in the vignette represent urolithiasis. Children who have urolithiasis require treatment for
the stone via either surgical extraction or extracorporeal shock wave lithotripsy under the
guidance of a pediatric urologist. The pediatrician needs to recognize the underlying risk factors
for stone development to aid in the prevention of a recurrence.
Urolithiasis in the pediatric patient usually involves the ureters and upper urinary tract, with a
few patients exhibiting stones within the bladder. The most common types of renal stones in
decreasing order of occurrence are calcium oxalate, calcium phosphate, mixed (calcium oxalate
and calcium phosphate), struvite (magnesium ammonium phosphate), cystine, and uric acid.
The evaluation for urolithiasis often begins with a 24-hour urine collection to look at urine volume
and measure creatinine as reference points to ensure adequate fluid intake and adequate 24hour urine collection. In addition, the 24-hour urine collection is aimed at measuring promoters of
stone formation (calcium, oxalate, uric acid, and cystine) and inhibitors of stone formation
(magnesium and citrate). The most common metabolic abnormality in a child who has stones is
hypercalciuria. If hypercalciuria is present, the child should have serum calcium, phosphorus,
ionized calcium, intact parathyroid hormone, and vitamin D (25-hydoxyvitamin D3 and 1,25dihydoxyvitamin D3) measured.
A child who has a renal stone also might have a urinary tract infection (UTI). Some urinary
pathogens that produce urease (especially Proteus sp) can lead to an environment favorable to
struvite stone formation. Also, even calcium-containing stones can obstruct urine flow and result
in the development of a UTI. However, a UTI is not likely in a patient who has no dysuria,
urgency, or frequency, such as the boy in the vignette. Risk factors for the development of renal
stones include hypercalciuria, hyperoxaluria, decreased urinary magnesium, and decreased
urinary citrate.
Although hypercalciuria is commonly associated with renal stones, hypercalcemia is not a
typical finding. One condition associated with hypercalciuria and renal stones is renal tubular
acidosis. Metabolic alkalosis is seen when calcium phosphate stones are formed, but it is far
less likely than hypercalciuria to be a risk factor for stone formation for the patient in the vignette.
References:
Alon US, Srivastava T. Urolithiasis. In: Kher KK, Schnaper HW, Makker SP, eds. Clinical
Pediatric Nephrology. 2nd ed. London, England: Informa Healthcare; 2007:539-551
Milliner DS. Urolithiasis. In: Avner ED, Harmon WE, Niaudet P, eds. Pediatric Nephrology. 5th ed.
Philadelphia, Pa: Lippincott Williams & Wilkins; 2004:1091-1111

Copyright 2009 by the American Academy of Pediatrics

page 503

2009 PREP SA on CD-ROM


Question: 239

A 12-year-old boy presents with a 3-year history of hay fever in the spring. He experiences daily
nasal congestion, sneezing, and rhinorrhea from March to May that worsens when he is outside.
He is asymptomatic for the remainder of the year, but his parents are concerned because his
symptoms interfere with outdoor sports activities. Use of over-the-counter first-generation
antihistamines resulted in undesirable sedation.
Of the following, the BEST initial medication to treat this patient is a(an)

A. intranasal corticosteroid
B. intranasal decongestant
C. nasal saline spray
D. oral decongestant
E. oral leukotriene antagonist

Copyright 2009 by the American Academy of Pediatrics

page 504

2009 PREP SA on CD-ROM


Critique: 239

Preferred Response: A

The boy described in the vignette presents with classic symptoms of allergic rhinitis. The
approach to medical management of allergic rhinitis should take into account the patient's
primary symptoms, past experiences with medications, and any coexisting medical problems.
Nasal corticosteroids have become a first-line therapy for allergic rhinitis, with studies
demonstrating improvement in nasal pruritus, sneezing, nasal obstruction, and rhinorrhea.
Further, nasal steroids typically are well tolerated and have minimal adverse effects. Nasal
decongestants may be purchased over the counter and markedly reduce nasal congestion, but
recurrent or prolonged use can result in rhinitis medicamentosa. Nasal saline is an adjunct for
nasal toilet, but it is inferior to nasal corticosteroids for the treatment of allergic rhinitis.
Oral decongestants may be taken separately or in combination with an oral antihistamine.
They can improve nasal congestion, but they may increase blood pressure or cause insomnia.
Oral leukotriene antagonists have been approved for allergic rhinitis and mild persistent asthma.
Although this class of medication generally has an excellent safety profile and is not a steroid, it
is inferior to nasal corticosteroids for treatment of nasal allergic rhinitis symptoms.
References:
Atkins D, Leung DYM. Principles of treatment of allergic disease. In: Kliegman RM, Behrman RE,
Jenson HB, Stanton BF, eds. Nelson Textbook of Pediatrics. 18th ed. Philadelphia, Pa:
Saunders Elsevier; 2007:942-948
Mahr TA, Sheth K. Update on allergic rhinitis. Pediatr Rev. 2005;26:284-289. Available at:
http://pedsinreview.aappublications.org/cgi/content/full/26/8/284

Copyright 2009 by the American Academy of Pediatrics

page 505

2009 PREP SA on CD-ROM


Question: 240

You are examining a 7-year-old boy who has a 2-day history of abdominal pain. The pain began
2 nights ago after he ate pizza with the rest of his family and initially was crampy and diffuse. No
one else became ill. He continued to complain of pain through the day yesterday, and this
morning he vomited once, prompting his mother to bring him to the office. The emesis was
nonbilious and nonbloody, and he has had no diarrhea, fever, or urinary symptoms. On physical
examination, the boy is afebrile, has normal vital signs, and has diminished bowel sounds with
involuntary guarding in the right lower quadrant. There are no peritoneal signs, and Rovsing,
obturator, and psoas signs are negative.
Of the following, the study MOST likely to confirm the diagnosis is

A. abdominal radiograph
B. complete blood count
C. computed tomography scan of the abdomen and pelvis
D. C-reactive protein determination
E. procalcitonin determination

Copyright 2009 by the American Academy of Pediatrics

page 506

2009 PREP SA on CD-ROM


Critique: 240

Preferred Response: C

Appendicitis is the most common abdominal surgical condition in children. Obstruction of the
appendix with fecal matter or other debris leads to edema, bacterial overgrowth, mucosal
breakdown, and appendiceal inflammation. Early diagnosis and surgical treatment are the keys
to favorable outcomes.
Diagnosing appendicitis in children can be challenging. When present, the highly sensitive
signs and symptoms of abdominal pain that migrates to the right lower quadrant, fever, and
abdominal rebound or guarding often are enough to prompt surgical exploration. However, the
presentation in children frequently is not classic. The difficulties encountered in evaluating
preverbal patients, the overlap of appendicitis with many other causes of childhood abdominal
pain, and the often atypical presentations contribute to the 28% to 57% appendicitis
misdiagnosis rate in children.
Laboratory and radiologic studies often are used as diagnostic adjuncts when appendicitis is
considered in a patient who has an atypical presentation, such as the child described in the
vignette. Abdominal computed tomography scan (Item C240) is the most useful for confirming
the diagnosis because its sensitivities and specificities approach 100%. There is debate in the
literature regarding the optimal use of oral or rectal contrast for such studies. Abdominal
radiographs are not helpful.
No laboratory tests are independently predictive of the presence or absence of appendicitis.
Although procalcitonin has not been studied in this setting, neither the total white blood count,
absolute neutrophil count, nor C-reactive protein measurement has been found to be adequately
sensitive or specific. However, with growing concern about the long-term effects of ionizing
radiation exposure in children, laboratory tests combined with clinical findings may prove useful
in identifying low-risk individuals for whom imaging can be avoided.
References:
Bundy DG, Byerly JS, Liles EA, Perrin EM, Katznelson J, Rice HE. Does this child have
appendicitis? JAMA. 2007;298:438-451. Abstract available at:
http://www.ncbi.nlm.nih.gov/pubmed/17652298
Kwok MY, Kim MK, Gorelick MH. Evidence-based approach to the diagnosis of appendicitis in
children. Pediatr Emerg Care. 2004;20:690-698
Wesson DE. Evaluation and diagnosis of appendicitis in childhood. UpToDate Online 15.3. 2008.
Available at:
http://www.utdol.com/utd/content/topic.do?topicKey=ped_surg/4980&selectedTitle=4~150&sourc
e=search_result

Copyright 2009 by the American Academy of Pediatrics

page 507

2009 PREP SA on CD-ROM


Question: 241

A 12-year-old boy who has a history of recurrent abdominal pain presents to your office for an
annual health supervision visit. The boy complains of periumbilical pain, unrelated to meals,
occurring twice a month and lasting 15 minutes. Physical examination findings are normal. Fecal
occult blood test results are negative. His father, who is a physician, asks if the boy should
undergo testing for Helicobacter pylori.
Of the following, a TRUE statement about H pylori infection is that

A. all children who have positive H pylori serologies should undergo endoscopy
B. antibiotic therapy for H pylori is most effective when combined with a proton pump inhibitor
C. H pylori is difficult to detect on gastric histology without special immunofluorescence staining
D. H pylori infection is less prevalent in children from the developing world
E. H pylori organisms rarely develop antibiotic resistance

Copyright 2009 by the American Academy of Pediatrics

page 508

2009 PREP SA on CD-ROM


Critique: 241

Preferred Response: B

Helicobacter pylori is a curved, gram-negative bacillus that has a causative role in peptic ulcer
disease. Approximately 50% of individuals worldwide are colonized with H pylori; the organism is
more prevalent in the developing world. Approximately 20% of people infected with H pylori
develop complications from the infection, but there is a long latency period between the time of
infection (usually in childhood) and the time that symptoms develop. The complications of longterm infection may be serious, however, and include gastric and duodenal ulcers, gastric
lymphoma (mucosal associated lymphoid tissue lymphoma [MALToma]), and atrophic gastritis.
If symptoms or clinical findings suggest the presence of H pylori, a number of diagnostic
tests can aid in establishing a diagnosis. The gold standard for H pylori diagnosis is upper
gastrointestinal endoscopy with biopsy of the gastric antrum. Endoscopy often shows nodular
inflammation of the antrum (Item C241A), and biopsy can demonstrate the characteristic
organisms in the gastric epithelium. Most often, organisms can be seen easily on conventional
hematoxylin and eosin stain; in cases involving few organisms, special staining is helpful (Item
C241B). Given the invasiveness of endoscopy, this procedure should not be performed on
every child who has abdominal pain or dyspeptic symptoms. If a clinician wants to rule out H
pylori as a cause of gastrointestinal symptoms, available tests include a serum H pylori
immunoglobulin G serology, a fecal sample for H pylori antigen, or a C13 urea breath test for H
pylori. The limitation of these tests is the occurrence of both false-positive and false-negative
results. In particular, the false-positive rate of serology is about 10%. Therefore, this test has a
poor positive predictive value and probably should not be ordered if the clinical suspicion of H
pylori infection is low (eg, in a child who has periumbilical or lower abdominal pain).
Given that chronic abdominal pain and dyspeptic symptoms are so common in children and
adults, the role of H pylori in chronic pain remains highly controversial. Even if a diagnostic test
for H pylori has a positive result, it may be hard to prove a definitive association between the
patient's symptoms and the H pylori. Therefore, the clinician and patient need to consider
whether to perform definitive testing (ie, endoscopy) or treat empirically. The "test versus treat"
question remains a clinical dilemma because empiric therapy is more cost-effective and less
invasive, but endoscopy provides a definitive diagnosis and excludes other conditions (eg, reflux
esophagitis).
Treatment of H pylori infection usually involves two antibiotics (eg, amoxicillin and
metronidazole or amoxicillin and clarithromycin) in conjunction with a proton pump inhibitor (PPI)
(eg, omeprazole, lansoprazole, pantoprazole). The PPI enhances antimicrobial eradication
because the organisms prefer the normal acid medium of the stomach. Unfortunately,
antimicrobial resistance is common in H pylori, especially to metronidazole. Therefore, if H pylori
is causing serious disease and has been treated, documentation of eradication is suggested
either by negative fecal antigen studies or by endoscopy.
References:
Ford A, McNulty C, Delaney B, Moayyedi A. Helicobacter pylori infection. BMJ Clinical Evidence.
2007. Available for subscription at:
http://clinicalevidence.bmj.com/ceweb/conditions/dsd/0406/0406.jsp
Gold BD, Colletti RB, Abbott M, et al; North American Society for Pediatric Gastroenterology and
Nutrition. Helicobacter pylori infection in children: recommendations for diagnosis and treatment.
J Pediatr Gastroenterol Nutr. 2000;31:490-497
Vilaichone RK, Mahachai V, Graham DY. Helicobacter pylori diagnosis and management.
Gastroenterol Clin North Am. 2006;35:229-247. Abstract available at:
http://www.ncbi.nlm.nih.gov/pubmed/16880064

Copyright 2009 by the American Academy of Pediatrics

page 509

2009 PREP SA on CD-ROM


Question: 242

A term newborn is delivered to a mother who has had a 5-day history of a nonspecific
gastroenteritis, some loose stools, generalized malaise, and low-grade fever. The infant had a
seizure at 6 hours of age and is ill, with an inspired oxygen requirement of 0.40, some petechiae,
and oozing from the umbilicus and phlebotomy sites. He is irritable on neurologic examination.
Laboratory findings include:
White blood cell count, 7.5x10 3/mcL (7.5x109/L)
Platelet count, 90.0x10 3/mcL (90.0x109/L)
Hematocrit, 45% (0.45)
Aspartate aminotransferase, 240.0 U/L
Alanine aminotransferase, 300.0 U/L
Fibrinogen, 90.0 mg/dL (2.6 mcmol/L)
Prothrombin time, 20 seconds
Partial thromboplastin time, 60 seconds
Internationalized Normalized Ratio (INR), 1.80
Serum glucose, 90.0 mg/dL (5.0 mmol/L)
A lumbar puncture reveals 35 white blood cells, with 50% polymorphonuclear cells and 50%
mononuclear cells; 1 red blood cell; glucose of 60.0 mg/dL (3.3 mmol/L); and protein of 100
mg/dL (1,000 g/L). No organisms are seen on cerebrospinal fluid (CSF) Gram stain.
Of the following, a TRUE statement about this patients meningitis is that

A. gram-negative organisms are unlikely to be causative


B. group B streptococcal meningitis is likely to be the cause
C. infection likely is related to maternal enteroviral infection
D. the abnormal CSF glucose and protein values indicate bacterial meningitis
E. the abnormal liver function test results and CSF cell counts indicate herpes simplex virus
infection

Copyright 2009 by the American Academy of Pediatrics

page 510

2009 PREP SA on CD-ROM


Critique: 242

Preferred Response: C

The infant described in the vignette has symptoms of respiratory distress, neurologic irritability,
and coagulopathy. The likely viral gastroenteritis suffered by his mother was of short enough
duration not to have conferred any protective antibody transfer to the fetus. His laboratory
findings are most concerning for thrombocytopenia, coagulopathy, and elevated hepatic
transaminases. His lumbar puncture results indicate a pleocytosis consistent with meningitis.
Neonatal meningitis of either viral or bacterial cause typically occurs in the face of a sepsis
syndrome that may be associated with known risk factors prenatally (maternal fever,
chorioamnionitis, prolonged rupture of membranes) or postnatally (bacteremia, urinary tract
infection, or respiratory distress), with attendant signs of apnea, lethargy, acidosis, and
hypoglycemia. The infection most often is bacterial. Physical examination findings can include a
bulging or tense anterior fontanelle, irritability, fever, emesis, coma, and seizures; neuromotor
tone may be increased or decreased. CSF values in bacterial meningitis generally include
elevated protein values to more than 150 mg/dL (1,500 g/L), white blood cell count of more than
100, and a glucose value less than 50% of that measured in the blood. These values may differ
greatly from those seen in viral (aseptic) meningitis of the newborn, such as that affecting the
infant described in the vignette. Neonatal enteroviral infection may follow maternal infection,
characteristically in the spring and summer seasons. Clinically, affected newborns manifest
illness with a sepsis syndrome, coagulopathy, and meningitis. Hepatopathy and respiratory
distress also may be significant.
The mortality rate of neonatal bacterial meningitis generally is cited at 10%, but both mortality
and late morbidity vary with the cause of meningitis in the newborn. The most common
pathogens for neonatal meningitis include group B Streptococcus, Escherichia coli, other gramnegative rods, and Listeria monocytogenes. Although neonatal group B streptococcal meningitis
is common, it generally is associated with later onset (after 7 days), a shocklike state, and CSF
findings consistent with bacterial meningitis, as noted previously. Gram-negative meningitis
varies with perinatal exposure, prior maternal or neonatal antibiotic use, comorbidities, and
neonatal intensive care unit (NICU) practices. S pneumoniae and Haemophilus influenzae are
uncommon causes of meningitis in the newborn. Staphylococcal infections may be increasing
causes of concern in the NICU. Methicillin-resistant S aureus meningitis may require aggressive
therapy and isolation. Herpes simplex viral meningoencephalitis is associated with a
hemorrhagic pleocytosis, apnea, seizures, coagulopathy, and hepatic transaminase values
generally greater than 1,000 U/L.
References:
Heath PT, Nik Yusoff NK, Baker CJ. Neonatal meningitis. Arch Dis Child Fetal Neonatal Ed.
2003;88:F173-F178. Abstract available at: http://www.ncbi.nlm.nih.gov/pubmed/12719388
Klinger G, Chin C-N, Beyene J, Perlman M. Predicting the outcome of neonatal bacterial
meningitis. Pediatrics. 2000;106:477-482. Available at:
http://pediatrics.aappublications.org/cgi/content/full/106/3/477
Miyairi I, Berlingieri D, Protic J, Belko J. Neonatal invasive group A streptococcal disease: case
report and review of the literature. Pediatr Infect Dis J. 2004;23:161-165. Abstract available at:
http://www.ncbi.nlm.nih.gov/pubmed/14872185
Moylett EH. Neonatal Candida meningitis. Semin Pediatr Infect Dis. 2003;14:115-122. Abstract
available at: http://www.ncbi.nlm.nih.gov/pubmed/12881799
Philip AGS. Neonatal meningitis in the new millennium. NeoReviews. 2003;4:e73-e80. Available
for subscription at: http://neoreviews.aappublications.org/cgi/content/full/4/3/e73
Polin RA, Harris MC. Neonatal bacterial meningitis. Semin Neonatol. 2001;6:157-172. Abstract
available at: http://www.ncbi.nlm.nih.gov/pubmed/11483021

Copyright 2009 by the American Academy of Pediatrics

page 511

2009 PREP SA on CD-ROM


Question: 243

A 5-month-old boy is brought to the emergency department by his mother because of


decreased activity and vomiting for 1 day. She reports occasional foul-smelling stools but no
recent changes in stool pattern. There has been no fever. As a neonate, the boy had difficulty
gaining weight and prolonged jaundice, but he has not required hospitalization. Physical
examination reveals an ill-appearing child who has mild dehydration, a heart rate of 120
beats/min, and otherwise normal vital signs. He appears somewhat cachectic, and his weight is
at the 3rd percentile. Laboratory values include a normal complete blood count and urinalysis,
sodium of 134.0 mEq/L (134.0 mmol/L), chloride of 86.0 mEq/L (86.0 mmol/L), potassium of 3.8
mEq/L (3.8 mmol/L), and carbon dioxide of 31.0 mEq/L (31.0 mmol/L). Blood urea nitrogen and
creatinine values are within normal limits.
Of the following, the MOST likely diagnosis is

A. Bartter syndrome
B. congenital adrenal hyperplasia
C. cystic fibrosis
D. Fanconi syndrome
E. hypertrophic pyloric stenosis

Copyright 2009 by the American Academy of Pediatrics

page 512

2009 PREP SA on CD-ROM


Critique: 243

Preferred Response: C

Cystic fibrosis is an autosomal recessive disorder affecting many children and adolescents. It is
caused by a defect in a chloride channel, the cystic fibrosis transmembrane conductance
regulator (CFTR), on the apical membranes of the linings of the airways, intestinal tract, vas
deferens, biliary tree, pancreatic ducts, and sweat ducts. The result is ineffective secretion of
fluids from affected areas and an increased sodium and chloride sweat concentration, the latter
being the basis for diagnostic testing. Affected children have varying degrees of mucoid airway
obstruction, with secondary bacterial infections, failure to thrive, intestinal obstruction, pancreatic
and biliary dysfunction, and infertility. Hypochloremic metabolic alkalosis with dehydration, as
described for the boy in the vignette, is a common feature because of the high salt loss from the
sweat glands. Foul-smelling stools, prolonged jaundice, and poor weight gain are additional
features of the disease. Therefore, the boy in the vignette should undergo testing for cystic
fibrosis (eg, sweat chloride testing and genetics testing).
Bartter syndrome results from a defect in chloride reabsorption in the loop of Henle. Clinical
features include failure to thrive, polyuria, and vomiting. Hypochloremia and metabolic alkalosis
can occur, but hypokalemia, which can be severe, is usual and due to urinary potassium
wasting. Infants who have the salt-losing form of congenital adrenal hyperplasia also may
present with vomiting and failure to thrive, but typical electrolyte abnormalities are hyponatremia,
hypochloremia, and hyperkalemia. Fanconi syndrome is characterized by abnormal proximal
renal tubule function. Excessive bicarbonaturia causes metabolic acidosis with hyperchloremia,
and hypokalemia may be seen. Clinical features in infancy can be similar to those of the
previously described diseases, with failure to thrive, vomiting, and polyuria being common.
Infants who have hypertrophic pyloric stenosis often have hypochloremic metabolic alkalosis,
but this condition is seen in younger infants and is the result of substantial vomiting of a long
duration.
References:
Boat TF, Acton JD. Cystic fibrosis. In: Kliegman RM, Behrman RE, Jenson HB, Stanton BF,
eds. Nelson Textbook of Pediatrics. 18th ed. Philadelphia, Pa: Saunders Elsevier; 2007:18031816
Davis PB. Cystic fibrosis. Pediatr Rev. 2001;22:257-264. Available at:
http://pedsinreview.aappublications.org/cgi/content/full/22/8/257

Copyright 2009 by the American Academy of Pediatrics

page 513

2009 PREP SA on CD-ROM


Question: 244

A 15-month-old girl presents to the emergency department with a temperature of 103F (39.5C)
during respiratory virus season. Physical examination reveals rhinorrhea and mild cough but no
other focus of infection. However, she has diffuse bruises in various stages of healing on her
abdomen, subscapular area, and both extensor and flexor surfaces of her extremities.
Laboratory studies reveal a white blood cell count of 9.2x103/mcL (9.2x109/L) with a normal
differential count, platelet count of 376.0x103/mcL (376.0x109/L), hemoglobin of 13.0 g/dL (130.0
g/L), and hematocrit of 39% (0.39).
Of the following, the BEST next step in the evaluation of this child is

A. computed tomography scan of the brain


B. measurement of factor VIII
C. measurement of von Willebrand factor
D. prothrombin time and partial thromboplastin time
E. radiographic skeletal survey

Copyright 2009 by the American Academy of Pediatrics

page 514

2009 PREP SA on CD-ROM


Critique: 244

Preferred Response: E

Bruises on the back (Item C244), abdomen, and flexor surfaces in various stages of healing, as
reported for the child in the vignette, are suspicious for nonaccidental trauma. Thus, the child
should undergo a bone survey to look for occult fractures.
Bruising in older infants and toddlers often results from the combination of increasing mobility
and developing coordination and stability. However, bruising that relates to falls associated with
creeping, crawling, cruising, and early walking is seen primarily on the front of the body,
especially on the forehead, forearms, and pretibial areas of the legs, where the child "meets"
obstacles in the environment.
The appearance of a bruise depends on its age, location, and depth as well as the child's
skin pigmentation. A review of the literature about "dating" bruises in children reveals that a
bruise may appear red, blue, or purple at any time from onset to resolution but often contains
yellow discoloration after 18 hours from onset. However, even multiple bruises resulting from the
same injury at the same time in the same person may "age" differently, so it may be difficult to
age bruises accurately.
Coagulation disorders such as hemophilia and von Willebrand disease may present with
bruising but often are associated with other physical findings and a history of mucosal bleeding.
Studies that can assess for a coagulation disorder, such as measuring prothrombin time, partial
thromboplastin time, von Willebrand factor, and factor VIII, are unlikely to yield positive results for
the child described in the vignette.
In the absence of signs of head contusion, irritability, focal neurologic signs, or
developmental delay, computed tomography scan of the brain is unlikely to be helpful and would
result in unnecessary exposure to ionizing radiation. Although not specifically addressed in this
vignette, some recommend evaluation for possible soft-tissue injury, including renal, hepatic, and
splenic contusions or laceration, with abdominal ultrasonography. Minor elevations in liver
enzymes and microscopic hematuria may be found with less severe injury. Ophthalmologic
examination may detect retinal hemorrhages, which indicates a potential need for brain imaging,
preferably with magnetic resonance imaging in a neurologically stable child.
Evaluation of the family and caretaker systems by social services and possibly a report for
evaluation by child protective services is clearly indicated when possible child abuse is
suspected. Development of a safety plan may improve the outcome for women who are in
abusive relationships with domestic partners, which is a risk factor for child abuse, and
decrease episodes of violence, at least in the short term.
References:
Barlow J, Johnston I, Kendrick D, Polnay L, Stewart-Brown S. Individual and group-based
parenting programmes for the treatment of physical child abuse and neglect. Cochrane
Database Syst Rev. 2006;3:CD005463. Available at:
http://www.mrw.interscience.wiley.com/cochrane/clsysrev/articles/CD005463/frame.html
Coulter K. In brief: bruising and skin trauma. Pediatr Rev. 2000;21:34-35. Available at:
http://pedsinreview.aappublications.org/cgi/content/full/21/1/34
Kellogg ND and the Committee on Child Abuse and Neglect. Evaluation of suspected child
physical abuse. Pediatrics. 2007;119:1232-1241. Available at:
http://pediatrics.aappublications.org/cgi/content/full/119/6/1232
Klevens J, Sadowski L. Intimate partner violence towards women (update). BMJ Clinical
Evidence. 2007. Available for subscription at:
http://clinicalevidence.bmj.com/ceweb/conditions/woh/1013/1013_I5.jsp
Labb J, Caouette G. Recent skin injuries in normal children. Pediatrics. 2001;108:271-276.
Available at: http://pediatrics.aappublications.org/cgi/content/full/108/2/271

Copyright 2009 by the American Academy of Pediatrics

page 515

2009 PREP SA on CD-ROM

Maguire S, Mann MK, Sibert J, Kemp A. Are there patterns of bruising in childhood which are
diagnostic or suggestive of abuse? A systematic review. Arch Dis Child. 2005;90:182-186.
Abstract available at: http://www.ncbi.nlm.nih.gov/pubmed/15665178
Sugar NF, Taylor JA, Feldman KW and the Puget Sound Pediatric Research Network. Bruises in
infants and toddlers: those who don't cruise rarely bruise. Arch Pediatr Adolesc Med.
1999;153:399-403. Available at: http://archpedi.ama-assn.org/cgi/content/full/153/4/399

Copyright 2009 by the American Academy of Pediatrics

page 516

2009 PREP SA on CD-ROM


Question: 245

You receive a telephone call from the physician mother of a 1-week-old patient who was born at
24 weeks' gestation. He is being treated in the neonatal intensive care unit and has been stable
on the ventilator. She is concerned because when she visited him this morning, his blood
pressure was 44/26 mm Hg. His mean arterial pressure was 30 mm Hg. She is worried that his
blood pressure is low and that this may be harmful.
Of the following, the MOST accurate statement regarding blood pressure in the preterm infant is
that

A. blood pressure values for preterm infants should be compared with those for term infants
B. blood pressure values vary indirectly with gestational age
C. mean arterial pressure should be no less than the corrected gestational age in weeks
D. patent ductus arteriosus narrows the pulse pressure by raising the diastolic pressure
E. systemic hypertension typically occurs coincidentally with pulmonary hypertension

Copyright 2009 by the American Academy of Pediatrics

page 517

2009 PREP SA on CD-ROM


Critique: 245

Preferred Response: C

Preterm birth can be associated with a number of morbidities, including hyper- or hypotension.
There are well-described normative values for blood pressure in a healthy term newborn, but
normal blood pressure values for the preterm infant are much less clear. In fact, there may be
no true correct or expected blood pressure for the preterm infant, particularly the child who is
born extremely preterm. Blood pressure increases during the first few days and weeks after
birth for each gestational age. Neonatologists generally agree that the mean arterial blood
pressure for a preterm infant should not be less than the corrected gestational age in weeks.
Thus, for example, a 26-week-gestation infant should have a mean arterial blood pressure in
excess of 26 mm Hg. Beyond this simple rule, a given blood pressure value can be considered
adequate if there is no evidence of metabolic acidosis, elevated lactate concentration, or
inadequate end-organ perfusion.
Just as infant and child blood pressure norms are not compared with adult norms, the
normative blood pressure for a term infant is not used to assess the blood pressure of the
preterm infant. As noted previously, blood pressure varies directly, not indirectly, with advancing
gestational age. A patent ductus arteriosus widens the blood pressure by allowing blood to be
diverted away from the higher-resistance systemic circulation and toward the lower-resistance
pulmonary circulation. As this "steal" becomes greater, the diastolic pressure decreases and the
pulse pressure (difference between the systolic and diastolic pressure) increases. Finally,
systemic hypertension usually does not occur with pulmonary hypertension. In fact, it is not
uncommon to note systemic hypotension with pronounced pulmonary hypertension as the failing
right ventricle leads to a decrease in cardiac output.
References:
Padbury JF. Neonatal hypotension and hypovolemia. In: Rudolph C, Rudolph A, eds. Rudolph's
Pediatrics. 21st ed. New York, NY: McGraw Hill Medical Publishing Division; 2003:137-140
Nwankwo MU, Lorenz JM, Gardiner JC. A standard protocol for blood pressure measurement in
the newborn. Pediatrics. 1997;99:E10. Available at:
http://pediatrics.aappublications.org/cgi/content/full/99/6/e10
Weindling AM, Subhedar NV. The definition of hypotension in very low-birthweight infants during
the immediate neonatal period. NeoReviews. 2007;8:e32. Available for subscription at:
http://neoreviews.aappublications.org/cgi/content/full/8/1/e32
Zubrow AB, Hulman S, Kushner H, Falkner B. Determinants of blood pressure in infants
admitted to neonatal intensive care units: a prospective multicenter study. Philadelphia Neonatal
Blood Pressure Study Group. J Perinatol. 1995;15:470-479. Abstract available at:
http://www.ncbi.nlm.nih.gov/pubmed/8648456

Copyright 2009 by the American Academy of Pediatrics

page 518

2009 PREP SA on CD-ROM


Question: 246

A 17-year-old boy who receives carbamazepine for epilepsy presents to the emergency
department after a 40-minute generalized tonic-clonic seizure. He has been well, and there is no
history of trauma. On physical examination, he answers a few questions, but he is sleepy and
confused. He is afebrile, and his vital signs are normal. Although he is uncooperative, he moves
all limbs spontaneously with good strength.
Of the following, the diagnostic test that is MOST likely to explain this seizure is

A. magnetic resonance imaging


B. noncontrast head computed tomography scan
C. prolonged electroencephalography
D. serum anticonvulsant measurement
E. serum electrolyte measurement

Copyright 2009 by the American Academy of Pediatrics

page 519

2009 PREP SA on CD-ROM


Critique: 246

Preferred Response: D

The adolescent described in the vignette has a prior diagnosis of epilepsy. He now is in a
postictal state after status epilepticus, but during the assessment in the emergency department
appears to be returning to normal consciousness. The most likely causes for a prolonged
seizure in this setting are his epilepsy and failure to take his medication. Accordingly, measuring
his serum drug concentrations should be most helpful in explaining the seizure.
Brief or prolonged seizures can be symptomatic of intracranial lesions such as ischemic
strokes, trauma, hemorrhages, neoplasms, or focal or generalized brain infections. They also
can occur due to metabolic derangements involving glucose or electrolytes. It is important to
consider this broad range of causes in any child presenting to the emergency department with
seizure, particularly one in status epilepticus, defined as any seizure lasting 30 minutes or
longer. If the seizure has a focal onset, there is a residual focal deficit, or the seizure is
accompanied by fever and prolonged confusion, imaging studies should be ordered emergently.
For infants or for those children whose histories suggest electrolyte disturbance, glucose and
electrolyte testing can be helpful. Electroencephalography (EEG) may be helpful if confusion is
prolonged because occasionally the prolonged confusion may be due to nonconvulsive status
epilepticus. In this case, however, the child's postictal state is resolving. Therefore, EEG will not
add anything useful to initial medical decisions.
For the child or adolescent who has a prior diagnosis of epilepsy, the most likely cause of
the prolonged seizure is noncompliance with medications. Illicit drug use also may be involved
and is worth considering in the differential diagnosis. In addition, excessively high serum drug
concentrations of carbamazepine, sometimes due to drug-drug interactions, occasionally can
precipitate seizures. Newer antiseizure medications are used more frequently in children.
Unfortunately, many hospital laboratories are unable to measure these rapidly. A history
obtained from the family ("He just ran out of medicine, and we couldn't get to the pharmacy") or a
phone call to the patient's neurologist ("This has happened before - he doesn't take his
medication and misses appointments") may help clarify the diagnosis in these cases.
References:
Riviello JJ Jr, Ashwal S, Hirtz D, et al. Practice parameter: diagnostic assessment of the child
with status epilepticus (an evidence-based review). Report of the Quality Standards
Subcommittee of the American Academy of Neurology and the Practice Committee of the Child
Neurology Society. Neurology. 2006;67:1542-1550. Available at:
http://www.neurology.org/cgi/content/full/67/9/1542

Copyright 2009 by the American Academy of Pediatrics

page 520

2009 PREP SA on CD-ROM


Question: 247

A mother brings in her 4-month-old baby because she is concerned about the infants head
shape. The baby is growing and developing normally. Physical examination findings are normal
except for a flat occiput and a wide biparietal diameter with a flat forehead. The head
circumference is normal, and the anterior fontanelle is small but patent.
Of the following, this babys findings MOST likely are caused by

A. bilateral coronal suture synostosis


B. bilateral lambdoid suture synostosis
C. metopic suture synostosis
D. positional plagiocephaly
E. sagittal suture synostosis

Copyright 2009 by the American Academy of Pediatrics

page 521

2009 PREP SA on CD-ROM


Critique: 247

Preferred Response: A

Craniosynostosis, the premature fusion of one or more cranial sutures, may have a pre- or
postnatal onset. The causes are varied and include mechanical forces, suboptimal brain growth,
gene mutations, and metabolic derangements. Craniosynostosis may be primary, in the case of
a genetic cause, or secondary, as can be seen with vitamin D deficiency or hyperthyroidism.
Sutural formation and skull growth are exceedingly complex. Histologically, craniosynostosis
begins at a single point and spreads along the suture. Skull growth typically occurs in a direction
perpendicular to each of the major sutures (Item C247A).
The infant described in the vignette demonstrates a short anteroposterior (AP) dimension to
the skull and a wide biparietal dimension with flattening of the forehead. These findings are
consistent with bilateral coronal synostosis, in which growth of the skull perpendicular to the
coronal sutures is limited (Item C247B). Coronal suture synostosis may be isolated but
frequently is associated with large thumbs or great toes and often is due to dominant mutations
in fibroblast growth factor receptor (FGFR) genes. For infants who have this presentation, it is
imperative to take a family history to determine if there are similarly affected relatives.
Metopic suture synostosis results in trigonocephaly, or a "keel-shaped" skull (Item C247C).
Sagittal suture synostosis causes the head shape to be scaphocephalic (long in the AP
dimension and narrow in the biparietal dimension) (Item C247D).
Posterior plagiocephaly (flattening of one or both sides of the occiput) that results from
premature closure of the lambdoid suture(s) is rare and often is confused with positional
plagiocephaly (Item C247E). Since the advent of the recommendation that infants be placed on
their backs to sleep, there has been a dramatic increase in positional plagiocephaly, which is
characterized by unilateral flattening of the occiput and compensatory prominence of the
forehead on the same side. Positional plagiocephaly can be ameliorated by the use of a helmet
molded to fit the infant's skull; ideally, this therapy should be initiated prior to 9 months of age.
References:
Cohen MM Jr. Fibroblast growth factor receptor mutations. In: Cohen MM Jr, MacLean RE, eds.
Craniosynostosis: Diagnosis, Evaluation, and Management. 2nd ed. New York, NY: Oxford
University Press; 2000:77-94
Cohen MM Jr. History, terminology, and classification of craniosynostosis. In: Cohen MM Jr,
MacLean RE, eds. Craniosynostosis: Diagnosis, Evaluation, and Management. 2nd ed. New
York, NY: Oxford University Press; 2000:103-111
Robin NH, Falk MJ, Hlademan-Englert CR. FGFR-related craniosynostosis syndromes.
GeneReviews. 2007. Available at:
http://www.geneclinics.org/servlet/access?db=geneclinics&site=gt&id=8888891&key=xAcWBcrj
mZrVo&gry=&fcn=y&fw=MR4U&filename=/profiles/craniosynostosis/index.html

Copyright 2009 by the American Academy of Pediatrics

page 522

2009 PREP SA on CD-ROM


Question: 248

The parents of a 14-year-old girl are concerned about her weight loss. Her weight today is 20 lb
less than a documented weight obtained 1 year ago at her camp physical examination. She
complains of frequent nausea, decreased appetite, and early satiety, even after eating very
small portions. She has no vomiting or diarrhea, but frequent constipation. She complains of
increased fatigue but is still able to participate in diving 5 days a week. She is doing well in school
academically. She attained menarche at 12 years of age and had monthly periods for about 18
months, but she has had no menses for the past 7 months. She has been a vegetarian for the
past 18 months and feels she is at a good weight currently. On physical examination, her body
mass index is 17.0. Her urine pregnancy test result is negative.
Of the following, the MOST likely diagnosis is

A. anorexia nervosa
B. depression
C. hypothalamic tumor
D. hypothyroidism
E. inflammatory bowel disease

Copyright 2009 by the American Academy of Pediatrics

page 523

2009 PREP SA on CD-ROM


Critique: 248

Preferred Response: A

The diagnosis of an eating disorder is not always straightforward, but the decreased appetite
and early satiety, large weight loss, amenorrhea, and disturbed body image with a low body
mass index reported for the girl in the vignette strongly suggest anorexia nervosa. Participating
in a body-conscious sport such as diving, becoming a vegetarian as a way to restrict the diet,
and the presence of constipation also may be associated with this condition.
The lack of diarrhea or hematochezia makes inflammatory bowel disease less likely. Weight
loss, doing well in school, and maintaining a strong exercise program are uncommon in
hypothyroidism. Depression can accompany an eating disorder, but the early satiety,
constipation, amenorrhea, and good school performance are unlikely to occur if depression is
the sole diagnosis. A hypothalamic tumor typically presents with neurologic symptoms or signs
of increased intracranial pressure.
A limited laboratory evaluation that includes a complete blood count; metabolic panel;
urinalysis; measurement of free thyroxine, thyroid-stimulating hormone, luteinizing hormone,
follicle-stimulating hormone, and prolactin; and electrocardiography may be useful in ruling out
other possible organic causes of this patient's weight loss and symptoms.
References:
Fisher M. Treatment of eating disorders in children, adolescents, and young adults. Pediatr Rev.
2006;27:5-16. Available at: http://pedsinreview.aappublications.org/cgi/content/full/27/1/5
Rome ES, Ammerman S, Rosen DS, et al. Children and adolescents with eating disorders: the
state of the art. Pediatrics. 2003;111:e98-e108. Available at:
http://pediatrics.aappublications.org/cgi/content/full/111/1/e98
Rosen DS. Eating disorders in children and young adolescents: etiology, classification, clinical
features, and treatment. Adolesc Med. 2003;14:49-59. Abstract available at:
http://www.ncbi.nlm.nih.gov/pubmed/12529190

Copyright 2009 by the American Academy of Pediatrics

page 524

2009 PREP SA on CD-ROM


Question: 249

You are evaluating an 8-month-old infant in preparation for administering chloral hydrate to
perform a sedated brainstem auditory evoked potentials test.
Of the following, a CONTRAINDICATION to proceeding with the planned sedation is

A. absence of intravenous access


B. history of delivery at 36 weeks estimated gestational age
C. ingestion of infant formula 3 hours ago
D. ingestion of apple juice 3 hours ago
E. no previous anesthetic administration

Copyright 2009 by the American Academy of Pediatrics

page 525

2009 PREP SA on CD-ROM


Critique: 249

Preferred Response: C

The goal of procedural sedation is to provide anxiolysis and analgesia and minimize patient
movement. Sedation should be viewed as a continuum; the American Academy of Pediatrics has
defined four levels of sedation. Minimal sedation (formerly anxiolysis) is defined as a druginduced state in which the patient retains the ability to respond normally to verbal commands,
and cardiovascular and respiratory functions are unaffected. Moderate sedation (formerly
conscious sedation) is a drug-induced depression of consciousness in which patients still
should respond to verbal and physical stimulation. Cardiovascular status is maintained, but the
clinician must be able to recognize and respond to potential airway compromise. Deep sedation
is defined as further depression of the level of consciousness, with partial or complete loss of
protective airway reflexes and the need for assistance with airway maintenance. General
anesthesia is the deepest level of sedation that is characterized by loss of consciousness and
airway protective reflexes. Impairment of respiratory and cardiovascular function is common.
Provision of safe and effective sedation requires appropriate patient selection, evaluation,
and preparation. Appropriate monitoring and resuscitation equipment, thorough documentation,
and trained personnel are essential. A contraindication to proceeding with the planned sedation
for the patient in the vignette is the ingestion of formula 3 hours ago, which increases the risk of
aspiration due to potential loss of airway reflexes. Children should not ingest formula or food
within 6 hours or human milk within 4 hours of sedation. Clear liquids such as juice are allowed
up to 2 hours prior to a planned elective sedation and may help reduce gastric contents. Neither
the history of birth at 36 weeks estimated gestational age nor the lack of previous anesthetic
administration are contraindications, but the preanesthetic history should explore any
complications at birth as well as any family history of difficulty with sedation or anesthesia.
Because preterm infants are at increased risk of apnea and bradycardia following sedation until
they have achieved a postconceptual age of 52 weeks, they deserve additional monitoring.
Pediatric sedations frequently are achieved with oral medications, but personnel and equipment
to obtain urgent intravenous access should be readily available.
References:
American Academy of Pediatrics, American Academy of Pediatric Dentistry, Cot CJ, Wilson S,
AAP Work Group on Sedation. Guidelines for monitoring and management of pediatric patients
during and after sedation for diagnostic and therapeutic procedures: an update. Pediatrics.
2006;118:2587-2602. Available at:
http://pediatrics.aappublications.org/cgi/content/full/118/6/2587
Koh JL, Palermo T. Conscious sedation: reality or myth? Pediatr Rev. 2007:28:243-248.
Available at: http://pedsinreview.aappublications.org/cgi/content/full/28/7/243
Wetzel R. Anesthesia and perioperative care. In: Kliegman RM, Behrman RE, Jenson HB,
Stanton BF, eds. Nelson Textbook of Pediatrics. 18th ed. Philadelphia, Pa: Saunders Elsevier;
2007:460-474

Copyright 2009 by the American Academy of Pediatrics

page 526

2009 PREP SA on CD-ROM


Question: 250

You observe a child entering the waiting room, accompanied by her mother. She looks at the
receptionist and says "Hi." While holding her doll, the child turns to her mother and says "juice."
The mother gives her a cup of juice, and the child says "doll" and tries to give the doll a drink.
The mother shakes her head, and the child says "no." The child then points to her own mouth,
smiles, and says "mouth." The mother takes a tissue to clean the dolls face. The child says
"me" and begins to imitate her mothers action with another wipe. The child looks at her mother,
says "ma ma," and gives her mother a hug.
Of the following, these developmental milestones are MOST typical for a child whose age is

A. 12 months
B. 15 months
C. 18 months
D. 24 months
E. 30 months

Copyright 2009 by the American Academy of Pediatrics

page 527

2009 PREP SA on CD-ROM


Critique: 250

Preferred Response: C

The developmental milestones described in the vignette are appropriate for a child of 18 months
of age. A healthy 18-month-old child may speak 7 to 10 words spontaneously and correctly, use
words for wants or needs, identify one or more body parts, and imitate household tasks.
Typically developing infants of 12 months of age can understand that a particular set of sounds
represents a certain object or action and may be just beginning to say their first words. A 15month-old child can say four to six specific words, use jargon, and indicate some desire or need
by pointing. An 18-month-old child may have a vocabulary of 20 to 29 words and will point to four
body parts on request. A 24-month-old child typically has a vocabulary of more than 50 words,
is beginning to use two- to three-word sentences, and will point to seven body parts and name at
least one part. A 30-month-old child can speak in eight- or nine-word sentences, identify his or
her sex, name seven body parts, and relate events that occurred 2 to 3 days ago.
When evaluating a child's language, cognitive, and behavioral development, the clinician
needs to determine if the language development is appropriate to the cognitive development and
if there are any atypical social behaviors.
References:
Dedrick C. Developmental milestones. Developmental Behavioral Pediatrics Online. 2005.
Available at: http://dbpeds.org/articles/detail.cfm?TextID=%20701
Knobloch H, Stevens FM, Malone AF. The revised developmental stages. In: Manual of
Developmental Diagnosis. Albany, NY: Developmental Evaluation Materials, Inc; 1987:17-120
Page Glascoe F. Developmental screening. In: Parker S, Zukerman B, Augustyn M, eds.
Developmental and Behavioral Pediatrics: A Handbook for Primary Care. 2nd ed. Philadelphia,
Pa: Lippincott Williams & Wilkins; 2005:41-50
Whitaker T, Palmer F. The developmental history. In: Accardo PJ. Capute & Accardo's
Neurodevelopmental Disabilities in Infancy and Childhood. Volume I: Neurodevelopmental
Diagnosis and Treatment. 3rd ed. Baltimore, Md: Paul H. Brookes Publishing Co; 2008:297-310

Copyright 2009 by the American Academy of Pediatrics

page 528

2009 PREP SA on CD-ROM


Question: 251

The hospital laboratory calls your office to tell you that the rapid plasma reagin (RPR) test on the
cord blood of a newborn you saw yesterday in the hospital is positive at 1:4. You recall that the
physical examination findings for the infant were normal.
Of the following, the MOST appropriate approach to this infant is to

A. begin penicillin therapy


B. obtain further maternal history
C. place the infant in contact isolation
D. repeat the RPR in 3 months
E. report the case to the Department of Health

Copyright 2009 by the American Academy of Pediatrics

page 529

2009 PREP SA on CD-ROM


Critique: 251

Preferred Response: B

Two serologic tests are used routinely to screen a patient for syphilis. The rapid plasma regain
(RPR) and the Venereal Disease Research Laboratory (VDRL) tests both are based on the
serologic detection of immunoglobulin G (IgG) antibody titers against lipoidal antigen from
Treponema pallidum. These tests are inexpensive, provide rapid results, and can be quantitated.
The quantitation tends to correlate with disease activity. Results of both of these nontreponemal
screening tests should be confirmed with a treponemal test such as the fluorescent treponemal
antibody absorption (FTA-ABS) or the T pallidum particle agglutination (TP-PA). The treponemal
tests are specific for the diagnosis of syphilis, are nonquantitative, and usually remain positive
for life.
Maternal passage of IgG antibody begins after 28 weeks of pregnancy. Therefore, results of
cord blood testing of a newborn, such as in the infant described in the vignette, reflect the
mother's status. Because the infant has a positive cord blood titer at 1:4 without signs or
symptoms of congenital syphilis, more maternal history is required. The RPR or VDRL can be
falsely positive due to concomitant viral infections (eg, Epstein-Barr virus), connective tissue
disease, malignancy, intravenous drug abuse, endocarditis, laboratory error, contamination of
the specimen by Wharton jelly when cord blood is used, or even pregnancy, thus necessitating
the need for more history.
Until more history is obtained, there is no indication for penicillin therapy. Unless the mother
has a history of untreated syphilis, there is no indication to make a report to the health
department. Newborns who have open skin lesions or snuffles (chronic rhinorrhea) (Item C251)
due to syphilis can be contagious, but because the patient in the vignette is asymptomatic, there
is no need for isolation. Waiting 3 months and repeating the testing is not the best approach
because if the mother does have syphilis without a history of treatment, the infant requires
therapy before leaving the hospital. For patients who acquire syphilis through sexual contact, a
positive RPR or VDRL with a positive confirmatory FTA-ABS or TP-PA confirms the diagnosis.
References:
American Academy of Pediatrics. Syphilis. In: Pickering LK, Baker CJ, Long SS, McMillan JA,
eds. Red Book: 2006 Report of the Committee on Infectious Diseases. 27th ed. Elk Grove
Village, Ill: American Academy of Pediatrics; 2006:631-644
Hyman EL, Adam HM. In brief: syphilis. Pediatr Rev. 2006;27:37-39. Available at:
http://pedsinreview.aappublications.org/cgi/content/full/27/1/37

Copyright 2009 by the American Academy of Pediatrics

page 530

2009 PREP SA on CD-ROM


Question: 252

You are speaking to a group of neonatal nurses about the laboratory methods that can be used
to make the diagnosis of human immunodeficiency virus infection/acquired immune deficiency
syndrome in high-risk infants.
Of the following, the test that is MOST likely to confirm the diagnosis is

A. cord blood culture


B. DNA polymerase chain reaction
C. neonatal specific immunoglobulin
D. p24 antigen
E. RNA polymerase chain reaction

Copyright 2009 by the American Academy of Pediatrics

page 531

2009 PREP SA on CD-ROM


Critique: 252

Preferred Response: B

Diagnosing human immunodeficiency virus (HIV) infection in neonates and infants, especially in
the perinatal period, is complicated by the prolonged persistence of maternal antibody following
delivery. Standard serologic tests for the detection of HIV are not reliable until infants are
approximately 18 months of age. In the neonatal period, infected infants produce only small
amounts of HIV-specific immunoglobulin G antibodies to a restricted number of antigens, and
these antibodies are obscured by maternal antibody, making them very difficult to measure.
The preferred test for the diagnosis of HIV infection in infants is HIV nucleic acid detection
by polymerase chain reaction (PCR) assay of DNA extracted from peripheral blood
mononuclear cells. About 30% of infants who are infected with HIV have a positive DNA PCR
assay result in samples obtained before 48 hours of age. A positive result identifies infants who
were infected in utero. Approximately 93% of infected infants have detectable HIV DNA by 2
weeks of age, and 100% of infected infants have positive HIV DNA PCR assay results by 1
month of age. A single HIV DNA PCR assay has a sensitivity of 95% and a specificity of 97% on
samples collected from infants 1 to 36 months of age. The HIV DNA PCR assay is more
sensitive on a single assay than viral culture.
Several alternative diagnostic methods have been suggested. Viral isolation by culture is
expensive, has limited availability, and requires up to 28 days for positive results. This test no
longer is recommended and has been replaced by the DNA PCR assay. The p24 antigen
detection test is substantially less sensitive than HIV DNA PCR assay or culture. Results may
vary, and the test is not recommended. HIV RNA PCR is not recommended for routine testing of
infants and children younger than 18 months of age because a negative result does not exclude
HIV infection definitely, especially if infection is acquired at the time of delivery, when HIV RNA
concentrations in the plasma may be quite low.
References:
American Academy of Pediatrics. Human immunodeficiency virus infection. In: Pickering LK,
Baker CJ, Long SS, McMillan JA, eds. Red Book: 2006 Report of the Committee on Infectious
Diseases. 27th ed. Elk Grove Village, Ill: American Academy of Pediatrics; 2006:378-401
Maldarelli F. Diagnosis of human immunodeficiency virus infection. In: Mandell GL, Bennett JE,
Dolin R, eds. Mandell, Douglas, and Bennett's Principles and Practice of Infectious Diseases. 6th
ed. Philadelphia, Pa: Elsevier Churchill Livingstone; 2005:1506-1526
Pollack H, Zhan MX, Ilmet-Moore T, Ajuang-Simbiri K, Krasinski K, Borkowsky W. Ontogeny of
anti-human immunodeficiency virus (HIV) antibody production in HIV-1-infected infants. Proc
Natl Acad Sci USA. 1993;90:2340-2344. Available at: http://www.pnas.org/cgi/reprint/90/6/2340
Read JS and the Committee on Pediatric AIDS. Diagnosis of HIV-1 infections in children younger
than 18 months in the United States. Pediatrics. 2007;120:e1547-e1562. Available at:
http://pediatrics.aappublications.org/cgi/content/full/120/6/e1547

Copyright 2009 by the American Academy of Pediatrics

page 532

2009 PREP SA on CD-ROM


Question: 253

The parents of a 3-year-old boy who has polyuria and polydipsia ask if anything can be done for
their childs symptoms and what the prognosis is for toilet training. After confirming a normal
serum glucose value and a negative urine culture, you arrange for a water deprivation test at the
hospital. The test begins at 9 am, and assessments are made hourly. At 0900, the patient weighs
14.1 kg, the serum osmolality is 290 mOsm/kg H2O, and the urine osmolality is 120 mOsm/kg
H2O. The measurements made over the course of the test are summarized in Item Q253. Per
protocol, the patient is given no food or fluids intravenously or orally. Aqueous vasopressin is
administered subcutaneously at 1101, immediately after the 1100 laboratory samples are taken.
After 4 hours, the test is stopped, and the patient is allowed to drink to prevent hypovolemia.
Of the following, the MOST likely diagnosis is

A. central diabetes insipidus


B. nephrogenic diabetes insipidus
C. no abnormalities in urinary concentrating ability
D. primary (psychogenic) polydipsia
E. inconclusive results necessitating repeat testing

Copyright 2009 by the American Academy of Pediatrics

page 533

2009 PREP SA on CD-ROM


Critique: 253

Preferred Response: B

When a child presents with polyuria and polydipsia, it is imperative for the clinician to exclude
diabetes mellitus. Once excluded by a normal glucose value, as occurred in the vignette, the
clinician should test for normal urinary concentrating ability, which usually is measured on a first
morning urine specimen, making it difficult to obtain in the incontinent child. In this situation, a
water deprivation test may be necessary to assess urinary concentrating ability appropriately.
The test is conducted in the hospital due to the risks of dehydration in the patient who has
diabetes insipidus (DI) and cannot concentrate urine adequately. Younger children who are not
yet toilet trained require a urinary catheter to monitor urine output and allow for urine osmolar
testing.
In a healthy person, the water deprivation test results in weight loss, increased serum
osmolality, and progressive increase in urine osmolality. The effective urinary concentrating
ability demonstrates normal antidiuretic hormone (ADH) production and normal renal response
to hypovolemia. In contrast, when the 4-hour test was administered to the boy in the vignette, he
lost weight, developed a progressive increase in serum osmolality, but maintained a fixed urine
osmolality. These findings are consistent with DI. A trial of aqueous vasopressin (VP) was given
in an effort to differentiate between central DI (VP-responsive) and nephrogenic DI (VPunresponsive). The patient's lack of response to VP indicates that he has nephrogenic DI (NDI).
NDI is an X-linked disorder in 90% of cases and is caused by mutations in the V2R gene
(located on the X chromosome), which encodes for the arginine vasopressin type 2 receptor
(V2R) that is expressed on the basolateral membrane of the collecting tubules. When circulating
antidiuretic hormone is presented to the V2 receptor, water channels (aquaporin-2 [AQP2])
translocate to the luminal surface of the collecting tubule, resulting in water reabsorption. Ten
percent of NDI cases are autosomal recessive and caused by mutations in the AQP2 gene.
Alternative clinical possibilities in the setting of polyuria and polydipsia include primary
(psychogenic) polydipsia, a diagnosis that is excluded readily by a water deprivation test.
Affected patients have normal ADH production and normal tubular response. Accordingly, when
the plasma osmolality reaches 295 mOsm/kg H2O, the urine osmolality exceeds 800 mOsm/kg
H2O. Results of the water deprivation test for the boy in the vignette are conclusive,
demonstrating an abnormality in urinary concentrating ability that failed to respond to volume
depletion (excluding primary polydipsia) and aqueous vasopressin (excluding central DI).
References:
Goodyer P. Disorders of tubular transport. In: Kher KK, Schnaper HW, Makker SP, eds. Clinical
Pediatric Nephrology. 2nd ed. London, England: Informa Healthcare; 2007:317-336
Knoers NVAM, Monnens LAH. Nephrogenic diabetes insipidus. In: Avner ED, Harmon WE,
Niaudet P, eds. Pediatric Nephrology. 5th ed. Philadelphia, Pa: Lippincott Williams & Wilkins;
2004:777-787
Rose BD, Post TW. Hyperosmolal states-hypernatremia. In: Clinical Physiology of Acid-base
and Electrolyte Disorders. 5th ed. New York, NY: McGraw-Hill Medical Publishing Division;
2001:746-793

Copyright 2009 by the American Academy of Pediatrics

page 534

2009 PREP SA on CD-ROM


Question: 254

An 18-year-old boy who has mild persistent asthma presents to the emergency department with
a 2-week history of coughing and wheezing that has not improved with twice-daily use of his
beta2 agonist metered dose inhaler (MDI). On physical examination, the teenager is breathing
comfortably but often coughs and has audible expiratory wheezing. His vital signs are
appropriate for age, but a room air pulse oximetry reading is 95%. Chest radiography shows
some peribronchial streaking but no infiltrate, no consolidation, and a normal cardiac silhouette.
His only other medication is a medium-dose inhaled corticosteroid.
Of the following, the MOST appropriate management step is to

A. begin a course of outpatient oral steroids


B. change his steroid inhaler to one that combines a steroid and long-acting beta2 agonist
C. double his steroid MDI dose
D. increase his rescue beta2 agonist to every 4 hours
E. observe the patient

Copyright 2009 by the American Academy of Pediatrics

page 535

2009 PREP SA on CD-ROM


Critique: 254

Preferred Response: A

The primary goal of managing asthma exacerbations is early treatment, with special attention
paid to infants and those who are at high risk for fatal asthma. Early treatment strategies include
use of a written asthma plan, recognition of symptoms, appropriate intensification of therapy, and
removal of possible allergens or irritants.
The teenager described in the vignette has a history of persistent asthma and presents with
a typical asthma exacerbation of moderate severity. The 2007 Expert Panel Report 3:
Guidelines for the Diagnosis and Management of Asthma (available at
www.nhlbi.nih.gov/guidelines/asthma/asthgdln.pdf) provides an excellent resource to determine
the appropriate course of action for such patients. Two primary therapies advocated for asthma
exacerbations are short-acting beta2 agonists (SABAs) and oral corticosteroids (evidence level
A). For patients who have mild asthma exacerbations, a 24- to 48-hour course of a SABA is
recommended. Patients who respond completely to this intervention need no further therapy.
Patients who have moderate, severe, or life-threatening exacerbations should seek medical
attention, and corticosteroids should be initiated promptly.
Neither increasing the patient's SABA to every 4 hours nor observing him is an appropriate
option because he has had symptoms for 2 weeks without improvement. Initiation of a
combination steroid/long-acting beta2 agonist inhaler (eg, fluticasone/salmeterol or
budesonide/formoterol) would not benefit him because it would only add a longer-duration beta2
agonist to his current medication regimen.
Some clinicians advocate doubling the inhaled steroid dose at the onset of asthma
symptoms. However, this approach has not been shown to be effective in reducing the severity
or preventing progression of exacerbations. An emergency department study has supported
quadrupling the dose of an inhaled corticosteroid for 7 days at the onset of worsening
symptoms. This may obviate the need for oral steroids, but does not have the level of evidence
as is present for oral corticosteroids.
References:
Harrison TW, Oborne J, Newton S, Tattersfield AE. Doubling the dose of inhaled corticosteroid
to prevent asthma exacerbations: randomised controlled trial. Lancet. 2004;363:271-275.
Abstract available at: http://www.ncbi.nlm.nih.gov/pubmed/14751699
Keeley D, McKean M. Asthma and other wheezing disorders in children. BMJ Clinical Evidence.
2006. Available for subscription at:
http://clinicalevidence.bmj.com/ceweb/conditions/chd/0302/0302.jsp538Q1
National Heart, Lung, and Blood Institute. National Asthma Education and Prevention Program.
Expert Panel Report 3: Guidelines for the Diagnosis and Management of Asthma. 2007.
Available at: www.nhlbi.nih.gov/guidelines/asthma/asthgdln.pdf

Copyright 2009 by the American Academy of Pediatrics

page 536

2009 PREP SA on CD-ROM


Question: 255

A 15-year-old boy presents to the emergency department after falling off of his skateboard. He
was skating downhill at high speed when he hit a bump and fell off his board. He fell onto his left
shoulder and struck his abdomen on the curb. He now complains of left shoulder pain. On
physical examination, his heart rate is 110 beats/min, respiratory rate is 24 breaths/min and
shallow due to pain, and blood pressure is 130/75 mm Hg. He refuses to move his left shoulder.
His lung sounds are clear, and his abdomen is diffusely tender. Radiographs of his left shoulder
are reported as normal. You order an abdominal computed tomography (CT) scan.
Of the following, the CT scan is MOST likely to demonstrate

A. abdominal aortic dissection


B. duodenal hematoma
C. pancreatic transection
D. retroperitoneal hemorrhage
E. splenic laceration

Copyright 2009 by the American Academy of Pediatrics

page 537

2009 PREP SA on CD-ROM


Critique: 255

Preferred Response: E

Blunt abdominal trauma in children is second only to head injury as a cause of injury-related
mortality. The most commonly injured intra-abdominal organs are the liver and spleen,
comprising 60% of childhood blunt abdominal injuries, with renal, pancreatic, and bowel injuries
occurring much less frequently. Most pediatric solid organ injuries are diagnosed by computed
tomography (CT) scan, although ultrasonography is being used increasingly as an initial
screening test. Most solid organ injuries are managed nonoperatively.
The patient described in the vignette demonstrates classic signs and symptoms of a splenic
injury, with left upper quadrant tenderness and left shoulder pain due to diaphragmatic irritation.
Blunt force from the fall onto the curb is the likely mechanism. The CT scan is the diagnostic
modality of choice because it can demonstrate the injury and extent of splenic damage (Item
C255). With the exception of patients who have massive splenic disruption and hemodynamic
instability, spleen preservation is preferred to splenectomy, with 90% to 98% of patients
recovering fully without surgery.
Although thoracic aortic dissection may be seen following blunt chest trauma (steering wheel
versus chest) or a high-velocity deceleration mechanism, abdominal aortic dissection is an
uncommon injury, especially in children, following blunt trauma.
Duodenal hematomas typically are seen after a focal blow to the upper abdomen (the socalled "handlebar injury") or after a deceleration mechanism in a restrained passenger following
a high-speed motor vehicle crash. Accounting for only 2% of blunt abdominal injuries in one
study, these injuries are notoriously difficult to diagnose. They often are missed on initial,
noncontrast abdominal CTs, and affected patients may present days after the initial trauma with
vomiting and diffuse abdominal pain. A high index of suspicion related to the mechanism of injury
is the key to timely diagnosis.
Pancreatic injuries are uncommon and occur from similar mechanisms as intestinal injuries.
These injuries also may be missed on initial CT scans. Elevated pancreatic enzyme values and
persistent upper abdominal pain should prompt evaluation for a pancreatic injury. Conservative
treatment in patients without pancreatic duct injury usually leads to full recovery.
Retroperitoneal hemorrhage may result from significant renal trauma. Physical examination
findings include flank ecchymoses and back pain. A CT scan should demonstrate both the
presence of retroperitoneal blood as well as its source in the trauma victim.
References:
Holmes JF, Sokolove PE, Brant WE, et al. Identification of children with intra-abdominal injuries
after blunt trauma. Ann Emerg Med. 2002;39:500-509. Abstract available at:
http://www.ncbi.nlm.nih.gov/pubmed/11973557
Wegner S, Colletti JE, Van Wie D. Pediatric blunt abdominal trauma. Pediatr Clin North Am.
2006;53:243-256. Abstract available at: http://www.ncbi.nlm.nih.gov/pubmed/16574524

Copyright 2009 by the American Academy of Pediatrics

page 538

You might also like